Sesap16 Volume 1
Sesap16 Volume 1
Sesap16 Volume 1
~
lVJ
~·~·
AMERICAN COLLEGE OF SURGEONS
Inspiring Quality:
Highest Standards, Better Outcomes
lOO+years
Contents
Foreword........................................................................................................... vi
Ajit K. Sachdeva, MD, FRCSC, FACS
Volume I
Category 1
Head and Neck
ltemsl-21 ................................................................................................... 1
Critiques 1-21 ............................................................................................. 8
Category 2
Breast
Items 1-49 ................................................................................................... 25
Critiques 1-49 ............................................................................................. 36
Category 3 - Part I
Alimentary Tract
Items 1-45 ................................................................................................... 63
Critiques 1-45 ............................................................................................. 74
Category 3 - Part II
Alimentary Tract
Items 1-40 ................................................................................................... 99
Critiques 1-40 ............................................................................................. 114
Category 3 - Part III
Alimentary Tract
Items 1-39 ................................................................................................... 149
Critiques 1-39 ............................................................................................. 164
Category 3 - Part IV
Alimentary Tract
Items 1-40 ................................................................................................... 197
Critiques 1-40 ............................................................................................. 211
Category 4 - Part I
Abdomen
Items 1-30 ................................................................................................... 245
Critiques 1-30 ............................................................................................. 250
iv
Category 4 - Part II
Abdomen
Items 1-28 ................................................................................................... 263
Critiques 1-28 ............................................................................................. 274
Category 5
Vascular
Items 1-39 ................................................................................................... 335
Critiques 1-39 ............................................................................................. 346
Category 6
Endocrine
Items 1-27 ................................................................................................... 375
Critiques 1-27 ............................................................................................. 381
Volume II
Category 7 - Part I
Tranma
Items 1-37 ................................................................................................... 397
Critiques 1-3 7 ............................................................................................. 408
Category 7 - Part II
Trauma
Items 1-37 ................................................................................................... 431
Critiques 1-37 ............................................................................................. 444
Category 8 - Part I
Perioperative Care
Items 1-50 ................................................................................................... 531
Critiques 1-50 ............................................................................................. 540
Category 8 - Part II
Perioperative Care
Items 1-51 ................................................................................................... 563
Critiques 1-51 ............................................................................................. 576
v
Category 9
Surgical Critical Care
Items 1-56 ................................................................................................... 667
Critiques 1-56 ............................................................................................. 679
Category 10
Problems in Related Specialties
Items 1-47 ................................................................................................... 709
Critiques 1-47 ............................................................................................. 724
Category 11
Oncology
Items 1-47 ................................................................................................... 761
Critiques 1-47 ............................................................................................. 77 4
Category 12
Skin/Soft Tissue
Items 1-23 ................................................................................................... 805
Critiques 1-23 ............................................................................................. 812
Category 13
Legal/Ethics
Items 1-16 ................................................................................................... 827
Critiques 1-16 ............................................................................................. 834
Category 14
Patient Safety/Systems of Care
Items 1-10 ................................................................................................... 845
Critiques 1-10 ............................................................................................. 848
Foreword
The American College of Surgeons (ACS) Division of Education is delighted to bring
you the 16th Edition of the Surgical Education and Seif-Assessment Program (SESAP"'
16). Under the superb leadership of John A. Weigelt, MD, DVM, FACS, Medical Director
for SESAP 16, the SESAP Advisory Committee and the eight authoring committees have
done an outstanding job in developing this exceptional educational program.
For the past 45 years, SESAP has remained a premier educational resource for practic-
ing surgeons. It includes an innovative self-assessment and guided cognitive skills educa-
tion model, which is specially designed to promote excellence and expertise. SESAP has
been a valuable educational resource for senior surgery residents as well. Concerted efforts
have been made to build on this stellar record spanning four decades. The evidence-based
content of SESAP primarily addresses the core competencies of medical knowledge and
patient care, and it also focuses on some of the other core competencies. The content
of SESAP has been selected to promote excellence and expertise in surgery. It should
also be helpful in addressing the self-assessment requirements of Part 2 of Maintenance
of Certification (MOC) and in preparing for the Recertification Examination to fulfill
requirements of Part 3 of MOC.
The content of SESAP 16 was developed through a rigorous, multistep process that
involved extensive peer review at each step. Each of the eight authoring committees
developed, peer reviewed, and submitted items using a process that generated more than
1200 items, along with critiques and references. These were reviewed further for clinical
significance and educational value by the SESAP Advisory Committee, composed of the
two Co-Chairs from each of the authoring committees. This committee selected the final
850 items in 14 major categories for inclusion in SESAP 16.
SESAP 16 uses the totally redesigned educational model introduced by the Division of
Education in concert with the leadership of SESAP. The iterative model reinforces learning
and supports mastery of the content. Participants can earn a maximum of 90 AMA PRA
Category 1 Credits™, all of which can be used to fulfill self-assessment requirements.
SESAP participants will be able to claim Continuing Medical Education (CME) Credits
for individual categories after answering 80% of the questions correctly; however, the final
SESAP certificate for 90 credits will be issued only after all 14 categories are completed.
Participants not interested in earning CME Credits will not be required to achieve a spe-
cific score prior to moving on to the next category.
SESAP 16 is available in a number of formats to offer participants several options based
on their individual educational needs. The CME Web version or one of the correspond-
ing mobile apps must be used to earn CME Credits. A supplemental print version is also
available with the Web version. The technological improvements made in the last edition
of SESAP have been enhanced further. Specifically, SESAP 16 is optimized for mobile
devices and offers mobile apps for iPads, iPhones, Android tablets, and Android phones.
In addition, users can now exit the program at any time; upon returning, they will be able
to resume their study at the same location. Users can also switch among devices and apps,
and the data will synchronize automatically.
Certain limitations of the program are worthy of note. The items and critiques have
been thoroughly reviewed by the SESAP authoring committees as well as the SESAP
Advisory Committee, and the content reflects the opinions of the authors and the com-
mittees; it does not necessarily reflect the official positions of ACS. The authors have
attempted to provide the most updated information; however, new information relating to
the topic may have become available since the time of content development. SESAP 16
should be a useful resource in preparation for the Recertification Examination; however,
additional reading and preparation for the examination are strongly recommended.
vii
The Division of Education owes an immense debt of gratitude to Dr. Weigelt and the
authors of SESAP 16 for their outstanding efforts and willingness to share generously their
expertise to create this outstanding resource. I would like to recognize Ms. Patrice Gabler
Blair, Associate Director of the Division of Education, Ms. Belinda Andry, Manager for
SESAP and Library Services, and Ms. Chrysa Cullather, SESAP and Review Courses
Medical Editor, for their invaluable contributions during the entire process of development
of SESAP 16.
Enrollment in SESAP has grown substantially over the past few years. We are delight-
ed that you have selected SESAP 16 and sincerely hope you will find this educational
resource valuable in your professional activities. We look forward t6 receiving your com-
ments and suggestions.
Introduction
SESAY'" is celebrating its 45 year anniversary! SESAP 16 will be introduced by the
American College of Surgeons (ACS) Division of Education in October 2016. Forty-eight
ACS Fellows spent the last two years writing and refining new questions for this edition.
Our finished product adds to the SESAP heritage as a valuable self-assessment tool for sur-
geons of all ages and interests. Whether it is preparing for the American Board of Surgery
(ABS) Certification or Recertification Examinations, obtaining Part 2 of Maintenance of
Certification (MOC), or just satisfying one's own educational needs, SESAP has become
an essential component of life-long learning for the general surgeon.
The process of developing SESAP content has remained essentially unchanged since the
first edition was published in 1971. Topic areas reflect current clinical practice and stay
relatively constant; as new areas of interest to general surgeons develop, they are incor-
porated. Topics align with the ABS categories listed for the Recertification Examination.
Every question for each SESAP edition is new. Each of the eight SESAP authoring com-
mittees meets in-person twice: once for question review and once for critique review.
Questions are developed based on supporting literature and are discussed by all members
of each committee at the first meeting before being accepted. Authors then write a critique
for each question to explain the correct answer as well as each wrong answer. The com-
mittees reconvene, and each critique is reviewed and discussed by its committee before it
is accepted. Authors try to incorporate the best evidence and experience as each question
and critique is written.
A constant concern is when new information should be incorporated into SESAP.
Authors ask themselves whether a topic is ready for inclusion in SESAP. Often the answer
is '"no'· as we wait to see how new data enter clinical practice. Another concern is whether
a question addresses an issue pertinent to clinical practice. We sometimes get criticized for
questions that deal with rare conditions; however, a question about a rare condition often
allows for a better discussion of closely related topics that are more common.
As you might expect with 850 questions, some material will be repeated; this is a fea-
ture of SESAP. Each question presents the topic slightly differently; rarely do the questions
describe exactly the same points. We try to avoid conflicting information between ques-
tions. This is no easy task, given the various opinions often found in the literature and in
practice. We strive to reach a high level of concordance or at least to explain differences
that occur with current evidence.
Once all questions are accepted by each committee, the committee Co-Chairs select the
final questions to be published. The Co-Chairs and Medical Director of SESAP review the
content again and work together to try to avoid duplication and ensure relevance. After all
questions are chosen, the program is edited and prepared for publication.
SESAP 16 is divided into 14 categories. Some categories are large and broken into
parts. The program continues to be a self-assessment exercise with three different modes.
The first is the self-assessment mode in which the questions and critiques are reviewed.
Once the self-assessment mode is completed, the continuing medical education (CME) test
mode can be accessed. A score of 80% is necessary for each part and category to be suc-
cessfully completed. If a score of 80o/o is not obtained, the user can review the questions
again in the self-assessment mode. The CME test mode can be re-entered, and you will
see only those questions you missed the first time. You are allowed three tries to attain the
80% score. Your scores are summed on each test mode until the 80% threshold is reached.
Additionally, you will be able to see the questions that you missed for further study. Once
IX
a category is complete, you can claim AMA PR4 Category 1 Credit™. The entire pro-
gram is worth 90 CME Credits. This process produces an accurate self-assessment of the
learner's current clinic_al knowledge. The overall goal is to provide general surgeons with
a broad-based educational experience using current evidence-based literature and practical
experience. It is truly a component of life-long learning, in which we all engage.
Each SESAP edition evolves slightly from its predecessor. SESAP 16 is no different.
Some changes came from user feedback of SESAP 15. Let me enumerate a few:
• The number of SESAP committees was increased from seven to eight, allowing selec-
tion of the final 850 questions from a larger pool of questions.
The program allows the user to logout and return to the same location in the program.
In the past~ each section had to be completed before leaving the program or the user
would have to start over. This new functionality occurs in all three modes: Self-
Assessment Mode, CME Test Mode, and Self-Assessment Study Mode.
• The online version of SESAP 16 is mobile optimized for tablets and smart phones.
Mobile apps are available for iPads, Android tablets, iPhones, and Android phones.
This will increase the portability of SESAP.
As Medical Director for SESAP 16, I had the privilege of working with each committee
and all 48 ACS Fellows as they developed their sets of questions. The discussions regard-
ing each question were valuable educationally and perhaps the essence of these discus-
sions will become part of future editions. I wish to thank the staff within the Division of
Education at the College without whom we could not produce such a wonderful educa-
tional product. We all believe SESAP 16 is another fine edition. I think you will find it
useful as you prepare yourself for maintaining or improving your knowledge base.
r
Good reading and studying,
a /J4df
John A. Weigelt, MD, DVM, FACS
SESAP 16 Medical Director
x
•
AMERICAN COLLEGE OF SURGEONS
Inspiring Quality: . DIVISION OF EDUCATION
Highest Standards, Better Outcomes . Aca.dlr.d wlrh Comfn<ndotlon by th<
• AcCl<dlrarlon Council for Continuing Medical Ed<Kot.on
IOO+years
PROGRAM OBJECTIVES
SESAp® 16 (Surgical Education and Self-Assessment Program) is intended for general
surgeons. Upon completion of SESAp® 16, participants should be able to:
• Compare their cognitive knowledge at the beginning of the program with evidence-
based surgical content for self-assessment purposes,
• Achieve and demonstrate mastery of surgical content to reach predetermined stan-
dards across all categories,
• Apply newly acquired knowledge to surgical practice,
• Keep current with surgical advances, and
• Prepare for Recertification Examinations.
XI
HARDWARE/SOFTWARE REQUIREMENTS
The minimum system requirements for SESAP 16 include the following:*
Operating systems: Mac 10.9+; Windows 7; iPad !OS 9
Browsers: Chrome 48+, Internet Explorer 11 and Edge+, Firefox 45+, Safari 8 and 9;
cookies and JavaScript must be enabled; cookies and pop-up windows must be enabled
Processor: 2 GHz or faster 32-bit or 64-bit
Memory: At least 4 GB
Screen resolution: I 024 x 768 for iPad; optimal screen resolution is 1280 x 1024 or better
Mobile Operating systems: Apple iOS 9.0; Android 4.0.3+
*Performance on other browsers or platforms may vary. While the site functions best in modem browsers. it is
possible to access the content in older browsers. For best performance and security, please upgrade your system
to meet the minimum browser requirements. Adobe Reader is also required.
Participants can claim CME credit for individual categories. If a categoiy has multiple
parts, each part must be completed at the 80% level to claim credit for the entire category.
Participants may prefer to claim CME credit once after completing all the categories.
After achieving a score of 80% for each category, participants will be asked to complete
an overall evaluation form and submit data to the ACS using the Internet. Upon successful
submission, a CME certificate will be available to print. SESAP 16 is certified for credit
through October 2019.
DISCLOSURE POLICY
In accordance with the ACCME Accreditation Criteria, the American College of Surgeons
must ensure that anyone in a position to control the content of the educational activity has
disclosed all relevant financial relationships with any commercial interest. Therefore, it
is mandatory that both the program planning committee and authors complete disclosure
forms. Members of the planning committee were required to disclose all financial relation-
ships and authors were required to disclose any financial relationship as it pertains to the
content of their contribution. The ACCME defines a 'commercial interest' as "any entity
producing, marketing, re-selling) or distributing health care goods or services consumed
by, or used on, patients." It does not consider providers of clinical service directly to
patients to be commercial interests. The ACCME considers "relevant'' financial relation-
ships as financial transactions (in any amount) that may create a conflict of interest and
occur within the 12 months preceding the time that the individual is being asked to assume
a role controlling content of the educational activity.
The ACCME also requires that ACS manage any reported conflict and eliminate the
potential for bias. The planning committee members and authors were contacted and the
conflicts listed below have been managed to our satisfaction. However, if you perceive a
bias) please advise us of the circumstances on the evaluation.
Xll
The requirement for disclosure is not intended to imply any impropriety of such relation-
ships, but simply to identify such relationships through full disclosure, and to allow the
audience to form its own judgments regarding the content.
Authors
Tawnya L. Bowles, MD, FACS (Nothing to disclose)
David R. Brenin, MD, FACS (Consultant-Insitec; Principal Investigator-Theraclion)
Clay Cothren Burlew, MD, FACS (Nothing to disclose)
Casey M. Calkins, MD, FACS (Nothing to disclose)
Jeremy W. Cannon, MD, SM, FACS (Nothing to disclose)
Jeffrey G. Chipman, MD, FACS (Nothing to disclose)
Quyen D. Chu, MD, FACS (Nothing to disclose)
Jay J. Doucet, MD, MSc, FRCSC, FACS, RDMS (Consultant honorarium-
Ethicon Endo-Surgery, Inc.)
Dan Eisenberg, MD, MS, FACS (Consultant honorarium-Intuitive Surgical, Inc.)
Dina M. Elaraj, MD, FACS (Nothing to disclose)
Charles M. Friel, MD, FACS, FASCRS (Nothing to disclose)
David A. Gerber, MD, FACS (Consultant honorarium-Pathfinder Therapeutics, Inc.)
Nicole S. Gibran, MD, FACS (Principal Investigator-M6lnlycke Health Care; Principal
Investigator-Fibrocell; Principal Investigator-Cytori Therapeutics, Inc.)
Kamal M. F. Itani, MD, FACS (Principal Investigator-Sanofi; Principal Investigator-
Merck & Co., Inc.; Principal Investigator-Cubist Pharmaceuticals; Research
Development and Advisory Board member-Davol, Inc.; Principal lnvestigator-
Dr. Reddy's Laboratories)
Tari A. King, MD, FACS (Nothing to disclose)
Richard P. M. Koehler, MD, FACS (Nothing to disclose)
COL Robert B. Lim, MD, FACS (Nothing to disclose)
Linda L. Maerz, MD, FACS, FCCM (Nothing to disclose)
David J. Milia, MD, FACS (Article Reviewer honorarium-Oakstone Publishing, LLC)
Forrest 0. Moore, MD, FACS (Editor-Wiley-Blackwell)
Leigh A. Neumayer, MD, FACS (Nothing to disclose)
Daryl P. Pearlstein, MD, FACS (Nothing to disclose)
Walter E. Pofahl II, MD, FACS (Nothing to disclose)
Matthew B. Rossi, MD, FACS (Nothing to disclose)
Paul J. Schenarts, MD, FACS (Nothing to disclose)
David G. Sheldon, MD, FACS (Consultant-Intuitive Surgical, Inc.)
Timothy R. Siegel, MD, FACS (Nothing to disclose)
Lance E. Stuke, MD, MPH, FACS (Nothing to disclose)
William W. Turner, Jr., MD, FACS (Nothing to disclose)
J. Patrick Walker, MD, FACS (Nothing to disclose)
Travis P. Webb, MD, MHPE, FACS (Nothing to disclose)
Planning Committee
John A. Weigelt, MD, DVM, FACS, Chair (Consultant honorarium-Pfizer, Inc.;
Consultant honorarium-Astellas Pharma US, Inc.; Consultant honorarium-
Decisio Health)
Richard M. Bell, MD, FACS (Nothing to disclose)
David C. Borgstrom, MD, FACS (Nothing to disclose)
David A, Bull, MD, FACS (Nothing to disclose)
Mary E. Fallat, MD, FACS (Nothing to disclose)
Jason B. Fleming, MD, FACS (Nothing to disclose)
Amy J. Goldberg, MD, FACS (Nothing to disclose)
John B. Hanks, MD, FACS (Nothing to disclose)
Lorrie A. Langdale, MD, FACS (Nothing to disclose)
xiii
Staff
Ajit K. Sachdeva, MD, FRCSC, FACS (Nothing to disclose)
Patrice Gabler Blair, MPH (Nothing to disclose)
Belinda A. Andry (Nothing to disclose)
Chrysa M. Cullather (Nothing to disclose)
Each category includes items~ corresponding critiques, correct answers, and references.
In the print syllabus, the items and critiques within an individual category can be eas-
ily located by means of the thumb tabs. White numbers on a black background denote
items; black numbers on a grey background denote critiques.
At the end of each critique, a bolded letter indicates the preferred response. Each
critique is followed by 2 or more references. A comprehensive subject index is also
included at the end of each of the volumes. Answers can be recorded in the enclosed
Self-Scoring Booklet.
Follow the Apple and Google guidelines for installing the apps on your devices. After
the SESAP 16 app is installed and opened, enter your usemame and password. which
were provided in the Order Confirmation e-mail sent to you after purchasing SESAP 16.
Users should be able to move seamlessly between devices.
CONTACT INFORMATION
For technical support or specific For education-related questions, contact:
questions about SESAP. contact:
Chrysa M. Cullather, MS
Administrator, SESAP SESAP & Review Courses Medical Editor
Division of Education Division of Education
American College of Surgeons American College of Surgeons
633 North Saint Clair Street, 24th Floor 633 North Saint Clair Street, 24th Floor
Chicago, IL 60611 USA Chicago, IL 60611 USA
Telephone: (312) 202-5419 Telephone: (312) 202-5175
Fax: (312) 202-5011 Fax: (312) 202-5011
E-mail: sesap@facs.org E-mail: ccullather@facs.org
Items 1-21
DIRECTIONS: Each of the questions or incomplete statements is followed by 5 suggested
answers or completions. Select the answer that is the BEST in each case and fill in the
space containing the corresponding letter on the answer sheet.
Items 2-3
A 35-year-old woman presents with a 2-month history of an enlarged left upper neck lymph
node. She is not a smoker. Physical exam is significant for an approximately 3-crn firm,
nontender, left upper jugular lymph node that is not mobile. Fine needle aspiration (FNA)
biopsy shows acid-fast bacilli.
3. She undergoes treatment for cervical tuberculous lymphadenitis. Which of the follow-
ing is the most appropriate management?
(A) Surgical debulking and 6 months of antituberculosis drug therapy
(B) Corticosteroids and 6 months of antituberculosis drug therapy
(C) Antitumor necrosis factor agent and 6 months of antituberculosis drug therapy
(D) 6 months of antituberculosis drug therapy
(E) External beam radiation
4. A 28-year-old woman has a 3-cm, right lobe. thyroid mass confirmed as papillary can-
cer by preoperative fine needle aspiration. Preoperative ultrasound shows 4-5 lymph
nodes measuring 8-9 mm with fatty hila in the central neck. Preoperative CT shows
no adenopathy but reports an incidental finding of a retroesophageal right subclavian
artery. The operating surgeon should consider
(A) preoperative angiography.
(B) performing a total thyroidectomy with central lymph node dissection.
(C) the possibility of a nonrecurrent right laryngeal nerve.
(D) positioning on the operating room table with minimal cervical extension.
(E) evaluating for midline congenital deformities.
CATEGORY 1 3
5. A surgeon plans to do a total thyroidectomy and left modified lymph node dissection
for thyroid cancer. She plans to use intraoperative neuromonitoring (IONM). Which
of the following statements is true?
(A) The left recurrent laryngeal nerve is more reliably found than the right, making
neuromonitoring unnecessary.
(B) IONM is associated with substantially lower rates of voice alterations in patients
with thyroid resection for malignancy.
(C) The most accepted method for IONM uses an endotracheal tube with
surface electrodes.
(D) IONM is no better than nerve visualization in terms of voice alteration.
(E) The positive predictive value of nerve damage with IONM approaches 100%.
6. During a modified radical neck dissection, the spinal accessory nerve is visualized
(A) cephalad to the great auricular nerve on posterior surface of the
stemocleidomastoid (SCM).
(B) anterior to the carotid sheath at the level of the cricoid cartilage.
(C) at the posterior junction of the omohyoid and SCM muscles.
(D) caudal to the great auricular nerve on the posterior surface of the SCM.
(E) posterior to the carotid sheath at the level of the cricoid cartilage.
7. A 28-year-old trauma patient with a severe head injury and an unstable cervical spine
fracture is in the intensive care unit. He has been mechanically ventilated via an endo-
tracheal tube for 10 days and is not expected to be a good weaning candidate. Which
of the following statements regarding a surgical airway is true?
(A) His neck should be extended to improve exposure to the trachea.
(B) Percutaneous tracheostomy has a higher rate of short- and long-term
complications than open tracheostomy.
(C) Bovie electrocautery is safe for the tracheotomy ifnitrous oxide is used.
(D) Surgical cricothyroidotomy is not used as an elective surgical airway.
(E) The optimal placement of the tracheotomy is at the level of the fourth
cartilaginous ring.
8. Regarding the use of fine needle aspiration histology for characterizing thyroid nod-
ules, which of the following statements is true?
(A) It can discriminate between benign and malignant nodules in up to 90o/o of
cases.
(B) It should always be performed with ultrasound guidance for maximum
reliability.
(C) It identifies cancer in 25-30% of thyroid nodules greater than 1 cm.
(D) A gene expression panel assay is indicated for indeterminate aspirates.
(E) Aspirates of thyroid nodules identified on 18-FDG-PET are rarely malignant.
4 CATEGORY 1
9. A 40-year-old man presents with a thyroid mass. Physical examination also reveals
raised lesions on the tongue and a large head circumference. Family history is positive
for "thyroid problems'" in multiple relatives, as well as breast, uterine, and colon can-
cer. Fine needle aspiration of the thyroid mass reveals malignant papillary cells. The
most likely inherited syndrome is
(A) multiple endocrine neoplasia type 2.
(B) Camey complex.
(C) Von Hippel-Lindau disease.
(D) familial adenomatous polyposis.
(E) Cowden syndrome.
10. A 36-year-old man presents with an enlarging, nontender, palpable node in the right
posterior neck. A fine needle aspiration is read as melanoma. The patient has no history
of melanoma, and a skin examination is negative for an obvious primary tumor. The
next step should be
(A) open lymph node biopsy.
(B) referral for an ophthalmologic examination.
(C) modified neck dissection.
(D) neck radiation.
(E) PET/CT scan.
11. A 35-year-old quadriplegic has a chronic tracheostomy and is hospitalized for sep-
sis. He develops ventilator-associated pneumonia and abrupt intratracheal bleeding.
Which of the following is the most appropriate first step in management?
(A) Replacement oftracheostomy with endotracheal tube
(B) Removal oftracheostomy tube and digital compression
(C) Immediate transport to the operating room
(D) Overinflation of the tracheostomy cuff
(E) CT-angiography scan of the neck
12. An 18-year-old man presents with a midline mass in his neck. His history is significant
for removal of an infected thyroglossal duct cyst when he was 6 years of age. Which
of the following statements is true?
(A) An extended course of intravenous antibiotics should be initiated.
(B) Further therapy requires surgery and external beam radiation.
(C) Histology from this mass would demonstrate papillary cells with swirled bands
of collagen.
(D) Neck CT should be performed.
(E) The primary recurrence rate after excision is approximately I 0%.
13. A 75-year-old male former heavy smoker complains of persistent hoarseness for sev-
eral months. He had a left carotid endarterectomy 5 years ago and 2 coronary stents
placed 3 years ago. Direct laryngoscopy showed left vocal cord paralysis. The re-
mainder of his physical exam is unremarkable. Which of the following studies is most
likely to elucidate the cause of his hoarseness?
(A) CT of his neck
(B) CT of his thorax
(C) PETscan
(D) Brain MRI
(E) Laryngeal electromyography
CATEGORY I 5
14. A patient who underwent a left modified radical neck dissection 2 days ago presents
with thick, white surgical drain output that was previously serous. The drain output is
200 m.L daily, and the left neck is flat without signs of infection. The patient is afebrile.
Which of the following is the best initial treatment option?
(A) Wound exploration
(B) Antibiotics
(C) Somatostatin
(D) Strict fat-free diet
(E) Removal of the surgical drain
15. Which of the following statements regarding oropharyngeal squamous cell carcinoma
(OPSCC) is true?
(A) OPSCC is less common than oropharyngeal adenocarcinoma.
(B) Human papillomavirus (HPV)-positive OPSCCs are epidemiologically similar
to HPV-negative OPSCCs.
(C) HPV subtype 1 accounts for the majority ofHPV-positive OPSCCs.
(D) The incidence of OPSCC is increasing.
(E) Survival rates of OPSCC are decreasing.
16. A 23-year-old woman presents with this painless mass beneath her left jaw, which has
been enlarging for 4 weeks (figure 16.1 ). The patient is HIV-positive but otherwise as-
ymptomatic. By palpation, the mass is firm, nontender, and mobile. Needle aspiration
returns no purulence, and the Gram stain and culture of the aspirate show no bacteria.
Her CD4 count is 850/µL (> 500 cells/µL), and her viral load is 50 copies/m.L. The
next best step is
(A) treatment with antiretroviral therapy.
(B) an empiric trial of antibiotics.
(C) repeat needle aspiration.
(D) surgical excision.
(E) observation.
Figure 16.1
17. A 72-year-old man presents with a basal cell carcinoma in the center of his forehead
confirmed by punch biopsy (figure 17.1 ). The lesion, which is his first, has been slowly
growing over the past 6 months. He has no cl inical adenopathy in the area. Which of
the following statements concerning this lesion is true?
(A) The lesion is commonly associated with basal cell nevus syndrome.
(B) The most common genetic mutation is in the k-ras gene.
(C) The lesion should be considered "high risk" for recurrence.
(D) Sentinel lymph node biopsy should be considered.
(E) A 3-mm clear surgical margin is adequate.
Figure 17.1
18. A 56-year-old woman presents with a 2-cm papillary thyroid cancer (right lobe). She
has biopsy-proven right central compartment nodal metastasis (level VI). Total thy-
roidectomy with right central neck node dissection is planned. Which of the following
describes the limits of the anatomic boundaries of a unilateral neck dissection for this
patient?
19. Before excision of a thyroglossal duct cyst. which of the following tests is recom-
mended?
(A) Fine needle aspiration
(B) Radioiodine scanning
(C) MRl
(D) Laryngoscopy
(E) Neck ultrasound
20. The most common finding related to a cranial nerve injury after carotid endarterec-
tomy consists of
(A) hoarseness.
(B) tongue deviation.
(C) lower lip drooping.
(D) impaired swallowing.
(E) shoulder droop.
21. A 60-year-old healthy woman had a right cheek melanoma completely resected 9
months ago. She had no lymphadenopathy at the time of the primary resection. She
now presents with a 1.5-cm right preauricular mass. Biopsy of the mass reveals meta-
static melanoma. Full-body imaging shows a 2-cm enlarged lymph node at level II of
the right neck but no distant metastatic disease. Which of the following is the next best
treatment option?
(A) Interferon-alpha
(B) Right superficial parotidectomy and right cervical lymph node dissection
(C) Ipilimumab
(D) Radiation to the right parotid and right neck
(E) Re-excision of the right cheek and sentinel lymph node biopsy
8 CATEGORY I
Critiques 1-21
CRITIQUE 1
The greater auricular nerve (GAN) originates from the cervical plexus and consists of
branches from C2 and C3. The GAN winds around the posterior border of the stemoclei-
domastoid muscle and ascends on that muscle beneath the platysma to the parotid gland,
where it divides into anterior and posterior branches. It is therefore most often injured or
sacrificed during parotid operations to expedite mobilization of the inferior pole of the
parotid or during facelifts. It is sensory to the preauricular and postauricular skin and parts
of the lower posterior ear, especially the lobular region (figure I. I). Injury leads not only
to numbness but also to functional deficits, such as difficulty shaving or wearing earrings
and an increased risk of neuromas, bums, and traumatic injuries. Nonetheless, numbness
associated with GAN injury usually resolves quickly, often within a few months; in I study,
57% of patients had normal sensation by I year.
Auricular branch
of the vagus nerve
Sensation to the upper ear is provided by the auriculotemporal nerve (ATN), which is a
branch of the trigeminal nerve. Frey syndrome (gustatory sweating) is caused by aberrant
regeneration of the parasympathetic fibers of the ATN after injury to that nerve, most com-
monly after parotidectomy.
References
I. George M, Karkos PD, Dwivedi RC, Leong SC, Kim D, Repanos C. Preservation of
greater auricular nerve during parotidectomy: sensation, quality of life, and morbidity
issues. A systematic review. Head Neck. 2014;36(4):603-608.
2. Hu J, Ye W, Zheng J, Zhu H, Zhang Z. The feasibi lity and significance of preservation
of the lobular branch of the great auricular nerve in parotidectomy. Int J Oral Maxillofac
Surg. 2010;39(7):684-689.
3. Witt RL, Pribitkin EA. How can Frey's syndrome be prevented or treated following
parotid surgery? Laryngoscope. 2013;123(7):1573-1574.
CATEGORY 1 9
CRITIQUE2
Tuberculous lymphadenitis of the cervical lymph nodes is the most common form of extra-
pulmonary tuberculosis and represents approximately 10% of cases of tuberculosis in the
United States. Most patients have no systemic symptoms and a normal chest x-ray. Tuber-
culosis is contagious even in patients without laryngeal or pulmonary involvement; there-
fore, a high index of suspicion is required when operating on patients with suspected tuber-
culous lymphadenitis, and airborne infection isolation precautions should be undertaken.
Patient factors that demand a higher index of suspicion include patients on immunosup-
pressive medications, with AIDS, and from countries where tuberculosis is endemic. If fine
needle aspiration biopsy is nondiagnostic, excisional biopsy is preferred over incisional
biopsy, because incisional biopsy is associated with an increased risk of fistula formation.
Answer: (A) The most common site is the cervical lymph nodes.
References
1. Roberts DS, Dowda!l JR, Winter L, Sulis CA, Grillone GA, Grundfast KM. Cervi-
cal tuberculosis: a decision tree for protecting healthcare workers. Laryngoscope.
2008; 118(8): 1345-1349.
2. Fontanilla JM, Barnes A, van Reyn CF. Current diagnosis and management of periph-
eral tuberculous lymphadenitis. C/in Irifect Dis. 201 ;53(6):555-562.
CRITIQUE3
The Infectious Disease Society of America recommends 4 months of isoniazid, rifampin,
pyrazinamide, and ethambutol, followed by 2 months of isoniazid and rifampin, for a total
of 6 months of therapy as treatment for tuberculous lymphadenitis. The expected cure rate
is approximately 90%. Corticosteroids are used for local discomfort, but no controlled
studies show a benefit of routine corticosteroid therapy with respect to cure or relapse rates.
Similarly, no controlled studies show a benefit of routine antitumor necrosis factor agents
with respect to cure or relapse rates. Surgical debulking can be considered for patients
who experience worsening of symptoms during medical treatment (known as paradoxical
upgrading reaction or PUR) or for those who fail medical treatment, but it is not a part of
the initial management. External beam radiation is not an appropriate treatment for tuber-
culous lymphadenitis.
References
I. Fontanilla JM, Barnes A, von Reyn CF. Current diagnosis and management of periph-
eral tuberculous lymphadenitis. C/in Infect Dis. 201;53(6):555-562.
2. Lienhardt C, Raviglione M, Spigelman M, et al. New drugs for the treatment of tuber-
culosis: needs, challenges, promise, and prospects for the future. J Infect Dis. 2012;205
(suppl 2):S241-S249.
CRITIQUE4
The anomaly of the origin and the course of the right subclavian artery was described more
than 200 years ago. In 1789, Bayford reported to the Medical Society of London a "curious
case of esophageal obstruction," which he linked to a right subclavian artery originating
from the left of the midline and crossing behind the esophagus. He originated the term
"dysphagia lusoria' describing a "quirk of nature." Soon this abnormal artery came to be
known as the arteria lusoria.
10 CATEGORY I
This patient presents with an incidental finding of a retroesophageal course of the right
subclavian artery. A nonrecurrent inferior laryngeal nerve usually occurs on the right side
and has a frequency ofless than 1%. This nonrecurrent course of the right nerve is typically
associated with the abnormal course of the right subclavian artery originating from a left
aortic arch.
Recent understanding of embryologic development explains the abnormal course of
the right nonrecurrent laryngeal nerve. During normal embryologic development, the right
nerve travels under the fourth right aortic arch, which will form the right subclavian artery.
In the case of atypical regression of the fourth right aortic arch, the nerve will then be as-
sociated with the third right aortic arch and will leave the vagus nerve at the level of the
carotid bifurcation and take a more direct lateral course into the larynx, thus forming the
nonrecurrent right laryngeal nerve.
No data justify the need for further evaluation with angiography. There is no adenopathy
on preoperative imaging; therefore~ central neck dissection is not necessary. Congenital
midline abnormalities, such as a thyroglossal duct cyst~ are not associated with nonrecur-
rent right laryngeal nerves. Finally, bed positioning will not help in optimal performance
of the surgery in this case.
References
I. Henry JF, Audiffret J, Denizot A, Plan M. The nonrecurrent inferior laryngeal nerve:
review of33 cases, including two on the left side. Surgery. 1988;104(6):977-984.
2. BrauckhoffM, Thanh NP, Dralle H. Images in thyroidology. Nervus laryngeus inferior
non recurrens and lusorial artery. Thyroid. 2004;14(1):79-81.
3. Avisse C, Marcus C, Delattre JF, et al. Right nonrecurrent inferior laryngeal nerve and
arteria lusoria: the diagnostic and therapeutic implications of an anatomic anomaly.
Review of 17 cases. Surg Radio! Anat. 1998;20(3):227-232.
4. Fellmer PT, Bohner H, Wolf A, Roher HD, Goretzki PE. A left nonrecurrent inferior
laryngeal nerve in a patient with right-sided aorta, truncus arteriosus communis, and an
aberrant left innominate artery. Thyroid. 2008;18(6):647-649.
5. Wang TS, Cheung K, Farrokhyar F, Roman SA, Sosa JA. A meta-analysis of the effect
of prophylactic central compartment neck dissection on locoregional recurrence rates in
patients with papillary thyroid cancer. Ann Surg Oneal. 2013;20(11):3477-3483.
CRITIQUES
This question of adjunctive use of intraoperative neuromonitoring (IONM) during the
course of thyroid resection has been a '"hot topic" among endocrine and otolaryngology
surgeons over the last decade. There is widespread agreement that nerve damage can lead
to significant and possibly life-threatening alterations in voice, laryngeal, and upper ain.vay
function. The incidence of voice alterations remains low but still occurs with a frequency
of0.2-0.8% in academic and educational hospitals. Several methodologies for IONM are
accepted: endotracheal tube with electrodes, glottic observation, and intramuscular vocal
cord electrodes. No methodology is clearly better.
A European collaborative group analyzing more than 16,000 consecutive operations at
multiple institutions identified risk factors for recurrent laryngeal nerve damage, including
recurrent benign and malignant diagnoses. Based on their data, visual nerve identification
was identified to be the gold standard for evaluating the recurrent laryngeal nerve during
thyroid surgery. Recurrent benign and malignant thyroid disease emerged as strong risk
factors.
Another European study done at approximately the same time evaluated 288 patients
and concluded that the incidence of recurrent laryngeal nerve lesions in benign, malignant,
CATEGORYl 11
and recurrent thyroid disease was not lowered by using IONM. A positive predictive value
of a test is the probability that the patients who test positive have a nerve injury. This
study pointed out several factors that produce alterations in the neuromonitoring signaling
that are not associated with voice changes: these would include traction or intraoperative
stretching of the nerve. In this study, positive predictive value for nerve injury using neuro-
monitoring was 33%. By contrast, the negative predictive value, indicating a negative test
and an intact recurrent laryngeal nerve, approached 1OOo/o.
A systematic review with meta-analysis of studies comparing IONM and visualization
alone during thyroidectomy summarized international results of more than 23,000 patients.
The meta~analysis demonstrated no statistically significant difference in the incidence of
recurrent laryngeal nerve palsy using IONM versus visualization during the time of sur-
gery.
Use of the IONM on one side or the other is not favored by any studies.
Answer: (D) IONM is no better than nerve visualization in terms of voice alteration.
References
I. Dralle H. Sekulla C, Haerting J, et al. Risk factors of paralysis and functional
outcome after recurrent laryngeal nerve monitoring in thyroid surgery. Surgery.
2004; 136(6): 1310-1322.
2. Beldi G, Kinsbergen T, SchlumpfR. Evaluation ofintraoperative recurrent nerve moni-
toring in thyroid surgery. World J Surg. 2004;28(6):589-591.
3. Pisanu A, Porceddu G, Podda M, Cois A, Uccheddu A. Systematic review with
meta-analysis of studies comparing intraoperative neuromonitoring of recurrent
laryngeal nerves versus visualization alone during thyroidectomy. J Surg Res.
2014;188(1 ):152-161.
CRITIQUE6
The spinal accessory nerve is a critical landmark during neck dissection. It separates level
IIA and IIB lymph nodes. Injury to the nerve is associated with shoulder debility. There-
fore, knowledge of its anatomic course is important to ensure adequate and appropriate
lymphadenectomy and to avoid injury.
The spinal accessory nerve exits the skull through the jugular foramen with cranial
nerves IX and X. It passes anterior to the jugular vein in the majority of cases. The nerve
then enters the posterior stemocleidomastoid muscle (SCM). It exits posteriorly in the
vicinity of Erb point, which is where the bundle of sensory nerves (including the great
auricular nerve) emerges from the posterior border of the SCM. This is a useful landmark
for locating the accessory nerve. In the referenced studies, the spinal accessory nerve was
noted to exit the SCM on average 1.5 cm above (cephalad) to Erb point.
Although the nerve does pass anterior to carotid sheath, it occurs much more cephalad
than at the level of the cricoid cartilage. The junction of the omohyoid and SCM muscles
is too low in the neck for the location of the nerve.
Answer: (A) cephalad to the great auricular nerve on the posterior surface of the stemo-
cleidomastoid (SCM).
References
I. Salgarelli AC, Landini B, Bellini P, Multinu A, Consolo U, Collini M. A simple
method of identifying the spinal accessory nerve in modified radical neck dissection:
anatomic study and clinical implications for resident training. Oral Maxillofac Surg.
2009; 13(2):69-72.
12 CATEGORY I
CRITIQUE7
Tracheostomy procedures and management of tracheal airways are important skills for the
general surgeon to master in the critically ill patient. Newer intensive care unit techniques
for percutaneous placement oftracheostomy tubes with bronchoscope guidance have good
results and offer the advantage of a bedside procedure. In large meta-analyses of data com-
paring open versus percutaneous airways, percutaneous airways are not associated with
a higher incidence of complications in critically ill patients but are associated with more
procedural technical difficulties.
Many techniques are possible for a surgical airway. Urgent airway creation with crico-
thyroidotomy is a necessary skill for the management of trauma patients and is a compo-
nent of the American College of Surgeons Advanced Trauma Life Support program skills
assessment. Although cricothyroidotomy can be a life-saving intervention in the airway-
compromised patient, conversion to tracheostomy when convenient in the subsequent
24--48 hours is recommended. This recommendation is made to avoid the possibility of
subglottic laryngeal stenosis.
A tracheostomy is performed after adequate exposure at the level of the second or third
tracheal ring; placement any lower in the trachea puts the patient at risk for erosion of the
tube or cuff into the innominate artery, resulting in a tracheoinnominate fistula. Placing
the incision in the first tracheal ring runs the risk of injuring the cricoid and thyroid carti-
lage, which can increase the risk of stenosis. Exposure of the trachea can be improved by
dividing the isthmus of the thyroid and extending the neck. Neck extension should not be
performed in the setting of a cervical spine fracture. The tracheotomy should be performed
sharply, without the use of electrosurgical instruments, because the high oxygen content
of inspired gasses can cause an ainvay fire. Even when nitrous oxide is used, oxygenation
concentrations in the ainvay will be adequate to ignite if a spark is generated.
References
I. Walts PA, Murthy SC, DeCamp MM. Techniques of surgical tracheostomy. Clin Chest
Med. 2003;24(3):413-422.
2. Whited RE. A prospective study of laryngotracheal sequelae in long-term intubation.
Laryngoscope. 1984;94(3):367-377.
3. Halum SL, Ting JY, Plowman EK, et al. A multi-institutional analysis of tracheotomy
complications. Laryngoscope. 2012;122(1):38-45.
4. Putensen C, TheuerkaufN, Guenther U, Vargas M, Pelosi P. Percutaneous and surgical
tracheostomy in critically ill adult patients: a meta-analysis. Crit Care. 2014; 18(6):544.
CRITIQUES
The use of fine needle aspiration biopsy (FNA) is standard in the management of thy-
roid nodules. However, 15-30% of thyroid nodules evaluated on FNA yield indeterminate
findings. Ultimately, only 5-15% of thyroid nodules larger than 1 cm are proven to be
malignant. FNA of palpable nodules is reliable in experienced hands without the use of
ultrasound. In nonpalpable thyroid nodules, ultrasound use maximizes the ability to obtain
adequate specimens.
CATEGORY I 13
The Bethesda System for Reporting Thyroid Cytopathology (table 8.1) is used to clas-
sify FNA biopsy results into 6 categories ranging from insufficient for diagnosis (Bethesda
category I) to definitely malignant (Bethesda category VI). Categories II (benign) and V
(suspicious for malignancy) can be managed accordingly based on FNAresults alone. Cat-
egories III (atypical, or follicular lesion of uncertain significance) and IV (follicular neo-
plasm) are indeterminate lesions and need further characterization for management. These
lesions historically led to surgical thyroid lobectomy, despite the fact that a large majority
were ultimately proven to be benign.
Bethesda
Category Description Management
I Insufficient for diagnosis
II Benign Managed accordingly based on FNA results alone
III Atypical, or follicular lesion Need further characterization for management
of uncertain significance
IV Follicular neoplasm Need further characterization for management
v Suspicious for malignancy Managed accordingly based on FNA results alone
VI Definitely malignant
Table 8.1 The Bethesda System for Reporting Thyroid Cytopathology.
A gene profile assay with the ability to analyze RNA from FNA biopsy specimens can
be applied to Bethesda category III and IV results to predict benign behavior accurately.
The assay has a high negative predictive value in histologically benign lesions and is useful
to avoid surgery in Bethesda category III and IV lesions.
Fluorodeoxyglucose-PET scanning demonstrates incidental thyroid abnormalities
in 2-3% of patients, but 14-47% of these thyroid abnormalities ultimately prove to
be malignant.
Answer: (D) A gene expression panel assay is indicated for indeterminate aspirates.
References
I. American Thyroid Association (ATA) Guidelines Taskforce on Thyroid Nodules and
Differentiated Thyroid Cancer; Cooper DS, Doherty GM, Haugen BR, et al. Revised
American Thyroid Association management guidelines for patients with thyroid
nodules and differentiated thyroid cancer [published correction appears in Thyroid.
20 I 0;20(8):942.]. Thyroid. 2009; 19(11):1167-1214.
2. Alexander EK, Kennedy GC, Baloch ZW, et al. Preoperative diagnosis of benign thyroid
nodules with indeterminate cytology. N Engl J Med. 2012;367(8):705-715.
3. Katz SC, ShahaA. PET-associated incidental neoplasms of the thyroid. J Am Coll Surg.
2008;207(2):259-264.
4. Cibas ES, Ali SZ; NCI Thyroid FNA State of the Science Conference. The Bethesda
System for reporting thyroid cytopathology Am J Clin Pathol. 2009;132(5):658-665.
CRITIQUE9
Thyroid cancers can be divided into medullary thyroid cancers (MTCs) and nonmedul-
lary thyroid cancers (NMTCs). Both can be inherited, depending on the type of germline
mutation. Most cases of familial thyroid cancer are NMTCs (mostly papillary) and present
in familial cancer syndromes such as familial adenomatous polyposis, Cowden syndrome
(multiple hamartomas, acral keratosis, oral papillomas, trichilemmomas, breast and uterine
tumors), Carney complex (myxomas, pituitary adenomas, adrenal hyperplasia, Cushing
syndrome, schwannomas, testicular tumors), Pendred syndrome, and Werner syndrome.
These syndromes are usually autosomal dominant with reduced penetrance.
14 CATEGORY I
The patient described has a family history and exam typical of Cowden syndrome. Such
patients have oral hamartomas, commonly on the tongue (figure 9. 1) and a large head circum-
ference. Patients with familial adenomatous polyposis do not have these phenotypic findings.
References
1. Ho TH, Jonasch E. Genetic kidney cancer syndromes. J Natl Compr Cane Netw.
2014: 12(9): 1347-1355.
2. Metzger R, Milas M. Inherited cancer syndromes and the thyroid: an update. Curr Opin
Oncol. 2014;26(1 ):51-6 1. PubMed
3. Richards ML. Familial syndromes associated with thyroid cancer in the era of personal-
ized medicine. Thyroid. 20 10;20(7):707- 713.
CRITIQUE IO
Although most patients with advanced melanoma have a known primary tumor (melanoma
of known primary; MKP), 2-6% of patients will present with no history or current evi-
dence of a primary (melanoma of unknown primary; MUP). A 1963 recommendation by
Das Gupta called for ophtbalmoscopy, otoscopy, nasopharyngoscopy, sigmoidoscopy, and,
in women, full gynecologic examination, to completely rule out an occult primary before
making a definitive diagnosis of MUP. A recent review showed that these tests are rarely
useful, and most melanoma centers recommend routine history and physical examination.
In particular, ophthalmologic examinations should be reserved for MUP patients with vis-
ceral metastases, especially to the liver.
CATEGORY I 15
Most patients with MUP present with nodal metastases; of these, the most common sites
in descending order are the axilla, groin, and head and neck. Per National Comprehensive
Cancer Network melanoma guidelines for nodal disease, fine needle aspiration is the pre-
ferred method of biopsy and should be followed by baseline imaging, such as a PET/CT
and brain MRI, to rule out distant metastatic disease. If none is found, the patient has stage
III disease and should be treated accordingly, with neck dissection and consideration for
adjuvant radiation and systemic therapy, on or off a clinical trial.
Four theories are usually proposed to explain the occurrence ofMUP: (1) the patient has
a clinically undetected primary, (2) the patient had a primary previously excised that was
either incorrectly diagnosed or not submitted for pathologic examination, (3) the patient
had a primary that underwent spontaneous involution, or (4) the melanoma arose de nova
in an intranodal melanocyte. The fourth theory may explain the fact that MUPs have the
same or a better stage-for-stage prognosis as MKPs.
References
I. Cormier JN, Xing Y, Feng L, et al. Metastatic melanoma to lymph nodes in patients with
unknown primary sites. Cancer. 2006;106(9):2012-2020.
2. National Comprehensive Cancer Network. NCCN Guidelines Melanoma. Version
2.2016. Fort Washington, PA: National Comprehensive Cancer Network.
3. Tos T, Klyver H, Drzewiecki KT. Extensive screening for primary tumor is redundant in
melanoma of unknown primary. J Surg Oneal. 2011; 104(7):724-727.
CRITIQUE 11
This patient has a possible tracheoinnominate artery fistula (TIF) with intratracheal bleed-
ing. TIF is a surgical emergency with a high mortality rate. Reported incidence is 0.1-1.0%
after tracheostomy, with peak incidence 3 days to 6 weeks after the procedure. Most pa-
tients manifest minor bleeding episodes before a massive bleed. TIF is usually fatal once
it massively bleeds. For the successful management of TIF, treatment should be initiated
immediately for hemorrhage control before definitive surgical management.
The following maneuvers can be considered to control bleeding and secure the airway
until the start of definitive surgical intervention:
• The tracheostomy cuff should be overinflated to provide compression of the fistula
bleeding.
• Flexible bronchoscopy should be done through the tracheostomy tube to clear and
secure the airway.
• Digital compression should be applied around or through the tracheostomy incision.
An attempt to manipulate the tracheostomy tube, like transoral endotracheal tube in-
tubation, is absolutely contraindicated, because it exacerbates the ongoing intratracheal
bleeding. CT imaging delays surgical intervention and is not recommended.
If bleeding can be locally controlled, operative intervention is the treatment. A me-
dian sternotomy is performed to gain access to the innominate artery. Most surgeons
advocate ligation and resection of the fistulous segment of the innominate artery with-
out vascular reconstruction.
References
!. Grant CA, Dempsey G, Harrison J, Jones T. Tracheo-innominate artery fistula after
percutaneous tracheostomy: three case reports and a clinical review. Br J Anaesth.
2006;96(1):127-131.
2. Komatsu T, Sowa T, Fujinaga T, Randa N, Watanabe H. Tracheo-innominate artery fistula:
two case reports and a clinical review. Ann Thorac Cardiovasc Surg. 2013;19(1):60-62.
CRITIQUE 12
Thyroglossal duct cysts originate from persistent epithelial remnants of the thyroglossal
duct. Initial presentation occurs in the proximity of the hyoid bone in approximately 60%
of the cases. The main aim of surgical treatment is the complete removal of the cyst and the
duct. The Sistrunk procedure includes resection of the thyroglossal duct cyst, tract, midline
portion of the hyoid bone, and cuff of the surrounding base of tongue musculature.
Preoperative infection is the most common complication of thyroglossal duct cysts,
with rates ranging from 10 to 70%. Even with appropriate surgery, the mean rate of re-
currence is 10. 7%. With a past medical history of a thyroglossal duct cyst diagnosis and
resection, there is no role for an extended course of intravenous antibiotics or adjunctive
external beam radiation. Having 2 or more preoperative infections is associated with an
elevated recurrence rate.
Malignant changes in thyroglossal duct cysts are very rare, occurring in less than I% of
cases. Malignancy would be suspected ifthe cyst were hard, fixed, or irregular or displayed
sudden expansion with palpable neck lymph nodes.
The diagnosis of recurrent thyroglossal duct cyst infection is frequently made postop-
eratively by histopathologic examination of the resected tissues. Routine preoperative neck
CT is not indicated.
Answer: (E) The primary recurrence rate after excision is approximately 10%.
References
1. Galluzzi F, Pignataro L, Gaini RM, Hartley B, Garavello W. Risk of recurrence in
children operated for thyroglossal duct cysts: A systematic review. J Pediatr Surg.
2013;48(1 ):222-227.
2. Choi YM, Kim TY, Song DE, et al. Papillary thyroid carcinoma arising from a thyro-
glossal duct cyst: a single institution experience. Endocr J. 2013;60(5):665--070.
3. Simon LM, Magit AE. Impact of incision and drainage of infected thyroglossal duct
cyst on recurrence after Sistrunk procedure. Arch Otolaryngol Head Neck Surg.
2012; 138( 1):20-24.
CRITIQUE 13
Commonly, unilateral vocal cord paralysis (UVCP) is brought to the attention of the physi-
cian because the patient complains of hoarseness, but its effect on voice, swallowing, and
even airway function may be significant. The most common cause of left-sided UVCP is
dysfunction of the left recurrent laryngeal nerve (RLN) caused by adjacent neoplasm, most
commonly lung or esophageal cancer. The evaluation of left UVCP depends on a detailed
knowledge of the anatomy of the RLN. The left RLN courses underneath the aortic arch in
the aortopulmonary window. Tumor at the left hilum of the lung or involving the aortopul-
monary window lymph nodes can directly invade the left RLN. A CT of the thorax is the
most effective imaging study to diagnose a lung or esophageal neoplasm.
A standard CT of the neck would be useful in the workup ofa palpable neck mass, par-
ticularly in a smoker, to evaluate for a head/neck neoplasm. A neck CT might also be useful
in the evaluation ofright-sided UVCP, because the anatomy of the right RLN is different;
CATEGORYl 17
it courses underneath the right subclavian artery. For example, an apical tumor in the right
lung could infiltrate the right RLN.
A PET scan may be useful to diagnose a lung or esophageal malignancy, but it should
not be the initial study due to high cost and lack of specificity. In addition, several studies
have shown that when UVCP is present, the contralateral normal vocal fold can have high
fluorodeoxyglucose uptake due to compensatory motion, potentially raising a misleading
concern for laryngeal malignancy. Brain MRI does not have a role in evaluating UVCP,
because the lesion is not in the central nervous system but rather is a result of damage to
the RLN. Laryngeal electromyography is a useful study to evaluate the prognosis of vocal
cord function returning but does not have a role in establishing the etiology of the UVCP.
References
1. Misono S, Merati AL. Evidence-based practice: evaluation and management of unilat-
eral vocal fold paralysis. Otolaryngol Clin North Am. 2012;45(5): 1083-1108.
2. Schwartz SR, Cohen SM, Dailey SH, et al. Clinical practice guideline: hoarseness
(dysphonia). Otolaryngol Head Neck Surg. 2009: 141 (3 suppl 2):S l-S3 l.
CRITIQUE 14
A postoperative chyle leak after neck dissection is most commonly due to injury to the
thoracic duct in the lower aspect of the left neck (level IV), where the duct empties into the
left subclavian vein. Initial management ofa low output (less than 1 L/day) chyle leak con-
sists of a strict fat-free diet. Medium-chain triglyceride diets are also used. A functioning
surgical drain should be left in place. Prophylactic antibiotics are not recommended. So-
rnatostatin may reduce the volume of chyle produced. The mechanism of action is poorly
understood, and studies supporting its use are lacking.
A strict, fat-free or medium-chain triglyceride diet will reduce the volume of chyle
produced. However, this diet will not provide adequate nutrition for the patient ifused for
a protracted period of time. For high output or persistent chyle leaks, parenteral nutrition
may be needed.
Wound exploration should be done only in the early postoperative setting if the surgical
drain is not adequately decompressing the leak or if there are signs of infection not respon-
sive to antibiotics.
Surgery is reserved for high-volume, persistent chyle leaks that have failed nonopera-
tive management. Options include open or thoracoscopic ligation of the thoracic duct in
the chest. Embolization of the thoracic duct by interventional radiologic techniques is pos-
sible too. Direct neck exploration for repair is often not successful due to the thin-walled,
delicate nature of the duct and associated postoperative inflammation.
References
1. Smith PW, Hanks JB. Evaluation of the isolated neck mass. In: Cameron JL, Camer-
on AM, eds. Current Surgical Therapy. 11th ed. Philadelphia, PA: Saunders Elsevier;
2014:718-723.
2. Brennan PA, Blythe JN, Herd MK, Habib A, Anand R. The contemporary manage-
ment of chyle leak following cervical thoracic duct damage. Br J Oral Maxillofac Surg.
2012;50(3):197-201.
18 CATEGORY 1
CRITIQUE 15
Of the head and neck malignancies arising from the upper aerodigestive tract (lip, oral cav-
ity, pharynx, and larynx), laryngeal cancer is the most common cause of death, but pharyn-
geal cancer has the highest incidence. There is a male preponderance in aerodigestive tract
malignancies. Tobacco and alcohol abuse are risk factors; combined abuse is multiplicative
with respect to odds ratio. Additional risk factors include human papilloma virus (HPV)
and Epstein-Barr virus infection, Plummer-Vinson syndrome, metabolic polymorphisms,
malnutrition, and occupational exposure to mutagenic agents. According to the National
Cancer Database, squamous cell carcinoma (SCC) is the most common histologic type of
head and neck tumor. The majority (89%) of cancers arising from the mucosa! surfaces of
the upper aerodigestive tract are secs.
HPV-positive and HPV-negative head and neck SCCs may be considered as 2 distinct
types of cancer. In particular, HPV causes an epidemiologically and clinically distinct form
of oropharyngeal squamous cell carcinoma (OPSCC). HPV-associated cancers affiict indi-
viduals who are younger in age and are less likely to be associated with alcohol or tobacco
abuse. The increased incidence of OPSCC is attributed to HPV infection secondary to
orogenital transmission among young adults.
HPV-positive patients have substantially better survival compared with HPV-negative
patients. High-risk HPV strains include subtype 16, which accounts for a majority (90-
95%) ofHPV-positive OPSCCs. Improved survival of patients with head and neck SCC in
recent years is associated with an increased incidence of HPV-positive tumors compared
with HPV-negative tumors. Therefore, increases in the incidence and survival of OPSCC
in the United States since 1984 may be attributed to HPV infection.
References
1. Cooper JS, Porter K, Mallin K, et al. National Cancer Database report on cancer of the
head and neck: I 0-year update. Head Neck. 2009;31(6):748-758.
2. Chaturvedi AK, Engels EA, Pfeiffer RM, et al. Human papillomavirus and rising oropha-
ryngeal cancer incidence in the United States. J Cl in Oncol. 2011;29(32):4294-4301.
CRITIQUE 16
The correct diagnosis of head and neck masses in HIV-infected patients is problematic,
particularly in lesions that are of presumed salivary origin. Generally, the parotid is the
most commonly involved salivary gland, similar to individuals with competent immune
systems. Etiologies encompass the entire spectrum from infectious, congenital, degenera-
tive processes, and neoplastic disease. Acute infectious or suppurative lesions are usually
accompanied by pain or at least discomfort. Violaceous skin discoloration over the lesion
is commonly associated with suppurative lesions, and purulent material expressed from the
intraoral duct orifice is generally pathognomonic. Painless lesions, as in this patient, sug-
gest a more chronic, indolent course.
The differential diagnosis includes cat scratch disease (Bartone/la henselae, a Gram-
negative rod), sarcoidosis (diagnosed by indirect fluorescent antibody testing and in-
creased serum IgG titers), Wegener granulomatosis (usually accompanied by additional
visceral involvement), Kimura disease (manifest by increased levels of IgE and peripheral
eosinophilia), and malignancy. Immunocompromised individuals are especially prone to
uncommon infections, including mycobacterium, particularly if there is a history of expo-
sure. Mycobacterial infection in salivary tissue is frequently confused with neoplastic dis-
ease because of its painless, indolent course. Atypical mycobacteria infections-Mycobac-
terium kansasii, M avium, and M. scrofulaceum-are common in immunocompromised
patients and children. Benign lymphoepithelial lesions are associated with HIV infection.
CATEGORY I 19
These lesions are usually cystic or microcystic in nature, and ultrasonography may aid in
the diagnosis. Aggressive antiretroviral therapy is indicated for these lymphoepithelial le-
sions.
Considering the divergent therapies for each of the possible conditions, a definitive
diagnosis is essential. Fine needle aspiration (FNA) is frequently nondiagnostic but may
be an appropriate first step. Considering the initial cytologic results obtained in this case,
repeat FNA is unlikely to be of benefit. A trial of antibiotics should not be considered unti l
a specific etiologic agent has been identified. Additionally, an acute infectious etiology is
unlikely given the absence of pain or skin discoloration. Observation only delays a defi ni-
tive diagnosis and appropriate treatment.
The increased .incidence of malignancy in immunocompromised individuals, particu-
larly when minor salivary tissue is involved, should prompt surgical excision for com-
prehensive histopathological evaluation and appropriate culture. Thickening of the gland
capsule due to chronic inflammation may make surgical extirpation difficult, but it is still
the preferred approach for lesions of the salivary glands. Injury to the lingual nerve, which
crosses the Wharton duct twice, and the hypoglossal nerve must be avoided. The marginal
mandibular branch of the facial nerve is particularly vulnerable, because it runs in the
subplatysmal plane within the superficial layer of the deep cervical fascia. This layer is
contiguous with the capsule of the submandibular gland (figure 16.2). To avoid injury
to the marginal mandibular nerve, the facial vein may be ligated and retracted superiorly
(Hays-Martin maneuver), which keeps the marginal mandibular nerve out of the area of
dissection. The Wharton duct should be ligated as close to the oral cavity as possible. In
this patient, acid-fast bacteria were identified by tissue staining.
Temporal branches
Pa rotid
gland
Zygomatic branches
Posterio r
auricular
n eNe ~~~-.:.......;:~._~__;~ Parotid duct
Main trunk of
Ne rve to posterior
belly of digastric
muscle and to
stylohyoid muscle
References
l. Brook I. The bacteriology of salivary gland infections. Oral Maxillofac Surg Clin North
Am. 2009;21(3):269-274.
2. Carlson ER. Diagnosis and management of salivary gland infections. Oral Maxillofac
Surg Clin North Am. 2009;21(3):293-312.
3. Beahm DD, Peleaz L, Nuss DW, et al. Surgical approaches to the submandibular gland:
a review ofliterature. Jnt J Surg. 2009;7(6):503-509.
4. Nadershah M, Salama A. Removal of parotid, submandibular, and sublingual glands.
Oral Maxillofac Surg Clin North Am. 2012;24(2):295-305.
5. O'Connell JE, George MK, Speculand B, Pahor AL. Mycobacterial infection of the parot-
id gland: an unusual cause ofparotid swelling. J Laryngol Oto/. 1993;107(6):561-564.
CRITIQUE 17
Basal cell carcinoma (BCC) is the most common of the nonmelanoma skin malignancies,
approximately 5 times as common as cutaneous squamous cell cancers. The true incidence
is unknown, because these tumors generally are not included in tumor registries. Estimates
place the number of nonmelanoma skin malignancies at more than 1 million per year in
the United States.
BCCs are most common in the head and neck region (>80%) and on sun-exposed and
sun-damaged skin. Although ultraviolet radiation from sunlight and tanning beds is impli-
cated in the etiology of squamous cell cancers, the relationship with BCC is not as clear.
Mutations are identified in 90% of sporadic BCC, including p53, fos, and k-ras, but the
cause-effect relationship remains inconclusive.
Most of these lesions are considered low-grade malignancies, but some are not. "'High-
risk" lesions are associated with head and neck locations~ particularly lesions greater than
6 mm on the mask area of the face; recurrent tumors; tumors that arise in areas previously
radiated for unrelated conditions; tumors with ill-defined borders; morpheaform (desmo-
plastic histopathology); aggressive growth patterns; and those that occur in immunocom-
promised individuals, especially patients with solid organ transplants. Transplant patients
have a risk of cutaneous malignancy that is 13-15 times that of the average population,
BCC being the most common.
Metastases can occur and are primarily to regional lymph nodes, followed by lung,
liver, and bone when they occur. Sentinel lymph node biopsy is not recommended in the
guidelines from the National Comprehensive Cancer Network (NCCN) expert panel in the
absence of clinically detected regional adenopathy.
High-risk lesions require a more aggressive surgical approach, primarily with wider sur-
gical margins-6 mm or greater if possible-compared with the 4-mm margins for low- or
intermediate-risk BCC. Mohs micrographic surgery has the advantage of tissue conserva-
tion in cosmetically sensitive areas. Some reports suggest a reduction in recurrence with
this technique. Regardless of the surgical approach, complete peripheral and deep margin
assessment should be performed, either by permanent or frozen section, to exclude the pos-
sibility ofresidual tumor. NCCN guidelines recommend that closure of the surgical defect
with local tissue transfer not be performed before complete margin assessment.
Alternative treatment options, radiation therapy, curettage and electrodessication, pho-
todynamic therapy, and topical therapy with 5-fluorouracil or immune modulating agents
(e.g., imiquimod) are generally reserved for special circumstances (e.g., the elderly, co-
morbidities that preclude a surgical approach, compromise of function, positive margins).
Answer: (C) The lesion should be considered "'high risk" for recurrence.
CATEGORY]
21
References
1. Goppner D, Leverkus M. Basal cell carcinoma: from the molecular understand-
ing of the pathogenesis to targeted therapy of progressive disease. J Skin Cancer.
2011:2011 :650258.
2. National Comprehensive Cancer Network. NCCN Guidelines Basal Cell and Squamous
Cell Skin Cancers. Version 2.2014. Fort Washington, PA: National Comprehensive
Cancer Network.
3. Smith V, Walton S. Treatment of facial Basal cell carcinoma: a review. J Skin Cancer.
2011:2011 :380371.
4. Rubin Al, Chen EH, Ratner D. Basal-cell carcinoma. N Eng!J Med 2005;353(2 I ):2262-2269.
5. Miller SJ, Alam M, Andersen J, et al. Basal cell and squamous cell skin cancers. J Nari
Compr Cane Netw. 2010;8(8):836-864.
6. Nakayama M, Tabuchi K, Nakamura Y, Hara A. Basal cell carcinoma of the head and
neck. J Skin Cancer. 2011;2011:496910.
7. Swanson NA. Mohs surgery. Technique, indications, applications, and the future. Arch
Dermatol. 1983;119(9):761-773.
CRITIQUE 18
Adequate lymphadenectomy requires knowledge of the anatomic boundaries of dissection.
These are based on the lymphatic drainage of the thyroid. Figure 18. l shows these ana-
tomic landmarks. Compartment-oriented dissection lessens the risk of recurrence and co-
morbidity associated with re-exploration. A consensus statement by the American Thyroid
Association Surgery Working Group clarified the nomenclature associated with central
neck dissection.
Figure 18.1 Schematic anterior view of the neck indicating boundaries of the central
neck compartment.
Consensus Statement on the Terminology and Classification of Central Neck Dissection for Thyroid Cancer:
The American Thyroid Association Surgery Working Group with Participation from the American Association
of Endocrine Surgeons. American Academy of Otolaryngology-Head and Neck Surgery. and American Head
and Neck Society. Sally E Carty. David S Cooper, Gerard M Doherty, et al. Thyroid. 2009, 19(11 ): 1153-1158.
22 CATEGORYl
The hyoid bone is the superior border of the dissection_ The hypoglossal nerve is too
cephalad, and cricoid cartilage too caudal. The lateral border of dissection should extend
only to the carotid artery. Any further lateral would unnecessarily dissect level II-IV nodes.
The medial extent for a unilateral central neck dissection is the tracheal midline. The nodal
packet surrounding the recurrent nerve is excised. The contralateral tracheoesophageal
groove would be dissected only as part of a bilateral central neck node dissection. Inferior
dissection to the level of the omohyoid muscle is inadequate.
Answer:
References
1. American Thyroid Association Surgery Working Group; American Association of
Endocrine Surgeons; American Academy of Otolaryngology-Head and Neck Surgery;
American Head and Neck Society; Carty SE, Cooper DS, Doherty GM, et al. Consensus
statement on the terminology and classification of central neck dissection for thyroid
cancer. Thyroid. 2009; 19(11):1153-1158. PMID: 19860578.
2. Giugliano G, Proh M, Gibelli B, et al. Central neck dissection in differentiated thyroid
cancer: technical notes. Acta Otorhinolaryngol Ital. 2014;34(1):9-14.
CRITIQUE 19
Thyroglossal duct cysts are common lesions. The main caveat is to make certain that this
is not the patient's only thyroid issue. Neck ultrasound demonstrating thyroid tissue in
its nonnal anatomic location ensures this. Fine needle aspiration biopsy is not routinely
indicated, because the diagnosis is rarely in doubt and the risk of malignancy is rare. Ra-
dioiodine scanning is unnecessary, because ultrasound can demonstrate thyroid tissue and
avoid radiation exposure. MRI is not indicated, because it will not provide information that
cannot be obtained by ultrasound. Laryngoscopy does not add to the evaluation or treat-
ment planning.
References
1. Joseph J, Lim K, Ramsden J. Investigation prior to thyroglossal duct cyst excision. Ann
R Coll Surg Engl. 2012;94(3):181-184.
2. Ahuja AT, Wong KT, King AD, Yuen EH. Imaging for thyroglossal duct cyst: the bare
essentials. Clin Radial. 2005;60(2):141-148.
CRITIQUE20
The reported incidence of injury to cranial nerves after carotid endarterectomy varies from
less than 5% to more than 30%, depending on whether the study was conducted prospec-
tively or retrospectively, on variation in definition of cranial nerve injury, and on sensitiv-
ity of the investigative methods. Cranial nerves at potential risk for injury during carotid
endarterectomy include VII (facial), IX (glossopharyngeal), X (vagus), XI (accessory), and
XII (hypoglossal).
The hypoglossal nerve appears to be most susceptible to injury, because of its proximity
to the carotid bifurcation. Traction applied to expose the area can result in nerve paresis
with resulting tongue deviation toward the ipsilateral side.
CATEGORY 1
23
The vagus nerve is the next most commonly injured cranial nerve. The vagus nerve
usually lies posterior and lateral to the internal and common carotid but may be positioned
differently in the carotid sheath. The recurrent laryngeal nerve can be injured if it follows
an anomalous nonrecurrent course. In both cases, vocal cord palsy is the result. Injury to
the superior laryngeal nerve can occur but is less common. Consequences of injury to the
superior laryngeal nerve include shallow voice, fatigue of the voice, and a tendency to
aspiration, but these often improve with observation.
The marginal mandibular branch of the facial nerve, located between the platysma and
deep carotid fascia after emerging from beneath the parotid gland, innervates the muscles
of the lower lip and angle of the mouth. Positioning of the head away from the operative
site may bring the branch lower into the superior neck, thus making the nerve more vul-
nerable to injury. Injuring this nerve can result in drooping of the mouth on the same side.
Mobilization of the glossopharyngeal nerve and division of the posterior belly of the di-
gastric muscle and styloid process to gain additional exposure can result in glossopharyn-
geal nerve injury. Damage to the glossopharyngeal nerve can have several effects. These
effects include loss of bitter and sour taste and impaired swallowing. The gag reflex is
impaired on the ipsilateral side of a glossopharyngeal nerve injury.
Injury to the accessory nerve during carotid endarterectomy is rare; traction on the ster-
nocleidomastoid muscle is the proposed mechanism. Injury can result in trapezius muscle
dysfunction and winging of the scapula, resulting in a drooping appearance of the shoulder.
References
1. Sajid MS, Vijaynagar B, Singh P, Hamilton G. Literature review of cranial nerve inju-
ries during carotid endarterectomy. Acta Chir Belg. 2007;107(1):25-28.
2. Theodotou B, Mahaley MS Jr. Injury of the peripheral cranial nerves during carotid
endarterectomy. Stroke. I 985;16(5):894-895.
3. Tucker JA, Gee W, Nicholas GG, McDonald KM, Goodreau JJ. Accessory nerve injury
during carotid endarterectomy. J Vase Surg 1987:5(3):440--444.
4. Rosenbloom M, Friedman SG, Lamparello PJ, Riles TS, Imparato AM. Glossopharyn-
geal nerve injury complicating carotid endarterectomy. J Vase Surg. I 987;5(3):469--471.
CRITIQUE21
Regional nodal recurrence of melanoma, in the absence of distant metastatic disease, is
treated with resection. In this patient, the preauricular mass suggests nodal recurrence in
the parotid gland, which is a common site of lymph node drainage in melanomas of the
scalp. face, and ear. Melanoma patients with palpable disease in the parotid have a 28-58%
chance of occult disease in the neck. Therefore, superficial parotidectomy and cervical
lymph node dissection are recommended. The extent of the neck dissection is debated,
with many surgeons advocating a selective cervical lymph node dissection based on the
location of the primary melanoma and the predicted lymphatic drainage pattern. Sentinel
lymph node biopsy is used to detect microscopic nodal metastases, and it is not used for
clinically palpable nodal disease.
Interferon-alpha is currently the only adjuvant treattnent for resected, node-positive
melanoma approved by the US Food and Drug Administration. It is also approved for ad-
juvant treatment of resected stage IIB and IIC melanomas, which are thick primary mela-
nomas without node involvement. In this patient, interferon-alpha may be considered but
only after complete resection. Radiation may also be used as adjuvant treatment after full
resection of the nodal recurrence. Ipilimumab is a monoclonal antibody that activates the
immune system by blocking CTLA-4, a protein receptor on I-cells that downregulates the
immune system. Ipilimumab is approved for use in melanoma patients with unresectable
24 CATEGORY]
node-positive melanoma and in patients with distant metastatic disease. Studies evaluat-
ing ipilimumab in the adjuvant setting for resected node-positive melanoma patients are
ongoing.
Answer: (B) Right superficial parotidectomy and right cervical lymph node dissection
References
I. National Comprehensive Cancer Network. NCCN Guidelines Melanoma. Version
2.2016. Fort Washington, PA: National Comprehensive Cancer Network.
2. Mack LA, McKinnon JG. Controversies in the management of metastatic melanoma to
regional lymphatic basins. J Surg Oneal. 2004;86(4): 189-199.
Category 2
Breast
Items 1--49
DIRECTIONS: Each of the questions or incomplete statements is followed by 5 suggested
answers or completions. Select the answer that is the BEST in each case and fill in the
space containing the corresponding letter on the answer sheet.
Items 1-2
2. She accepts your recommendation for breast conservation but wants to know if she
needs a preoperative MRI. Current evidence indicates that preoperative MRI is associ-
ated with
(A) lower rates of positive margins.
(B) lower local recurrence rates.
(C) higher mastectomy rates.
(D) improved disease free and overall survival.
(E) lower rates of conversion to mastectomy.
6. A 75-year-old woman notices changes in her left nipple (figure 6.1 ). A mammogram
shows fatty-replaced breasts and no lesions. Physical examination is negative except
for the changes on the left nipple. The next step in her management is
(A) shave biopsy of the nipple.
(B) core needle biopsy of the subareolar tissue.
(C) full-thickness nipple biopsy.
(D) partial mastectomy.
(E) simple mastectomy.
Figure 6.1
7. Which of the following increases a woman's future risk of developing breast cancer?
(A) Sclerosing adenosis
(B) Fat necrosis
(C) Complex sclerosing lesions
(D) Intraductal papilloma
(E) Atypical ductal hyperplasia
28 CATEGORY2
8. A 25-year-old woman presents with 2 days of pain in the left lateral breast. On exami-
nation, you note a tender, cord-like structure without an associated breast mass. Which
of the following statements is true of this condition?
(A) Further evaluation for cancer is necessary.
(B) Treatment includes anti-inflammatory medications.
(C) Men are affected by this condition more often than women.
(D) Antibiotics are indicated in the majority of patients.
(E) Pulmonary embolism must be ruled out.
10. A 67-year-old man with a history of hypertension, prostate cancer, and gastroesopha-
geal reflux disease presents with progressive and painful bilateral breast swelling. The
patient takes spironolactone, ftutamide, and a proton pump inhibitor (PP!). Examina-
tion shows bilaterally enlarged breasts with symmetric, firm, tender subareolar tissue
without nipple discharge, dominant masses, or adenopathy. The best next step is
(A) liposuction.
(B) change the PP! to an Hz-receptor blocker.
(C) breast biopsy.
(D) a trial oftamoxifen.
(E) bilateral total mastectomy.
11. A 40-year-old woman presents with newly diagnosed breast cancer. Her family history
is positive for breast cancer in a maternal aunt at age 55, extremity soft tissue sarcoma
in her brother at age 25, and childhood astrocytoma in her sister. Her father died of
adrenocortical carcinoma at age 42. She most likely has a mutation of which of the
following genes?
(A) BRCA I
(B) BRCA2
(C) p16
(D) p53
(E) PTEN
12. The need to return to the operating room for re-excision after lumpectomy for breast
cancer
(A) occurs at a constant rate nationwide.
(B) is reduced by intraoperative frozen section.
(C) is expected to increase in the future.
(D) does not affect morbidity.
(E) does not affect cosmetic results.
13. You are seeing a 32-year-old woman in your office. Her aunt was recently diagnosed
with breast cancer at age 75. Your patient wishes to know about the influence of
hormone therapy on her risk of breast cancer. Which of the following statements is
most accurate?
CATEGORY:?: 29
(A) Oral contraceptive agents do not increase the risk of breast cancer.
(B) The risk for breast cancer should be based on female hormone levels.
(C) Hormone replacement therapy increases the risk of breast cancer in women.
(D) Combination hormone replacement therapy is safer than estrogen only.
(E) Hormone replacement therapy reduces the risks of cardiovascular disease
despite the effect on breast cancer risks.
14. A 65-year-old woman presents with a 1.5-cm ER-positive invasive ductal cancer and
extensive ductal carcinoma in situ (DCIS) of the left breast that requires mastectomy.
She has no family history of breast cancer. She has heard that the risk of the cancer
"going to the other breast'' is very high. She requests a bilateral mastectomy. The pa-
tient should be advised that
(A) because she has extensive DCIS in the left breast, she is likely to develop the
same type of breast cancer in the right breast.
(B) there is no increased risk of complications when comparing bilateral
mastectomy with unilateral mastectomy.
(C) bilateral mastectomy will improve her chances of survival.
(D) bilateral mastectomy will reduce her risk of local recurrence.
(E) endocrine therapy will decrease her risk of a contralateral cancer.
15. A 32-year-old female smoker, 8 months postpartum who is not breast feeding, presents
with a painful, tender breast. She reports a 6-month history of chronic waxing and
waning pain, erythema, and swelling under her left nipple. Yesterday, she noted puru-
lent drainage at the border of the areola with improvement of symptoms. The patient
described most likely is suffering from
(A) a peripheral, nonlactational breast abscess.
(B) a puerperal abscess.
(C) hidradenitis suppurativa.
(D) granulomatous lobular mastitis.
(E) a periductal fistula.
16. A 58-year-old woman undergoes a lumpectomy and sentinel node biopsy for invasive
breast cancer, followed by routine whole breast radiation therapy. Her risk of develop-
ing measurable upper-extremity lymphedema
(A) is not affected by radiation therapy.
(B) can be stratified based on preoperative lymphoscintigraphy.
(C) is 7%, 6 months after surgery.
(D) is increased by weight lifting.
(E) is unaffected by sentinel node biopsy.
17. A 56-year-old woman undergoes excision and sentinel lymph node biopsy for a 3-cm
breast mass. Pathology examination reveals a high-grade lesion that is ER-negative,
PR-negative, HER2-negative by fluorescence in situ hybridization (FISH). The sen-
tinel lymph node is negative for tumor. In addition to radiation therapy, which of the
following adjuvant treatment options would be appropriate for this patient?
(A) Tamoxifen
(B) Letrozole
(C) Trastuzumab
(D) Cytotoxic chemotherapy
(E) No systemic adjuvant treatment
30 CATEGORY 2
18. Which of the following is a risk factor for breast cancer in a male patient?
(A) Occupation in a cold environment
(B) Asian heritage
(C) History of gynecomastia
(D) Alcoholism
(E) Exposure to electromagnetic fields
19. A 28-year-old woman tested positive for a BRCA2 mutation. She does not have a
personal history of breast cancer. In counseling her, which of the following statements
is correct?
(A) Prophylactic bilateral mastectomy will improve her overall survival.
(B) Skin-sparing mastectomy is contraindicated.
(C) Prophylactic bilateral salpingo-oophorectomy will not reduce her risk of
developing breast cancer.
(D) Prophylactic bilateral salpingo-oophorectomy will improve her overall survival.
(E) Neither prophylactic bilateral mastectomy nor salpingo-oophorectomy will
improve her overall survival.
20. A 55-year-old woman presented with the following breast mass (figure 20.1) and a
palpable axillary lymph node. A core needle biopsy demonstrated an ER-negative in-
filtrating ductal carcinoma. Which of the following is the next step in management?
(A) Modified radical mastectomy
(B) Radiation therapy
(C) Neoadjuvant endocrine therapy
(D) Sentinel lymph node biopsy
(E) Neoadjuvant chemotherapy
Figure 20.1
CATEGORY 2 31
21. Which of the following statements is true regard ing intraductal papilloma of
the breast?
(A) It is the most common benign breast mass.
(B) It should be excised with a 1-cm negative margin.
(C) It can be distinguished from malignant d isease on ductography.
(D) lt may occur peripherally, spari ng the main ducts.
(E) It is a risk factor for the developme nt of breast cancer.
22. A 32-year-old patie nt in her second trimester is di agnosed with an in vasive 2-cm
ductal carcino ma of her left breast, ER/PR-positive. Whic h of the following sta te-
me nts is true?
(A) Surgery sho uld be pe rformed only in the second trimester.
(B) Modifi ed rad ical mastectomy is the preferred option. regardless of
gestational age.
(C) Therapy with tamoxife n should be deferred until after pregnancy.
(D) Che motherapy with doxorubicin or cyclophosphamide sho uld be avoided during
pregnancy.
(E) Sentinel node biopsy is contraindicated during pregnancy.
23. A 31-year-old woman presents with tenderness and swelling of her left breast w ith
prog ressive surrounding erythe ma, as shown in figure 23. l . She is currently breast-
feeding. An ultrasound of this area demonstrates a 2-cm x 2.5-cm simple subareolar
fluid collection. In addition to pain control. which of the following is the next best step
in management?
(A) Apply warm compresses to the affected area.
(B) Avo id breastfeedi ng fro m either breast.
(C) Administer broad-spectrum intravenous antibiotics.
(D) Aspirate the fluid and administer antibiotics for Gram-positive organisms.
(E) Perform wide surgical incision and drainage.
Figure 23.1 Used with permission of Daniel Rosenthal MD FACS FASCRS DMCC, Col. MC
USA(Ret).
32 CATEGORY 2
24. A 35-year-old woman (G1P1AB0) is seen with refractory mastalgia. Her pain is bi-
lateral and has been present for many years. It typically begins about 2 weeks before
the onset of menses and abates gradually after menses begin. She has tried avoiding
xanthines, reducing dietary fat, adding vitamin supplements, taking nonsteroidal anti-
inflammatory drugs, and other conservative treatments without success. The pain is so
severe that it disrupts her social and sexual life and causes her to miss work 3--4 days
each month. She asks you to consider bilateral mastectomy if the pain cannot be con-
trolled. Her physical examination and breast ultrasound identify no dominant mass.
The best treatment at this point is
(A) danazol.
(B) tamoxifen.
(C) evening primrose oil.
(D) bromocriptine mesylate.
(E) bilateral mastectomy.
27. A 50-year-old postrnenopausal woman with no family history of breast cancer under-
went bilateral breast reduction surgery and was found to have lobular carcinoma in
situ in the reduction specimen from the left breast. Her preoperative mammogram was
normal, and she has no history of prior breast abnormalities. The patient should be
advised that lobular carcinoma in situ
CATEGORY2
33
28. A 30-year-old woman presents with a palpable mass in the upper outer quadrant of the
right breast. On physical exam, the mass is nontender, round, and mobile and measures
1.0 cm. There are no other findings on exam. Ultrasound-guided core biopsy reveals
infiltrating ductal carcinoma, wh.ich is ER-negative, PR-negative, and HER2-negative.
Her mother was also diagnosed with breast cancer at the age of35. Which of the fol-
lowing is the most appropriate next step in her management?
(A) Neoadjuvant chemotherapy
(B) Mastectomy
(CJ Genetic counseling
(D) Excisional breast biopsy
(E) Preoperative radiation therapy followed by mastectomy
29. The survival benefit of women who undergo prophylactic contralateral mastectomy in
the setting of unilateral breast cancer
(A) is less than I%.
(B) is less beneficial in younger women compared with older women.
(C) increases by stage.
(D) decreases with positive estrogen receptor status.
(E) decreases if the patient has a BRCA mutation.
Items 30-32
(A) Atypical lobular hyperplasia
(B) Atypical ductal hyperplasia
(CJ Both
(D) Neither
Items 33-37
(A) Aromatase inhibitors
(B) Tamoxifen
(C) Both
(D) Neither
33. Used as adjuvant therapy for postmenopausal women with ER-positive breast cancer
34. Used as adjuvant therapy for premenopausal women with ER-positive breast cancer
36. Reduces the risk of contralateral breast cancer in women with ER-positive breast cancer
37. Reduces the risk of local recurrence in women with ER-positive breast cancer
Items 38-43
(A) Fibroadenoma
(B) Phyllodes tumor
(C) Both
(D) Neither
41. Enucleation
Items 44-46
(A) BIRADS-5 mammogram
(B) BIRADS-4B mammogram
(C) Both
(D) Neither
Items 47-49
(A) Fibroadenoma
(B) Atypical ductal hyperplasia
(C) Both
(D) Neither
Critiques 1-49
CRITIQUE 1
In the United States, rates of contralateral breast cancer among women treated for unilater-
al breast cancer have been on the decline since 1985. Surveillance, Epidemiology, and End
Results (SEER) data reported in 2011 demonstrate that the annual hazard for contralateral
breast cancer ranges from 0.3 to 0.5o/o per year for women with an ER-positive first breast
cancer. As such, 3-5% of women treated for an ER-positive unilateral breast cancer will
develop contralateral breast cancer in the subsequent I 0 years.
Rates of contralateral breast cancer are affected by treatment of the index lesion. In the
setting of ER-positive breast cancer, the use of tamoxifen in premenopausal women and
aromatase inhibitors in a postmenopausal woman contribute to decreasing rates of contra-
lateral breast cancer. Rates of contralateral breast cancer for women with a ER-negative
first breast cancer are slightly higher but remain under I% per year in all but the youngest
patients (age <30).
Answer: (B) 5%
References
I. Nichols HB, Berrington de Gonzalez A, Lacey JV Jr, Rosenberg PS, Anderson WF.
Declining incidence of contralateral breast cancer in the United States from 1975 to
2006. J Clin Oneal. 2011;29(12):1564--1569.
2. Hartman M, Czene K, Reilly M, et al. Incidence and prognosis of synchronous and
metachronous bilateral breast cancer.JC/in Oneal. 2007;25(27):4210-4216.
3. Baum M, Budzar AU, Cuzick J, et al; ATAC Trialists' Group. Anastrozole alone or in
combination with tamoxifen versus tamoxifen alone for adjuvant treatment of post-
menopausal women with early breast cancer: first results of the ATAC randomised
trial [published correction appears in Lancet. 2002;360(9344):1520]. Lancet.
2002;359(9324):2131-2139.
4. Davies C, Godwin J, Gray R, et al. Early Breast Cancer Trialists' Collaborative Group
(EBCTCG). Relevance of breast cancer hormone receptors and other factors to the effi-
cacy of adjuvant tamoxifen: patient-level meta-analysis of randomised trials. Lancet
2011;378(9793):771-784.
5. Early Breast Cancer Trialists' Collaborative Group (EBCTCG), Davies C, Godwin J,
Gray R, et al. Relevance of breast cancer hormone receptors and other factors to the
efficacy of adjuvant tamoxifen: patient-level meta-analysis of randomised trials. Lancet.
2011 ;378(9793):771-784.
CRITIQUE2
The increased sensitivity of MRI generated great enthusiasm for its use in selecting pa-
tients for breast conservation. However, cumulative data from prospective trials, several
well-designed case-control cohort studies, and an individual person-level meta-analysis
have all failed to demonstrated that MRI results in lower rates of positive margins; decreas-
es the need for secondary surgical procedures (re-excision or conversion to mastectomy);
or results in improved rates of local recurrence, disease-free, or overall survival. The use of
preoperative MRI consistently results in increased use of mastectomy.
References
1. Turnbull L, Brown S, Harvey I, et al. Comparative effectiveness of MRI in breast cancer
(CO MICE) trial: a randomized controlled trial. Lancet. 2010;375(9714):563-571.
2. Pilewskie M, King TA. Magnetic resonance imaging in patients with newly diagnosed
breast cancer: a review of the literature. Cancer. 2014;120(14):2080-2089.
3. Houssami N, Turner R, Morrow M. Preoperative magnetic resonance imaging in breast
cancer: meta-analysis of surgical outcomes. Ann Surg. 2013;257(2):249-255.
4. Houssami N, Turner R, Macaskill P, et al. An individual person data meta-analysis of
preoperative magnetic resonance imaging and breast cancer recurrence. J Clin Oncol.
20l4;32(5):392-401.
CRITIQUE3
Patients with lifetime risk exceeding 20-25% should receive a screening MRI of the breast.
This patient population typically includes women with a strong family history of breast
or ovarian cancer, as well as patients treated with mantle irradiation for Hodgkin disease.
Screening MRI is also indicated for women with BRCA mutation, as well as those not
tested who have a first-degree relative with BRCA mutation.
Current evidence is insufficient to recommend MRI for screening women with lifetime
risk ofless than 15-20%, as defined by various models that largely depend on family histo-
ry. Other conditions that do not warrant MRI screening include lobular carcinoma in situ or
atypical lobular hyperplasia (ALH), atypical ductal hyperplasia (ADH), heterogeneously
or extremely dense breast on mammography, or women with a personal history of breast
cancer, including ductal carcinoma in situ (DCIS).
References
1. Bevers TB, Anderson BO, Bonaccio E, et al; National Comprehensive Cancer Network.
NCCN clinical practice guidelines in oncology: breast cancer screening and diagnosis
[published correction appears in J Natl Compr Cane Net»: 2010;8(2):xxxvii]. J Natl
Compr Cane Netw. 2009;7(10):1060-1096.
2. Saslow D, Boetes C, Burke W, et al; American Cancer Society Breast Cancer Advisory
Group. American Cancer Society guidelines for breast screening with MRI as an adjunct
to mammography [published correction appears in CA Cancer J Clin. 2007;57(3):185].
CA Cancer J Clin. 2007;57(2):75-89.
3. Miller BT, Abbott AM, Tuttle TM. The influence of preoperative MRI on breast cancer
treatment. Ann Surg Oneal. 2012; 19(2):536-540.
4. Houssami N, Turner R, Macaskill P, et al. An individual person data meta-analysis of
preoperative magnetic resonance imaging and breast cancer recurrence. J Clin Oneal.
20l4;32(5):392-401.
CRITIQUE4
Pleomorphic lobular carcinoma in situ (PLCIS) is a recently described entity with distinct
clinical, histologic, and imaging findings; it differs from classic lobular carcinoma in situ.
PLCIS shares many histomorphic, genetic, and biomarker characteristics with invasive
lobular cancer and ductal carcinoma in situ. PLCIS is frequently associated with an inva-
sive cancer, so current recommendations are for excision when it is found on core needle
biopsy. The risk of PLCIS found on core needle biopsy being upgraded to invasive can-
cer is up to 25o/o of patients when an excisional biopsy is performed: therefore, observa-
tion with a 6-month follow-up mammogram is not indicated. An excisional biopsy/partial
38 CATEGORY2
mastectomy is indicated. Given that the lesion was found on mammogram, a localization
technique will be needed.
Chemoprevention using tamoxifen or an aromatase inhibitor can be considered if the
only finding is PLCIS, but it is important to exclude an invasive cancer with an excisional
biopsy before considering this therapy. Trastuzumab would not be indicated, because the
patient does not have an invasive cancer and therefore is not a candidate for chemotherapy
or targeted therapy.
References
1. Moran MS, Schnitt SJ, Giuliano AE, et al. Society of Surgical Oncology-American
Society for Radiation Oncology consensus guideline on margins for breast-conserving
surgery with whole-breast irradiation in stages I and II invasive breast cancer. Int J
Radial Oneal Biol Phys. 2014:88(3):553-564.
2. Downs-Kelly E, Bell D, Perkins GH, Sneige N, Middleton LP. Clinical implications of
margin involvement by pleomorphic lobular carcinoma in situ. Arch Patho! Lab Med.
2011; 135(6):737-743.
3. Chivukula M, Haynik DM, Brufsky A, Carter G, Dabbs DJ. Pleomorphic lobular carci-
noma in situ (PLCIS) on breast core needle biopsies: clinical significance and immuno-
profile. Am J Surg Pathol. 2008;32( 11):1721-1726.
CRITIQUES
The use of sentinel lymph node biopsy (SLNB) after neoadjuvant chemotherapy in a pa-
tient with a prechemotherapy node-positive breast cancer remains controversial. Two re-
cent clinical trials evaluated the use of SLNB in this setting. The ACOSOG Zl071 trial,
a prospective single-ann nonrandomized trial, showed a false-negative rate of more than
I Oo/o, leading the authors to recommend against using SLNB after neoadjuvant chemo-
therapy. In subgroup analysis, however, the authors found that the false-negative rate was
less than 10% if both radiolabeled tracer and blue dye were used.
The SENTINA trial was a randomized trial with a completely different design. This trial
from Germany enrolled patients with both clinically negative and clinically positive axil-
lae. Patients with clinically negative axillae underwent preneoadjuvant SLNB. If the senti-
nel node was positive, a second SLNB was performed after neoadjuvant therapy (group A).
If the sentinel node was negative, no further surgery was performed on the axilla (group B).
In the clinically node-positive group (no biopsy required), patients whose nodes converted
during chemotherapy to clinically negative underwent an SLNB and completion axillary
node dissection at the end of chemotherapy (group C). If the nodes did not convert with
chemotherapy, the patient underwent an axillary lymph node dissection (group D). Patients
who underwent a prechemotherapy SLNB (groups A and B) had a detection rate of 99.1 %,
whereas those who underwent SLNB after chemotherapy (group C) had a detection rate of
80.1 %. The patients in group C were treated most similarly to those in theACOSOG Zl 071
and had a similar false-negative rate(> 10%). However, again when both radiolabeled and
blue dye were used, the false-negative rate dropped below 10%. In many multidisciplinary
practices, with the agreement of all disciplines taking care of the patient, an SLNB might
be offered to the patient in this question, especially ifthe patient had a good response to the
chemotherapy (i.e., would not be leaving grossly positive nodes behind). If it is the agreed
upon management, the surgeon should use 2 types of dye, and the team should detennine
whether there will be additional surgery ifthe sentinel lymph nodes retrieved after chemo-
therapy are positive.
Whether SLNB is considered definitive treatment of the axilla in a patient with a histo-
logically positive node is also controversial. Most would recommend additional treatment
CATEGORY'.!
39
including extended radiation fields to treat the lymphatic basins (axillary. internal mam-
mary and supraclavicular). SLNB can be performed if the patient undergoes mastectomy.
Answer: (B) is most accurate using radiolabeled tracer and blue dye.
References
I. Kuehn T. Bauerfeind I. Fehm T. et al. Sentinel-lymph-node biopsy in patients with
breast cancer before and after neoadjuvant chemotherapy (SENTTNA): a prospecti ve.
multicentre cohort study. lancet Oncol. 2013; 14(7):609-6 18.
2. Boughey JC, Suman VJ, Mittendorf EA, et al: Alliance for Clinical Trials in Oncol-
ogy. Sentinel lymph node surgery after neoadjuvant chemotherapy in patients with
node-positive breast cancer: the ACOSOG Z I 071 (Alliance) clinical trial. JAMA.
2013;9;3 I0( 14): 1455-146 1.
CRITIQUE6
The scaly erythematous appearance of this nipple is suspicious for Paget disease of the
breast (figure 6.2). To make an accurate diagnosis, a full-thickness biopsy is recommended.
A core needle biopsy of the subareolar tissue w ill not help diagnose this dermal-based le-
sion, especially because the mammogram was negative. Therefore, the core needle biopsy
would not have anything in particular to target.
The treatment of the lesion shown is dependent on the full-thickness biopsy. maki ng a
shave biopsy also unhelpful. A partial or simple mastectomy is not indicated without bisto-
logic diagnosis of the nipple lesion. If the nipple lesion is diagnosed as Paget disease. fur-
ther treatment such as a partial or total mastectomy should be considered. because 85-92%
of women with Paget disease of the nipple have an underlyi ng breast cancer.
References
1. Sandoval-Leon AC, Drews-Elger K, Gomez-Fernandez CR, Yepes MM, Lippman ME.
Paget's disease of the nipple. Breast Cancer Res Treat. 2013;141(1):1-12.
2. Karakas C. Pagers disease of the breast. J Carcinog. 2011;10:31.
CRITIQUE?
Breast cancer is the most common cancer in women and the second leading cause of
cancer-related deaths. The lifetime risk of developing breast cancer in women is 1 in 8.
Certain breast conditions or findings on imaging increase a women's risk of developing
breast cancer. Of the choices listed, only atypical ductal hyperplasia (ADH) is a risk factor
for the development of breast cancer.
Fibrocystic changes encompass a spectrum of findings that result in breast pain, tender-
ness, and nodularity. These changes affect the breast stroma and epithelium and are the
result of circulating hormones and growth factors. In addition to cysts noted histologically,
solid elements may include epithelial hyperplasia. If epithelial hyperplasia is present, fi-
brocystic changes are classified as nonproliferative, proliferative without atypia, or prolif-
erative with atypia. ADH is proliferation of atypical ductal epithelial cells, and this confers
a 4-fold increased risk of breast cancer. The annual risk for developing breast cancer in
patients with ADH is 0.5-1 % per year.
Sclerosing adenosis may be found in patients with fibrocystic changes and refers to an
increased number of small terminal ductules associated with stromal tissue proliferation.
These lesions may be confused with carcinoma; however, there is no increased cancer risk.
Observation is appropriate when concordance exists between imaging and pathologic find-
ings on biopsy.
Fat necrosis of the breast may follow a traumatic event or a surgical procedure. Fat ne-
crosis may form a palpable mass or a mammographic finding that contains calcifications,
but the condition does not confer an increased breast cancer risk.
Complex sclerosing lesions originate at sites of terminal duct branching where charac-
teristic histologic changes radiate from a central area of fibrosis. Lesions larger than 1 cm
are referred to as complex sclerosing lesions, whereas lesions smaller than 1 cm are termed
radial scars. Neither increases a woman's risk of breast cancer; however, distinguishing
them from carcinoma may be difficult and often requires a vacuum-assisted biopsy or
surgical excision.
lntraductal papillomas are polyps of epithelial-lined breast ducts and may be adjacent to
the areola or in the periphery. Bloody nipple discharge is common with papillomas close
to the nipple. Peripheral papillomas may be confused with invasive papillary carcinoma.
Intraductal papillomas should undergo excisional biopsy but are not associated with an
increased risk of breast cancer.
References
I. Hunt KK, Robertson JFR, Bland KI. The breast. In: Brunicardi FC, Andersen DK, Billiar
TR, et al, eds. Schwartz's Principles of Surgery. 10th ed. New York, NY: McGraw-Hill;
2015: 497-564.
2. Hunt KK, Green MC, Buchholz TA. Diseases of the breast. In: Townsend CM Jr, Beau-
champ RD, Evers BM, Mattox KL, eds. Sabiston Textbook of Surgery: The Biological
Basis of Modern Surgical Practice. 19th ed. Philadelphia, PA: Elsevier Saunders; 2012:
824-869.
CATEGORY2
41
CRITIQUES
Mondor disease is a superficial thrombophlebitis of the breast and anterior chest wall and
abdomen. It typically produces a tender, palpable cord of the lateral thoracic vein, thora-
coepigastric vein, or the superficial epigastric vein. The diagnosis is made by history and
clinical examination, and ultrasonography is the diagnostic modality of choice. Etiologies
may include previous breast surgery or trauma, infections, excess physical activity, preg-
nancy, large and pendulous breasts, blood dyscrasias, rheumatoid arthritis, or intravenous
drug abuse. This process is a self-limited benign disease of women, and treatment includes
warm compresses and anti-inflammatory medications.
Biopsy is indicated when there is a mass adjacent to the cord or the diagnosis is uncer-
tain. Because this is an inflammatory condition, antibiotics are not indicated unless there
is a suspicion of infection. Treatment for infected thrombophlebitis requires excision of
the vein. Thrombotic events, such as pulmonary embolism, are not associated with this
disease; therefore, there is no need for further workup of thromboembolic phenomena.
References
1. Hunt KK, Robertson JFR, Bland KL The breast. In: Brunicardi FC, Andersen DK, Billiar
s
TR, et al, eds. Schwartz Principles ofSurgery. 10th ed. New York, NY: McGraw-Hill;
2015: 497-564.
2. Salemis NS, Merkouris S, Kimpouri K. Mondor's disease of the breast. A retrospective
review. Breast Dis. 2011;33(3):103-107.
CRITIQUE9
Granulomatous mastitis is an idiopathic inflammatory condition of the breast that occurs
worldwide, mostly in nonwhite women of childbearing age. It is being seen more often
in the United States due to the increasing Hispanic population. Granulomatous mastitis
often presents with abscesses and fistulas. Standard surgical drainage is usually followed
by recurrence, and a diagnosis by core biopsy is preferred when possible. The disease is
linked in many cases to an underlying rheumatic or autoimmune condition; therefore, a
course of oral steroids may reduce the disease burden, rendering operative intervention
smaller or unnecessary. The role of surgery versus steroids is currently controversial,
further complicated by the observation that in many cases, the disease spontaneously
''burns out" after several months. Most recent reviews recommend starting with core
biopsy, followed by observation or steroids for mild cases and surgical excision only for
steroid-refractory patients.
References
1. Mohammed S, Statz A, Lacross JS, et al. Granulomatous mastitis: a 10 year experience
from a large inner city county hospital. J Surg Res. 2013; 184(1):299-303.
2. Pandey TS, Mackinnon JC, Bressler L, Millar A, Marcus EE, Ganschow PS. Idiopathic
granulomatous mastitis-a prospective study of 49 women and treatment outcomes
with steroid therapy. Breast J. 2014;20(3):258-266.
42 CATEGORY2
CRITIQUE 10
Gynecomastia is a common benign condition related to an absolute or relative deficiency
in androgen or to estrogen excess. In adults, such conditions are most commonly created
by a variety of medications; liver, thyroid, or kidney disease; obesity; endocrine tumors; or
treatment for prostate cancer.
The most common medications implicated in gynecomastia include spironolactone, cal-
cium channel blockers~ antipsychotics_. and gastrointestinal agents. Both proton pump in-
hibitors and H2-receptor blockers are implicated, so changing one for another in the patient
presented will likely not improve the gynecomastia and may actually worsen it. Patients
with prostate cancer often develop gynecomastia as a result of antiandrogen therapy. The
risk is highest with estrogen therapy (>70o/o), intermediate with antiandrogens (such as
flutamide or bicalutamide; up to 50%), and lowest after bilateral orchiectomy (10%).
Routine mammograms, breast ultrasounds, or biopsies are not recommended. These
tests can be considered ifthe enlargement is unilateral. Although gynecomastia can resolve
by itself or after removal of the underlying cause, treatment is indicated in men with pain
and tenderness, especially in patients with several risk factors that cannot be remediated,
such as the patient presented.
First-line treatment involves decreasing estrogen levels by the use of androgens or an-
tiestrogens, such as tamoxifen. Tamoxifen (10-20 mg/day, off-label use) has been studied
in prospective randomized trials and is effective in resolving gynecomastia in 60-90%
of cases, including men with prostate cancer. Surgery can be offered to patients who do
not respond to medical treatment. Surgical approaches include liposuction or removal of
glandular breast tissue through a periareolar incision. The goals of operation are to remove
the painful tissue while restoring normal breast contour and appearance. Total mastectomy,
especially with removal of the nipples, is rarely indicated except in massive gynecomastia.
References
I. Bowman JD, Kim H, Bustamante JJ. Drug-induced gynecomastia. Pharmacotherapy.
2012;32(12): 1123-1140.
2. Morcos RN, Kizy T. Gynecomastia: when is treatment indicated? J Fam Pract.
2012;61(12):719-725.
3. Narula HS, Carlson HE. Gynaecomastia-pathophysiology, diagnosis and treatment.
Nat Rev Endocrinol. 2014; 10( 11 ):684-698.
CRITIQUE 11
Several germline genetic mutations are linked to hereditary breast cancer, and the family
history of types of cancers usually gives a clue as to what gene may be mutated. The pa-
tient presented has a family history suggestive ofLi-Fraumeni syndrome (LFS), a rare but
highly penetrant familial cancer syndrome caused by germline mutations in the p53 tumor
suppressor gene. p53 is an extensive, encompassing regulator of the cell cycle, often called
"the guardian of the genome." The hallmark of p53 mutations is, therefore, a myriad of
cancers occurring earlier than the usual age. LFS is autosomal dominant and nearly 100%
penetrant by age 70. The most frequent cancers in LFS are premenopausal breast cancer,
bone and soft tissue sarcomas, adrenal cortical carcinomas, and brain tumors. A host of
other cancers are less frequently associated with LFS, including ovarian cancer, which can
cause confusion with breast-ovarian cancer syndrome.
The most common mutations associated with hereditary breast cancer are in the
BRCA 1 and BRCA2 genes. Mutations in these genes usually cause breast and ovarian
cancer (breast-ovarian cancer syndrome) but are actually much less often associated
with other cancers. BRCA2 mutations are associated with breast cancer in men. In a re-
CATEGORY2
43
cent large study, the incidences of pancreas cancer, sarcomas, and melanomas in patients
with BRCA mutations were statistically higher compared with the population risk, but
they were still small.
Germline mutations in PTEN, located on chromosome I Oq, cause Cowden syndrome:
benign and malignant breast and thyroid diseases, gastrointestinal polyps, and lipomas
(usually occurring as papillomatous growths of the oral mucosa), central nervous system
defects, and macrocephaly. In addition to breast cancers, Cowden patients occasionally
develop cancers of the thyroid, endometrium, and colon.
Germline mutations in p 16 (chromosome 9p) are most commonly associated with the
familial atypical mole and malignant melanoma (FAMMM) syndrome. This syndrome is
associated with an elevated risk of melanoma and pancreas cancer; breast cancer is some-
times associated, as are cancers of the colon, stomach, nasopharynx, and tongue, leukemia,
retinoblastoma, and squamous cell cancers of the skin.
References
1. Czajkowski R, Placek W, Drewa G, Czajkowska A, Uchat\ska G. FAMMM syndrome:
pathogenesis and management. Dermato/ Surg. 2004;30(2, pt 2):291-296.
2. Merino D, Malkin D. p53 and hereditary cancer. Subcell Biochem. 2014;85:1-16.
3. Mersch J, Jackson MA, Park M, et al. Cancers associated with BRCAl and BRCA2
mutations other than breast and ovarian [published correction appears in Cancer.
2015;121(14):2474-2475]. Cancer. 2015;121(2):269-275.
4. Mester J, Eng C. Cowden syndrome: recognizing and managing a not-so-rare hereditary
cancer syndrome. J Surg Oneal. 2015; 111(1): 125-130.
CRITIQUE 12
Achieving a negative surgical margin in patients undergoing lumpectomy for breast cancer
is imperative; local recurrence rates are higher among patients with a persistently positive
surgical margin. Further, the best cosmetic results are obtained during the first procedure.
Re-excision procedures not only reduce cosmesis but also increase infection rates, costs,
and patient anxiety. Rates of re-excision of positive margins vary widely nationally, inter-
nationally, and between surgeons and institutions; they vary between 5 and 70o/o and aver-
age between 20 and 40o/o. Accordingly, rates of secondary procedures after initial lumpec-
tomy are used as a quality measure by groups such as the National Quality Forum in the
United States and the European Union of Breast Cancer Specialists.
A large recent meta-analysis supported the use of intraoperative margin assessment us-
ing either imprint cytology or frozen section to reduce the need for re-excision. Rates of
re-excision after lumpectomy are expected to decrease due to the use of these techniques
and new guidelines stating that close but negative margins are acceptable.
Specifically, in the past, many re-excisions were done for negative margins because of
a lack of consensus regarding safe margins for radiation. A recent consensus statement is-
sued by the Society for Surgical Oncology (SSO) and the American Society of Therapeutic
Radiology (ASTRO) was endorsed by the American Society of Breast Surgeons and the
American Society of Clinical Oncology. The statement indicates that "margins off the ink'"
are adequate for lumpectomy. The SSO-ASTRO statement will likely reduce both variabil-
ity and rates of re-excision after lumpectomy.
References
!. Schnitt SJ, Moran MS, Houssami N, Morrow M. The Society of Surgical Oncology-
American Society for Radiation Oncology Consensus Guideline on Margins for Breast-
Conserving Surgery With Whole-Breast Irradiation in Stages I and II Invasive Breast
Cancer: Perspectives for Pathologists. Arch Pathal Lab Med. 2015;139(5):575-577.
2. Esbona K, Li Z, Wilke LG. Intraoperative imprint cytology and frozen section pathol-
ogy for margin assessment in breast conservation surgery: a systematic review. Ann
Surg Oneal. 2012;19(10):3236-3245.
3. Landercasper J, Whitacre E, DegnimAC,Al-Hamadani M. Reasons for re-excision after
lumpectomy for breast cancer: insight from the American Society of Breast Surgeons
Mastery(SM) database. Ann Surg Oneal. 2014;21(10):3185-3191.
CRITIQUE 13
One of the more common questions women ask is whether there is an increased risk of
breast cancer associated with hormone use. Women who take oral contraceptives have a
slightly increased risk of breast cancer than women who have never used them. Ten years
after cessation of birth control pills, the risk returns to baseline. The same is true of inject-
able depot-medroxyprogesterone acetate. Using combined hormone therapy (progesterone
plus estrogen) after menopause increases the risk of breast cancer. The Women's Health
Initiative enrolled 161,809 postmenopausal women ages 50-79 during the years of 1993
and 1998. A cohort of more than 16,000 women with intact uteri were randomized to
receive conjugated estrogens and progesterone versus placebo. This limb of the trial was
stopped early because of increased mortality as well as an increase in breast cancer of26%.
Increased density on mammography produced more biopsies. The use of estrogen alone
seems to have no effect on breast cancer risk. Female hormone replacement therapy may
actually increase cardiovascular risk.
Answer: (C) Hormone replacement therapy increases the risk of breast cancer in women.
References
1. Marchbanks PA, McDonald JA, Wilson HG, et al. Oral contraceptives and the risk of
breast cancer. N Engl J Med. 2002;346(26):2025-2032.
2. Santen RJ, Boyd NF, Chlebowski RT, et al. Critical assessment of new risk factors for
breast cancer: considerations for development of an improved risk prediction model.
Endocr Relat Cancer. Jun 2007;14(2):169-187.
3. Santen RJ, Boyd NF, Chlebowski RT, et al; Breast Cancer Prevention Collaborative
Group. Critical assessment of new risk factors for breast cancer: considerations for devel-
opment ofan improved risk prediction model. Endocr Relat Cancer. 2007; 14(2): 169-187.
4. Anderson GL, Chlebowski RT, Aragaki AK, et al. Conjugated equine oestrogen and
breast cancer incidence and mortality in postmenopausal women with hysterectomy:
extended follow-up of the Women's Health Initiative randomised placebo-controlled
trial. Lancet Oneal. 20!2;13(5):476-486.
5. Schairer C, Lubin J, Troisi R, Sturgeon S, Brinton L, Hoover R. Menopausal estrogen
and estrogen-progestin replacement therapy and breast cancer risk [published correction
appears in JAMA. 2000;284(20):2597]. JAMA. 2000;283(4):485-491.
6. Hsia J, Langer RD, Manson JE, et al; Women's Health Initiative Investigators. Conju-
gated equine estrogens and coronary heart disease: the Women's Health Initiative
[published correction appears in Arch Intern Med. 2006;166(7):759]. Arch Intern Med.
2006; 166(3):357-365.
7. Rossouw JE, Anderson GL, Prentice RL, et al; Writing Group for the Women's Health
Initiative Investigators. Risks and benefits of estrogen plus progestin in healthy post-
menopausal women: principal results from the Women's Health Initiative randomized
controlled trial. JAMA. 2002;288(3):321-333.
CATEGORY2 45
CRITIQUE 14
Although prophylactic mastectomy is an established option for risk reduction in women
with a genetic predisposition or elevated breast cancer risk, this patient is requesting a con-
tralateral prophylactic mastectomy (CPM) in the absence of a substantially elevated risk
ofcontralateral breast cancer. The use ofCPM in the United States has increased dramati-
cally in the last decade. In a single institution study, rates of CPM increased from 7 to 24%
between 1997 and 2005. Several factors are associated with the increasing use of CPM
such as patient age, family history of breast cancer, the use of preoperative MRI, and the
availability of immediate breast reconstruction. However, most patients pursuing CPM are
not at increased risk for contralateral breast cancer.
CPM reduces the incidence of contralateral breast cancer by at least 90%; however,
the patient in question only has a 3-5% risk of contralateral breast cancer over the next
10 years and this risk is not modified by the absence or presence of extensive DCIS. Her
risk of contralateral breast cancer will be affected by the use of endocrine therapy to treat
her index breast cancer; therefore, she will receive very little benefit in risk reduction from
undergoing CPM.
There is no biologic basis to support the notion that CPM would reduce the risk oflocal
recurrence for the treated breast cancer. Further, a 2010 Cochrane review and several large
population-based analyses demonstrated little evidence to suggest a survival benefit for the
use of CPM in patients at average to low risk of contralateral breast cancer. CPM is also
associated with increased complication rates (hazard ratio 2- to 3-fold higher) compared
with unilateral mastectomy in both single institution series and a report from the National
Surgical Quality Improvement Program® database.
Answer: (E) endocrine therapy will decrease her risk of a contralateral cancer.
References
1. King TA, Sakr R, Patil S, et al. Clinical management factors contribute to the decision
for contralateral prophylactic mastectomy. J Clin Oneal. 2011;29(16):2158-2164.
2. Lostumbo L, Carbine NE, Wallace J. Prophylactic mastectomy for the prevention of
breast cancer. Cochrane Database Syst Rev. 2010;(11 ):CD002748.
3. Portschy PR, Kuntz KM, Tuttle TM. Survival outcomes after contralateral prophylactic
mastectomy: a decision analysis. J Natl Cancer Inst. 2014; 106(8).
4. Nichols HB, Berrington de Gonzalez A, Lacey JV Jr, Rosenberg PS, Anderson WF.
Declining incidence of contralateral breast cancer in the United States from 1975 to
2006. J Clin Oneal. 2011;29(12): 1564-1569.
5. Baum M, Budzar AU, Cuzick J, et al; ATAC Trialists' Group. Anastrozole alone or in
combination with tamoxifen versus tamoxifen alone for adjuvant treatment of post-
menopausal women with early breast cancer: first results of the ATAC randomised
trial [published correction appears in Lancet. 2002;360(9344):1520]. Lancet.
2002;359(9324):2131-2139.
6. Miller ME, Czechura T, Martz B, et al. Operative risks associated with contralat-
eral prophylactic mastectomy: a single institution experience. Ann Surg Oneal.
2013;20(13):4113-4120.
7. Osman F, Saleh F, Jackson TD, Corrigan MA, Ci! T. Increased postoperative complica-
tions in bilateral mastectomy patients compared to unilateral mastectomy: an analysis
of the NSQIP database. Ann Surg Oneal. 2013;20(10):3212-3217.
8. Yao K, Winchester DJ, Czechura T, Huo D. Contralateral prophylactic mastectomy and
survival: report from the National Cancer Data Base, 1998-2002. Breast Cancer Res
Treat. 2013;142(3):465-476.
46 CATEGORY2
CRITIQUE 15
A periareolar mammary duct fistula is characterized by a chronically discharging nipple
lesion, most often at the border of the areola but sometimes at the base of the papilla, which
tracks to a major subareolar milk duct (figure 15.1 ). The etiology of this process is thought
to be related to obstruction ofa major milk duct secondary to squamous metaplasia, result-
ing in accumulation of secretions that become infected, forming an acute subareolar ab-
scess. The abscess drains along the route ofleast resistance, commonly at the border of the
areola. The fistula then heals, but in many cases, a recurrent abscess develops and drains
along the same route, establishing a chronic fistula.
Treatment of a periareolar fistula should consist of excision of the entire fistula tract
from its origin in the periareolar skin through the subareolar space, together with the major
milk duct it originates from as it traverses the papilla of the nipple. If the procedure is timed
properly, when there is no acute infection, the wound can usually be closed primarily with
minimal risk of infection.
The lesion described is central, thus it is not a peripheral, nonlactational breast abscess.
A puerperal abscess occurs during lactation. Hidradenitis suppurativa typically affects mul-
tiple areas of the body and is not isolated to the nipple. Granulomatous lobular mastitis
usually results in numerous, chronic infections at multiple locations in the breast.
Incision and drainage can temporize an acute exacerbation of chronic periareolar infec-
tion associated with a mammary duct fistula, but the process will usually recur ifthe fistula
tract is not excised. Corticosteroid therapy can be an effective treatment for granulomatous
lobular mastitis but would not be used to treat a periareolar mammary duct fistula. Ciga-
rette smoking is strongly associated with chronic subareolar abscess and fistula formation.
References
I. Klimberg VS. Incision and drainage of an abscess. In: Klimberg VS, ed. Atlas a/Breast
Surgical Techniques. 1st ed. Philadelphia, PA: Saunders Elsevier; 2010: 10-17.
2. Grobmyer SR, Massoll N, Copeland EM 3rd. Clinical management of mastitis and
breast abscess and idiopathic granulomatous mastitis. In: Bland Kl, Copeland EM 3rd,
eds. The Breast: Comprehensive management of benign and malignant disease. 4th ed.
Philadelphia, PA: Saunders Elsevier;2009: 145-150.
3. Meguid MM, Kort KC, Numann PJ, Oler A. Subareolar breast abscess: the penultimate
stage of the mammary duct-associated inflammatory disease sequence. In: Bland Kl,
Copeland EM 3rd, eds. The Breast: Comprehensive management of benign and malig-
nant disease. 4th ed. Philadelphia, PA: Saunders Elsevier;2009: I 07-144.
CRITIQUE 16
Sentinel node biopsy results in a far lower hazard of complications compared with routine
axillary dissection for the treatment for patients with invasive breast cancer, but it is not
without risk. Evaluation of the complications in patients participating in the American Col-
lege of Surgeons Oncology Group ZOOlO trial showed that 7% of patients who underwent
sentinel node biopsy demonstrated proximal upper-extremity lymphedema, defined as a
change from baseline arm circumference of more than 2 cm.
Axillary radiation therapy is associated with an increased risk of lymphedema by at
least 20%. As of 2009, no studies identified a subset of patients at risk of developing
postmastectomy lymphedema by means of a preoperative lymphoscintigram. A random-
ized controlled trial evaluating the effect of weight lifting on lymphedema risk in 154
breast cancer patients who underwent axillary surgery demonstrated that a program of
slowly progressive weight lifting compared with no exercise did not increase incidence
of lymphedema.
References
I. Sakurai T, Endo M, Shimizu K, et al. Axillary reverse mapping using fluorescence
imaging is useful for identifying the risk group of postoperative lymphedema in breast
cancer patients undergoing sentinel node biopsies. J Surg Oneal. 2014;109(6):612-615.
2. Schmitz KH, Ahmed RL, Troxel AB, et al. Weight lifting for women at risk for breast
cancer-related lymphedema: a randomized trial. JAMA. 2010;304(24):2699-2705.
3. Wilke LG, McCall LM, Posther KE, et al. Surgical complications associated with senti-
nel lymph node biopsy: results from a prospective international cooperative group trial.
Ann Surg Oneal. 2006;13(4):491-500.
4. Kaur P, Puleo CA, Cox CE. Lymphedema in the postmastectomy patient: pathophysiol-
ogy, prevention, and management. In: Bland KJ, Copeland EM 3rd, eds. The Breast:
Comprehensive management ofbenign and malignant disease. 4th ed. Philadelphia, PA:
Saunders Elsevier:2009:897-914.
CRITIQUE 17
Breast cancer that does not express hormone receptors or hormone epidermal growth fac-
tor receptor 2 (HER2) is labeled "triple-negative" breast cancer (TNBC). TNBCs represent
12-24o/o of all breast cancers. These are more aggressive and are more likely to affect
younger people, African Americans, Hispanics, and those with BRCAl mutation. TNBCs
tend to not have microcalcifications. When biopsied, most are invasive ductal cancers with
high-grade features such as central necrosis and pushing borders.
48 CATEGORY2
References
1. Schmadeka R, Harmon BE, Singh M. Triple-negative breast carcinoma: current and
emerging concepts. Am J Clin Pathol. 2014;141(4):462-477.
2. de Ruijter TC, Veeck J, de Hoon JP, van Engeland M, Tjan-Heijnen VC. Characteristics
of triple-negative breast cancer. J Cancer Res C/in Oneal. 20II;137(2): 183-192.
CRITIQUE 18
Breast cancer in men is rare and accounts for less than I% of all patients with the dis-
ease, peaking in incidence at age 71. Risk factors generally suggest changes in the
estrogen:androgen ratio, noting that more than 90o/o are hormone receptor-positive. Ex-
amples would include Klinefelter syndrome (relative risk 50 times), testicular abnormali-
ties, cirrhosis, and obesity. There is a familial disposition; at least 20o/o of men with breast
cancer have first-degree female relatives with breast cancer and BRCA2 genetic mutations
(cumulative risk estimated at 6.8% by age 70). An association with BRCAl variance is less
frequent (cumulative risk by age 70of1.2%) and was only recently described.
Although occupational risks, including high-temperature environments and exposure
to radiation, carcinogens, and exhaust fumes have been identified as nongenetic risk fac-
tors for male breast cancer, electromagnetic field exposure has not been implicated. Men
of Ashkenazi Jewish ancestry are the only group at increased risk. As is the case for Asian
women, Asian men have a lower incidence of the disease. Although several risk factors,
including alcoholism~ are common to the development of gynecomastia and breast cancer,
patients with a history of gynecomastia can be reassured that their risk of breast cancer is
not increased.
References
1. Charlot M, Beatrix 0, Chateau F, et al. Pathologies of the male breast. Diagn Interv
Imaging. 2013;94(1):26-37.
2. Fentiman IS, Fourquet A, Hortobagyi ON. Male breast cancer [published correction
appears in Lancet. 2006;367(9525): 1818]. Lancet. 2006;367(9510):595-604.
3. Ottini L, Palli D, Rizzo S, Federico M, Bazan V, Russo A. Male breast cancer. Crit Rev
Oncol Hematol. 2010;73(2):141-155.
CRITIQUE 19
Hereditary breast cancer represents 5-10% of all cases of breast cancer. By far, BRCAl
and BRCA2 are the 2 most common inherited breast cancer genes; both are tumor sup-
pressor genes thought to be involved with DNA repair. In general, tumor suppressor genes
encode protein(s) that acts as "brakes" of the cell cycle, and mutation ofa tumor suppressor
gene is considered a loss-of-function mutation. The inherited defect results in one function-
ing gene and one nonfunctioning gene. The affected individual has a lifetime risk of having
the one functioning gene mutating, leading to a complete loss of tumor suppressor function
CATEGORY2 49
or loss of the ''brakes." For patients who inherited the BRCA 1 or BRCA2 gene, such a
complete loss is thought to lead to uncontrolled cellular growth, resulting in the develop-
ment of breast cancer as well as other cancers, such as ovarian cancer.
An important point of distinction is that although mutations of tumor suppressor genes
are said to be recessive at the level of the individual cells, the pattern of inheritance is au-
tosomal dominant. Fifty percent of the offspring ofBRCAl/BRCA2 carriers will inherit
the genetic mutation. Patients with BRCAl mutation have a 57--05% chance of developing
breast cancer and a 40% chance of developing ovarian cancer. Those with BRCA2 muta-
tion have a 45-49o/o chance of developing breast cancer and an 18% chance of developing
ovarian cancer.
Management options for patients with BRCA1/BRCA2 mutations include vigilant
screening or preventive steps with either chemoprevention or prophylactic operations.
Prophylactic operations include bilateral prophylactic mastectomy and bilateral salpingo-
oophorectomy. Bilateral mastectomy reduces the risk of breast cancer by more than 90%,
whereas bilateral salpingo-oophorectomy reduces the risk of breast cancer by up to 50%
and reduces the risk of ovarian cancer by 96%. In addition, bilateral salpingo-oophorectomy
leads to a significant improvement in overall survival, although current studies do not
demonstrate an overall survival benefit with bilateral prophylactic mastectomy.
Total mastectomy, skin-sparing mastectomy, as well as nipple-sparing mastectomy are
acceptable options for women who elect to undergo bilateral prophylactic mastectomy.
References
1. Domchek SM, Friebel TM, Singer CF, et al. Association of risk-reducing surgery
in BRCAl or BRCA2 mutation carriers with cancer risk and mortality. JAMA.
2010;304(9):967-975.
2. Peled AW, Irwin CS, Hwang ES, Ewing CA, Alvarado M, Esserman LJ. Total skin-
sparing mastectomy in BRCA mutation carriers. Ann Surg Oneal. 2014;2 l (1 ):37-41.
CRITIQUE20
This patient presents with a locally advanced breast cancer. Locally advanced breast cancer
represents a heterogeneous group of cancers that can be categorized into 2 major groups:
inflammatory breast cancer (!BC) and non-IBC locally advanced. Patients with !BC gener-
ally present with a rapid onset of symptoms and signs such as diffuse erythema and edema
(peau d'orange), often without having clinical evidence of a breast mass. Criteria for diag-
nosis of!BC include (I) a rapid onset of breast erythema, edema, or peau d' orange or warm
breast with or without an underlying palpable mass; (2) duration of history of no more than
6 months; (3) erythema occupying at least one-third of the breast; (4) and histologic confir-
mation of invasive carcinoma. Tumor emboli in the dermal lymphatic channels is pathog-
nomonic of !BC, although its absence does not exclude the diagnosis of!BC, because only
60% of patients with !BC have this pathognomonic histologic feature.
Conversely, patients with non-IBC locally advanced typically present with a large, ul-
cerative, or fungating breast mass secondary to a long-standing and often neglected breast
cancer. Although patients with locally advanced breast cancer have at least a 50% risk of
developing distant disease compared with early breast cancer, by definition, they have no
evidence of distant disease (MO) at the time of diagnosis and therefore should be consid-
ered to have stage III breast cancer.
Initial treatment is neoadjuvant chemotherapy, which generally consists of 4-{) cycles of
anthracyclines (i.e., doxorubicin, epirubicin) and taxanes (i.e., paclitaxel, docetaxel) over
4-6 months before surgery. Breast conservation therapy is an option for patients with non-
50 CATEGORY2
!BC locally advanced, but not for those with !BC. For patients with !BC, a modified radical
mastectomy should be offered after a clinical response to neoadjuvant therapy. Postopera-
tive radiation remains an integral part of the treatment regimen, regardless of the responses
to neoadjuvant chemotherapy.
The role of neoadjuvant endocrine therapy for patients with hormone-positive tumor is
not as well defined as that for neoadjuvant chemotherapy. Most of the neoadjuvant endo-
crine therapy trials were performed on postmenopausal women and, therefore, neoadjuvant
endocrine therapy should not be considered in young women unless within a clinical trial.
Regardless, this patient's tumor is ER-negative, which means that neoadjuvant endocrine
therapy is not an option.
Sentinel lymph node biopsy is not indicated for patients with locally advanced breast
cancer before neoadjuvant chemotherapy.
References
I. Kaufmann M, von Minckwitz G. Mamounas EP, et al. Recommendations from an inter-
national consensus conference on the current status and future of neoadjuvant systemic
therapy in primary breast cancer. Ann Surg Oneal. 2012;19(5): 1508-1516.
2. Lee MC, Newman LA. Management of patients with locally advanced breast cancer.
Surg Clin North Am. 2007;87(2):379-398, ix.
CRITIQUE21
The most common benign breast mass is fibroadenoma. Intraductal papilloma is the most
common papillary lesion of the breast, accounting for up to 5o/o of benign disease. Intra-
ductal papilloma is suspected in patients with nipple discharge. Intraductal papillomas with
no evidence ofatypia do not increase a woman's risk of future breast cancer.
Typically, no mass is palpable on physical exam in a patient with an intraductal pap-
illoma. lntraductal papilloma may be localized to the periphery, sparing the main duct.
Ductography may localize the area of the abnormal duct, allowing the excision of the main
duct to be spared in women of child-bearing age who may wish to breastfeed in the future.
Ductography is not adequate to distinguish benign from malignant disease. Benign lesions
do not require 1-cm margins.
References
1. Li X, Weaver 0, Desouki MM, et al. Microcalcification is an important factor in the
management of breast intraductal papillomas diagnosed on core biopsy. Am J Clin
Pathol. 2012;138(6):789-795.
2. Moncrief RM, Nayar R, Diaz LK, Staradub VL, Morrow M, Khan SA. A comparison
of ductoscopy-guided and conventional surgical excision in women with spontaneous
nipple discharge. Ann Surg. 2005;241(4):575-581.
3. Lewis JT, Hartmann LC, Vierkant RA, et al. An analysis of breast cancer risk in women
with single, multiple, and atypical papilloma. Am J Surg Pathol. 2006;30(6):665-672.
CATEGORY2
51
CRITIQUE22
Pregnancy-associated breast cancer, defined as breast cancer diagnosed during pregnancy
or in the first postpartum year, occurs in approximately 123 per 100,000 pregnancies. Ap-
proximately 3500 cases are reported annually. In a cohort study, survival by stage was
no different between women diagnosed with breast cancer during pregnancy and stage-
matched nonpregnant women. Estrogen receptor and HER2/neu statuses were comparable
between the pregnant patients and nonpregnant women with breast cancer. During sentinel
node biopsy, the entire radioisotope stays trapped at the site of injection or within the lym-
phatics until decay occurs-the half-life is 6 hours. Therefore, the fetus is not exposed to
dangerous amounts of radiation. In a study exploring the safety of lymphatic mapping in
pregnant women with breast cancer, no adverse effects on the fetus were reported.
Surgery is the first-line treatment of pregnancy-associated breast cancer. Surgery can
be performed safely in any stage of pregnancy with minimal complications. Surgeons may
prefer to wait until after the first trimester to minimize the risk of a spontaneous abor-
tion. However, data demonstrating a higher rate of spontaneous loss compared with the n
general population are limited. Both mastectomy and breast conservation surgery are op- "
;;
tions. Chemotherapy should be administered after the first trimester and can be adjuvant ""·~
0
or neoadjuvant. Anthracycline-based chemotherapy, doxorubicin and cyclophosphamide
(AC regimen) or 5-fluorouracil, doxorubicin, and cyclophosphamide administered during
the second and third trimesters are the most common regimens. Therapy with tamoxifen
or other selective estrogen receptor modulators in women with hormone receptor-positive
breast cancer should be deferred until after delivery. Tamoxifen has the potential to disturb
the hormonal environment of pregnancy and is associated with birth defects, including
craniofacial malformations and ambiguous genitalia.
Answer: (C) Therapy with tamoxifen should be deferred until after pregnancy.
References
1. Krishna I, Lindsay M. Breast cancer in pregnancy. Obstet Gynecol Clin North Am.
2013;40(3):559-571.
2. Cardonick E, Dougherty R, Grana G, Gilmandyar D, Ghaffar S, Usmani A. Breast
cancer during pregnancy: maternal and fetal outcomes. Cancer J 2010: 16(1 ):76-82.
CRITIQUE23
This patient has a simple subareolar breast abscess (figure 23.2). Breast abscesses can form
in both lactating and nonlactating women. Mastitis can occur in up to 24% ofbreastfeeding
women, and up to 11 % of these cases can progress to a puerperal breast abscess if treatment
of mastitis is delayed. Small skin lacerations or stagnation within ectatic mammary ducts
permit bacterial proliferation within the breast tissue. The most common causative organ-
ism is Staphylococcus aureus, with community-acquired methicillin resistant S. aureus
(MRSA) becoming increasingly common. Ultrasound evaluation of the affected area can
help differentiate mastitis from a breast abscess.
Treatment depends on whether the patient has mastitis alone, a simple abscess, or a
multiloculated abscess. Antibiotics administered to cover Gram-positive organisms are in-
dicated in all cases. Broad-spectrum antibiotics are not indicated for a puerperal abscess
but may be considered for nonpuerperal or recurrent abscesses. Antibiotics should be con-
tinued until complete clinical resolution.
For a simple abscess, supportive care with analgesics (e.g., ibuprofen) and warm com-
presses is useful but not sufficient. Aspiration of the fluid often results in abscess resolu-
tion. Ultrasound guidance can be used to ensure complete drainage. Surgical incision and
drainage is indicated for multiloculated abscesses, when there is overlying skin necrosis,
and following recurrence after multiple aspirations.
52 CATEGORY:!
The patient should be encouraged to continue to drain the affected breast with breast
feeding or pumping. Follow-up through complete resolution is critical to ensure the patient
does not have inflammatory breast cancer.
Answer: (D) Aspirate the fluid and administer antibiotics for Gram-positive organisms.
References
1. Amin AL, Purdy AC, Mattingly JD, Kong AL, Termuhlen PM. Benign breast disease.
Surg Clin North Am. 2013 ;93(2):299- 308.
2. Chen CY, Anderson BO, Lo SS, Lin CH, Chen HM. Methicillin-resistant Staphylococ-
cus aureus infections may not impede the success of ultrasound-guided drainage of
puerperal breast abscesses. J Am Coll Surg. 2010;210(2): 148-154.
CRITIQUE 24
Mastalgia is the most common complaint among women presenting to specialty breast
clinics and is a benign condition in more than 90% of cases. Jn the majority of cases, the
discomfort is bothersome yet not incapacitating. After confirming a negative history and
comprehensive clinical breast examination, with or without indicated imaging studies, best
management is reassurance and symptomatic support.
The majority of women report an increase in breast tenderness, fullness, and lumpiness
during the luteal phase (progesterone dominant) of the menstrual cycle, called cyclical mas-
talgia. Breast enlargement of up to 100 mL in total volume is documented in the absence of
overt breast pathology and is considered a normal response to the hormonal milieu. Most
women will relate a breast pain score of2-4 on a 10-point scale for 3-4 days each month, re-
solving at the onset or shortly after the beginning menses. Cyclical mastalgia nearly always
abates during pregnancy and after menopause. Cyclical mastalgia is virtually unheard of in
association with malignancy. The precise etiology of cyclical mastalgia is poorly understood
and is ascribed to diet (dietary fats, methylxanthines), fibrocystic mastopathy which on bi-
opsy is consistent with fibrocystic change, anxiety/psychological distress, iodine deficiency,
CATEGORY2 53
References
I. Amin AL, Purdy AC, Mattingly JD, Kong AL, Termuhlen PM. Benign breast disease.
Surg Clin North Am. 2013:93(2):299-308.
2. Salzman B, Fleegle S, Tully AS. Common breast problems. Am Fam Physician.
2012;86(4):343-349.
3. Smith RL, Pruthi S, Fitzpatrick LA. Evaluation and management of breast pain. Mayo
Clin Proc. 2004;79(3):353-372.
4. Srivastava A, Mansel RE, Arvind N, Prasad K, Dhar A, Chabra A. Evidence-
based management of mastalgia: a meta-analysis of randomised trials. Breast.
2007; 16(5):503-512.
CRITIQUE ZS
Atypical ductal hyperplasia (ADH) is an epithelial proliferative lesion of the terminal duct
lobular unit that confers an increased risk of breast cancer 4-fold higher than the general
population. As such, it is considered a high-risk breast lesion. ADH is a risk factor for both
premenopausal and posttnenopausal women, and the risk of breast cancer appears to be
steady at approximately 0.5-1 % per year over a woman's lifetime. It is considered a mark-
er for increased breast cancer risk, but it is not considered a precursor to breast cancer, be-
cause only a minority (-15%) of women with ADH will eventually develop breast cancer.
The risk of developing breast cancer for a patient with a BRCA mutation is substantially
higher than that for a patient with ADH, ranging from 50-80%.
Answer: (C) associated with an increased relative risk of breast cancer that is 4-fold
higher than the general population.
54 CATEGORY2
References
1. Degnim AC, Hartmann LC. Refining risk assessment in women with atypical hyperpla-
sia. Curr Breast Cancer Rep. 2009; 1(3):167-174.
2. Whiffen A, El-Tamer M, Taback B, Feldman S, Joseph KA. Predictors of breast cancer
development in women with atypical ductal hyperplasia and atypical lobular hyperpla-
sia. Ann Surg Oneal. 2011; 18(2):463-467.
3. Page DL, Dupont WD, Rogers LW, Rados MS. Atypical hyperplastic lesions of the
female breast. A long-term follow-up study. Cancer. 1985;55( 11 ):2698-2708.
CRITIQUE26
Although young women with breast cancer are typically considered to be at higher risk of
local recurrence than their older counterparts, data from recent population based studies
demonstrate that with appropriate attention to surgical margins and appropriate use of ad-
juvant therapy, 5-year rates of local recurrence for women 40 or younger are less than 5%.
The rate of local recurrence was investigated in a study of 3799 women who underwent
lumpectomy and whole breast irradiation with or without adjuvant therapy in 5 National
Adjuvant Breast and Bowel Project protocols. In this study, the 12-year cumulative inci-
dence of ipsilateral breast tumor recurrence and other locoregional recurrence in patients
treated with adjuvant therapy was 6.6% and 1.8%, respectively. These studies also demon-
strated that the incidence of local recurrence is affected by both the use of chemotherapy
and antiestrogen therapy in the setting of ER-positive disease.
The incidence oflocoregional recurrence after mastectomy ranges from 5 to 20o/o. These
estimates vary depending on age, axillary nodal status, breast cancer subtype, adjuvant
systemic therapy, and the use ofpostmastectomy radiation. However, lifetime rates of local
recurrence after mastectomy in a 40-year-old woman will always exceed 1%. A recent pop-
ulation-based study with long-term follow-up demonstrated no difference in local recur-
rence-free survival, locoregional recurrence-free survival, disease-free survival, or overall
survival comparing mastectomy to breast conservation therapy in women 40 or younger.
Contralateral prophylactic mastectomy decreases rates of contralateral breast cancer by
approximately 90-95o/o. However, it is not a guarantee that a young woman will not get
contralateral breast cancer and there are no data that mastectomy or contralateral prophy-
lactic mastectomy improves survival in young women with ER-positive breast cancer. In
fact, a recent population-based study from the California cancer registry reported inferior
survival for women younger than 40 treated with unilateral mastectomy or bilateral mas-
tectomy compared with breast conservation therapy.
Answer: (A) After margin-negative lumpectomy, whole breast radiation therapy, and
appropriate systemic therapy, her risk oflocal recurrence in 10 years will be less than 10%.
References
1. Anderson SJ, Wapnir I, Dignam JJ., et al. Prognosis after ipsilateral breast tumor recur-
rence and locoregional recurrences in patients treated by breast-conserving therapy in
five National Surgical Adjuvant Breast and Bowel Project protocols of node-negative
breast cancer. J Clin Oneal. 2009;27(15):2466-2473.
2. Jatoi I, Proschan MA. Randomized trials of breast-conserving therapy versus mastec-
tomy for primary breast cancer: a pooled analysis of updated results. Am J Clin Oneal.
2005;28(3):289-294.
3. Fisher B. Anderson S, Bryant J, et al. Twenty-year follow-up of a randomized trial
comparing total mastectomy~ lumpectomy, and lumpectomy plus irradiation for the
treatment of invasive breast cancer. N Engl J Med. 2002;347( 16): 1233-1241.
CATEGORY2 55
4. Arriagada R, Le MG, Guinebretiere JM, Dunant A, Rochard F, Tursz T. Late local recur-
rences in a randomised trial comparing conservative treatment with total mastectomy in
early breast cancer patients. Ann Oneal. 2003;14(1l):1617-1622.
5. Portschy PR, Kuntz KM. Tuttle TM. Survival outcomes after contralateral prophylactic
mastectomy: a decision analysis. J Natl Cancer Inst. 2014; 106(8).
6. Kurian AW, Lichtensztajn DY, Keegan TH, Nelson DO, Clarke CA, Gomez SL. Use of
and mortality after bilateral mastectomy compared with other surgical treatments for
breast cancer in California, 1998-2011. JAMA. 2014;312(9):902-914.
7. van der Leest M, Evers L, van der Sangen MJ, et al. The safety of breast-conserving
therapy in patients with breast cancer aged< or~ 40 years. Cancer. 2007;109(10): 1957-
1964.
8. van Laar C, van der Sangen MJ~ Poortmans PM~ et al. Local recurrence following
breast-conserving treatment in women aged 40 years or younger: trends in risk and
the impact on prognosis in a population-based cohort of 1143 patients. Eur J Cancer.
2013;49(15):3093-3101.
9. Cao JQ, Truong PT, Olivotto IA, et al. Should women younger than 40 years of age with
invasive breast cancer have a mastectomy? 15-year outcomes in a population-based
cohort. Int J Radial Oneal Biol Phys. 2014;90(3):509-517.
CRITIQUE27
Since the original description of lobular carcinoma in situ (LCIS), confusion has existed
regarding the most appropriate management. LCIS is a proliferative lesion of the terminal
duct lobular unit that is typically diagnosed as an incidental finding in the surrounding
breast tissue of otherwise benign breast biopsy specimens. Historically, the morphologic
and cytologic similarities between LCIS and invasive lobular carcinoma (!LC) led to the
hypothesis that LCIS was a precursor to !LC and should be treated with mastectomy. Sub-
sequently, retrospective data demonstrated that the risk of breast cancer after a diagnosis
of LCIS was approximately 1-2% per year and was conferred equally to both breasts. This
finding led to the acceptance ofLCIS as a "risk factor" for the subsequent development of
invasive breast cancer, and mastectomy fell out of favor.
A diagnosis of LCIS confers an increased risk of breast cancer that is 8- to 10-fold
higher than the general population. This risk is conferred equally to both breasts. As such,
there is no role for surgical excision to negative margins and there are no data that radia-
tion therapy reduces the risk of breast cancer in patients with LCIS. Patients with LCIS
should be counseled regarding their increased risk of breast cancer and are candidates for
chemoprevention.
Recent studies examining LCIS and synchronous breast cancers demonstrated similar
alterations in DNA copy number and common mutations in LCIS, DCIS, and !LC. These
studies reopened the debate regarding the significance ofLCIS as a precursor to breast can-
cer and led to current thinking that LCIS is both a risk factor and a "nonobligate precursor'·
lesion. A nonobligate precursor implies that the lesion may or may not progress to cancer.
Answer: (D) is a risk factor for the future development of breast cancer in both breasts.
References
1. Oppong BA. King TA. Recommendations for women with lobular carcinoma in situ
(LCIS). Oncology (Williston Park). 2011 ;25(11):1051-1056, 1058.
2. Rosen PP, Kosloff C, Lieberman PH, Adair F. Braun DW Jr. Lobular carcinoma in situ
of the breast. Detailed analysis of99 patients with average follow-up of24 years. Am J
Surg Pathol. l 978;2(3):225-251.
3. Anderson BO, Calhoun KE, Rosen EL. Evolving concepts in the management oflobu-
lar neoplasia. J Natl Compr Cane Netw. 2006;4(5):5! I-522.
--~·--------------------
56 CATEGORY:?:
CRITIQUE28
Triple-negative breast cancers (TNBC) are high-grade, poorly differentiated malignancies
that comprise approximately 15% of invasive breast cancer in the United States. TNBC
is more common in young patients and may be associated with a BRCAI mutation (up to
40% of women under the age of 40 with TNBC). In the context of a suspicious clinical
breast examination, a family history of premenopausal breast cancer and a positive core
biopsy result, there is no indication for excisional breast biopsy to confirm the diagnosis of
TNBC in a young woman.
Current National Comprehensive Cancer Network guidelines recommend genetic risk
assessment and counseling for any patient diagnosed with TNBC under the age of 60. In
addition, guidelines recommend genetic risk assessment and counseling for any patient
diagnosed with breast cancer under the age of 45, regardless of family history or breast
cancer subtype. Thus, this young woman has 2 indications for genetic referral, and the
possibility of the presence of BRCA mutation in a young woman should be considered in
surgical decision making.
Compared with non-TNBC, TNBC is associated with higher rates of local recurrence
after breast conservation; however, it is also associated with higher rates of local recur-
rence after mastectomy. Therefore, mastectomy is not superior to breast conservation. Fur-
thermore, in recent series TNBC was not an independent predictor of local recurrence
when controlling for other prognostic factors. No data indicated that mastectomy improves
survival in young women or in women with TNBC.
Neoadjuvant therapy is associated with rates of pathological complete response ap-
proaching 50o/o in TNBC; however, there is no survival advantage to neoadjuvant che-
motherapy over standard adjuvant chemotherapy. Therefore, unless neoadjuvant chemo-
therapy is indicated to downstage the primary tumor to improve cosmesis after breast
conservation, it is not indicated in this setting. No data support the use of preoperative
radiation therapy in a young women with a clinical Tl breast cancer.
References
I. Gonzalez-Angulo AM, Timms KM, Liu S, et al. Incidence and outcome of BRCA muta-
tions in unselected patients with triple receptor-negative breast cancer. Clin Cancer Res.
2011;17(5):1082-1089.
2. Abdulkarim BS, Cuartero J, Hanson J, Deschenes J, Lesniak D, Sabri S. Increased risk
of locoregional recurrence for women with Tl-2NO triple-negative breast cancer treated
with modified radical mastectomy without adjuvant radiation therapy compared with
breast-conserving therapy. J Cl in Oneal. 2011 ;29(21 ):2852-2858.
3. Adkins FC, Gonzalez-Angulo AM, Lei X, et al. Triple-negative breast cancer is not a
contraindication for breast conservation. Ann Surg Oneal. 2011; 18(11 ):3164-3173.
4. Zumsteg ZS, Morrow M, Arnold B, et al. Breast-conserving therapy achieves locore-
gional outcomes comparable to mastectomy in women with Tl-2NO triple-negative
breast cancer. Ann Surg Oneal. 2013;20(11):3469-3476.
5. Lowery AJ, Kell MR, Glynn RW, Kerin MJ, Sweeney KJ. Locoregional recurrence after
breast cancer surgery: a systematic review by receptor phenotype. Breast Cancer Res
Treat. 2012;133(3):831-841.
6. Newman LA, Reis-Filho JS, Morrow M, Carey LA, King TA. The 2014 Society of
Surgical Oncology Susan G. Komen for the Cure Symposium: triple-negative breast
cancer. Ann Surg Oneal. 2015;22(3):874-882.
CATEGORY2
57
CRITIQUE29
Prophylactic mastectomy in the setting of unilateral breast cancer is a controversial topic
fueled by anecdote, media reporting, and strong emotion. There is no "'one-size-fits-all"
answer for breast cancer patients, and the risk of concurrent or subsequent disease in the
contralateral breast depends on numerous factors, including genetics, pathologic type, hor-
mone receptor status, patient age, tumor stage, and family history.
A study that used decision-tree modeling and reported outcome probabilities demon-
strated that women with stage I and II breast cancer without BRCA mutations had less
than a 1% absolute survival difference at 20 years. This Markov decision analysis did not
identify any group of patients that had even a I% improvement in 20-year survival with
contralateral prophylactic mastectomy.
Compared with older women, contralateral mastectomy in younger patients was associ-
ated with increased survival; this increase in survival was due in part to the decreased life
expectancy of the older cohort. Prophylactic contralateral mastectomy may be beneficial
in patients with estrogen receptor-positive status or BRCA mutations, because the risk of
developing a contralateral breast cancer is higher. The benefit of contralateral mastectomy
in stage II disease is lower because of the worse survival prognosis from the primary breast
cancer rather than a decreased survival risk from the potential of developing cancer in the
contralateral breast.
References
1. Lostumbo L. Carbine NE, Wallace J. Prophylactic mastectomy for the prevention of
breast cancer. Cochrane Database Syst Rev. 2010;(1 l):CD002748.
2. Portschy PR, Kuntz KM, Tuttle TM. Survival outcomes after contralateral prophylactic
mastectomy: a decision analysis. J Natl Cancer Inst. 2014; 106(8).
CRITIQUE 30-32
Despite their similar sounding names, atypical ductal hyperplasia (ADH) and atypical
lobular hyperplasia (ALH) have very different clinical features. ADH is a site-specific
premalignant breast lesion that meets some, but not all, of the histologic features of ductal
carcinoma in situ (DCIS). The lesion is usually located near microcalcifications on mam-
mography and may appear as a microlobulated mass with mild hypoechogenicity and no
acoustic enhancement or shadowing on ultrasound. Core biopsies of ADH underdiagnoses
DCIS or invasive cancer I 0- 60% of the time; therefore, follow-up open excisional biopsy
of ADH is usually recommended to exclude cancer.
By contrast, ALH is a non-site-specific risk lesion with no distinctive associated imag-
ing findings. It is often an incidental finding on core biopsies performed to assess other
lesions. Collectively, ALH and lobular carcinoma in situ (LCIS) are termed lobular neo-
plasia. The lobular neoplasia phenotype is likely caused by decreased or absent expres-
sion of e-cadherin, a plasma membrane cell-adhesion molecule. This molecular aberration
is thought to be the cause of the characteristic dyshesiveness of lobular neoplasia cells
("the single filing" seen on histology). Although National Comprehensive Cancer Network
guidelines recommend surgical consultation if lobular neoplasia is diagnosed on core bi-
opsy, recent prospective series found that the rates of missed cancer are only 2-3o/o and
concluded that with a multidisciplinary approach correlating pathology with imaging find-
ings, many patients can be observed without open biopsy.
58 CATEGORY2
Answers:
30. (B) Atypical ductal hyperplasia
31. (A) Atypical lobular hyperplasia
32. (A) Atypical lobular hyperplasia
References
I. King TA, Reis-Filho JS. Lobular neoplasia. Surg Oneal Clin N Am. 2014:23(3):487-503.
2. Mesurolle B, Perez JC, Azzumea F, et al. Atypical ductal hyperplasia diagnosed at sono-
graphically guided core needle biopsy: frequency, final surgical outcome, and factors
associated with underestimation. AJR Am J Roentgenol. 20l4;202(6):1389-1394.
3. Neal L, Sandhu NP, Hieken TJ, et al. Diagnosis and management of benign, atypi-
cal, and indeterminate breast lesions detected on core needle biopsy. Mayo Clin Proc.
20 l 4;89(4):536-547.
CRITIQUE 33-37
Both tamoxifen (selective estrogen receptor modulator) and aromatase inhibitors act as
effective adjuvant therapy for postmenopausal women with ER-positive breast cancer.
However, the side effect profile favors the use of aromatase inhibitors in postmenopausal
women with an intact uterus. Aromatase inhibitors are not sufficient as adjuvant therapy for
premenopausal women, because they do not inhibit estrogen production from the ovaries.
If used in premenopausal women, aromatase inhibitors must be used in combination with
ovarian suppression.
Multiple studies report the effectiveness oftamoxifen for reducing rates of ER-positive
breast cancer in the setting of primary prevention and after adjuvant therapy. The NSABP-
Pl trial demonstrated that tamoxifen reduces the risk of ER-positive breast cancer in both
premenopausal and postmenopausal women at increased risk of breast cancer (defined as
age >65 years, 5-year Gail risk> 1.6, or having a diagnosis of atypia or lobular carcinoma
in situ). The trial randomized 13,388 women to 5 years oftamoxifen or placebo and dem-
onstrated that tamoxifen reduced the risk of invasive and noninvasive ER-positive breast
cancer approximately 50o/o but had no effect on rates of ER-negative breast cancer.
In the ATAC (anastrozole, tamoxifen alone, or in combination) trial, investigators
compared the safety and effectiveness oftamoxifen and anastrozole (aromatase inhibitor)
and demonstrated thatneithertamoxifen nor aromatase inhibitors resulted in an improvement
in disease-free survival for women with ER-negative breast cancer; however, both reduced
rates of contralateral breast cancer and local recurrence in women with ER-positive breast
cancer. More recently, the Early Breast Cancer Trialists' Collaborative Group (EBCTCG)
conducted a meta-analysis of individual patient data from 20 trials (n = 21,457) of early
breast cancer patients and confirmed that 5 years of adjuvant tamoxifen reduces 15-
year risks of both local and distant breast cancer recurrence and death in patients with
ER-positive disease.
Answers:
33. (C) Both
34. (B) Tamoxifen
35. (D) Neither
36. (C) Both
37. (C) Both
References
1. Early Breast Cancer Trialists' Collaborative Group (EBCTCG); Davies C, Godwin J,
Gray R. et al. Relevance of breast cancer hormone receptors and other factors to the
efficacy of adjuvant tamoxifen: patient-level meta-analysis of randomised trials. Lancet.
2011 :378(9793):771-784.
CATEGORY2 59
2. Vogel VG, Costantino JP, Wickerham DL, et al; National Surgical Adjuvant Breast and
Bowel Project. Update of the National Surgical Adjuvant Breast and Bowel Project
Study ofTamoxifen and Raloxifene (STAR) P-2 Trial: preventing breast cancer. Cancer
Prev Res (Phi/a). 2010;3(6):696-706.
3. Fisher B, Costantino J, Redmond C, et al. A randomized clinical trial evaluating tamoxi-
fen in the treatment of patients with node-negative breast cancer who have estrogen-
receptor-positive tumors. N Engl J Med. l 989;320(8):479-484.
4. Baum M, Budzar AU, Cuzick J, et al; ATAC Trialists' Group. Anastrozole alone or in
combination with tamoxifen versus tamoxifen alone for adjuvant treatment of post-
menopausal women with early breast cancer: first results of the ATAC randomised
trial [published correction appears in Lancet. 2002;360(9344):1520]. Lancet. 2002;
359(9324):2131-2139.
5. Fisher B, Costantino JP, Wickerham DL, et al. Tamoxifen for prevention of breast
cancer: report of the National Surgical Adjuvant Breast and Bowel Project P-1 Study.
J Natl Cancer Inst. 1998;90(18): 1371-1388.
CRITIQUE 38-43
Fibroepithelial lesions of the breast include fibroadenoma and phyllodes tumors. Fibroad-
enoma, the most common breast mass in younger women, is truly benign, whereas phyl-
lodes tumors represent a spectrum from benign to various degrees of malignancy. Phyl-
lodes tumors are rare, representing less than 0.5% of all breast tumors. They can occur in
women of all ages, yet are most typically seen in premenopausal women.
Fibroadenoma are generally well encapsulated and demonstrate a smooth interface with
the surrounding tissue. This makes enucleation possible and the appropriate management
for fibroadenoma that require excision. Phyllodes tumors are typically excised with a wider
excision. The general recommendation is 1-cm margins; however, for benign phyllodes tu-
mors, the evidence that margin width impacts recurrence is limited, at best. For malignant
phyllodes, a margin of at least 1 cm should be obtained, yet breast conservation remains an
appropriate option for all phyllodes tumors, and the need for mastectomy, which is rarely
necessary, is based only on the size of the lesion and cosmetic issues.
Fibroadenoma and benign phyllodes tumors are so histopathologically similar that dif-
ferentiating between these lesions can be problematic when limited tissue is available,
such as with core needle biopsy. Both lesions contain epithelial and stromal elements, yet
the mitotic count is low, and neither contain fat. By contrast, malignant phyllodes tumors,
like other breast carcinomas, may show stromal overgrowth and necrosis. For cellular le-
sions where a conclusive diagnosis cannot be made on limited tissue, complete excision
is recommended. Mitotic counts of at least l 0/10 high-powered fields and a high Ki-67
index are helpful in identifying malignant phyllodes tumors. Molecular analysis can dif-
ferentiate between benign and malignant phyllodes tumors. In contrast to fibroadenomas,
phyllodes tumors do carry a risk of both local and distant recurrence; however, the latter
is very uncommon. Local recurrence is as high as 50o/o in some series, whereas rates of
distant recurrence are less than 10%. Metastasis from phyllodes tumors is usually via the
hematogenous route not via lymphatic spread. Therefore, sentinel lymph node biopsy is not
recommended, and axillary lymph nodes dissection for phyllodes tumors is reserved for
clinically palpable axillary adenopathy.
---··------·····--·-----------------------
60 CATEGORY2
Answers:
38. (B) Phyllodes tumor
39. (C) Both
40. (B) Phyllodes tumor
41. (A) Fibroadenoma
42. (D) Neither
43. (A) Fibroadenoma
References
I. Giri D. Recurrent challenges in the evaluation of fibroepithelial lesions. Arch Pathol
Lab Med. 2009;133(5):713-721.
2. Grobmeyer SR, Copeland EM 3rd, Simpson JF, Page DL. Benign, high-risk and prema-
lignant lesions of the breast. Jn: Bland KI, Copeland EM 3rd, eds. The breast: compre-
hensive management of benign and malignant diseases. 4th ed. Philadelphia, PA: Saun-
ders·Elsevier;2009: 169-183.
3. Khosravi-Shahi P. Management of non metastatic phyllodes tumors of the breast: review
of the literature. Surg Oneal. 2011;20(4):el43-el48.
4. Spitaleri G, Toesca A, Botteri E, et al. Breast phyllodes tumor: a review of liter-
ature and a single center retrospective series analysis. Crit Rev Oneal Hematol.
2013 ;88(2):427-436.
5. Yang X, Kandi! D, Cosar EF, Khan A. Fibroepithelial tumors of the breast: pathologic
and immunohistochemical features and molecular mechanisms. Arch Pathol Lab Med.
2014; 138(1 ):25-36.
CRITIQUE 44-46
The Breast Imaging Reporting and Data System (BI-RADS) is a quality assurance tool
designed to provide a standardized system of breast imaging reporting (table 44-46.1). It
was developed and regularly updated by the American College of Radiology. Its use for
mammographic reporting is ubiquitous, and a clear understanding of its terms is important
for any practitioner who orders breast imaging studies. Biopsy is not required for lesions
classified as BI-RADS-3. A biopsy is required for BI-RADS-4A, B, or C lesions, which
may be a mass, calcifications, or architectural distortion, and appropriate action should be
taken as determined by biopsy results. A BI-RADS 4B has a moderate risk for malignancy
and ifthe biopsy is negative, observation is acceptable. BI-RADS-5 lesions found to be be-
nign on core biopsy are considered ""benign-discordant" and should be surgically excised.
Minimally invasive biopsy (i.e., core biopsy) is appropriate for the vast majority of
breast lesions, and the use of excisional biopsy should be avoided except when core biopsy
is not technically feasible.
Answers:
44. (C) Both
45. (B) BIRADS-4B mammogram
46. (A) BIRADS-5 mammogram
CATEGORY2
61
Category 4: Suspicious l
--ca~~ii~~Y-3:i~~~ifilr.j~~~~~~~==~~==~~=~=~=.==~~~-~~~~~=-===~=-~~~~~~~=~~=~==~~~~~~=
Mammography Category 4A: low suspicion for malignancy
----···------··-···------·-····--- ···------~'..~-~~::~------~~;:;;~-;~~-~l~~:;~~;!~r~~;-~~~;;_~~;anc~--------·-----·----------
·--~-t_i:s!?..'1_~.:-~i9~!Y..!_u_~.s.~E~~-~~-~~-!i9_~~~x_-··--------·---------·---------···------··-----------------------···----------·---
Category 6: Known Biopsy-Proven Malignancy
References
I. Sickles, EA, D'Orsi CJ, Bassett LW, et al. ACR BI-RADS® Mammography. In: ACR
BI-RADS® Atlas, Breast Imaging Reporting and Data System. Reston, VA, American
College of Radiology; 2013.
2. Levin DC, Parker L, Schwartz GP, Rao VM. Percutaneous needle vs surgical breast
biopsy: previous allegations of overuse of surgery are in error. J Am Coll Radio/.
2012;9(2): 137-140.
3. Linebarger JH, Landercasper J, Ellis RL, et al. Core needle biopsy rate for new cancer
diagnosis in an interdisciplinary breast center: evaluation of quality of care 2007-2008.
Ann Surg. 2012;255(1):38--43.
CRITIQUE 47--49
With increased awareness of breast cancer and advances in screening programs and imag-
ing studies, the number of suspicious lesions detected radiologically has also risen. Image-
guided core needle biopsy. instead of surgical excision, is the procedure of choice to inves-
tigate mammographically •·suspicious" lesions of the breast.
Fibroadenomas are solid benign breast tumors that occur mainly in women under the
age of 30. They present as smooth, finn, rubbery, and well-circumscribed round or oval
masses on physician exam and mammography. Diagnosis can be established with a fine-
needle aspiration or a percutaneous core biopsy. Fibroadenomas are not considered a risk
factor for the development of breast cancer.
Atypical ductal hyperplasia (ADH) is found in up to 15% of core needle biopsy speci-
mens done for abnonnal mammographic findings. Although it is not breast cancer, ADH
is associated with an increased risk for the development of breast cancer. After a diagnosis
of ADH on a core needle biopsy, surgical excision is recommended, because up to 18%
of the lesions can be upstaged to either infiltrating carcinoma (5%) or ductal carcinoma
in situ (13%). If the final excisional specimen demonstrates ADH only, close observation,
chemoprevention, or prophylactic risk-reducing surgery should be considered.
Because neither fibroadenoma nor ADH is invasive breast cancer, sentinel lymph node
biopsy or adjuvant chemotherapy is not necessary.
Answers:
47. (D) Neither
48. (B) Atypical ductal hyperplasia
49. (A) Fibroadenoma
·-·---,---
_______ __________________
,
62 CATEGORY 2
References
1. Moore MM, Hargett CW 3rd, Hanks JB, et al. Association of breast cancer with the find-
ing ofatypical ductal hyperplasia at core breast biopsy. Ann Surg. 1997;225(6):726-731.
2. Dupont WD, Page DL, Parl FF, et al. Long-term risk of breast cancer in women with
fibroadenoma. N Eng!J Med. 1994;331(1):10-15.
3. McGhan LJ, Pockaj BA, WasifN, Giurescu ME, McCullough AE, Gray RJ. Atypical
ductal hyperplasia on core biopsy: an automatic trigger for excisional biopsy? Ann Surg
Oneal. 2012;19(10):326~3269.
Category 3 - Part I
Alimentary Tract
Items 1--45
DIRECTIONS: Each of the questions or incomplete statements is followed by 5 suggested
answers or completions. Select the answer that is the BEST in each case and fill in the
space containing the corresponding letter on the answer sheet.
3. Which of the following statements regarding the management of a patient who pres-
ents with portal hypertension and bleeding esophageal varices is true?
(A) Intravenous octreotide should be administered during resuscitation.
(B) One dose of intravenous antibiotics should be given before endoscopy.
(C) Endoscopic sclerotherapy has fewer complications than endoscopic variceal
ligation when used to control acute hemorrhage.
(D) Transjugular intrahepatic portosystemic shunt increases the hepatic venous
pressure gradient.
(E) Follow-up endoscopy after initial hemorrhage control is unnecessary.
4. A 52-year-old woman has a I-cm sessile polyp removed from her cecum during a
colonoscopy. Forty-eight hours later, the patient presents with right lower quadrant
pain. Her temperature is 38°C, her heart rate is 72 beats per minute, and her blood
pressure is 130/80 mm Hg. She is focally tender in the right lower quadrant. Her
white blood count is 14,200/mml (3600-11.200/mml) A CT scan shows inflammation
around her cecum without extraluminal air. The final pathology showed a benign vil-
lous adenoma. All margins are free of adenomatous tissue. Which of the following is
the best management of this patient?
(A) Intravenous antibiotics and observation
(B) Endoscopic closure ofpolypectomy site
(C) Laparoscopic peritoneal lavage
(D) Cecal resection with primary anastomosis
(E) Cecal resection with ileostomy
CATEGORY 3 - PART I 65
S. A 38-year-old woman who had laparoscopic adjustable gastric band placement 3 years
ago reports a several-week history of worsening solid food intolerance and reflux. She
has severe left upper quadrant abdominal pain. She is febrile to 38.3°C, her heart rate
is 120 beats per minute, and her a blood pressure is 100/50 mm Hg. A fluoroscopic
image is shown in figure 5.1. Which of the following is the most appropriate next step
in management of this patient?
(A) Removal of saline from the gastric band
(B) Upper gastrointestinal series
(C) Endoscopic dilatation
(D) Laparoscopy
(E) Nasogastric tube
Figure 5.1
6. A 55-year-old man initially presents with bright red blood per rectum. On colonoscopy,
he has a 4-cm friable mass 6 cm from the dentate line. Pathology reveals moderately
differentiated adenocarcinoma. Pelvic MRI demonstrates invasion into the perirectal
fat with no invasion into surrounding structures or irregular lymph nodes. CT scan of
the chest, abdomen~ and pelvis reveals no evidence of metastatic disease. The next best
step in the management of this patient is
(A) low anterior resection with total mesorectal excision followed by
adjuvant chemoradiation.
(B) neoadjuvant chemoradiation followed by low anterior resection with total
mesorectal excision.
(C) low anterior resection with total mesorectal excision only.
(D) local excision only.
(E) local excision followed by adjuvant chemoradiation.
·--·-------
66 CATEGORY 3 - PART J
Items 8-12
(A) Crohn disease
(B) Ulcerative colitis
(C) Both
(D) Neither
Items 13-16
(A) Surgical resection
(B) Observation
(C) Both
(D) Neither
CATEGORY 3 - PART I 67
13. Proximal esophageal squamous cell carcinoma with clinical complete response after
chemoradiotherapy
14. Distal esophageal adenocarcinoma with clinical complete response after chemoradio-
therapy
16. Distal esophageal adenocarcinoma with lung metastasis after definitive chemoradio-
therapy
Items 17-19
(A) Lynch syndrome
(B) Familial adenomatous polyposis coli
(C) Both
(D) Neither
DIRECTIONS: Each group of items that follow consists of 5 lettered headings or illustra-
tions followed by a list of numbered words, phrases, or illustrations. For each numbered
word, phrase, or illustration, select one lettered heading or illustration that is most closely
associated with it and fill in the space containing the corresponding letter on the answer
sheet. Each lettered heading or illustration may be selected once, more than once, or not
at all.
Items 20-22
(A) Endoscopic clips
(B) High-dose intravenous proton pump inhibitors
(C) Somatostatin
(D) Metoclopramide
(E) Epinephrine injection
20. Most effective therapy for high-risk nonvariceal upper gastrointestinal hemorrhage
Items 23-25
(A) Open appendectomy
(B) Laparoscopic appendectomy
(C) Intravenous antibiotics alone
(D) Percutaneous drain placement
(E) Oral antibiotics alone
23. 23-year-old woman with localized pain, white blood cell count= 16,000/mm3 (3600-
l l ,200/mm3); CT scan shows uncomplicated, acute appendicitis
24. 26-year-old man with minimal diffuse abdominal pain and vomiting, white blood cell
count = 26,000/mm3 (3600-l l ,200/mm3); CT scan shows perforated appendicitis
with a 6-cm abscess
25. 32-year-old man with localized right lower quadrant pain, nausea~ and vomiting, white
blood cell count= 16,000/mm3 (3600-11,200/mm3); CT scan shows 8-cm periappen-
diceal phlegmon
Items 26--28
(A) Gastric antral vascular ectasia
(B) Peptic ulcer
(C) Dieulafoy lesion
(D) Gastric varices
(E) Aortoenteric fistula
Items 29-32
(A) Observation
(B) Local wound care
( C) Infliximab
(D) Stoma resiting
(E) Local stoma revision
Items 33-35
(AJ Calcium channel blockers
(B) Botulinum toxin injection
(CJ Pneumatic dilatation
(D) Peroral endoscopic myotomy (POEM)
(EJ Laparoscopic Heller myotomy
33. Division of the inner circular muscle layer of the esophagus via a submucosal tunnel
34. Mechanical tear of the muscle fibers of the lower esophageal sphincter
36. A 59-year-old woman presents with painless jaundice and is found to have a mass
within the head of her pancreas on triple-phase CT. Endoscopic retrograde cholangio-
pancreatoscopy confirms the presence of a pancreatic mass, and brushings are posi-
tive for adenocarcinoma. Which of the following CT findings would preclude surgical
resection in this patient?
(AJ Occlusion of the mid portion of the portal vein
(BJ Complete encasement of the gastroduodenal artery
(C) Tumor abutment of the hepatic artery
(D) Distortion of the entire superior mesenteric/portal vein axis
(E) Three-quarters encasement of the superior mesenteric artery
37. Which of the following statements regarding intraductal papillary mucinous neo-
plasms (IPMNJ of the pancreas is true?
(A) The tubular variety of carcinoma arising in an IPMN indicates a better prognosis
than the colloidal type.
(BJ Main-duct IPMN have greater malignant potential than do the branch
duct type.
(C) IPMN is more common in men than in women.
(D) Cystic lesions larger than 3 cm should undergo surgical resection.
(E) Branch-duct !PMN should undergo resection regardless of size.
38. A 52-year-old man with no family history of colorectal cancer undergoes screening
colonoscopy. A 3-cm sessile polyp at the hepatic flexure is removed with piecemeal
snare cautery technique. The pathology comes back as tubular adenoma. The next
follow-up colonoscopy should be performed in
(A) 3 months.
(B) 1 year
(CJ 3 years.
(DJ 5 years.
(EJ I 0 years.
··---~--~-~---------
72 CATEGORY 3 - PART l
39. A45-year-old man has a recent history ofa 13.6-kg weight loss, back pain, jaundice,
and pruritus. He is found to have a pancreatic mass and liver lesions. Biopsy of one of
the liver lesions demonstrates metastatic adenocarcinoma consistent with a pancreatic
primary. Which of the following statements is true regarding palliation in this patient?
(A) Percutaneous transhepatic catheter placement provides optimal palliation.
(B) Gastrojejunostomy reduces the incidence of later gastric outlet obstruction
by 50%.
(CJ An endoscopically placed duodenal stent will prevent later gastric outlet
obstruction.
(D) Endoscopic biliary stenting is indicated to relieve the pruritus.
(E) Pancreaticoduodenectomy will optimally palliate this fit individual.
43. A 62-year-old man has acute onset of crampy abdominal pain. His CT scan is consis-
tent with a high-grade small bowel obstruction with a 4-cm solid tumor obstructing
the terminal ileum. At laparotomy, there are no detectable metastases or lymphade-
nopathy. The tumor is removed via ileocolectomy, and the patient has an unremarkable
recovery in hospital from surgery. Pathology reveals a gastrointestinal stromal tumor
(GIST) with negative margins, intact pseudocapsule and 5 negative lymph nodes, and
approximately 2/50 mitoses per high-power-field (T2NOMO). Immunohistochemistry
shows CD 117 and DOG 1 positivity. Which of the following statements is true?
(A) This patient should receive adjuvant imatinib mesylate.
(B) Recurrence is likely to be extra abdominal.
(C) The tumor recurrence rate is approximately 50%.
(D) Prognosis is better ifthe tumor is located in the stomach.
(E) Mutational analysis should be done on the tumor.
44. A 65-year-old woman undergoes an MRI for back pain. The MRI scan shows a 2-cm
simple intrapancreatic cyst in the mid pancreas, emanating from a branch of the main
pancreatic duct, without a solid component. The cyst wall is not thickened. The main
pancreatic duct is 4 mm. She has no prior history of pancreatic abnormality. The next
step in the evaluation and management of this cyst is
(A) repeat MRI scan in I year.
(B) endoscopic ultrasound now.
(C) endoscopic ultrasound in 6 months.
(D) diagnostic laparoscopy.
(E) distal pancreatectomy.
45. A 56-year-old man presents with a 36-hour history of a retained foreign body of the
low rectum, which is palpable. Plain abdominal radiographs demonstrate no free air.
He is afebrile, and an abdominal exam reveals a slightly distended, tympanitic abdo-
men without peritoneal irritation. Which of the following should be the next step in
his management?
(A) CT scan of the abdomen and pelvis
(B) Digital retrieval
(C) Exploratory laparotomy
(D) Endoscopic retrieval
(E) Retrieval with Foley catheter/balloon inflation in the rectosigmoid
74 CATEGORY 3 - PART I
Critiques 1--45
CRITIQUE 1
Patients diagnosed with Barrett esophagus with high-grade dysplasia (BE-HGD) should
undergo definitive treatment, because the risk of the lesion progressing to adenocarcinoma
is high (6-19% per year). There is also a high likelihood of occult carcinoma, with some
reports suggesting that up to 40o/o ofBE-HGD harbor invasive cancer. Nissen fundoplica-
tion is not an acceptable treatment strategy for BE-HGD, because it does not excise or
ablate the lesion. Observation with repeat endoscopy in 3 months is also not appropriate.
Currently, endoscopic treatment is preferred over esophagectomy for BE-HGD. Endo-
scopic therapies are effective and safe in the treatment of patients with BE-HGD. Equally
important, these endoscopic therapies preserve the esophagus and avoid the morbidity of
esophagectomy, particularly in elderly or fragile patients. The 2 most common endoscopic
treatments for BE-HGD are radiofrequency ablation (RFA) and endoscopic mucosa! resec-
tion (EMR).
RFA is delivered endoscopically using a 3-crn long radio-frequency balloon with a coil
mounted on the balloon surface that contacts the esophageal mucosa circumferentially.
Radiofrequency energy is delivered through the coil in less than 1 second and creates a
superficial burn to the esophageal mucosa. The balloon can be moved up or down along the
length of the esophagus, with multiple doses ofRFA applied to completely ablate long seg-
ments of Barrett esophagus. Although these patients need careful surveillance to evaluate
for persistent or recurrent disease, overall results are excellent. Many large studies show
complete eradication of dysplasia in more than 90% of patients. Complications and post-
treatment strictures are extremely infrequent.
EMR uses a '"cap-and-snare" technique, with or without a rubber band, to create a suc-
tion polyp followed by a cauterized snare to resect down to the submucosal level while
simultaneously achieving hemostasis. Ideally, EMR should be reserved for lesions smaller
than 2 cm. Resection of lesions larger than 2 cm requires a ''piecemeal" approach and can
leave behind some dysplastic mucosa. In addition, a circumferential EMR over a lengthy
segment can result in posttreatment stricture. For these reasons, RFA is generally preferred
over EMR for segments of Barrett esophagus longer than 2 cm. However, both endoscopic
procedures are in current guidelines to treat BE-HGD.
Photodynamic therapy (PDT) was used to treat BE-HGD, but it has fallen out of favor.
PDT is a suboptimal treatment modality, because the photosensitizer that the patients re-
ceive before therapy is associated with extreme sunlight toxicity that can persist up to 90
days after treatment. In addition, because most patients have to undergo several courses of
PDT to eradicate the lesion, it is associated with a high rate of stricture (31 %).
References
1. Davila ML. Hofstetter WL. Endoscopic management of Barrett's esophagus with
high-grade dysplasia and early-stage esophageal adenocarcinoma. Thorac Surg C!in.
20 I 3;23(4):479-489.
2. Roorda AK, Marcus SN, Triadafilopoulos G. Early experience with radiofrequency
energy ablation therapy for Barrett's esophagus with and without dysplasia. Dis Esoph-
agus. 2007:20(6):516-522.
3. Wang KK, Tian JM, Gorospe E, et al. Medical and endoscopic management of high-
grade dysplasia in Barrett's esophagus. Dis Esophagus. 2012;25(4):349-355.
CATEGORY 3 - PART I 75
CRITIQUE2
Understanding gastric anatomy and the physiology of acid secretion is important in deter-
mining the diagnosis and management of peptic ulcers. Currently. 5 types of gastric ulcers
are described based on location and acid secretion. Type I ulcers are located on the lesser
curvature of the stomach and are associated with normal or decreased acid secretion. Type
II ulcers are gastric ulcers associated with an active or quiescent duodenal ulcer. Type III
ulcers are located in the prepyloric region. Both type II and III are associated with normal
or increased acid production. Type IV ulcers occur near the gastroesophageal junction in
the setting of normal or decreased acid production. Type V ulcers are medication induced
and may be located anywhere within the stomach.
Nonsteroidal anti-inflammatory drugs (NSA!Ds) are a common cause of type V ulcers.
NSAIDS inhibit mucus secretion in the stomach, making the mucosa more susceptible to
gastric acid, but they do not increase acid secretion. In the United States, 24% of cases are
caused by NSAID use. and 48% are associated with Helicobacter pylori infection. Overall.
analysis of the normal stomach epithelial cells reveals that 13% are parietal cells, 44% are
chief cells, 40% are mucous cells, and 3% endocrine cells. In general, the antrum produces
gastrin but not acid, and the proximal stomach produces acid but not gastrin. Parietal cells
are stimulated to secrete acid when I or more of 3 membrane receptors are stimulated by
acetylcholine (vagus nerve), gastrin (D cells), or histamine (enterochromaffin-like cells).
Answer: (C) Gastrin hypersecretion leads to increased parietal cell production of acid.
References
1. Dempsey DT. Stomach. In: Brunicardi F, Andersen DK, Billiar TR, et al, eds. Schwartz s
Principles of Surgery. 9th ed. New York, NY: McGraw-Hill;2010:889-948.
2. Lagoo J, Pappas TN, Perez A. A relic or still relevant: the narrowing role for vagotomy
in the treatment of peptic ulcer disease. Am J Surg. 2014;207(1): 120-126.
CRITIQUE3
Varices from portal hypertension are the cause of upper gastrointestinal bleeding in 5-1 Oo/o of
patients. Treatment starts with standard resuscitation principles for fluid and blood. Because
many patients with portal hypertension will have abnormal bleeding profiles, clotting studies
to assess their coagulation status should be done early. Additionally, these patients require
prophylactic antibiotics, which should be started as part of the resuscitation and continued for
7 days. These antibiotics will decrease the risk of bacterial infections and increase survival.
Unlike bleeding from a peptic ulcer, patients with a variceal hemorrhage should be
started on octreotide, somatostatin, terlipressin, or vasopressin, which will reduce portal
blood flow and thus portal pressure. Of these agents, only octreotide and vasopressin are
available in the United States. The recommended dose of octreotide is a 50-µg intravenous
bolus followed by a 50-µg/hour infusion. If variceal hemorrhage is confirmed on endos-
copy, this therapy should be continued for 3-5 days after the endoscopy
Endoscopic therapy with either sclerotherapy or variceal banding is used to halt bleed-
ing varices. Endoscopic sclerotherapy controls active esophageal variceal bleeding in more
than 90% of patients and reduces the frequency and severity of recurrent variceal hemor-
rhage. Sclerotherapy complications, mucosal ulceration, bleeding, esophageal perforation,
and mediastinitis occur in 15-20% of patients, with an associated mortality rate of 2%.
Banding has a lower complication rate that sclerotherapy. Once endoscopic treatment has
begun, it should be continued until varices are eradicated. A suggested frequency of repeat
endoscopy is 1- to 8-week intervals for endoscopic variceal banding.
Medical management fails in 5-10% of patients with bleeding esophageal varices. Al-
though surgical shunts are possible, they have been supplanted by transjugular intrahepatic
portosystemic shunt (TIPS). TIPS is not without complications; 20% of patients develop
encephalopathy, and 50% may occlude their shunt within 1 year.
76 CATEGORY 3 - PART I
References
1. Garcia-Tsao G, Bosch J. Management of varices and variceal hemorrhage in cirrhosis
[published correction appears in N Engl J Med. 2011 ;364(5):490. Dosage error in article
text.]. N Engl J Med. 2010;362(9):823-832.
2. Hwang JH, Shergill AK, Acosta RD, et al; American Society for Gastrointestinal Endos-
copy. The role of endoscopy in the management of variceal hemorrhage. Gastrointest
Endosc. 20 l 4;80(2):221-227.
CRITIQUE4
Perforation after colonoscopy is an uncommon complication, ranging from 0.01 % to 0.6%,
depending on the indication for the procedure and whether the procedure is diagnostic or
therapeutic. Perforations can result from mechanical forces from the colonoscope, baro-
trauma, or directly from the energy necessary for therapeutic maneuvers, such as polypec-
tomy. Perforations from mechanical forces usually result in a large defect mandating surgi-
cal intervention. If recognized immediately, surgery can be either a resection or a primary
closure, depending on the extent and location of the injury. More recently, endoscopic clips
are being used to close small defects recognized immediately.
However, not all injuries require surgical intervention. Thermal energy used during a
polypectomy can cause a full-thickness injury to the bowel wall. Collectively, these inju-
ries are classified as postpolypectomy coagulation syndrome. Typically, patients will pres-
ent shortly after a therapeutic colonoscopy with pain at the site of the polypectomy site.
The patient may have tenderness on exam, fever, and a leukocytosis. The presentation may
be very similar to a patient with a frank perforation; therefore, further imaging is neces-
sary to identify these patients. A CT scan is used to exclude significant extraluminal air
and may show significant inflammation at the site of the polypectomy. As long as the pa-
tient is hemodynamically normal and without evidence of diffuse peritonitis, nonoperative
management with intravenous antibiotics, bowel rest, and observation is appropriate. Even
in the setting of some extraluminal air, suggestive of a perforation, perforations are often
small and seal quickly. Therefore, without evidence of systemic toxicity, close observa-
tion is appropriate. In a recent study, 88% of the perforations from a diagnostic procedure
mandated surgery, whereas 91 o/o of the perforations after a therapeutic procedure were suc-
cessfully managed nonoperatively. Serial exams are mandated if a nonoperative approach
is attempted. With clinical deterioration, surgery is required.
In this scenario, the patient has evidence of inflammation associated with the polypec-
tomy site. She does not have a large perforation evident on her CT scan, making surgery
unnecessary at this time. Endoscopic closure is not helpful with a delayed presentation.
Intravenous antibiotics and observation, therefore, is the most appropriate therapy.
References
1. Avgerinos DV, Llaguna OH, Lo AY, Leitman IM. Evolving management of colono-
scopic perforations. J Gastrointest Surg. 2008; 12(10): 1783-1789.
2. Cha JM, Lim KS, Lee SH, et al. Clinical outcomes and risk factors of post-polypectomy
coagulation syndrome: a multicenter, retrospective, case-control study. Endoscopy.
20 l 3;45(3):202-207.
3. Waye JD, Lewis BS, Yessayan S. Colonoscopy: a prospective report of complications.
JC/in Gastroenterol. 1992;15(4):347-351.
CATEGORY 3 - PART I 77
CRITIQUES
This patient has a gastric prolapse or slippage of her gastric band (figure 5.2). Slippage
of the gastric band may be either anterior or posterior, with the posterior type being more
common. Band slippage occurs in 2-3% of patients. A properly placed band will have an
approximately 45° upward angle from the horizontal plane on a plain film of the abdomen
(figure 5.3). Figure 5.4 shows another slipped band. The band appears in a 30° downward
angle from horizontal. The contrast demonstrates a large gastric prolapse. A patient suf-
fering from slippage of the gastric band will complain of oral intake intolerance, reflux,
and vomiting. In most patients, a slipped band is not an urgent situation, and removal of
fluid from the band will give some symptomatic relief until an elective procedure can be
scheduled. The patient will need a revision of the band to correct the slippage or prolapse.
There is no role for endoscopic treatments in these patients. There are case series reports
of endoscopic removal of the band to address band erosion. The role of an upper gastroin-
testinal series is to diagnose a suspected slip in a hemodynamically normal setting.
This patient has slippage of a previously placed gastric band and is hemodynamically
abnormal. A diagnosis of gastric incarceration must be considered. Even though this condi-
tion is rare, this diagnosis can lead to an abdominal disaster. A CT scan of the abdomen will
note the prolapse and any signs of perforation. If the stomach is thought to be incarcerated,
then urgent surgical repair is needed. The procedure could be started laparoscopically. Na-
sogastric tube placement would not solve the problem of the incarceration.
References
I. Cunneen SA, Phillips E, Fielding G, et al. Studies of Swedish adjustable gastric band
and lap-band: systematic review and meta-analysis. Surg Obes Re/at Dis. 2008;4(2):
174-185.
2. Fischer G, Myers JA. Huang W, Shayani V. Gastric migration and strangulation after
adjustable gastric banding. Obes Surg. 2008:18(6):753-755.
3. Snow JM, Severson PA. Complications of adjustable gastric banding. Surg Clin North
Am. 2011:91(6):1249-1264.
CRITIQUE6
This patient has T3, NO, MO rectal cancer. With a T3 lesion, he is not a candidate for trans-
anal local excision, because negative margins cannot be assured with local excision only.
The addition of adjuvant chemotherapy to a local excision would not be considered the
standard of care either. because it is not known whether the adjuvant therapy could over-
come the suboptimal resection of the tumor. Per the National Comprehensive Cancer Net-
work guidelines and the German Rectal Cancer Study, the best choice is for neoadjuvant
therapy with chemotherapy and radiation therapy followed by surgery, in this case a low
anterior resection with total mesorectal excision. The advantages of preoperative chemora-
diation therapy are that it reduces the 5-year local relapse from 13% to 6% and it decreases
chemoradiation toxicity from 40% to 27%. The 5-year overall survival is the same for both
preoperative and postoperative adjuvant therapy.
Answer: (B) neoadjuvant chemoradiation followed by low anterior resection with total
mesorectal excision.
References
1. Folkesson J, Birgisson H, Pahlman L, Cedermark B. Glimelius B, Gunnarsson U. Swed-
ish Rectal Cancer Trial: long lasting benefits from radiotherapy on survival and local
recurrence rate. J Clin Oneal. 2005;23(24):5644-5650.
2. Sauer R, Becker H, Hohenberger W, et al; German Rectal Cancer Study Group. Preop-
erative versus postoperative chemoradiotherapy for rectal cancer. N Engl J Med
2004:351(17): 1731-1740.
3. National Comprehensive Cancer Network. NCCN Guidelines Rectal Cancer. Version
1.2016. Fort Washington, PA: National Comprehensive Cancer Network.
so CATEGORY 3 - PART 1
CRITIQUE7
Acute appendicitis, traditionally managed with appendectomy, is a common problem
worldwide. The rate of major complications is low, but wound infection and intra-abdominal
abscess are not uncommon. Nonsurgical management of uncomplicated appendicitis,
although not a new idea, is gaining popularity. Early studies noted medical management to
be successful in approximately 90% of cases ofuncomplicated appendicitis, with a relapse
rate of less than 15% at 1 year. Modem studies suggest a failure rate of approximately 6%,
with only 3% of treatment failures progressing to perforation or abscess. Nonoperative
treatment is associated with a longer length of stay.
The overall complication rate, however, is no different between surgical and nonsurgi-
cal groups. The risk of presentation with perforation or abscess after nonoperative manage-
ment is no different from the general population (-3%).
Answer: (D) Although the risk of treatment failure with nonoperative management is
higher than operative management, the overall complication rate is no higher.
References
I. Di Saverio S, Sibilio A, Giorgini E, et al. The NOTA Study (Non Operative Treat-
ment for Acute Appendicitis): prospective study on the efficacy and safety of antibiotics
(amoxicillin and clavulanic acid) for treating patients with right lower quadrant abdomi-
nal pain and long-term follow-up of conservatively treated suspected appendicitis. Ann
Surg. 20 l 4;260(1):109-117.
2. McCutcheon BA, Chang DC, Marcus LP, et al. Long-term outcomes of patients with
nonsurgically managed uncomplicated appendicitis. J Am Coll Surg. 2014;218(5):
905-913.
3. Salminen P, Paajanen H, Rautio T, et al. Antibiotic therapy vs appendectomy for treat-
ment of uncomplicated acute appendicitis: The AP PAC randomized clinical trial. JAMA.
2015;313(23):2340-2348.
CRITIQUE 8-12
Crohn disease is characterized by segmental (discontinuous) involvement of the entire
gastrointestinal tract~ whereas ulcerative colitis is characterized by diffuse (continuous)
inflammation beginning in the rectum and extending proximally. Crohn disease most com-
monly affects the terminal ileum and right colon. Strictures are common in Crohn disease
and uncommon in ulcerative colitis.
!leocolonoscopy is used to obtain biopsies taken from the terminal ileum and at least 5
sites along the colon and rectum. At least 2 biopsies should be taken from each site. Histo-
logically, Crohn disease is characterized by transmural inflammation) whereas ulcerative
colitis affects only the mucosa and occasionally the submucosa. Sometimes expert patholo-
gists have difficulty distinguishing Crohn colitis from ulcerative colitis histologically.
Both Crohn disease and ulcerative colitis are associated with an increased risk for
colorectal cancer, and risk increases with extent and duration of disease. Patients with
Crohn disease are also at increased risk for developing small bowel cancer.
Both Crohn disease and ulcerative colitis may be treated with corticosteroids; 5-amino-
salicylates are a mainstay for treating active ulcerative colitis but are typically not used for
Crohn disease. The effect of 5-aminosalicylates is primarily intraluminal; it can act directly
on the mucosal changes of ulcerative colitis and therefore does not affect the transmural
inflammation ofCrohn disease. Multiple other immunosuppressant medications, including
thiopurines, methotrexate, calcineurin inhibitors, and antitumor necrosis factor agents, are
also used; other biological agents are being studied.
CATEGORY 3 - PART I
81
Answers:
8. (C) Both
9. (A) Crohn disease
10. (B) Ulcerative colitis
11. (A) Crohn disease
12. (C) Both
References
1. Annese V, Dapemo M, Rutter MD, et al; European Crohn's and Colitis Organisation.
European evidence based consensus for endoscopy in inflammatory bowel disease.
J Crohns Colitis. 2013;7(12):982-1018.
2. Bernstein CN. Treatment ofIBD: where we are and where we are going. Am J Gastro-
enterol. 2015;110(1):114--126.
3. Bae SL Kim YS. Colon cancer screening and surveillance in inflammatory bowel
disease. Clin Endosc. 2014;47(6):509-515.
CRITIQUE 13-16
Advanced gastrointestinal cancers of the esophagus or rectum are difficult-to-treat tumors
associated with significant diminution of quality of life related to treatment. Combined
modality therapy in a neoadjuvant manner plays an important role. Not infrequently, com-
plete pathologic response to chemoradiotherapy can be achieved; thus, risk-to-benefit as-
sessment must be undertaken on a patient-by-patient basis before committing to radical
surgery. In the case of proximal carcinomas of the esophagus with complete response to
therapy, observation and palliative interventions as needed usually out\veigh any benefit of
radical surgery in these frequently debilitated patients.
Distal adenocarcinoma of the esophagus, however, has a high recurrence rate, despite
excellent initial responses to combined modality therapy. Evidence suggests that observa-
tion followed by salvage esophagectomy is equally efficacious as neoadjuvant chemora-
diotherapy followed by radical surgery in clinical complete responders to treatment
After any neoadjuvant therapy for either esophageal or rectal cancers, restaging should
be undertaken before considering surgical intervention or surveillance. If evidence of per-
sistent disease is found~ usually surgery is indicated if disease-free or long-term survival
is the goal of such treatment. If restaging demonstrates progression of disease or distant
disease, radical surgery is not indicated, and palliative chemotherapy or palliative care op-
tions should be explored.
Answers:
13. (B) Observation
14. (C) Both
15. (A) Surgical resection
16. (D) Neither
References
I. Cunningham D, Allum WH, Stenning SP, et al; MAGIC Trial Participants. Periopera-
tive chemotherapy versus surgery alone for resectable gastroesophageal cancer. N Engl
J Med. 2006;355(1):11-20.
2. Sudo K, Xiao L, Wadhwa R, et al. Importance of surveillance and success of salvage
strategies after definitive chemoradiation in patients with esophageal cancer. J Clin
Oncol. 2014;32(30):3400-3405.
3. Swisher SG, Winter KA, Komaki RU, et al. A Phase II study of a paclitaxel-based
chemoradiation regimen with selective surgical salvage for resectable locoregionally
advanced esophageal cancer: initial reporting of RTOG 0246. Int J Radial Oncol Biol
Phys. 20I2;82(5):1967-1972.
82 CATEGORY 3 - PART I
4. Roh MS, Colangelo LH, O'Connell MJ, et al. Preoperative multimodality therapy
improves disease-free survival in patients with carcinoma of the rectum: NSABP R-03.
JC/in Oneal. 2009;27(31):5124-5130.
CRITIQUE 17-19
Colorectal cancer is currently the fourth leading cause of cancer death worldwide. Stage
at presentation is the single most important predictor of survival, ranging from a 5-year
survival of 90% for stage 1 disease to approximately 10% for stage 4 tumors. Although
sporadic disease is by far the most common form of colorectal cancer, approximately 3-5%
of these cancers are hereditary in nature. Of these, the Lynch syndrome and familial adeno-
matous polyposis coli (FAP) are the most common forms.
Lynch syndrome is a form of hereditary nonpolyposis colorectal cancer (HNPCC) in-
volving a gennline mutation in the DNA mismatch repair system, resulting in microsatel-
lite instability. It is autosomal dominant in nature and confers a 50-60% lifetime risk of
developing colorectal cancer. Patients typically present in their 40s, and cancers develop in
adenomatous polyps rather than serrated polyps.
FAP is associated with a germline mutation in the APC gene. Like Lynch syndrome, it
is an autosomal dominant disorder; unlike Lynch syndrome, it has 100% lifetime risk of
colorectal cancer due to the development of hundreds to thousands of polyps within the
colon. De nova mutations are responsible for 25o/o of FAP presentations, and patients can
manifest polyps throughout the gastrointestinal tract as well as thyroid cancers, desmoids
tumors, and hepatoblastomas.
The CpG island methylator phenotype (CIMP) is a unique epigenetic phenotype of
colorectal cancer found in sporadic microsatellite instability tumors. It is associated with
older age, poor differentiation, proximal location, B-type Raf (BRAF) mutation, MLHl
methylation, and cancers arising in sessile serrated polyps. It is not present in Lynch syn-
drome tumors.
Answers:
17. (C) Both
18. (A) Lynch syndrome
19. (B) Familial adenomatous polyposis coli
References
1. Legolvan MP, Taliano RJ, Resnick MB. Application of molecular techniques in the
diagnosis, prognosis and management of patients with colorectal cancer: a practical
approach. Hum Pathol. 2012;43(8): 1157-1168.
2. Brenner H, Kloor M, Pox CP. Colorectal cancer. Lancet. 2014;383(9927):1490-1502.
3. Colas C, Coulet F, Svrcek M, et al. Lynch or not Lynch? ls that always a question? Adv
Cancer Res. 2012;113:121-166.
CRITIQUE 20-22
Although the mortality rates from nonvariceal upper gastrointestinal hemorrhage have de-
clined in the past decade, they are still between 10 and 14%. Initial treatment is directed at
resuscitation of any hypovolemic shock and determination of risk stratification via prog-
nostic scales with urgent therapeutic endoscopy, which is performed within 24 hours of
presentation. The use of endoscopic clips or thermocoagulation with or without epineph-
rine injection is recommended therapy; epinephrine alone for hemostatic control of non-
variceal upper gastrointestinal hemorrhage is not recommended. Second-look endoscopy
is undertaken only in selected high-risk lesions, which would include a visible vessel or
adherent clot.
CATEGORY 3 - PART I 83
Answers:
20. (A) Endoscopic clips
21. (B) High-dose intravenous proton pump inhibitors
22. (C) Somatostatin
References
1. Barkun A, Bardou M, Marshall JK; Nonvariceal Upper GI Bleeding Consensus Confer-
ence Group. Consensus recommendations for managing patients with nonvariceal upper
gastrointestinal bleeding. Ann Intern Med. 2003; 139(10):843-857.
2. Laine L, McQuaid KR. Endoscopic therapy for bleeding ulcers: an evidence-based
approach based on meta-analyses of randomized controlled trials. Clin Gastroenterol
Hepato/. 2009;7(1):33-47.
3. British Society ofGastroenterology Endoscopy Committee. Non-variceal upper gastro-
intestinal haemorrhage: guidelines. Gut. 2002;51(suppl4):ivl-iv6.
4. Greenspoon J, Barkun A. The pharmacological therapy ofnon-variceal upper gastroin-
testinal bleeding. Gastroenterol Clin North Am. 2010;39(3):419-432.
CRITIQUE 23-25
Uncomplicated appendicitis in adults continues to be managed with surgery. Laparoscopic
appendectomy is now preferred to open procedures for most cases due to its lower mor-
bidity. Several recent studies report suspected acute appendicitis may be safely managed
with antibiotics alone, with a less than 14% recurrence rate; this nonoperative approach for
uncomplicated appendicitis, however, remains controversial.
Management of complicated appendicitis can be a more vexing problem. In patients
with a clearly defined abscess from perforated appendicitis, placement of a percutaneous
drain is the optitnal choice and usually requires a short course of antibiotics. For patients
with an inflammatory phlegmon, treatment with intravenous, not oral, antibiotics has a
13.5% morbidity rate compared with a 35.6% morbidity rate for immediate surgery. Con-
servative management in this patient population is associated with fewer wound infections,
intra-abdominal abscesses, and ileus/bowel obstructions compared with patients undergo-
ing appendectomy. Patients with complicated appendicitis managed in this manner have a
reported failure rate of approximately 7o/o. Factors associated with failure of nonoperative
management include tobacco use, tachycardia at presentation, generalized abdominal ten-
derness, and abscesses smaller than 5 cm. These initial findings have yet to be validated in
prospective studies.
For patients managed non operatively, a discussion of interval appendectomy typically oc-
curs at the follow-up clinic visit. Although once considered routine, the complication rate of
an interval appendectomy (2~23%) must be weighed against the risk of recurrent appendicitis
(<I 0%). Risk of recurrence may be higher when an appendicolith is present. If appendec-
tomy is not performed, colorectal cancer or other disease must be excluded, particularly in
patients over the age of 40. Colonoscopy or barium enema is recommended in these patients.
84 CATEGORY 3 - PART l
Answers:
23. (B) Laparoscopic appendectomy
24. (D) Percutaneous drain placement
25. (C) Intravenous antibiotics
References
1. Maxfield MW, Schuster KM, Bokhari J, McGillicuddy EA, Davis KA. Predictive
factors for failure of nonoperative management in perforated appendicitis. J Trauma
Acute Care Surg. 2014;76(4):976-981.
2. Andersson RE, Petzold MG. Nonsurgical treatment of appendiceal abscess or phleg-
mon: a systematic review and meta-analysis. Ann Surg. 2007;246(5):741-748.
3. Simillis C, Symeonides P, ShorthouseAJ, Tekkis PP. A meta-analysis comparing conser-
vative treatment versus acute appendectomy for complicated appendicitis (abscess or
phlegmon). Surgery. 2010;147(6):818-829.
4. Ingraham AM, Cohen ME, Bilimoria KY, Pritts TA, Ko CY, Esposito TJ. Comparison
of outcomes after laparoscopic versus open appendectomy for acute appendicitis at 222
ACS NSQIP hospitals. Surgery. 201O;148(4):625-635; discussion 635-63 7.
5. Tiwari MM, Reynoso JF, Tsang AW, Oleynikov D. Comparison of outcomes of lapa-
roscopic and open appendectomy in management of uncomplicated and complicated
appendicitis. Ann Surg. 2011 ;254(6):927-932.
6. McCutcheon BA, Chang DC, Marcus LP, et al. Long-term outcomes of patients with
nonsurgically managed uncomplicated appendicitis. J Am Coll Surg. 2014;218(5):
905-913.
7. Di Saverio S, Sibilio A, Giorgini E, et al. The NOTA Study (Non Operative Treat-
ment for Acute Appendicitis): prospective study on the efficacy and safety ofantibiotics
(amoxicillin and clavulanic acid) for treating patients with right lower quadrant abdomi-
nal pain and long-term follow-up of conservatively treated suspected appendicitis. Ann
Surg. 2014;260(1):109-117.
8. Salminen P, Paajanen H, Rautio T, et al. Antibiotic therapy vs appendectomy for treat-
ment of uncomplicated acute appendicitis: the AP PAC randomized clinical trial. JAMA.
2015;313(23):2340-2348.
CRITIQUE 26-28
Dieulafoy lesions are vascular malformations along the lesser curvature of the stomach~
typically within 6 cm of the gastroesophageal junction. These lesions result from erosion
of gastric mucosa overlying submucosal vessels found in this area, often leading to life-
threatening arterial hemorrhage. The mucosal defect overlying the vessel is often small and
difficult to identify unless actively bleeding, resulting in the mistakenly normal appearance
of the mucosa on esophagogastroduodenoscopy.
Peptic ulcer disease is the most common cause of upper gastrointestinal bleeding, ac-
counting for 40-47% of all cases. Approximately 10-15% of patients with peptic ulcer
disease will go on to develop bleeding. Bleeding remains the most frequent indication for
intervention and the primary cause for death in patients affected by this disease process.
Watermelon stomach, also known as gastric antral vascular ectasia, is best described as
a series of dilated vessels appearing as longitudinal linear red streaks on the antrum mucosa
with the appearance of a watermelon (figure 26-28.1). Acute severe hemorrhage is rare,
and most patients present with persistent iron deficiency anemia from continued occult
blood loss. Endoscopic therapy is indicated for persistent, transfusion-dependent bleeding
and is successful in up to 90% of patients. Patients failing endoscopic therapy should be
considered for antrectomy.
CATEGORY 3 - PART I
85
Gastric varices are found in the cardia of the stomach associated with portal hyperten-
sion. Aortoenteric fistulas are characterized by a distal duodenal bleeding site, which is an
ulceration through the mucosa from the aortic graft in patients who have had prior place-
ment of an aortic graft.
Answers:
26. (C) Dieulafoy lesion
27. (B) Peptic ulcer
28. (A) Gastric antral vacular ectasia
References
1. Munoz-Navas M, Ibanez MB, Femandez-Urien Sainz I. Chronic gastrointestinal bleed-
ing. In: Ginsberg GG, Kochrnan ML, Norton ID, Gostout CJ, eds. Clinical Gastrointes-
inal Endoscopy. 2nd ed. St. Louis, MO: Elsevier Saunders:20 12:213-238.
2. Rockall TA, Logan RF, Devlin HB, Northfield TC; Steering Committee and members
of the National Audit of Acute Upper Gastrointestinal Haemorrhage. Incidence of and
mortality from acute upper gastrointestinal haemorrhage in the United Kingdom. BMJ.
I 995;31 1(6999):222- 226.
3. Squires RA, Postier RG. Acute abdomen. In: Townsend CM, Beauchamp RD, Evers
BM, Mattox KL, eds. Sabiston Textbook of Surgery: The Biological Basis of Modern
Surgical Practice. 19th ed. Philadelphia, PA: Elsevier Saunders;2012: 1141 - 1160.
4. Tavak.kolizadeh A, Ashley SW. Acute gastrointestinal hemorrhage. In: Townsend
CM, Beauchamp RD, Evers BM, Mattox KL, eds. Sabiston Textbook of Surgery: The
Biological Basis ofModern Surgical Practice. 19th ed. Philadelphia, PA: Elsevier Saun-
ders;2012: 11 60-1 181.
86 CATEGORY 3 - PART I
CRITIQUE 29-32
Stoma Discussion
Given that 40-50% of"temporary" stomas are never reversed, care and attention to their
construction is important to the patient's quality of life. In addition to the disease process
necessitating stoma creation, age, body habitus, sex, and use of steroids are all factors that
influence the relatively high risk of developing postoperative stomal complications.
The most common error in stoma creation is poor siting. Preoperative assessment should
identify a site away from previous or planned incisions, skin creases, and bony prominenc-
es, ideally with 5 cm of flat skin around the site for an appliance to fit without leak. The
patient must be able to see it to care for it while standing or sitting, which can be a problem
in an obese patient or patients with a spinal cord injury. Clothing should not be irritating to
the stoma, which is avoided ifthe stoma is placed below or above a belt line. Assessment
with the patient standing and sitting and clothes on and off allows for an acceptable site to
be chosen and marked. Although management of stomal complications is often relegated to
a stomal therapist, he or she may also be helpful in the preoperative education of patients
undergoing elective surgery.
At operation, attention to maturation technical details decreases later complications. An
opening that accommodates bowel and mesentery without tension is required. Avoid ori-
enting the stoma with bowel mesentery at the inferior aspect to prevent leakage when the
patient is standing. A Brooke maturation with eversion of the bowel to a height of at least a
centimeter will also decrease later irritant contact dermatitis due to stool pH and chemical
exposure. The stoma pictured is an excellent example of a " good" stoma (figure 26-32.5).
Stoma Complications
Stoma l. This stoma shows evidence of granulation tissue at skin level surrounding
an ileostomy. This local inflammatory response is a common problem with an ill-fitting
stoma appliance and can be addressed with local wound care. Inexperienced caregivers
often create progressively larger wafer openings in an attempt to alleviate the irritation.
This approach perpetuates the skin exposure, worsening the problem. Silver nitrate may
be applied to the granulation tissue. Application of zinc oxide will aid skin repair while
providing a barrier to the associated skin irritation and may require a change in appliance
(figure 26-32.6).
CATEGORY 3 - PA RT I 87
Stoma 2. Tbis stoma is prolapsed. In effect, it is a late intussusception and is best man-
aged with stoma revision. Although it may be manually reduced, prolapse will recur and
may interfere with adherence of the appliance. Where bowel edema and engorgement in-
hibit reduction, application of table sugar or hyaluronidase injection reduces edema. Al-
though dramatic and disconcerting, prolapse is rarely a surgical emergency. Revision can
often be accomplished with a local procedure, separation of the bowel margin from the
skin. resection of the redundant bowel and rematuration. Intra-abdominal reduction may
be necessary but is not usually required (figure 26-32.7).
Stoma 3. Vascular compromise of a new stoma may be localized to the mucosa or ex-
tend below the level of the fascia. This stoma shows venous congestion without overt ne-
crosis. The size of the abdominal opening, mobilization of the bowel, and orientation of the
mesentery are contributing factors that may be addressed at the time of the original opera-
tion. Excessive trimming of the epiplo ic fat and mesentery should be avoided. Because the
upper abdominal wall has less subcutaneous fat, higher placement of the stoma in an obese
patient may offer a technical advantage. Transillumination of the mucosa will distinguish
postoperative congestion from progressive ischemia and underlying necrosis. Examina-
tion of the deeper stoma can be accomplished by placing a clear test tube in the lumen and
looking at the mucosa! v iability down to the fascia or beyond. A lthough superficial muco-
sa! sloughing may be observed, deeper injury requires reoperation. Even relatively minor
ischemia in a stoma may result in poor stomal function in the future (figure 26-32.8).
Stoma 4. Pyoderma gangrenosurn may present as ulcerated skin in association with in-
flammatory bowel disease, arthritis, multiple rnyeloma, and malignancy. The skin lesions
start as small pustules and progress to ulcers with necrotic edges, as shown here. Patients are
frequently treated with topical steroids or antibiotics. The addition of systemic steroid has
some utility. Clinical trials of infliximab have documented variable responses, ranging from
a 21 % complete response to a 31% nonresponse. In the case of inflammatory bowel d isease,
removal of the involved bowel does speed skin healing time, but recurrence with re-siting of
the stoma also occurs. Closure of the stoma is encouraged, if possible (fig ure 26-32.9).
Answers:
29. (B) Local wound care
30. (E) Local stoma revision
31. (A) Observation
32. (C) Infliximab
CATEGORY 3 - PART I
89
References
1. Bafford AC, Irani JL. Management and complications of stomas. Surg Clin North Am.
2013;93(1): 145-166.
2. Kwiatt M, Kawata M. Avoidance and management of stomal complications. Clin Colon
Rectal Surg. 201 3;26(2): 112-121.
3. Wu XR, Shen B. Diagnosis and management of parastomal pyoderma gangrenosum.
Gastroenterol Rep (Oxj). 2013;1( 1):1-8.
CRITIQUE 33-35
The word achalasia means "failure to relax'" and refers to any sphincter that remains in
a constant state of tone with periods of relaxation. Achalasia is the best understood of
all esophageal motility disorders. Pathogenesis is believed to be idiopathic or infectious
neurogenic degeneration. Associated factors include emotional stress, trauma, significant
weight loss. and Chagas disease (parasitic infection with Trypanosoma cruzi). The muscle
of the esophagus and lower esophageal sphincter (LES) are affected. A prevailing theory is
that destruction of the nerves to the LES is the primary pathology and that degeneration of
the neuromuscular function of the body of the esophagus is secondary. Hypertension of the
LES and failure of the LES to relax result in pressurization of the esophagus, esophageal
dilatation, and loss of progressive peristalsis.
Pharmacological management of esophageal achalasia plays a minor role. Calcium
channel blockers can reduce LES pressure and relieve dysphagia, but the results are usually
unsatisfactory and incomplete. Side effects include headache, dizziness, and pedal edema.
Botulinum toxin is derived from Clostridium botulinum and causes paralysis of volun-
tary and involuntary muscles. Local injection into the LES lowers sphincter tone, which
results in symptom relief. The main limitation of this therapy is short duration of effect.
which lasts only 6-9 months. Currently, recommendations are for use in elderly patients
and those who are high-risk for invasive procedures due to comorbid disease.
Pneumatic dilatation entails forceful bougie dilatation, resulting in mechanical tear of the
muscle fibers of the LES. This is the first choice for nonsurgical treatment of achalasia.
Repeated dilation based on symptom recurrence can achieve long-term remission. Two or
more dilatations are required in 20-75% of patients. Complications are uncommon; the most
90 CATEGORY 3 - PART 1
severe is perforation, which occurs in 1-2% of patients. Reflu.'X symptoms after pneumatic
dilatation are usually minor, transient, and easily controlled with proton pump inhibitors.
Laparoscopic Heller myotomy is the most durable treatment for achalasia at the ex-
pense of increased reflux-associated complications. Both longitudinal and circular muscle
fibers of the lower esophagus and gastric cardia are divided above and below the sphincter
mechanism. Currently, most surgeons perform this procedure with a variety offundoplica-
tion procedures in the majority of patients.
Peroral endoscopic myotomy (POEM) is performed by dissection and division of the in-
ner circular muscle layer of the esophagus through a submucosal tunnel created endoscopi-
cally via a small proximal opening of the esophageal mucosa. The longitudinal muscle
fibers are spared. In addition, POEM does not disrupt the suspensory ligaments of the
esophagus, which may contribute to maintenance of anatomic reflux barriers. Compared
with surgical myotomy, POEM may result in a longer myotomy, if desired. Initial data in-
dicate that POEM is comparable with laparoscopic Heller myotomy for safe and effective
treatment of achalasia, but long-term efficacy and safety will require ongoing assessment.
Answers:
33. (D) Peroral endoscopic myotomy (POEM)
34. (C) Pneumatic dilatation
35. (E) Laparoscopic Heller myotomy
References
1. Chuah SK, Chiu CH, Tai WC, et al. Current status in the treatment options for esopha-
geal achalasia. World J Gastroenterol. 2013; 19(33):5421-5429.
2. Bhayani NH, Kurian AA, Dunst CM, Sharata AM, Rieder E, Swanstrom LL. A compara-
tive study on comprehensive, objective outcomes of laparoscopic Heller myotomy with
per-oral endoscopic myotomy (POEM) for achalasia. Ann Surg. 20l4;259(6):1098-1103.
CRITIQUE36
In the United States, pancreatic ductal adenocarcinoma is the fourth leading cause of can-
cer death. Risk factors include African American descent, male sex, obesity, and smoking.
Surgical resection remains the only curative option, with 5-year survival rates approaching
25%. Preoperative determination ofresectability, especially for lesions involving the head
of the pancreas, is essential. This assessment is done via a 3-phase pancreatic protocol CT
scan to assess the extent of disease, vascular involvement, and resectability, as well as to
determine the stage of the disease. Metastasis is most commonly present in the liver.
In the absence of metastatic disease, the mesenteric vessels (i.e., the portal vein [PV],
the superior mesenteric vein [SMV], the superior mesenteric artery [SMA], the hepatic
artery [HA], and the celiac artery [CA]) are evaluated for tumor abutment or invasion.
Vascular involvement does not necessarily preclude surgical resection but may alter op-
erative planning. Unresectable disease is defined as tumor abutting the CA/trunk, HA, or
SMA by more than 180° or encasement/occlusion of the SMV/PV. Borderline resectability
is defined as tumor causing venous distortion of the SMV/PV axis including short segment
venous occlusion with sufficient proximal and distal vessel length to allow for safe recon-
struction, encasement of the gastroduodenal arte:ry up to the HA with either short segment
or direct abutment of the HA without CA involvement, or less than 180° abutment of the
SMA. Neoadjuvant chemotherapy is often used to improve resectability.
References
I. Edi! BH, McCarter M, Gajdos C, Schulick RD. Periampullary carcinoma. In: Cameron
JL, Cameron AM, eds. Current Surgical Therapy. 11th ed. Philadelphia, PA: Elsevier
Saunders: 2014:471-476.
2. Bockhorn M, Uzunoglu FG, Adham M, et al; International Study Group of Pancreatic
Surgery. Borderline resectable pancreatic cancer: a consensus statement by the Interna-
tional Study Group orPancreatic Surgery (ISGPS). Surgery. 2014;155(6):977-988.
3. Wesson RN, Cameron AM. Vascular reconstruction during the Whipple operation. In:
Cameron JL, Cameron AM, eds. Current Surgical Therapy. 11th ed. Philadelphia, PA:
Elsevier Saunders: 2014:477-480.
CRITIQUE37
The reported incidence of cystic lesions of the pancreas ranges from 2.6o/o in asymptomatic
adults, up to 8% in patients over the age of 80, and up to nearly 25% in autopsy series.
The increased use of cross-sectional radiographic imaging is suspected to be the reason
for the increased identification of these lesions; before 1989, only 3% of pancreatic resec-
tions were performed for cystic neoplasms, whereas today the number approaches 50%.
Our knowledge of the natural history of these lesions is in evolution, and there is no evi-
dence from randomized trials to provide evidenced-based guidelines for their management.
Hence, current recommendations are from expert consensus panels, the first in 2006 known
as the Sendai consensus guidelines and more recently updated in 2012 (Fukuoka criteria).
Changes in the Fukuoka criteria were designed to improve the sensitivity of determining
the malignant potential of these lesions before invasion, allowing earlier surgical inter-
vention in surgically fit patients, with the goal of improving survival. The changes to the
criteria involve a reduction in the size of the main pancreatic duct from greater than 10 mm
to greater than 5 mm for a diagnosis of main duct intraductal papillary mucinous neoplasm
(IPMN) and the addition of"high-risk stigmata of malignancy" and ·'worrisome features."
High-risk stigmata of malignancy include clinical jaundice, duct size of at least 10 mm
and enhanced solid components. Worrisome features include duct size 5-9 mm; cyst size
greater than 3 cm; thickened, enhanced cyst walls; nonenhanced mural nodules; and rapid
enlargement of duct width coupled with distal atrophy. Patients meeting these criteria who
are surgically fit should be offered an oncologic pancreatic resection. Surgical resection
with clear margins for mucinous pancreatic neoplasms is considered curative, and lymph
node metastasis are so rare that regional lymphadenectomy is not necessary. Despite surgi-
cal margins for IPMN that show no malignancy or dysplasia, recurrence is reported in more
than 10%, so life-long surveillance is necessary.
MRI/magnetic resonance cholangiopancreatography is the preferred imaging modality.
Better imaging is obtained with this modality, which allows the determination of com-
munication of these lesions with the main pancreatic duct, identification of mural nod-
ules, and protection from radiation exposure. It is also the preferred method for surveil-
lance. Endoscopic ultrasound and fine needle aspiration are recommended for the initial
workup for both mucinous cystic pancreatic neoplasms (MCP) and IPMN, particularly
those with worrisome features. This allows obtaining fluid for the determination of mucin
levels, chemical analysis (amylase), tumor markers (CEA), cytology, and, more recently,
genetic analysis (KRAS) for excluding serous cystadenomas. These studies are essential in
differentiating between these cystic lesions and for planning treatment. The general char-
acteristics of these neoplasms are presented in table 37.1. IPMN are of3 types: main duct
(MD), branch duct (BD), and mixed, which contains characteristic of both. Mixed type-
IPMN behave like MD-IPMN and are grouped together. Additional epithelial subtyping
(i.e., oncocytic, gastric, and pancreaticobiliary) is linked with prognosis. Survival rates at
5 years are 75% for intestinal type (colloidal) compared with 20% for pancreaticobiliary
type (tubular), respectively.
92 CATEGORY 3 - PART 1
Answer: (B) Main-duct IPMN have greater malignant potential than do the branch
duct type.
References
I. Goh BK, Tan DM, Thng CH, et al. Are the Sendai and Fukuoka consensus guidelines
for cystic mucinous neoplasms of the pancreas useful in the initial triage of all suspected
pancreatic cystic neoplasms? A single-institution experience with 317 surgically treated
patients. Ann Surg Oneal. 2014;21(6):1919-1926.
CATEGORY 3 - PART I 93
CRITIQUE38
Surveillance after polypectomy should be based on risk stratification. Surveillance should
be directed to those most likely to benefit. Those who are less likely to benefit, but who
would be placed at risk for complications from removal of small polyps, should have a
reduced surveillance intensity.
If a sessile polyp is removed piecemeal, the patient should be followed at 2-6 month
intervals to document complete removal and no regrowth of the polyp. In such cases, the
polypectomy site should be tattooed to allow proper identification of the site at endoscopic
follow-up. Once complete removal is established, subsequent surveillance is individual-
ized based on the physician's judgment. Both endoscopic and pathologic assessments are
used to assess completeness of removal.
Other findings in patients that would benefit from a short follow-up, within 6 months,
include numerous adenomas, a malignant adenoma (with invasive cancer), a large sessile
adenoma, or an incomplete colonoscopy. Actual time for follow-up should again be based
on the physician's clinical judgment.
References
I. Winawer SJ, Zauber AG, Fletcher RH, et al. Guidelines for colonoscopy surveillance
after polypectomy: a consensus update by the US Multi-Society Task Force on Colorec-
tal Cancer and the American Cancer Society. CA Cancer J Cl in. 2006;56(3): 143-159.
2. Winawer S, Fletcher R, Rex D, et al; Gastrointestinal Consortium Panel. Colorectal
cancer screening and surveillance: clinical guidelines and rationale-Update based on
new evidence. Gastroenterology. 2003;124(2):544-560.
3. Robertson DJ, Greenberg ER, Beach M, et al. Colorectal cancer in patients under close
colonoscopic surveillance. Gastroenterology. 2005; 129(1 ):34-41.
94 CATEGORY 3 - PART l
CRITIQUE39
This young man is presenting with stage IV adenocarcinoma of the pancreas and signifi-
cant nutritional disability. He is not a candidate for curative therapy. Establishing his goals
of care and options for optimal palliation are paramount. The immediate opportunities
for palliation include eliminating the pruritus and minimizing his pain. Endoscopic bili-
ary stents are the standard and preferred method for obtaining biliary decompression and,
therefore, palliation of pruritus. Although some controversy surrounds whether plastic or
metal stents should be deployed initially, it is clear that endoscopic approaches are effec-
tive in more than 95% of patients and minimize overall morbidity and cost. Percutaneous
transhepatic catheters will relieve jaundice and pruritus at the expense of a chronic external
tube that requires routine maintenance (e.g., flushes and dressing changes).
Future gastric outlet obstruction is a concern but will occur in fewer than 1Oo/o of all
patients and frequently as a premorbid finding. The risk is a function of the size and loca-
tion of the primary tumor as well as the patient's life expectancy. In an asymptomatic indi-
vidual, prophylactic surgical gastric bypass is not effective and is indicated only for those
who are not candidates for endoscopic duodenal stenting. Duodenal stents are associated
with a 90% success rate in patients with pancreatic cancer and should be placed with the
onset of symptoms.
Some authors have written about palliative Whipple operations (i.e., performing the op-
eration outside of curative intent). Such an approach is clearly contraindicated in this case
and should be considered only in rare and extraordinary circumstances when less morbid
solutions are not available.
References
1. Tonozuka R, Itoi T, Sofuni A, Itokawa F, Moriyasu F. Endoscopic double stenting for
the treatment of malignant biliary and duodenal obstruction due to pancreatic cancer.
Dig Endosc. 2013;25 (suppl 2):100-108.
2. ASGE Standards of Practice Committee, Fukami N, Anderson MA, et al. The role
of endoscopy in gastroduodenal obstruction and gastroparesis. Gastrointest Endosc.
2011;74(1):13-21.
3. Boulay BR, Parepally M. Managing malignant biliary obstruction in pancreas cancer:
choosing the appropriate strategy. World J Gastroenterol. 2014;20(28):9345-9353.
4. Piesman M, Kozarek RA, Brandabur JJ, et al. Improved oral intake after palliative
duodenal stenting for malignant obstruction: a prospective multicenter clinical trial. Am
J Gastroenterol. 2009; 104( 10):2404--2411.
CRITIQUE40
Aberrant hepatic arteries can be of major surgical significance in operations of the upper
intestinal tract, the gallbladder, and pancreas; therefore, knowledge of the likely variations
is critical to performing safe surgery. In 1966, Michels used data from an autopsy series of
200 dissections to propose an internationally recognized classification of hepatic arterial
anatomy. The most commonly observed distribution was classified as type I and is consid-
ered ''normal" with a common hepatic artery arising from the celiac trunk becoming the
proper hepatic artery anatomy after giving off the gastroduodenal artery. A right and left
hepatic artery then branch from the proper hepatic. Variants of this pattern occur in up to
49% of cases.
Recent series used arteriogram images and confirmed the normal type as dominant, with
more frequent variations as follows. The common hepatic artery is consistently found to
arise from the superior mesenteric artery (SMA) and not the celiac trunk in 7-9% of cases.
A replaced or accessory right hepatic artery (RHA) arising from the SMA is consistently
CATEGORY 3- PART I
95
present in I 0-12% of cases. The aberrant RHA, which typically arises from the SMA, runs
upward through the pancreatic head and posterior to the portal vein into the hepatic pedicle.
Because of its location, a replaced RHA is at risk of injury during pancreaticoduodenec-
tomy. This is an important surgical feature, because the right hepatic artery is the origin
of the cystic artery and the dominant arterial supply to the biliary tree, and the loss of this
inflow often results in biliary strictures.
The left gastric artery is derived as a branch off the left hepatic artery in 4-5% of cases,
which can be important information if the left gastric artery is divided during gastrectomy.
Other variants of the celiac trunk, double hepatic arteries branching at the celiac trunk, or
hepatic arteries arising directly from the aorta, occur in 4.1 o/o of cases.
Answer: (C) Right hepatic artery arising from the superior 1nesenteric artery
References
L Koops A, Wojciechowski B, Broering DC, Adam G, Krupski-Berdien G. Anatomic
variations of the hepatic arteries in 604 selective celiac and superior mesenteric angiog-
raphies. Surg Radio/ Anal. 2004;26(3):239-244.
2. Hiatt JR, Gabbay J, Busuttil RW. Surgical anatomy of the hepatic arteries in I 000 cases.
Ann Surg. !994;220(1):50-52.
CRITIQUE41
Pancreatic neuroendocrine tumors (PNET) are rising in incidence and prevalence. Seventy
percent of malignant PNET are nonfunctional, without any accompanying syndrome of
hormonal hypersecretion, and their presentation is usually with vague abdominal symp-
toms from the mass effect of the tumor. Many are identified on abdominal CT studies as
incidental hyperintense masses in the pancreas.
Circulating biomarkers lack sufficient diagnostic accuracy~ when present, a biopsy is
still required. Chromogranin A (CgA) is considered the best blood biomarker. An elevated
CgA has a reported sensitivity of67.9% for PNET and a specificity of85.7%.
The grade and stage of the PNET determine patient prognosis. Determining tumor grade
is a diagnostic necessity. The number of mitoses per high-powered fields or the Ki-67
proliferation score can each be used to determine the grade as I (low), 2 (moderate), or 3
(high). High-grade (grade 3) tumors have a mitotic count greater than 20/10 high-powered
fields or a Ki-67 proliferation index greater than 20%. In the SEER database, low-grade
tumors (grades 1-2) yielded a median survival of223 months for patients with localized
disease and high-grade tumors yielded a survival pf only 34 months.
The primary therapy for a resectable PNET is surgery. Enucleation is appropriate for
small PNET of low grade and minimal metastatic potential. Larger lesions require distal
pancreatectomy or pancreaticoduodenectomy, depending on their location. Everolimus,
streptozocin, and sunitinib are used for metastatic PNET. Everolimus is a small molecule
inhibitor ofmTOR approved for use based on studies demonstrating improved outcomes.
References
L Halperin DM, Kulke MH, Yao JC. A tale of two tumors: treating pancreatic and extra-
pancreatic neuroendocrine tumors. Annu Rev Med. 2015;66: 1-16.
2. Kulke MH, Benson AB 3rd, Bergsland E, et al; National Comprehensive Cancer
Networks. Neuroendocrine tumors. J Natl Compr Cane Netw. 2012;10(6):724-764.
3. Yao JC, Hassan M, Phan A, et al. One hundred years after ·'carcinoid": epidemiology of
and prognostic factors for neuroendocrine tumors in 35,825 cases in the United States.
JC/in Oncol. 2008;26(18):3063-3072.
96 CATEGORY 3 - PART I
4. Kulke MH, Anthony LB, Bushnell DL, et al; North American Neuroendocrine Tumor
Society (NANETS). NANETS treatment guidelines: well-differentiated neuroendo-
crine tumors of the stomach and pancreas. Pancreas. 2010;39(6):735-752.
CRITIQUE42
After resection for a rectal cancer, follow-up involves periodic examination of the rectum
every 3~6 months for the first 2 or 3 years after low anterior resection. These examinations
are done by rigid or flexible proctoscopy or endoscopic ultrasound. Colonoscopy is done
every year for the first 2-3 years to detect metachronous lesions. Recurrences after colon
cancer resection are 2-4o/o, but local recurrence rates for rectal cancer are noted to be 10
or more times higher. Surgical technique is the most common reason given to explain this
difference.
Total mesorectal excision (TME) technique was introduced to reduce this high recur-
rence rate. TME involves sharp dissection of the rectum and its surrounding adventitia in
the mesorectal fascia, and the technique is associated with local recurrence rate of 10%
or less. TME is also reported to have higher rates of successful low anterior resection
and lower rates of postoperative sexual dysfunction in men. Preoperative (neoadjuvant)
chemoradiation therapy has also reduced local recurrence rates in rectal cancer.
The rectal examinations are in addition to the colonoscopic examinations, which are
aimed at detecting metachronous lesions in the colon. Unfortunately, none of the rectal
examination approaches appear to improve survival.
Fecal occult blood testing is discouraged in patients undergoing colonoscopic surveil-
lance. CT colonography (virtual colonoscopy) is not established as a surveillance modality.
Answer: (B) examination of the rectum every 3--{) months and colonoscopy at I year.
References
I. Rex DK, Kahi CJ, Levin B, et al. Guidelines for colonoscopy surveillance after cancer
resection: a consensus update by the American Cancer Society and US Multi-Society
Task Force on Colorectal Cancer. CA Cancer J Cl in. 2006;56(3): 160-167.
2. Figueredo A, Rumble RB, Maroun J, et al; Gastrointestinal Cancer Disease Site Group
of Cancer Care Ontario's Program in Evidence-Based Care. Follow-up of patients with
curatively resected colorectal cancer: a practice guideline. BMC Cancer. 2003;3:26.
CRITIQUE43
Gastrointestinal stromal tumor (GISTs) make up less than I% ofall gastrointestinal tumors
but are the most common mesenchymal tumor in the gastrointestinal tract. Although tyro-
sine kinase inhibitors (TKI), such as imatinib, have revolutionized the treatment of GIST,
surgery remains the therapy of choice for resectable tumors without evidence of metastasis.
TKis are used for nonresectable and metastatic tumors.
Risk stratification of primary GISTs is done by mitotic index, tumor size, and tumor
location. Tumors with less than 5/50 mitoses per high-powered field and less than 5 cm in
size are considered to have a ·•tow'' chance of recurrence (less than 1Oo/o ). Tumors com-
pletely resected that are 5 cm or less are associated with a disease-free survival of 95%.
Patients with completely resected, low-risk primary GISTs do not require adjunctive TKls.
Tumor location has an effect on risk of recurrence: gastric GISTs have lower risk than
duodenal, rectal, or jejunal/ileal GISTs. Mutational analysis ofunresectable GIST tumors
may help to exclude nonsensitive mutations before imatinib cytoreduction therapy or to de-
termine whether a tumor harbors a KIT exon 9 mutation, which may require an increase in
initial imatinib dosing, but is not indicated in this patient who does not require TKI therapy.
Most GISTs recur intra-abdominally.
CATEGORY 3 ~PART I 97
References
1. Demetri GD, von Mehren M, Antonescu CR, et al. NCCN Task Force report: update on
the management of patients with gastrointestinal stromal tumors. J Natl Compr Cane
Nern: 2010;8 (suppl 2):Sl-S41.
2. Judson I. Demetri G. Advances in the treatment of gastrointestinal stromal tumours. Ann
Oneal. 2007;18 (suppl 10):x20-x24. -
CRITIQUE44
Cystic lesions of the pancreas can be classified as infectious cysts, cystic variants of solid
tumors (cystic neuroendocrine tumors), nonneoplastic cysts (pseudocysts, cysts associated
with syndromes), and neoplastic cysts. Neoplastic cysts can be nonmucinous (primary se-
rous cystic neoplasm, solid pseudopapillary neoplasm) or mucinous (intraductal papillary
mucinous neoplasm [IPMN] or mucinous cystic neoplasm).
This patient is presenting with imaging findings consistent with IPMN. Most IPMN
are diagnosed between 60 and 70 years of age. Although they can be seen anywhere in the
pancreas, they are most frequently seen in the head of the gland.
Evaluation of an IPMN requires identification of the duct involved, evaluation for high-
risk stigmata of malignancy, and evaluation for worrisome features if high-risk stigmata
are absent. IPMN can involve the main duct or a branch duct, and treatment decisions are
based on making this distinction. High-risk stigmata include obstructive jaundice, enhanc-
ing solid component within the cyst, and a main pancreatic duct that is 10 mm or greater in
size. Worrisome features include cysts larger than 3 cm, thickened or enhancing cyst wall,
main duct 5-9 mm, and nonenhancing mural nodule.
Main duct IPMN (MD-IPMN) has a high likelihood of malignancy, with approximately
60% being malignant at the time of resection. For this reason, the current recommendation
is for all MD-IPMN to undergo resection regardless of other features. However, recent data
suggest that MD-IPMN with no worrisome features except for a main duct size of5-9 mm
can be safely observed with CT or MRI.
Branch duct IPMN (BD-IPMN) has a lower likelihood of malignancy, averaging 25%
in studies. For this reason, further characterization allows for identification of lesions that
can be safely observed. If high-risk stigmata are identified, surgery should be considered.
If high-risk stigmata are absent, evaluation for worrisome features should be performed. If
any of these are present, endoscopic ultrasound should be performed. If direct duct involve-
ment or mural nodule is seen, or if cytology reveals malignant cells, resection is indicated.
If none of these features is seen, the cyst can be safely observed. For cysts 2 cm or smaller,
yearly CT scan or MRI would be used to follow lesion size and characteristics. Lesions
larger than 2 cm require imaging alternating with endoscopic ultrasound every 3--6 months.
References
1. Tanaka M. Fernandez-de! Castillo C, Adsay V, et al; International Association of
Pancreatology. International consensus guidelines 2012 for the management of IPMN
and MCN of the pancreas. Pancreatology. 2012;12(3):183-197.
2. Roch AM, DeWitt JM, Al-Haddad MA, et al. Nonoperative management of main
pancreatic duct-involved intraductal papillary mucinous neoplasm might be indicated
in select patients. J Am Coll Surg. 2014;2l9(1):122-129.
3. Roch AM, Ceppa EP, Al-Haddad MA. et al. The natural history of main duct-involved,
mixed-type intraductal papillary mucinous neoplasm: parameters predictive of progres-
sion. Ann Surg. 2014;260(4):680-688; discussion 688--{)90.
CRITIQUE45
A patient with a rectal foreign body must always be assessed for a colonic perforation.
Plain abdominal x-rays are helpful and often show the position of the foreign body, espe-
cially if it is above or below the rectosigrnoid junction. If perforation has occurred, im-
mediate laparotomy is indicated. If there are no signs of perforation, an ordered, stepwise
approach is used to remove the foreign object.
Digital removal of the object should be attempted first. This may need to be attempted
with the patient in different positions. Bimanual manipulation is the next step and if still
unsuccessful, endoscopy with attempts at grasping the foreign body is tried. Placing cathe-
ters above the foreign object to help break the vacuum, which will often release the foreign
object, is often tried at this point.
If the object is still stuck, full relaxation of the anal sphincter muscles should be done.
This can be accomplished by local, spinal, or general anesthesia. Local injections might
help with the previous attempts, but at this time usually spinal or general anesthesia is used
in the operating room. Bimanual manipulation under anesthesia can be successful at times.
Consenting patients for laparotomy before examination under anesthesia is reasonable,
because noninvasive techniques may ultimately prove unsuccessful.
If conservative treatment fails, laparoscopic or open approaches are indicated. An at-
tempt to maneuver the foreign body into the rectum should be made. Colotomy is a last
resort. Sigmoidoscopy is indicated after successful removal to exclude perforation. The
likelihood of operative intervention is associated with objects that have been present for
more than 2 days~ objects longer than 10 cm, and objects proximal to the rectum.
References
1. Koomstra JJ, Weersma RK. Management of rectal foreign bodies: description ofa new
technique and clinical practice guidelines. World J Gastroenterol. 2008;14(27):4403-
4406.
2. Lake JP, Essani R, Petrone P, Kaiser AM, Asensio J, Beart RW Jr. Management of
retained colorectal foreign bodies: predictors of operative intervention. Dis Colon
Rectum. 2004;47(10):1694--1698.
Category 3 - Part II
Alimentary Tract
Items 1-40
DIRECTIONS: Each of the questions or incomplete statements is followed by 5 suggested
answers or completions. Select the answer that is the BEST in each case and fill in the
space containing the corresponding letter on the answer sheet.
Figure 1.1
2. A 42-year-old woman presents with right upper and epigastric quadrant pain that ra-
diates to her back and is associated with nausea and vomiting. She has no other past
medical or surgical history. On physical examination, she is afebrile, hemodynamical-
ly normal, with a positive Murphy sign and a tender epigastrium. Her laboratory test-
ing shows a white blood cell count of 14,000/mm3 (3600-l 1,200/mm3), an amylase of
600 units/L (23-85 U/L), and a lipase of 900 units/L (adults 60 and younger 10-140
U/L). Her other laboratory values are normal. A right upper quadrant ultrasound shows
gallstones, a gallbladder wall of 3 mm~ no pericholecystic fluid, and a normal common
bile duct. When is the most appropriate time to perform her cholecystectomy?
(A) Immediately
(B) After 6-8 weeks
(C) After 7 days of antibiotics
(D) After patient's abdominal exam has normalized.
(E) After an endoscopic retrograde cholangiopancreatography with sphincterotomy
CATEGORY 3 - PART 11
101
5. A 65-year-old man with a history of endovascular graft repair for a 6-cm abdominal
aortic aneurysm but no previous abdominal surgeries presents with symptoms consis-
tent with a small bowel obstruction. He has no obvious hernias on exam, and his rectal
exam is normal. He had a colonoscopy 2 years ago, which was normal. A CT scan
of his abdomen and pelvis is shown in figure 5.1 with a transition point in the mid to
distal jejunum. Which of the following statements is true?
(A) The most common location for these tumors is the duodenum.
(B) Survival in patients with metastatic disease is improved with chemotherapy after
a complete resection.
(C) Systemic symptoms are more readily apparent in hindgut disease.
(D) Small bowel obstruction is most often caused by the tumor itself.
(E) Octreotide is indicated for perioperative carcinoid crisis.
Figure 5.1
102 CATEGORY 3 - PART II
6. A 49-year-old obese man presents with acute left lower quadrant peritoneal irritation
and leukocytosis (serum white blood cell count 12,400/mml, 3600-11.200/mml). He
has no prior bowel history and has never had a colonoscopy. He has mild nausea without
vomiting and is tolerant of a liquid diet without pain. CT scanning is shown in figure 6.1.
Which of the following is the most appropriate treatment for this patient?
(A) Urgent colectomy
(BJ Admission, bowel rest, and intravenous antibiotics
(C) Elective colectomy
(DJ Outpatient management with oral antibiotics
(E) Percutaneous drainage
Figure 6.1
8. A 28-year-old woman with a history of major depressive disorder and previous suicide
attempts presents with a reported history of ingestion of a household cleaning agent.
Which of the following statements is true regarding her management?
CATEGORY 3 - PART II 103
(A) The ingested agent is likely an acidic substance, because strong alkaline agents
are uncommon household products.
(B) Antibiotics and steroids are recommended early in the ingestion period to
prevent early infectious complications.
(C) Ingestion of acidic substances causes a liquefaction necrosis.
(D) Endoscopy within the first 24 hours of ingestion is recommended.
(E) Only patients with extensive necrosis are at risk for stricture development.
9. A 56-year-old man with a history of esophageal varices and alcoholic cirrhosis pres-
ents with a I-hour history of hematemesis. He is awake and alert. His vitals are re-
markable for a respiratory rate of 24. heart rate of 120 beats per minute. systolic blood
pressure of 80 mm Hg, and Sp02 of 92%. After placement of adequate intravenous
access and administration of fluids~ he is admitted to the intensive care unit. Which of
the following statements is true regarding the early care of this patient with a presumed
variceal bleed?
(A) He should be urgently intubated.
(B) He should be transfused with packed red blood cells to a goal hemoglobin
greater than 10 g/dL.
(C) Prophylactic antibiotics are not needed.
(D) The preferred treatment is early endoscopy with band ligation coupled with
somatostatin.
(E) Sclerotherapy has fewer complications than band ligation ofvarices.
10. A 70-year-old man has a parastomal hernia after abdominal perinea! resection. Which
of the following approaches minimizes the risk of recurrence?
(A) Primary repair with absorbable suture
(B) Re-siting of the stoma
(C) Primary repair with permanent suture
(D) Repair with biologic mesh
(E) Repair with synthetic mesh
11. A 45-year-old man presents with an impacted food bolus in the esophagus from the
previous evening. He has no chest or neck pain, but he is drooling saliva. Which of the
following initial tests or intenrentions is most appropriate?
(A) Nasogastric tube
(B) Intravenous glucagon, repeated every 5 minutes until the bolus passes
(C) Barium swallow
(D) Flexible endoscopy with biopsy
(E) Rigid esophagoscopy
12. Which of the following factors is most predictive of the need for operative interven-
tion in pneumatosis intestinalis?
(A) Lactic acid greater than 2 mmol/L
(B) Portal venous gas
(C) Corticosteroid use
(D) Age greater than 60
(E) Acute lung injury
104 CATEGORY 3 - PART 11
13. Inpatient mortality associated with peptic ulcer disease (PUD) has decreased as the
result of
(A) performing therapeutic endoscopy to control bleeding only after the patient is
hemodynamically normal.
(B) intravenous proton pump inhibitors before and after endoscopy.
(C) definitive operation for PUD in the urgent setting.
(D) surgical intervention for recurrent bleeding.
(E) vagotomy in the surgical treatment of perforated or bleeding PUD.
14. Which of the following is the most appropriate treatment for a patient with cecal
volvulus?
(A) Right hemicolectomy with primary ileocolic anastomosis
(B) Tube cecostomy
(C) Colonoscopic detorsion
(D) Exploratory laparotomy with detorsion
(E) Exploratory laparotomy, detorsion, and cecopexy
15. A 24-year-old man presents with a 4-month history of postprandial nausea and ab-
dominal pain~ vomiting, and 6.8-kg weight loss. A CT is obtained (figures 15.1 and
15.2). Despite conservative treatment, the patient does not improve. After rehydration
and correction of his electrolyte abnormalities, surgical management should be
(A) release of Ligament ofTreitz.
(B) gastrojejunostomy.
(C) duodenojejunostomy.
(D) resection of the third and fourth portions of duodenum and duodenojejunostomy.
(E) pancreaticoduodenectomy (Whipple procedure).
Figure 15.1
CATEGORY 3 - PART II
105
Figure 15.2
17. A 57-year-old man presents with an upper gastrointestinal hemorrhage. Upper endos-
copy shows an ulcerated mass on the greater curvature of the stomach. Biopsies show
a high-grade sarcoma positive for KIT (CD 117). Which of the following statements is
true regarding the use of imatinib if the tumor is proven to be initially nonresectable?
(A) It should be continued postoperatively for 12 months.
(B) More than 90% of patients will respond radiographically.
(C) More than 60% of patients can receive an RO resection after treatment.
(D) Patients who fail should be treated with doxorubicin hydrochloride.
(E) Daily intravenous access is required.
18. The most appropriate diagnostic test in the workup of a symptomatic patient with
lower gastrointestinal bleeding who required transfusion with intermittent periods of
hypotension is
(A) unprepped colonoscopy.
(B) catheter-based angiography.
(C) CT angiography.
(D) exploratory laparotomy.
(E) tagged red blood cell bleeding scan.
106 CATEGORY 3 - PART 11
19. The best patient for transanal resection of a rectal cancer has a
(A) 2-cm Tl tumor with lymphovascular invasion.
(B) !-cm Tl tumor with perineural invasion.
(C) 3-cm tumor with grade 3 histology.
(D) 3-cm Tl tumor with 25% involvement of the bowel circumference.
(E) !-cm T2 tumor 10 cm from the anal verge.
20. A 59-year-old female patient presents with intermittent nausea, early satiety, and bloat-
ing. She has a 12-year history of type II diabetes mellitus. She underwent laparoscopic
cholecystectomy 6 years ago. A clinical diagnosis of gastroparesis is made. The best
initial treatment is
(A) metoclopramide.
(B) domperidone.
(C) erythromycin.
(D) small, low-fat, frequent meals.
(E) an implantable neurostimulator.
21. A 42-year-old woman presents with flushing, nausea, vomiting, and weight loss. Up-
per endoscopy shows a near obstructing tumor of the antrum. Biopsies are positive
for carcinoid tumor. CT scan shows several lesions in both lobes of the liver consis-
tent with metastatic disease. Which of the following surgical procedures should be
planned?
(A) Total gastrectomy with regional lymphadenectomy
(B) Total gastrectomy with resection of the hepatic metastasis
(C) Antrectomy with gastroduodenostomy
(D) Antrectomy with cholecystectomy
(E) Wedge resection with vagotomy
23. A 51-year-old man has a 2-cm villous adenoma endoscopically removed from his
cecum by piecemeal resection. Seventy-two hours later, he is seen in the emergency
department passing maroon stools. His temperature is 37.!°C, his heart rate is 100
beats per minute, and his blood pressure is 110/60 mm Hg. His abdomen is soft and
not tender. His white blood cell count is 10,000/mm3 (3600-ll,200/mm3) and his
hematocrit is 24% (43-52%). He is given 2 units of packed red blood cells and admit-
ted to the hospital. Twenty-four hours later, he continues to pass maroon stools, and
his hematocrit remains 24o/o with unchanged vital signs. After transfusing additional
packed red blood cells, the next best step in his management is
CATEGORY 3 - PART II 107
24. A 39-year-old man has significant left lower quadrant pain associated with a fever of
38.3°C. His white blood cell count is 15,000/mm3 (3600-ll,200/mm3). A CT scan
shows inflammation around the sigmoid colon without evidence of extraluminal air
or fluid. He is given a course of antibiotics with complete resolution of his symptoms.
Eight weeks after the episode, a colonoscopy reveals moderate diverticulosis in the
sigmoid colon and mild diverticulosis throughout the rest of his colon. Which of the
following is the next best step in management?
(A) Observation
(B) Sigmoid colectomy
(C) Total abdominal colectomy
(D) 5-aminosalicylic acid
(E) Probiotic therapy
26. A 53-year-old woman is diagnosed with a rectal cancer 14 cm from the anal verge by
rigid proctoscopy. It cannot be palpated on physical exam. A biopsy is consistent with
a moderately differentiated adenocarcinoma. A pelvic MRI confirms the tumor loca-
tion. The tumor is confined to the bowel wall without associated lymphadenopathy.
There is no evidence of metastatic disease on a chest, abdomen, and pelvic CT scan.
Which of the following is the most appropriate next step in management?
(A) Chemoradiation
(B) Transanal excision
(C) Resection with a total mesorectal excision
(D) Resection with a 5-cm distal margin
(E) Resection with a 2-cm distal margin
108 CATEGORY 3 - PART 11
27. Which of the following clinical scenarios for a patient with a gastrointestinal stromal
tumor (GIST) is associated with the highest risk for tumor recurrence after surgery?
28. During a screening colonoscopy, a 53-year-old woman has a 2-cm pedunculated polyp
removed from her sigmoid colon. The final pathology is a tubulovillous adenoma with
a focus of invasive moderately differentiated adenocarcinoma confined to the head of
the polyp. The stalk is free of adenomatous changes. No lymphovascular invasion or
tumor budding is evident. The most appropriate management is
(A) colonoscopy in 3 months.
(B) colonoscopy in 5 years.
(C) colonoscopy in 10 years.
(D) intraoperative colotomy with excision.
(E) sigmoid colectomy.
Figure 29.1
30. Which endoscopic gastric finding has the highest malignant potential and warrants
aggressive endoscopic surveillance?
(A) Fundic gland polyp
(B) Inflammatory fibroid polyp
(C) Adenomatous polyp
(D) Hyperplastic polyp
(E) Ectopic pancreas
32. A 94-year-old farmer with a history of diabetes and hypertension lives independently
in spite of prior amputation of the thumb and index finger on his dominant hand. He
now presents with 7 days of constipation, nausea, and abdominal distention without
tenderness. Plain films and CT scan with rectal contrast are shown in figures 32.1-
32.3. Flexible sigmoidoscopy confirms a malignant obstructing mass lesion in the
distal sigmoid colon. Which of the following would be the most appropriate initial
management?
(A) Laparotomy, sigmoid resection, and end colostomy
(B) Laparotomy, sigmoid resection, and primary anastomosis
(C) Cecostomy
(D) Diverting loop colostomy
(E) Endoscopic stent placement
Figure 32.1
Figure 32.2
CATEGORY 3- PART II
111
Figure 32.3
33. Which of the following statements regarding repair of large type III paraesophageal
hernias is true?
(A) Fundoplication is rarely performed.
(B) A transthoracic approach is necessary.
(C) Biologic mesh repair reduces long-term hernia recurrence rates.
(D) Primary repair of the crura is rarely possible.
(E) Complete reduction of the hernia sac is advised.
34. A previously healthy 92-year-old woman sustains an iatrogenic injury of the middle
third of her esophagus during endoscopic retrograde cholangiopancreatography for a
biliary stricture. Gastrografin esophagogram reveals a perforation with focal contained
extravasation of contrast. She is hemodynamically normal and complains only of mild
chest discomfort. The best course of initial treatment in addition to intravenous antibi-
otics and nothing by mouth would be
(A) thoracotomy with primary repair and wide drainage.
(B) thoracotomy and drainage only.
(C) nasogastric tube and repeat esophagogram in 1 week.
(D) esophagostomy and gastrostomy.
(E) removable esophageal stent.
112 CATEGORY 3 - PART II
35. A 53-year-old man presents with severe dysphagia and heartburn 10 years after suc-
cessful laparoscopic Nissen fundoplication. Endoscopy reveals an unremarkable
esophagus with the Z line at 37 cm from the incisors. A 96-hour capsule pH study
shows severe acid reflux. Esophageal manometry reveals normal motility, increased
lower esophageal sphincter (LES) length and pressure, and incomplete LES relaxation
with swallows. Barium esophagogram is shown in figure 35.1. The most likely cause
for the patient's symptoms is
(A) disrupted fundoplication.
(B) achalasia.
(C) pseudo achalasia.
(D) paraesophageal hernia.
(E) slipped fundoplication.
39. Which of the following statements is true regarding marginal ulceration after gastric
bypass (GBP)?
(A) Ulcers most commonly present as painless upper gastrointestinal bleeding.
(B) Smoking is a common risk factor.
(C) Ninety percent of marginal ulcerations after GBP are caused by
Helicobacter pylori.
(D) Nonabsorbable suture use for the gastrojejunostomy will reduce the risk of
marginal ulceration after GBP.
(E) The majority of patients present within 3 months of their index GBP.
Critiques 1-40
CRITIQUE 1
Resection is the treatment of choice for colorectal liver metastases (CLM; figure 1.2).
Resection of CLM is performed with the intent to cure patients of their disease. Currently,
this operation is the only hope for complete elimination of disease. However, patient char-
acteristics prognostic for long-term survival are not well defined.
In a large study of patients who had survived more than 10 years after resection of
CLM, pre- and postoperative prognostic factors were defined. The majority of patients
who survived 10 years after resection of CLM had fewer than 3 metastases. Only 7% of
patients who survived I 0 years after CLM resection had a preoperative carcinoembryonic
acid (CEA) level greater than 200 ng/mL. The majority of patients with levels that high
survived less than 2 years after surgery. Metastatic tumor sizes greater than 5 cm and lymph
node positive for primary disease were associated with a poor prognosis. Most importantly,
no patients with positive margins were alive at I 0 years after surgery in this study. This
finding is incorporated into the expert consensus statement regarding the evaluation of
patients for resection of CLM. This recommendation is that patients are considered for
surgery only if a margin-negative (RO) resection is expected to be feasible.
References
1. Tomlinson JS, Jarnagin WR, De Matteo RP, et al. Actual 10-year survival after resection
of colorectal liver metastases defines cure. J Clin Oneal. 2007;25(29):4575--4580.
2. Adams RB, Aloia TA, Loyer E, Pawlik TM, Taouli B, Vauthey JN; Americas Hepato-
Pancreato-Biliary Association: Society of Surgical Oncology; Society for Surgery of the
Alimentary Tract. Selection for hepatic resection of colorectal liver metastases: expert
consensus statement. HPB (Oxford). 2013;15(2):91-103.
CATEGORY 3 - PART II 115
CRITIQUE2
The timing of cholecystectomy in the setting of mild pancreatitis is an area of controversy.
Mild pancreatitis is defined as having fewer than 3 Ranson criteria. These patients will
typically respond to fluid resuscitation. Patients who have more than 3 Ranson criteria,
have evidence of organ failure, or worsen within the first 24 hours after presentation are
classified as having moderate to severe pancreatitis. These patients would not likely benefit
from early, same admission, laparoscopic cholecystectomy.
For mild pancreatitis, proponents of delaying cholecystectomy for a subsequent admis-
sion suggest that the procedure is technically easier with less inflammation present and is
associated with less morbidity because the pancreatitis has resolved. A systematic review
of the literature, however, shows that complication rates, conversion to an open technique,
and mortality do not differ when comparing early versus delayed cholecystectomy. Fur-
thermore, delayed cholecystectomy was associated with an 18% readmission rate within an
average of 40 days for recurrent pancreatitis, acute cholecystitis, or biliary colic.
A prospective study concluded that there was no difference in morbidity or conversion
to an open procedure in patients who underwent a laparoscopic cholecystectomy within
48 hours compared with those who were delayed until the abdominal pain resolved or the
laboratory values normalized. However this study was criticized for being small (n = 50)
and for having a high-risk of bias. Subsequent articles advocated caution with early cho-
lecystectomy but acknowledged growing literature supporting an early laparoscopic cho-
lecystectomy when the patient is clinically normal, implying a normal abdominal exam.
There is clearly a decreased length of stay in the hospital with early laparoscopic cholecys-
tectomy, but for other outcomes, the evidence is very low or low quality.
In summary, waiting several weeks before performing laparoscopic cholecystectomy
is associated with a high-risk of readmission. Performing an early laparoscopic cholecys-
tectomy in patients with mild pancreatitis is associated with decreased hospital stay and is
not associated with increased morbidity. This patient has mild gallstone pancreatitis and
thus laparoscopic cholecystectomy should be done when the patient's abdominal exam is
normal, regardless of the amylase and lipase values, and during the initial hospitalization.
There is no indication for the use of antibiotics in mild acute pancreatitis. The use of endo-
scopic retrograde cholangiopancreatography in the face of mild pancreatitis is not recom-
mended because the gallstone likely has passed as evidenced by the clinical resolution of
the pancreatitis.
References
I. AboulianA, Chan T, Yaghoubian A, et al. Early cholecystectomy safely decreases hospi-
tal stay in patients with mild gallstone pancreatitis: a randomized prospective study. Ann
Surg. 2010;251(4):615--619.
2. van Baal MC, Besselink MG, Bakker OJ, et al; Dutch Pancreatitis Study Group. Timing
of cholecystectomy after mild biliary pancreatitis: a systematic review. Ann Surg.
20 I 2;255(5):860-866.
3. Demehri FR, Alam HB. Evidence-based management of common gallstone-related
emergencies. J Intensive Care Med. 2016;31(1):3-13.
4. Gurusamy KS, Nagendran M, Davidson BR. Early versus delayed laparoscop-
ic cholecystectomy for acute gallstone pancreatitis. Cochrane Database Syst Rev.
20l3;9:CDO10326.
116 CATEGORY 3 - PART II
CRITIQUE3
This patient has achalasia and although endoscopic techniques are useful, the most durable
treatment is surgical. Treatment of achalasia involves endoscopic therapies: balloon dila-
tion and botulinum toxin type A injection or a surgical myotomy of the lower esophageal
sphincter (LES), called a Heller Myotomy, An additional endoscopic therapy called per
oral endoscopic myotomy (POEM) essentially performs an endoluminal myotomy by go-
ing through the esophageal mucosa to get to the hypertrophied muscle of the LES.
The Heller Myotomy is most often done laparoscopically and has the best long-term
results in terms of symptom resolution. POEM has promising potential but as yet is not ac-
cepted universally as a primary treatment, mostly because it requires advanced endoscopic
skills that the typical gastroenterologist or general surgeon does not possess. Balloon dila-
tion is successful 70% of the time in resolving symptoms of achalasia but has a significant
recurrence rate within 2 years. Botulinum toxin type A injection is successful in approxi-
mately 50% of cases, and it too has a high recurrence rate. In addition, injection therapy
tends to create inflammation and scarring around the LES, making surgical therapy more
difficult. As such, botulinum toxin type A is generally reserved for patients who are poor
surgical candidates or who refuse surgical treatment.
The myotomy consists of dividing the inner circular and outer longitudinal muscles
of the distal esophagus wall. This division is carried out at least 5 cm onto the esophagus
and 2 cm on the stomach. Many surgeons opt to perform a concomitant antireflux opera-
tion, but this is not mandatory. Further, there is no consensus on what antireflux operation
should be done in this situation. Finally, although robot-assisted laparoscopic surgery of-
fers a theoretical advantage of improved visualization, improved maneuverability in a nar-
row mediastinal area, and improved surgeon comfort because the surgeon is not standing
but sitting. no data show outcomes improve with the aid of the robot platform_
This patient's symptoms may also be indicative of gastroesophageal reflux disease
(GERD); in fact, they are considered the typical symptoms of GERD. Atypical symptoms
include reflux with exercise, acidic taste, hoarse voice, difficulty lying supine due to re-
flux, and adult-onset asthma. In distinguishing achalasia from GERD or other esophageal
diseases, an upper gastrointestinal (UGI) series should be ordered to look for reflux or a
hiatal/diaphragmatic hernia. With achalasia, a ''bird's beak" appearance is often present
on an UGI series. However, an UGI series does not confirm the diagnosis. Longstanding
achalasia may also be associated with dilation of the proximal esophagus and even "'sig-
moidization" of the proximal esophagus as the dilated esophagus forms a curvy path to
accommodate the delayed passage of food.
Next, an esophagogastroduodenoscopy (EGD) should be ordered to look for evidence
of esophagitis and to rule out Barrett esophagitis. In patients with achalasia, the muco-
sa may look entirely normal. Achalasia would also be suggested by retained food in the
esophagus, a dilated esophagus, and a narrowed area of the distal esophagus that is difficult
to pass the scope through.
To confirm the diagnosis of achalasia, however, a manometry study is needed. An el-
evated LES pressure (normal 10-15 mm Hg), a nonrelaxing LES, and discordant esopha-
geal peristaltic waves are diagnostic of achalasia. A manometry study can rule out primary
esophageal pathology like diffuse esophageal spasm, Sjogren syndrome, and nutcracker
esophagus.
Calcium channel blockers and proton pump inhibitors have no role in the treatment of
achalasia.
Answer: (C) Compared with endoscopic therapy, surgical management has better long-
term results.
CATEGORY 3 - PART II
117
References
1. Vaziri K, Soper NJ. Laparoscopic Heller myotomy: technical aspects and operative
pitfalls. J Gastrointest Surg. 2008; 12(9): 1586-1591.
2. Moonen A, Boeckxstaens G. Current diagnosis and management of achalasia. J Clin
Gastroenterol. 2014;48(6):484-490.
CRITIQUE4
The sleeve gastrectomy is the most popular bariatric procedure performed in the United
States today. In general, its efficacy and risk are between those of the adjustable gastric
band and the Roux-en-Y gastric bypass (RYGB). Weight loss at 2 years is generally 60-
65%, whereas that of the RYGB is approximately 70-75%. The sleeve gastrectomy is
associated with a lower overall complication rate, is less technically demanding, and has a
lower incidence of postoperative malnutrition. The sleeve gastrectomy does have a higher
leak rate, 5-9o/o, and a higher incidence of esophageal reflux, as high as 30%, than the
RYGB. The RYGB leak rate is 0-5%, and the new onset reflux disease rate is less than 1%.
Finally, the durability of the sleeve gastrectomy beyond 5 years is not known.
In many respects, the sleeve gastrectomy provides equivalent results compared with the
RYGB. A prospective controlled trial compared the efficacy of sleeve gastrectomy to the
RYGB and best medical management. The sleeve gastrectomy was equal to the RYGB in
eliminating a patient's type II diabetes mellitus.1 Approximately 40% of patients with type
II diabetes were cured of their disease within I year with both the sleeve gastrectomy and
the RYGB.
References
1. Schauer PR, Kashyap SR. Wolski K, et al. Bariatric surgery versus intensive medical
therapy in obese patients with diabetes. N Engl J Med. 2012;366(17):1567-1576.
2. Pallati PK, Shaligram A, Shostrom VK, Oleynikov D, McBride CL, Goede MR.
Improvement in gastroesophageal reflux disease symptoms after various bariatric
procedures: review of the Bariatric Outcomes Longitudinal Database. Surg Obes Relat
Dis. 2014;10(3):502-507.
3. Aurora AR, Khaitan L, Saber AA. Sleeve gastrectomy and the risk ofleak: a systematic
analysis of 4,888 patients. Surg Endosc. 2012;26(6): 1509-1515.
CRITIQUES
In adult patients who present with a bowel obstruction, without any evidence of hernias
and without a history of abdominal surgery, a malignancy must be ruled out. For patients
who present with a small-bowel obstruction, the differential diagnosis includes adenocar-
cinoma of the small bowel, lymphoma, metastatic melanoma. carcinoid tumor, and stromal
tumors. The most common cause is a carcinoid tumor, which occurs in 1-2 per 100,000
adults in the United States. Carcinoid tumors of the small bowel most often occur in the
ileum, followed by the jejunum and then the duodenum. Figure 5.2 shows the fibrosis in
the mesentery, classic findings for carcinoid tumor.
118 CATEGORY 3- PART II
Figure 5.2 Fibrosis in the mesentery, classic findings for carcinoid tumor.
Carcinoid tumors of the gastrointestinal tract can present with flushing and diarrhea due
to the hormones released by the active endocrine cells within the gastrointestinal tract. The
location of the tumor, however, plays a role in the symptoms produced. Hindgut carcinoids
present more often with hematochezia and are otherwise rarely symptomatic. Foregut car-
cinoids do not present with typical symptoms because of the multitude of biologically ac-
tive compounds they produce. Midgut tumors present with flushing and diarrhea due to the
high levels of serotonin secretion, but these usually occur with bulky or metastatic disease.
Systemic symptoms are more apparent in foregut and hindgut disease. A 24-hour urine
level for the serotonin metabolite 5-hydroxyindoleacetic acid (5-HIAA) can often confirm
the diagnosis of a carcinoid tumor.
Pati;nts with midgut tumors will more likely present with a small bowel obstruction due
to the intense desmoplastic reaction caused by the tumor. For patients who present with a
small bowel obstruction, the best treatment is surgical resection, which includes resection
of the fibrotic tissue of the desmoplastic reaction. An oncologic resection and exploration
for other disease within the small bowel and in the liver should be done. These patients
should be monitored for signs of a carcinoid crisis after surgery. The signs of a carcinoid
crisis include facial flushing, diarrhea, tachycardia, arrhythmias, hypotension, and mental
status changes due to the release of serotonin. These symptoms are usually refractory to
fluid resuscitation and vasopressor medications. Carcinoid crisis can be treated effectively
with the administration of octreotide.
In patients with metastatic disease, there is little role for chemotherapy. Hepatic resection
can be considered as well as cryotherapy, radioablative therapy, and ethanol injection therapy.
References
1. Kulke MH. Clinical presentation and management of carcinoid tumors. Hematol Oncol
Clin North Am. 2007:21(3):433--455.
2. Landry CS, Brock G, Scoggins CR, McMasters KM, Martin RC 2nd. A proposed stag-
ing system for small bowel carcinoid tumors based on an analysis of 6,380 patients. Am
J Surg. 2008;196(6):896-903.
CATEGORY 3 - PART !I
119
CRITIQUE6
Randomized, controlled data in uncomplicated sigmoid diverticulitis indicate that in ap-
propriately selected patients, outpatient management of acute diverticulitis is appropriate
and cost-effective. Figure 6.2 demonstrates sigmoid colon inflammation without sugges-
tion of complicated diverticulitis such as abscess or free perforation (Hinchey class O: table
6.1 ). The patient is tolerant ofliquids without pain, which is predictive of successful outpa-
tient management of sigmoid diverticulitis, making ad1nission unnecessary. Less than 5%
of patients with uncomplicated diverticulitis ultimately require colectomy. Percutaneous
drainage is used to treat an abscess that is accessible based on imaging and is usually done
in conjunction with a course of antibiotics.
l"l
"'~= ~
"
Figure 6.2 Sigmoid colon inflammation without sugges-
tion of complicated diverticulitis such as abscess or free
perforation.
-._...,, "°..,
-o
w
References
I. Janes S, Meagher A, Faragher JG, Shedda S, Frizelle FA. The place of elective surgery
following acute diverticulitis in young patients: when is surgery indicated? An analysis
of the literature. Dis Colon Rectum. 2009;52(5):1008-1016.
2. Broderick-Villa G, Burchette RJ, Collins JC, Abbas MA, Haigh PI. Hospitaliza-
tion for acute diverticulitis does not mandate routine elective colectomy. Arch Surg.
2005; 140(6):576--583.
3. Biondo S, Golda T, Kreisler E, et al. Outpatient versus hospitalization management
for uncomplicated diverticulitis: a prospective, multicenter randomized clinical trial
(DIVER Trial). Ann Surg. 2014;259(1):38-44.
4. Salem TA, Molloy RG, O'Dwyer PJ. Prospective, five-year follow-up study of
patients with symptomatic uncomplicated diverticular disease. Dis Colon Rectum.
2007;50(9): 1460-1464.
5. Wasvary H, Turfah F, Kadro 0, Beauregard W. Same hospitalization resection for acute
diverticulitis. Am Surg. l 999;65(7):632-636.
CRITIQUE 7
In the absence of any suggestion of adenomatous polyps or advanced neoplasia, American
Gastroenterology Association Guidelines for surveillance of rectosigmoid or rectal hyper-
plastic polyps less than 10 mm confer no substantial risk for subsequent colon cancer, and
recommended colonoscopic surveillance interval is 10 years. Tattooing of a polypectomy
site would be appropriate for any polyp thought to be incompletely removed or that ap-
peared suspicious for malignancy, even with removal.
Dietary modification with fiber supplementation is an appropriate recommendation for
this patient with asymptomatic diverticulosis. Surgical intervention would not be appropriate.
References
I. Lieberman DA, Rex DK, Winawer SJ, Giardiello FM, Johnson DA, Levin TR; Unit-
ed States Multi-Society Task Force on Colorectal Cancer. Guidelines for colonoscopy
surveillance after screening and polypectomy: a consensus update by the US Multi-
Society Task Force on Colorectal Cancer. Gastroenterology. 2012;143(3):844-857.
2. Maconi G, Barbara G, Bosetti C, Cuomo R, Annibale B. Treatment of diverticular
disease of the colon and prevention of acute diverticulitis: a systematic review. Dis
Colon Rectum. 2011;54(10): 1326-1338.
CRITIQUES
Although accidental ingestions are common in the pediatric population, the majority of
caustic ingestions in the adult population are intentional. Agents of choice are typically
alkaline as acidic substances cause severe pain. There are several alkaline household
cleaning and laundry agents, and most contain either sodium or potassium hydroxide.
Although ingestion of acidic substances causes a coagulative necrosis leading to vessel
thrombosis and scar formation, alkaline agents cause a liquefactive necrosis. This occurs
as the agent bonds to tissue protein, which can also produce injury outside the esophagus
and into the mediastinum.
The mainstay of treatment involves airway protection and resuscitation. Endoscopy
within 24 hours once the patient is stabilized is recommended. Caution must be used not
to judge the extent of injury by the condition of the oropharynx. Endoscopy helps to grade
the injury and predict prognosis.
CATEGORY 3 - PART JI 121
The stricture rate increases with worsening injury (table 8.1). Grade 2B is focal or cir-
cumferential deep ulceration, and 3A is focal necrosis; both demonstrate a 70-100% stric-
ture rate. Neither steroids nor antibiotics reduce the risk of strictures. Intravenous antibiot-
ics should be administered for Grade 3 injuries to reduce infectious complications. Steroids
increase the risk of infectious complications without any positive effect on the stricture rate
and should be avoided.
References
1. Tohda G, Sugawa C, Gayer C, Chino A, McGuire TW, Lucas CE. Clinical evaluation
and management of caustic injury in the upper gastrointestinal tract in 95 adult patients
in an urban medical center. Surg Endosc. 2008;22(4): 1119-1125.
2. Park KS. Evaluation and management of caustic injuries from ingestion of acid or alka-
line substances. C/in Endosc. 2014;47(4):301-307.
CRITIQUE9
Variceal gastrointestinal bleeding carries an overall mortality of approximately I 0-20%
in patients with cirrhosis. Modem therapy, including advances in endoscopy, critical care,
and interventional radiology, reduced this mortality several fold over the past 3 decades.
The rapid stabilization and management of patients presenting with this condition is criti-
cal in maximizing their survival. Endoscopic band ligation should be perfonned within
12 hours of presentation. Band ligation has fewer complications than sclerotherapy and is
the recommended endoscopic treatment. Sclerotherapy should be reserved for when band
ligation is technically not feasible. Several studies demonstrated endoscopic band ligation,
used in combination with a vasoactive agent such as somatostatin, is more effective than
either treatment alone.
Other studies demonstrated that a restrictive transfusion policy with a goal hemoglobin
of7-8 g/dL is associated with a reduction in rebleeding and increased overall survival. Al-
though airway protection is of primary importance, patients who are awake, alert. and able
to protect their airway do not require prophylactic intubation. In such patients, intubation
does not reduce the incidence of aspiration or pneumonia. Patients with delirium related
to shock, uncontrolled hematemesis, and ongoing hemodynamic instability should be intu-
bated before endoscopy.
122 CATEGORY 3 - PART ll
Infectious complications, early rebleeding, and overall mortality are reduced when pa-
tients are administered prophylactic antibiotics. This is specific to variceal gastrointestinal
bleed and does not apply to patients with nonvariceal gastrointestinal bleeds.
Answer: (D) The preferred treatment is early endoscopy with band ligation coupled with
somatostatin.
References
I. Elmunzer BJ, Young SD, lnadomi JM, Schoenfeld P, Laine L. Systematic review of the
predictors of recurrent hemorrhage after endoscopic hemostatic therapy for bleeding
peptic ulcers. Am J Gastroenterol. 2008; I 03(10):2625-2632
2. Cremers I, Ribeiro S. Management of variceal and nonvariceal upper gastrointestinal
bleeding in patients with cirrhosis. Therap Adv Gastroenterol. 2014;7(5):206-216.
CRITIQUE 10
The incidence ofparastomal hernia occurrence after enterostomy is high, with quoted rates
ranging from 5 to 40o/o. Several techniques to address these hernias are described in the
literature. Primary suture repair, with absorbable or permanent sutures, has the worst out-
comes, with recurrence rates ranging from 50 to 1OOo/o. Re-siting the stoma is not only
associated with an unacceptably high recurrence rate (20-90%) but also leaves the patient
with 3 possible sites for postoperative hernia formation: new stoma, old stoma site, and
laparotomy incision.
Repair with synthetic mesh, whether performed laparoscopically or open, has the lowest
recurrence risk in the literature today. The risk of mesh infection is low, and the document-
ed recurrence rates range from 7 to 17%. Several types of mesh repairs are described in the
literature, including onlay, sublay, and intraperitoneal. None appears to be better in large
studies except for the laparoscopic approach in which the "Sugarbaker" approach (figure
IO.I) shows improved outcomes over the "keyhole" technique (figure 10.2).
The use of biologic mesh increases recurrence outcomes and offers no benefit over
lightweight, macroporous synthetic mesh from an infection standpoint.
References
I. Hansson BM, Slater NJ, van der Velden AS, et al. Surgical techniques for parastomal
hernia repair: a systematic review of the literature. Ann Surg. 2012;255(4):685-695.
2. Shah NR, Craft RO, Harold KL. Parastomal hernia repair. Surg C/in North Am.
2013;93(5):1185-1198.
3. Shabbir J, Chaudhary BN, Dawson R. A systematic review on the use of prophylactic
mesh during primary stoma formation to prevent parastomal hernia formation. Colorec-
tal Dis. 2012;14(8):931-936.
CRITIQUE 11
With prolonged impaction, prevention of aspiration is of utmost importance. Flexible en-
doscopy is safe and highly effective in the diagnosis and management of esophageal food
impaction. Contrast exam and blind nasogastric tube insertion are not recommended. Al-
though a trial of glucagon to lower the esophageal-gastric junction sphincter tone is rea-
sonable, with repeated dosing, however, glucagon is associated with nausea and vomiting,
imposing a risk of aspiration or perforation of the esophagus.
Most food bolus impactions pass spontaneously without intervention, but 10-20% of im-
pactions require interventions. Most esophageal food impactions (88-97%) are related to
underlying esophageal pathology, most of which can be diagnosed and possibly managed
with flexible endoscopy. Peptic strictures, tumors, Schatzki rings, and eosinophilic esophagi-
tis can be diagnosed with endoscopy. Biopsy, as appropriate, is a benefit of intervention with
flexible endoscopy. Up to 50% of young patients with esophageal food impaction may have
histologic evidence of eosinophilic esophagitis, a cause of esophageal stricture occurring
with increasing prevalence. Rigid esophagoscopy is not the best initial intervention.
References
1. ASGE Standards of Practice Committee, Ikenberry SO, Jue TL, Anderson MA, et al.
Management of ingested foreign bodies and food impactions. Gastrointest Endosc.
2011;73(6): 1085-1091.
124 CATEGORY 3 - PART II
CRITIQUE 12
Pneumatosis intestinalis refers to the presence of gas within the bowel wall of the gastroin-
testinal tract. It is associated with numerous conditions ranging from benign to life threat-
ening. Found on abdominal imaging, pneumatosis intestinalis is associated with intestinal
ischemia, inflammatory bowel disease, chemotherapy, connective tissue disease, trauma,
transplantation, and other conditions. Differentiating a benign etiology from a pathological
cause is quite difficult, and as such, many patients with a benign source undergo unneces-
sary abdominal exploration. A comprehensive evaluation must be performed and includes
a history and physical examination and laboratory and radiographic studies. Pneumatosis
remains a complex process, and determining the appropriate treatment is a challenge.
Risk factors for pathologic pneumatosis intestinalis include lactic acid greater than 2
mmol/L (0.5-2.3 mmol/L), portal venous gas, corticosteroid use, age greater than 60, and
acute lung injury. Of the choices listed, only lactic acid greater than 2 mmol/L was found on
multivariate analysis to be an independent predictor of the need for operative intervention
(i.e., intestinal ischemia). Other independent risk factors include hypotension or pressor use,
peritonitis, acute kidney injury, mechanical ventilation, and absent bowel sounds.
References
1. DuBose JJ, Lissauer M, Maung AA, et al; EAST Pneumatosis Study Group. Pneuma-
tosis Intestinalis Predictive Evaluation Study (PIPES): a multicenter epidemiologic
study of the Eastern Association for the Surgery of Trauma. J Trauma Acute Care Surg.
2013;75(1): 15-23.
2. Duron VP, Rutigliano S, Machan JT, Dupuy DE, Mazzaglia PJ. Computed tomographic
diagnosis ofpneumatosis intestinalis: clinical measures predictive of the need for surgi-
cal intervention. Arch Surg. 2011;146(5):506-510.
CRITIQUE 13
Peptic ulcer disease (PUD) is the most common cause of upper gastrointestinal bleeding.
Advances in the understanding of the pathophysiology of this disease process and Helico-
bacter pylori infection have changed management dramatically. Despite an overall mortal-
ity of 5-10% for patients with PUD, inpatient mortality decreased from 3.8% in 1993 to
2. 7% in 2006. Improvements in mortality have resulted from using therapeutic endoscopy
to control bleeding earlier in the initial stages of resuscitation when the patient may be he-
modynamically abnormal versus waiting until the patient is fully resuscitated and hemody-
namically normal. Infusing intravenous proton pump inhibitors before and after endoscopy
has resulted in decreased mortality secondary to a decreased risk of ulcer rebleeding and
need for urgent surgery. An additional decrease in in-hospital mortality is due to increased
endoscopic retreatment for recurrent bleeding versus surgery. The mortality rate of surgical
patients decreased from 12.4% to 11.0% and is attributable to the lower mortality associ-
ated with gastrotomy and oversewing of the bleeding ulcer instead of vagotomy with py-
loroplasty or antrectomy. Although the latter has a lower rebleeding rate, both procedures
have a higher mortality when compared with oversewing the bleeding ulcer.
Answer: (B) intravenous proton pump inhibitors before and after endoscopy
CATEGORY 3 - PART II 125
References
I. Wang YR, Richter JE, Dempsey DT. Trends and outcomes of hospitalizations for peptic
ulcer disease in the United States, 1993 to 2006. Ann Surg. 2010;251(1):51-58.
2. Gralnek IM, Barkun AN, Bardou M. Management of acute bleeding from a peptic ulcer.
N Engl J Med. 2008;359(9):928-937.
3. Abraham NS. Proton pump inhibitors: potential adverse effects. Curr Opin Gastroen-
terol. 20 I 2;28(6):615--020.
4. Sachar H, Vaidya K, Laine L. Intermittent vs continuous proton pump inhibitor therapy
for high-risk bleeding ulcers: a systematic review and meta-analysis. JAMA Intern Med.
2014;174(11):1755-1762.
CRITIQUE 14
Cecal volvulus is the second most common type of volvulus and accounts for 25-40% of
all volvulus of the colon. Variants of cecal volvulus involve the terminal ileum, cecum, and
proximal right colon. The most common variant is the clockwise axial rotation of these
structures around their mesentery. Late embxyogenesis is likely the underlying etiology
for this type of cecal volvulus and results in incomplete fixation of the mesentery of the
tenninal ileum., cecum, and proximal right colon to the retroperitoneum.
Cecal bascule involves anterosuperior folding of the cecum without axial rotation; this
is therefore less likely to cause vascular compromise and intestinal ischemia. A cecal bas-
cule occurs when a redundant pelvic cecum flops into the right upper quadrant., kinking
the right colon. Risk factors for cecal volvulus include chronic constipation. distal colon
obstruction, high-fiber diets, and ileus.
Patients present similarly to patients with a distal small bowel obstruction with nausea,
vomiting, obstipation, abdominal pain, and distention. If the volvulus progresses to in-
testinal ischemia, then patients will have peritonitis and possibly sepsis. Plain abdominal
radiographs may reveal the classic "coffee bean" sign, an air-filled cecum pointing to the
left upper quadrant (figure 14.1). Alternatively, a water-soluble enema may be performed
that shows a ''bird's beak" sign, which is progressive tapering of the ascending colon. A CT
scan of the abdomen and pelvis may show both signs (figure 14.2).
Figure 14.2 CT scan of the abdomen and pelvis showing ·•coffee bean''
and ''bird's beak" signs.
References
1. Bullard Dunn KM, Rothenberger DA. Colon, rectum, and anus. In: Brunicardi FC,
s
Andersen DK, Billiar TR, et al, eds. Schwartz Principles of Surgery. 10th ed. New
York, NY: McGraw-Hill;2015:1175-1240.
2. Gingold D, Murrell Z. Management of colonic volvulus. Clin Colon Rectal Surg.
2012;25(4 ):236-244.
CATEGORY 3 - PART Il 127
CRITIQUE 15
The textbook scenario for superior mesenteric artery (SMA) syndrome is often a young,
otherwise healthy person who experiences significant weight loss, often after a major op-
eration or illness, and then develops abdominal pain, nausea, and vomiting. The most com-
mon proposed pathophysiology is that rapid weight loss leads to loss of the duodenal fat
pad. This causes narrowing of the aortomesenteric angle and obstruction of the duodenum
as it passes under the SMA (figure 15.3). The normal aortomesenteric angle is 38°-{)5°.
An angle less than 25° is considered the most sensitive measure on diagnostic imaging,
with near 100% sensitivity and specificity, if it is accompanied by at least 1 symptom and
a decrease of the aortomesenteric distance to less than 8 mm (figure 15.4).
Initial treatment is often medical. Sometimes patients can maintain sufficient oral in-
take if they eat leaning forward. If this approach fails, they may require bowel rest, gastric
decompression, and nutritional support with either a nasoenteric feeding tube distal to the
SMA or parenteral nutrition. The hope is that with weight regain~ the aortomesenteric angle
will increase back to normal, and the obstruction will subside.
When medical management fails, SMA syndrome can be treated with several surgical
options. Duodenojejunostomy provides the most reliable surgical treatment and is per-
formed with or without division of the fourth portion of the duodenum. This procedure can
also be performed laparoscopically. Release of Ligament of Treitz (Strong procedure) is
associated with a lower success rate, while gastrojejunostomy is associated with a higher
complication rate, including ongoing duodenal obstruction, blind loop syndrome, and mar-
ginal ulcers. Some surgeons advocate resection of the affected portion of the duodenum
with the theory being that this a primary motility disorder. There are few data to support
this, and thus, resection would be considered excessive.
Figure 15.3 CT scan showing narrowing of the aortomesenteric angle and obstruction of
the duodenum as it passes under the superior mesenteric artery.
128 CATEGORY 3 - PART II
Figure 15.4
References
I. Munene G, Knab M, Parag B. Laparoscopic duodenojejunostomy for superior mesen-
teric artery syndrome. Am Surg. 2010;76(3):321-324.
2. Merrett ND, Wilson RB, Cosman P, Biankin AV. Superior mesenteric artery syndrome:
diagnosis and treatment strategies. J Gastrointest Surg. 2009;13(2):287-292.
CRITIQUE 16
Enhanced recovery after surgery (ERAS) programs are designed to accelerate postopera-
tive recovery and shorten time to discharge from the hospital without compromising patient
safety. The basic hallmarks of ERAS programs are evidence-based, standardized, preop-
erative assessment and teaching, as well as intraoperative and postoperative management.
Preoperative teaching involves setting expectations for the expected postoperative course
and criteria for discharge. Intraoperative management foc uses on minimizing intravenous
fluids, maintaining gastrointestinal function, and providing early, active pain management.
Postoperative care focuses on pain management, early mobility, maintenance of gastroin-
testinal tract function, and early resumption of oral intake.
In general, ERAS programs decrease hospital stay by just more than 2 days. These pro-
grams are associated with a lower overall complication rate, although there is no difference
in surgical complications or mortality. Concerns that earlier discharges would result in
more readmissions or increased use of home healthcare resources have not been borne out.
1n addition, patient satisfaction and health-related quality of life are not negatively affected
by the reduced hospital stay associated with ERAS programs. These programs are equally
effective in the elderly; several studies have shown no differences in outcomes in elderly
patients compared with younger patients for hospital stay, complications, or mortality.
CATEGORY 3 - PART Il 129
References
1. Lawrence JK, Keller DS, Samia H, et al. Discharge within 24 to 72 hours of colorectal
surgery is associated with low readmission rates when using enhanced recovery path-
ways. J Am Coll Surg. 2013;216(3):390-394.
2. Khan S, Wilson T, Ahmed J, Owais A, MacFie J. Quality oflife and patient satisfaction
with enhanced recovery protocols. Colorectal Dis. 201O;12( 12): 1175-1182.
3. Zhuang CL, Ye xz, Zhang XD, Chen BC, Yu z. Enhanced recovery after surgery
programs versus traditional care for colorectal surgery: a meta-analysis of randomized
controlled trials. Dis Colon Rectum. 2013;56(5):667--678.
4. Bagnall NM, Malietzis G, Kennedy RH, Athanasiou T, Faiz 0, Darzi A. A systematic
review of enhanced recovery care after colorectal surgery in elderly patients. Colorectal
Dis. 2014; 16(12):947-956.
CRITIQUE 17
Gastrointestinal stromal tumors (GISTs) arise in the muscularis of the gastrointestinal tract:
70% are of spindle cell type, 20% are epithelioid, and 10% are mixed. Regardless of histol-
ogy, 75-80% ofGISTs will harbor an activating mutation in the KIT oncogene, and many
others have mutations in platelet-derived growth factor. Accordingly, the use of tyrosine
kinase inhibitors that target these mutations, such as imatinib, in the treatment of advanced
GISTs is now considered standard of care.
Surgical resection alone (without the use of tyrosine kinase inhibitors) with clear surgi-
cal margins is associated with a 54% long-term GIST survival. For tumors initially con-
sidered unresectable or marginally resectable, therapy should begin with imatinib. It is
available in 100-mg and 400-mg tablets. Dosing is 400 mg orally once a day unless the
tumor is positive for KIT exon 9 mutation, in which case patients should receive 800 mg
per day. The most common side effect is edema/fluid retention (>70% with prolonged
therapy). Approximately 60% of patients will respond to treatment with imatinib. Patients
with initially nonresectable disease should have repeat staging after 6 months of therapy,
because 60% of these patients are able to come to RO resection. Resected patients should
remain on imatinib for 24--36 months.
Failure of imatinib should be followed by a second-line tyrosine kinase inhibitor (e.g.,
sunitinib 50 mg per day orally) rather than drugs used previously, such as doxorubicin. In
patients unable to undergo a RO resection, debulking may improve survival.
Answer: (C) More than 60% of patients can receive an RO resection after treatment.
References
I. DeMatteo RP, Lewis JJ, Leung D, Mudan SS, Woodruff JM, Brennan MF. Two hundred
gastrointestinal stromal tumors: recurrence patterns and prognostic factors for survival.
Ann Surg. 2000;231(1):51-58.
2. Shrikhande SV, Marda SS, Suradkar K, et al. Gastrointestinal stromal tumors: case
series of 29 patients defining the role of imatinib prior to surgery. World J Surg.
2012;36(4):864--871.
130 CATEGORY 3 - PART II
3. Joensuu H, Eriksson M, Sundby Hall K, et al. One vs three years of adjuvant imatinib
for operable gastrointestinal stromal tumor: a randomized trial. JAMA. 2012;307(12):
1265-1272.
4. Demetri GD, von Mehren M, Blanke CD, et al. Efficacy and safety ofimatinib mesylate
in advanced gastrointestinal stromal tumors. N Engl J Med. 2002;347(7):472-480.
CRITIQUE 18
In the hemodynamically normal patient, colonoscopy should be performed as the initial
test. Rapid lavage bowel preparation is safe and provides better results than unprepped
colonoscopy. Upper endoscopy should be obtained at the same time, because 10-15% of
patients thought to have a lower gastrointestinal source will be found to be bleeding from
the upper gastrointestinal tract. Radionuclide labeled red blood cells are effective in local-
izing gastrointestinal bleeding; however, the test takes hours to perform and should not
be used in a hemodynamically abnormal patient. CT angiography is now recognized as
the most appropriate initial study in the patient with ongoing hemorrhage. If the study is
negative, catheter-based angiography is likely to be of little use. Only rarely should these
patients be taken to the operating room without a diagnosis. In the patient with continued
massive bleeding without a diagnosis, rapid endoscopy can be performed at the time of
laparotomy in an attempt to localize the area of bleeding.
References
I. Laine L, Shah A. Randomized trial ofurgent vs. elective colonoscopy in patients hospi-
talized with lower GI bleeding. Am J Gastroenterol. 201O;105(12):2636-2641.
2. Jacovides CL, Nadolski G, Allen SR, et al. Arteriography for lower gastrointestinal
hemorrhage: role of preceding abdominal computed tomographic angiogram in diagno-
sis and localization. JAMA Surg. 2015;150(7):650-656.
3. Lightner AL, Russell MM. The evolving role of computed tomographic angiography for
lower gastrointestinal hemorrhage. JAMA Surg. 2015;150(7):657.
4. Davila RE, Rajan E, Adler DG, et al; Standards of Practice Committee. ASGE Guide-
line: the role of endoscopy in the patient with lower-GI bleeding. Gastrointest Endosc.
2005;62(5):656-{)60.
5. A Scottish Intercollegiate Guidelines Network (SIGN). Management of acute upper
and lower gastrointestinal bleeding. A national clinical guideline. Edinburgh, Scotland:
SlGN publication; 2008: 105.
CRITIQUE 19
All patients with rectal carcinoma should have a chest, abdominal, and pelvic CT scan. The
pelvis and rectal area should be evaluated with either endorectal ultrasound or MRI. PET
scanning or PET/CT is not indicated. In the absence of distant metastases or local inva-
sion on imaging, all patients with early rectal cancer should be evaluated for a potentially
curative transanal resection. Factors in favor of local excision are a tumor smaller than 3
cm; Tl status; grade 1 or 2 histology; no lymphatic, neural, or venous invasion, less than
30% involvement of bowel wall; a mobile, nonfixed lesion; proximity within 8 cm of the
anal verge; no evidence of suspicious adenopathy on pretreatment imaging; and negative
margins. Factors in favor of an abdominal-perinea} resection include size greater than 3
cm, grade 3 histology, lymphovascular invasion, or positive margins.
Transanal resection and transanal endoscopic microsurgery should involve excision of
the tumor into the perirectal fat. Margins are generally defined as positive if there is less
than 1 mm of normal tissue, although some authors suggest greater than 3 mm of margin.
CATEGORY 3 - PART !I 131
Answer: (D) 3-cm Tl tumor with 25% involvement of the bowel circumference.
References
I. Nash GM, Weiser MR, GuiUem JG, et al. Long-term survival after transanal excision of
Tl rectal cancer. Dis Colon Rectum. 2009;52(4):577-582.
2. National Comprehensive Cancer Network. NCCN Guidelines Rectal Cancer. Version
1.2016. Fort Washington, PA: National Comprehensive Cancer Network.
3. Liebig C, Ayala G, Wilks J, et al. Perineural invasion is an independent predictor of
outcome in colorectal cancer.JC/in Oncol. 2009;27(31):5131-5137.
4. Stitzenberg KB, Sanoff HK, Penn DC, Meyers MO, Tepper JE. Practice patterns and
long-term survival for early-stage rectal cancer.JC/in Oneal. 2013;31(34):4276-4282.
CRITIQUE20
Diabetic gastroparesis is an underdiagnosed complication of diabetes mellitus. It is thought
to be a presentation of diabetic neuropathy; therefore, standard procedures to slow the pro-
gression of diabetic neuropathy should be part of the management. Tight, stable glycemic
control is of the utmost importance. Workup of disease processes that may have similar
symptoms is necessary, because the diagnosis is often one of exclusion. Upper endoscopy
is used to rule out obstruction or active ulcer disease. Radiographic studies, both with con-
trast and radiolabeled meals, can demonstrate poor gastric emptying. Biliary tract disease
and pancreatic disease may also present with bloating and early satiety that could be con-
fused with diabetic gastroparesis.
Initial management should begin with dietary education and include the institution of
small, low-fat, frequent meals. In patients who fail dietary management, medical manage-
ment is often necessary and can involve agents such as erythromycin, metoclopramide,
domperidone (restricted use in the United States), and tegaserod (with US Food and Drug
Administration permission). Surgical treatment should be considered in patients who fail
medical therapy and can include a feeding jejunostomy or a gastric pacing device.
References
1. Camilleri M, Parkman HP, Shafi MA, Abell TL, Gerson L; American CoUege of Gastro-
enterology. Clinical guideline: management of gastroparesis. Am J Gastroenterol.
2013:108(1): 18-37.
2. Lacy BE, Weiser K. Gastric motility, gastroparesis~ and gastric stimulation. Surg Clin
North Am. 2005;85(5):967-987, vi-vii.
3. Olausson EA, Storsrud S, Grundin H, lsaksson M,AttvaU S, Simren M. A small particle
size diet reduces upper gastrointestinal symptoms in patients with diabetic gastropare-
sis: a randomized controlled trial. Am J Gastroenterol. 2014;109(3):375-385.
4. Guerci B, Bourgeois C, Bresler L, Scherrer ML, Bohme P. Gastric electrical stimulation
for the treatment of diabetic gastroparesis. Diabetes Metab. 2012;38(5):393-402.
5. Ross J, Masrur M, Gonzalez-Heredia R, EUi EF. Effectiveness of gastric neurostimula-
tion in patients with gastroparesis. JSLS. 2014;18(3).
132 CATEGORY 3 ~PART Il
CRITIQUE21
This patient presented should have only an antrectomy to prevent obstruction, because
metastatic disease is already present. Cholecystectomy should be performed at any pro-
cedure done for neuroendocrine tumors, because the patient is highly likely to receive a
long-term somatostatin analog (octreotide or lanreotide), which commonly produces bili-
ary stasis and cholecystitis. The other options are all inappropriate.
The liver metastases can be dealt with via thermal ablation or catheter-based embolism/
therapy. In selected patients, the hepatic metastasis can be surgically excised.
References
I. Yao JC, Hassan M, Phan A, et al. One hundred years after "carcinoid": epidemiology of
and prognostic factors for neuroendocrine tumors in 35,825 cases in the United States.
JC/in Oncol. 2008;26(18):3063-3072.
2. National Comprehensive Cancer Network. NCCN Guidelines Neuroendocrine Tumors.
Version 1.2015. Fort Washington, PA: National Comprehensive Cancer Network.
3. Hur S, Chung JW, Kim HC, et al. Survival outcomes and prognostic factors of trans-
catheter arterial chemoembolization for hepatic neuroendocrine metastases. J Vase
Interv Radio!. 2013;24(7):947-956.
4. Saxena A, Chua TC, Perera M, Chu F, Morris DL. Surgical resection of hepatic metasta-
ses from neuroendocrine neoplasms: a systematic review. Surg Oneal. 2012;2l(3):e131-
e I 4 l.
5. Landry CS, Brock G, Scoggins CR, McMasters KM, Martin RC 2nd. A proposed stag-
ing system for gastric carcinoid tumors based on an analysis of 1,543 patients. Ann Surg
Oneal. 2009;16(1):51--{;0.
CRITIQUE22
Esophageal perforation is associated with an extremely high mortality without treatment.
A delay in diagnosis of more than 24 hours is associated with a doubling of overall mortal-
ity from 14 to 27%. The diagnosis can be made by contrast study. Gastrografin should be
given initially. If negative, this should be followed by thin barium. If a CT scan is obtained
as the initial study and is negative, fluoroscopic esophagography is unnecessary.
Perforation of the gut is a rare but real complication associated with upper gastrointes-
tinal endoscopy. A recent study showed perforations occurred in 72 of217,507 esophago-
gastroduodenoscopy (EGD) procedures (incidence, 0.033%). The esophagus was injured
most commonly (51 %), followed by the duodenum (32%), jejunum (6%), stomach (3%),
and common bile duct (3%). Overall mortality after perforation was 17% with a morbid-
ity rate of 40%. Thirty-eight patients (49%) were initially treated nonoperatively; 7 (18%)
failed nonoperative management.
Mortality rates vary by the location of the perforation within the esophagus: cervical
(6%), thoracic (34%), and intra-abdominal (29%). The mortality rates also vary by etiol-
ogy: spontaneous (39%), iatrogenic (19%), and traumatic (9%). This is probably because
of the delay in diagnosis in patients having spontaneous and iatrogenic causes.
One study compared the use of stenting versus open surgical repair. Between 2009 and
2012, 60 patients undergoing esophageal stent placement or surgical repair were matched
by propensity scoring. Significant differences in morbidity (4% vs 43%; p ~ .02), mean
length of stay (6 vs 11 days;p ~ .0007), time to oral intake (3 vs 8 days;p ~ .0004), and cost
($91,000 vs $142,000;p <.0001) were identified in the esophageal stent cohort compared
with patients receiving surgical repair. Mortality was the same.
Stenting should be considered in all patients who have a perforation while undergoing
upper endoscopy. It is less expensive and has less morbidity than open repair. Stenting is
CATEGORY 3 - PART II 133
contraindicated in long tears (>6 cm) and in those with a delayed presentation (>24 hours)
resulting in substantial tissue necrosis. The stents are difficult to place in the proximal
esophagus (posterior pharynx) and in the distal esophagus/gastroesophageal junction (the
distal flair will not attach to the stomach itself). Stents may be especially useful in the pres-
ence of malignancy.
References
I. Lindenmann J, Matzi V, Neuboeck N, et al. Management of esophageal perforation in
120 consecutive patients: clinical impact ofa structured treatment algorithm. J Gastro-
intest Surg. 2013;17(6): I 036-1043.
2. Vallb6hmer D, Holscher AH, Holscher M, et al. Options in the management of esopha-
geal perforation: analysis over a 12-year period. Dis Esophagus. 2010;23(3): 185-190.
3. Eroglu A, Turkyilmaz A, Aydin Y, Yekeler E, Karaoglanoglu N. Current management of
esophageal perforation: 20 years experience. Dis Esophagus. 2009;22(4):374-380.
4. Wu CH, Chen CM, Chen CC, et al. Esophagography after pneumomediastinum with-
out CT findings of esophageal perforation: is it necessary? AJR Am J Roentgenol.
20 l 3;201 (5):977-984.
5. MercheaA, Cullinane DC, Sawyer MD, et al. Esophagogastroduodenoscopy-associated
gastrointestinal perforations: a single-center experience. Surgery. 20IO;148(4):876-
880; discussion 881-882.
6. Freeman RK, Ascioti AJ, Giannini T, Mahidhara RJ. Analysis of unsuccessful esopha-
geal stent placements for esophageal perforation, fistula, or anastomotic leak. Ann
Thorac Surg. 2012;94(3):959-964; discussion 964-965.
7. BrinsterCJ, Singhal S, Lee L, Marshall MB, Kaiser LR, KucharczukJC. Evolving options
in the management of esophageal perforation. Ann Thorac Surg. 2004;77(4): 1475-1483.
8. Freeman RK, Herrera A, Ascioti AJ, Dake M, Mahidhara RS. A propensity-matched
comparison of cost and outcomes after esophageal stent placement or primary
surgical repair for iatrogenic esophageal perforation. J Thorac Cardiovasc Surg.
2015;149(6): 1550-1555.
CRITIQUE23
Bleeding after colonoscopy is uncommon, ranging from 0. 1% to 0.6o/o. However, of all
the colonoscopic complications, bleeding is the most common. The risk increases when a
polypectomy is performed. Other risk factors include the size of the polyp removed, the
number of polyps removed, and the periprocedural use of anticoagulation by the patient.
The bleeding is often delayed, presenting several days after the procedure. Most bleeding
is self-limited and will stop without any specific intervention. However, the bleeding can
be substantial, and as many as half of patients may need a blood transfusion.
For patients who continue to bleed, repeat colonoscopy is the best therapeutic proce-
dure. In these cases, the site of bleeding is usually known, being the site of the previous
polypectomy. Therefore, localizing the site of bleeding, critical to management of patients
presenting with other types of lower gastrointestinal bleed, is typically unnecessary. A
rapid colonic cleanse can be performed, followed by a colonoscopy. Once the site is iden-
tified, the bleeding can be treated with thermal energy, epinephrine injections, clips, or a
combination of these therapies. Using these techniques, the bleeding is usually well con-
trolled. Rebleeding after intervention is uncommon. However, if the patient does rebleed
and remains hemodynamically normal, an additional attempt at therapeutic colonoscopy
is probably warranted. Surgery should be limited to those rare patients who have hemody-
namically significant bleeding, who continue to bleed, or who have another indication for
surgery, such as a colonic mass.
134 CATEGORY 3 - PART 11
Tagged red blood cell scans are primarily used to localize the source of gastrointestinal
bleeding, which is not necessary in this scenario. Angiography can also be done to localize
bleeding. Angiography can allow for embolization to treat gastrointestinal bleeding, but it
is invasive, carries the risk of colonic ischemia, and would not be the procedure of choice
in this situation.
References
1. Waye JD, Lewis BS, Yessayan S. Colonoscopy: a prospective report of complications.
J Cl in Gastroenterol. 1992; 15(4):347-35 L
2. Sawhney MS, Salfiti N, Nelson DB, Lederle FA, Bond JH. Risk factors for severe
delayed postpolypectomy bleeding. Endoscopy. 2008;40(2): 115-119.
3. Sonnenberg A. Management of delayed postpolypectomy bleeding: a decision analysis.
Am J Gastroenterol. 2012;107(3):339-342.
CRITIQUE24
Historically, diverticulitis was considered a progressive disease, with each attack being
more severe than the previous episodes. It seemed, therefore, that early surgical interven-
tion was warranted to prevent the complications associated with diverticulitis.
However, more recent population studies have provided a better understanding of the
natural history of diverticulitis, altering the guidelines for surgical intervention. Surgery
is indicated for complicated diverticulitis, including free perforation, symptomatic stric-
ture, and fistula formation. For uncomplicated diverticulitis, which frequently responds to
medical therapy, the indications for surgery are far less clear. Surgery is not recommended
after a single bout of uncomplicated diverticulitis. In addition, the previously accepted
guidelines for surgery after the second bout of uncomplicated diverticulitis are no longer
accepted. The current guidelines from the American Society of Colon and Rectal Surgeons
(ASCRS) read as follows: "The decision to recommend elective sigmoid colectomy after
recovery from uncomplicated acute diverticulitis should be individualized.'' This decision
will include an assessment of the operative risk, the severity of the attacks, the frequency
of the attacks, and the presence oflingering or "smoldering" symptoms.
These changing guidelines are particularly true in younger patients, who, historically,
were thought to have a more aggressive and virulent disease. Because of these beliefs,
surgery was routinely offered after the first attack for younger patients. More recently,
several studies suggested that the clinical course in younger patients is not more virulent.
Age is not an independent predictor of poor outcome or of disease recurrence. Therefore,
recommendations for surgery should be independent of age and based on other clinical
circumstances as described here. This younger man has a first course of uncomplicated
diverticulitis that has completely resolved. Therefore, surgery is not indicated.
There is evolving literature on the use of 5-aminosalicylic acid and probiotics for the
treatment of uncomplicated diverticulitis. However, the routine use of these compounds is
not yet fully recommended and should not be considered for routine maintenance therapy.
References
1. Feingold D, Steele SR, Lee S, et aL Practice parameters for the treatment of sigmoid
diverticulitis. Dis Colon Rectum. 2014;57(3):284-294.
2. Anaya DA, Flum DR. Risk of emergency colectomy and colostomy in patients with
diverticular disease. Arch Surg. 2005; 140(7):681-685.
CATEGORY 3 - PART II 135
CRITJQUE25
Lynch syndrome affects approximately 2-3o/o of patients with colon cancer. Identification
of these patients is critical to optimize surgical management. Historically, a family history
was the primary method of identifying patients at risk for Lynch syndrome. More recently,
universal testing has emerged as an alternative strategy to identify Lynch syndrome. One
effective strategy is to use immunohistochemistry as a screening tool. The tumor can be
tested for the absence of the mismatch repair proteins associated with Lynch syndrome. If
all the proteins are intact, the patient is unlikely to have Lynch syndrome. The absence of
any of these proteins suggests the possibility of Lynch syndrome, and more testing is war-
ranted, including genetic testing.
In addition to colon cancer, Lynch syndrome patients are at risk for several extracolonic
malignancies. The most common are endometrial cancer (risk about 40-50%) and ovarian
cancer (risk about 6-12%). For patients with known Lynch syndrome and a diagnosis of
colon cancer, the operation of choice is a total abdominal colectomy to reduce the risk of
metachronous lesions, which can approach 60% at 30 years. Extended resections, while
not eliminating the risk of a metachronous colon cancer, can substantially reduce the risk.
Conversely, the risk does not seem to justify a complete proctocolectomy, as is indicated
for familial adenomatous polyposis.
In this scenario, the patient has genetic testing indicating the absence of the MSH2 gene.
She therefore has Lynch syndrome. She is young and healthy and expected to Jive a long
time. To reduce the risk of developing a second colon cancer while maintaining adequate
bowel function, a total colectomy is optimal. To prevent both endometrial and ovarian
cancer in this post- or perimenopausal woman, a hysterectomy and bilateral salpingo-
oophorectomy should also be done.
References
I. Kalady MF, Church JM. Prophylactic colectomy: Rationale, indications, and approach.
J Surg Oneal. 2015; 111(1): 112-117.
2. Natarajan N, Watson P, Silva-Lopez E, Lynch HT. Comparison of extended colec-
tomy and limited resection in patients with Lynch syndrome. Dis Colon Rectum.
20 I 0;53(1 ):77-82.
3. Crispens MA. Endometrial and ovarian cancer in lynch syndrome. Clin Colon Rectal
Surg. 2012;25(2):97-102.
CRITIQUE26
This patient has a T2NOMO (stage I) upper rectal/rectosigmoid cancer. In general, neoadju-
vant chemoradiation is indicated for stage II and III mid and low rectal cancers (i.e., tumors
within 10 cm from the anal verge), so chemoradiation would not be indicated, and primary
surgery should be done. A total mesorectal excision (TME) is the standard operative tech-
nique for mid and low rectal cancers. TME ensures complete resection of the lymph node
basin for most mid and low rectal cancers and significantly reduces the rate of recurrent
disease. However, it is unusual to find metastatic tumor deposits within the mesorectum
more than 5 cm from the primary tumor. Therefore, a complete mesorectal excision is
136 CATEGORY 3 - PART II
References
1. Lopez-Kastner F, Lavery IC, Hool GR, Rybicki LA, Fazio VW. Total mesorectal exci-
sion is not necessary for cancers of the upper rectum. Surgery. 1998;124(4):612-617;
discussion 617-618.
2. Zaheer S, Pemberton JH, Farouk R, Dozois RR, Wolff BG, Ilstrup D. Surgical treatment
of adenocarcinoma of the rectum. Ann Surg. l 998;227(6):800-811.
CRITIQUE27
Gastrointestinal stromal tumors (GISTs) represent 1% of all gastrointestinal malignancies
and are the most common sarcoma of the gastrointestinal tract. These tumors arise from
the interstitial cells of Cajal, which are the neural pacemaker cells responsible for smooth
muscle activity in the gastrointestinal tract. This cell of origin explains the identification of
GISTs throughout the alimentary tract.
Operation is the treatment of choice for localized tumors, and the 5-year actuarial sur-
vival rate is 54% for a ROe resection. Recurrence of disease after resection usually involves
the original tumor site, peritoneum, and liver. Patient outcome is predicated on anatomic
site, size of the tumor, number of mitoses observed per high-powered microscopic field
(HPF), and molecular characteristics. Four size categories are used: less than or equal to 2
cm, 2-5 cm, 5-10 cm, and greater than 10 cm. Mitotic rate is less than or greater than 5/HPF.
Tumor size is segregated into the risk of recurrence after surgical resection. Risk of
recurrence is lowest for gastric tumors that are less than or equal to 2 cm with less than 5
mitoses/HPF. Conversely, patients with a 10 cm tumor with more than 5 mitoses/HPF in
the small intestine have a more than 90% chance of cancer progression.
Activating mutations of the c-KIT tyrosine kinase receptor are found in more than 60-
70% of GIST tumor cells. The most common mutations occur in exon II and exon 9.
Answer:
References
1. Halperin DM, Kulke MH, Yao JC. A tale of two tumors: treating pancreatic and extra-
pancreatic neuroendocrine tumors. Annu Rev Med. 2015;66: 1-16.
2. Miettinen M, Lasota J. Histopathology of gastrointestinal stromal tumor. J Surg Oneal.
2011; l 04(8):865-873.
3. Lasota J, Miettinen M. Clinical significance of oncogenic KIT and PDGFRA mutations
in gastrointestinal stromal tumours. Histopathology. 2008;53(3):245-266.
CRITIQUE28
The management of the malignant polyp is focused on 2 underlying principles. The first
is to ensure that the lesion was completely removed from the bowel wall. The second is to
assess the risk of associated cancer in the surrounding lymphatic tissue. If the lesion was
completely resected AND the risk of cancer in the associated lymph nodes is negligible,
radical surgery is not indicated.
The risk of lymph node metastases is related to the depth of invasion. The Haggitt clas-
sification is used to describe pedunculated polyps with a focus of invasive cancer. Level 1
is in the head of the polyp, level 2 into the neck of the polyp, level 3 into the stalk of the
polyp, and level 4 into the bowel wall below the stalk. The depth of the invasive disease is
within the submucosa for all of these levels. Sessile polyps, therefore, are all considered
Haggitt level 4 polyps. The risk of nodal metastases in Haggitt level 1, 2, and 3 polyps
without aggressive features is less than 1%. Under these circumstances, therefore, a radical
lymphadenectomy is unnecessary.
By contrast, Haggitt level 4 malignant polyps have positive lymph nodes in 12-25% of
cases, indicating the need for an oncologic resection of the colon. As originally described,
sessile polyps were all considered Haggitt level 4 polyps, mandating resection. More re-
cently, sessile polyps were divided into 3 categories, depending on the depth of invasion
(sml =superficial third ofsubmucosa, sm2 =middle third ofsubmucosa, sm3 =deep third
of the submucosa). This subclassification translates into obtaining a deep margin of at least
2 mm. Therefore, the risk oflymph node metastases in a sessile polyp removed with at least
a 2-mm deep margin is also negligible, and surgery would not be necessary.
In addition to depth of invasion, it is critical to assess other aggressive features that
increase the risk of lymph node metastases. These include poor differentiation, lympho-
vascular invasion, and tumor budding. If any of these aggressive features are present, the
risk of lymph node metastases increases to nearly 30%, independent of tumor depth of
invasion, and mandates resection.
In summary, not all polyps that contain microscopic cancer require a fonnal resection. If
the cancer does not have aggressive features (poor differentiation, lymphovascular invasion
or tumor budding) and has at least a 2-mm margin, observation is appropriate. This would
include early endoscopic visualization to ensure there is no recurrence in the bowel wall.
However, if any aggressive features are noted, the margin is less than 2 mm, or ifthe margin
is unclear (e.g., in a piecemeal resection), a formal surgical resection should be done.
In this scenario, the malignant polyp is confined to the head of the pedunculated polyp
(Haggitt level 1) and is completely excised. No other histologically aggressive features are
present; therefore, surgery is not indicated. An intraoperative colotomy would be inappro-
priate and unnecessary. Early endoscopic surveillance is important to ensure there is no re-
sidual lesion within the bowel wall. Therefore, colonoscopy at 3 months is the best answer.
References
1. Di Gregorio C, Bonetti LR, de Gaetani C, Pedroni M, Kaleci S, Ponz de Leon M. Clini-
cal outcome of low- and high-risk malignant colorectal polyps: results of a popula-
tion-based study and meta-analysis of the available literature. Intern Emerg Med.
20l4;9(2):151-160.
2. Ramirez M, Schierling S, Papaconstantinou HT, Thomas JS. Management of the malig-
nant polyp. Clin Colon Rectal Surg. 2008;21(4):286-290.
3. Hassan C, Quintero E, Dumonceau JM, et al; European Society of Gastrointestinal
Endoscopy. Post-polypectomy colonoscopy surveillance: European Society of Gastro-
intestinal Endoscopy (ESGE) Guideline. Endoscopy. 2013;45( 10):842-851.
CRITIQUE29
The patient was found incidentally to have multiple duodenal adenomas. There is a signifi-
cant association in patients with familial adenomatous polyposis (FAP) syndrome and the
development of duodenal adenomas. Over time, duodenal adenomas can be seen in 50-90%
of these patients. The risk oftransfopnation and cancerous changes is also time related.
More than 20 years ago, a classification system was developed to determine the appro-
priate management of these patients. The Spigelman scoring system is based on the number
of polyps, the polyp size, the histological type, and the severity of dysplasia (see table
29.1). Using this approach guides further evaluation and management in these patients.
The patient cited here needs repeat endoscopy with biopsies of all the lesions, looking
for higher-grade disease. The specific number of polyps, their size, histologic typing, and
dysplasia will determine the need for subsequent intervention.
Number of Points
1 2 3
Number of polyps 1--4 5-20 >20
Polyp size (mm) 1--4 5-10 >10
Histology Tubulous Tubulovillous Villous
Dysplasia Mild Moderate Severe
Stage Spigelman score*
0 0
I 1--4
III 5-6
III 7-8
IV 9-12
"'Addition of points.
Approximately 80-85% of patients fall into Spiegelman class 0-3; the remainder fall
into class 4. Surveillance endoscopy is appropriate for early disease: every 5 years for class
0 and I, every 3 years for class 2, and every 1-2 years for class 3. There is no indication for
a Whipple procedure without Spiegelman class 4 disease. Pancreas-sparing duodenectomy
is not an effective means of managing this disease when it progresses to needing surgical
CATEGORY 3 - PART I! 139
resection. There is no indication for endoscopic ultrasound in this patient. Repeat endos-
copy in 12 months would be appropriate after appropriate biopsies are done and specific
characterization of the disease and if it is Spiegelman classification stage 3 disease.
Answer: (A) Repeat endoscopy with biopsy looking for higher-grade disease
References
1. Vasen HF, M6slein G, Alonso A, et al. Guidelines for the clinical management of famil-
ial adenomatous polyposis (FAP). Gut. 2008;57(5):704--713.
2. Drini M, Speer A, Dow C, CollierN, Bhathal P, Macrae FA. Management of duodenal
adenomatosis in FAP: single centre experience. Fam Cancer. 2012;11(2): 167-173.
3. Spigelman AD, Williams CB, Talbot IC, Domizio P, Phillips RK. Upper gastrointestinal
cancer in patients with familial adenomatous polyposis. Lancet. l 989;2(8666):783-785.
4. Jagelman DG, DeCosse JJ, Bussey HJ. Upper gastrointestinal cancer in familial adeno-
matous polyposis. Lancet. 1988; 1(8595):1149-1151.
CRITIQUE30
Gastric polyps are found in 6% of upper endoscopies; more than 90% are asymptomatic.
Most gastric polyps are benign and have a similar endoscopic appearance, so classification
depends primarily on histologic results from biopsy.
Gastric adenomatous polyps are precursors to malignancy. While most common in ar-
eas with endemically high rates of gastric cancer (East Asia), they can account for up
to 10% of all gastric polyps in Western populations. Histologically, they are classified
in a similar method to colon polyps with tubular, villous, and tubulovillous distinctions.
They are most commonly located in the antrum, but they can be located anywhere in the
stomach. Polyps greater than 2 cm in size with villous histology have the highest risk of
neoplasia (28-40%). No association with Helicobacter pylori infection is documented,
although atrophic gastritis and intestinal metaplasia are frequently associated with these
polyps. The presence of high-grade dysplasia is associated with an increased risk of cancer
both within the polyp and in other areas of the stomach. Recommendations for treatment
include complete endoscopic excision of the adenoma and examination of the remaining
gastric mucosa for abnormalities, which should also be biopsied if abnormal. Additional
endoscopic sunreillance is recommended at 6-12 months. Operative resection should be
considered for polyps that cannot be excised endoscopically.
Fundic gland polyps (FGPs) are the most common gastric polyp detected in Western
countries. FGPs may occur sporadically, in association with proton pump inhibitor (PP!)
use, or in patients with familial adenomatous polyposis (FAP) syndrome. Sporadic FGPs
are small (<l cm) and typically located in the body or fundus. Patients on long-term PP!
therapy (>5 years) have a 4-times higher prevalence of FGP. In addition, withdrawal of
PP! therapy leads to a regression of FGP. FGPs are common in patients with FAP. Unlike
the sporadic FGP, those associated with FAP often carpet the body of the stomach and can
show low-grade dysplasia. Workup for FAP should be considered in any young patient with
multiple FGPs noted on endoscopy. Malignant potential for all FGPs is low.
Hyperplastic polyps are associated with H. pylori infection and chronic gastritis. They
are most frequently located in the antrum and are often multiple. Eradication of H. pylori
results in regression of hyperplastic polyps in up to 70% of patients. The clinical course
of gastric hyperplastic polyps should not be confused with that of the colonic hyperplastic
polyp, despite having a similar nomenclature. The overall prevalence of dysplasia is less
than 2o/o and is most common in those polyps larger than 2 cm. Treatment of these large
polyps is eradication of H. pylori and endoscopic excision of any remaining polyps. A re-
peat endoscopy at I year can be considered .
.,----------~---·------·'
140 CATEGORY 3 - PART 11
Inflammatory fibroid polyps (IFPs), also known as Vanek tumors, comprise less than I%
of all gastric polyps. IFPs are typically small (<1.5 cm), solitary lesions found in the antral
and pyloric areas of the stomach. They rarely cause symptoms. They have no malignant po-
tential, and no endoscopic follow-up is recommended after initial histologic confirmation.
Ectopic pancreas is most commonly located in the stomach and has no anatomic or
vascular connection with the main body of the pancreas. These lesions are small and found
incidentally. They have extremely low malignant potential.
References
1. Islam RS, Patel NC, Lam-Himlin D, Nguyen CC. Gastric polyps: a review of clini-
cal, endoscopic, and histopathologic features and management decisions. Gastroenterol
Hepatol (N l'.). 2013;9(10):640-65 I.
2. Carmack SW, Genta RM, Graham DY, Lauwers GY. Management of gastric polyps:
a pathology-based guide for gastroenterologists. Nat Rev Gastroenterol Hepatol.
2009;6(6):331-341.
CRITIQUE31
The decision to use adjuvant imatinib after resection of a gastrointestinal stromal tumor
(GIST) is based on estimates of recurrence. Tumors greater than 3-5 cm in size, with a
mitotic rate at least 5/50 high-powered fields, or located in areas other than the stomach
are associated with the highest risk of recurrence. This patient would be a candidate for
adjuvant imatinib based on his mitotic rate of 15/50 high-powered fields. In addition, some
studies indicate benefit from adjuvant therapy in GISTs of at least 3 cm.
Although resection to negative margins (RO resection) was considered optimal treat-
ment for a GISTs in the past, recent studies have demonstrated no difference in recurrence-
free survival for those resected to microscopically positive margins (RI resection). No data
suggest that patients who have an Rl resection benefit from re-excision to clear margins.
Observation would not be appropriate in this patient, who is at high risk for recurrence.
Radiation therapy and octreotide are not a part of the treatment strategy for GISTs.
References
1. Raut CP, Pawlik TM. Gastrointestinal stromal tumors. In: Cameron JL, Cameron AM, eds.
Current Surgical Therapy. 11th ed. Philadelphia, PA: Elsevier Saunders;2014:96-l 03.
2. Dematteo RP, Ballman KV, Antonescu CR, et al; American College of Surgeons Oncol-
ogy Group (ACOSOG) Intergroup Adjuvant GIST Study Team. Adjuvant imatinib
mesylate after resection of localised, primary gastrointestinal stromal tumour: a
randomised, double-blind, placebo-controlled trial [published correction appears in
Lancet. 2009;374(9688):450]. Lancet. 2009;373(9669): 1097-1104.
3. McCarter MD, Antonescu CR, Ballman KV, et al; American College of Surgeons Oncol-
ogy Group (ACOSOG) Intergroup Adjuvant Gist Study Team. Microscopically positive
margins for primary gastrointestinal stromal tumors: analysis of risk factors and tumor
recurrence. J Am Coll Surg. 20I2;2I5(I):53-59; discussion 59-60.
CRITIQUE32
This patient has a malignant large bowel obstruction (MLBO). The plain films demonstrate
a distended colon with a cutoff in the sigmoid region. The CT scan with rectal contrast re-
veals the typical "apple core" lesion ofMLBO (figure 32.4). Malignancy by far is the most
common cause of large bowel obstruction.
CATEGORY 3 ~PART II
141
Figure 32.4 CT scan with rectal contrast showing the typical "apple
core' lesion of a malignant large bowel obstruction.
The traditional approach to this patient with unprepped and dilated bowel would be seg-
mental colectomy with end colostomy. Elective colostomy closure would follow months
later. Patients who undergo emergency laparotomy for MLBO have significantly higher
rates of morbidity and mortality, because they tend to be of advanced age and have as-
sociated comorbidities, as is the case with this patient. Furthermore, this patient's inde-
pendence is threatened due to partial amputation of his dominant hand, making colostomy
management more difficult, if not impossible.
Self-expanding metal stents (SEMS) are used successfully in MLBO as a bridge to sur-
gery, allowing decompression of the colon, optimization of the patient's medical condition,
proper bowel preparation, and subsequent 1-stage colectomy with primary anastomosis.
Large randomized studies comparing emergency colectomy and colostomy to SEMS with
primary anastomosis are lacking; however, multiple reports support the safety of SEMS
in the properly selected patient. Immediate technical success rates exceed 90%. The need
for multiple stents is a risk for failure. The ideal patient will have a focal, distal, left-sided
lesion. The SEMS is deployed through the operating channel of the colonoscope with the
aid of fluoroscopy. The stent should not be balloon dilated, because this may increase the
risk of colonic perforation. Generally, the stent will open gradually over the ensuing 12-48
hours after placement (figure 32.5). Surgical resection should be planned as soon as the
patient's medical condition is optimized and the bowel is properly prepped, because there
is a significant incidence of stent obstruction or migration (figure 32.6). SEMS-related
perforation of the colon is more likely as the dwell time of the stent increases.
Use ofSEMS for definitive palliative treatment of malignant colonic obstruction should
be approached with caution, because complications such as obstruction, perforation, and
stent migration are significant.
In this patient, cecostomy or colostomy would be very difficult for the patient to manage
and likely could be avoided with SEMS placement. Emergency laparotomy and sigmoid
resection with primary anastomosis in dilated, unprepped colon would be expected to have
an unacceptably high complication rate.
References
1. Ghazal AH, El-Shazly WG, Bessa SS, El-Riwini MT, Hussein AM. Colonic endolu-
minal stenti ng devices and elective surgery versus emergency subtotal/total colectomy
in the management of malignant obstructed left colon carcinoma. J Gastrointest Surg.
2013; 17(6): 1123-1129.
2. Cheung DY, Lee YK, Yang CH. Status and literature review of self-expandable metallic
stents for malignant colorectal obstruction. Clin Endosc. 20 14;47(1):65- 73.
3. van den Berg MW, Ledeboer M, Dijkgraaf MG, Fockens P, ter Borg F, van Hooft JE.
Long-term results of palliative stent placement for acute malignant colonic obstruction.
Surg Endosc. 20 15;29(6): 15 80-1 585.
4. Quereshy FA, Poon JT, Law WL. Long-term outcome ofstenting as a bridge to surgery
for acute left-sided malignant colonic obstruction. Colorectal Dis. 2014(10):788- 793.
CATEGORY 3 - PART !l
143
5. van den Berg MW, Sloothaak DA, DijkgraafMG, et al. Bridge-to-surgery stent place-
ment versus emergency surgery for acute malignant colonic obstruction. Br J Surg.
2014; 101 (7):867-873.
6. Kim JH, Kwon KA, Lee JJ, et al. Surgical failure after colonic stenting as a bridge to
surgery. World J Gastroenterol. 20l4;20(33):11826--11834.
7. van Hoof! JE, van Halsema EE, Vanbiervliet G, et al; European Society of Gastrointes-
tinal Endoscopy. Self-expandable metal stents for obstructing colonic and extracolonic
cancer: European Society of Gastrointestinal Endoscopy (ESGE) clinical guideline.
Endoscopy. 2014;46(11):990-1053.
CRITIQUE33
Paraesophageal hernia repair remains a formidable challenge. Due to the very high rates
of radiologically identifiable hiatal hernia recurrence after primary crural repair, mesh re-
inforcement is used. It is now clear that synthetic mesh should not be used in this location
due to the unacceptably high rates of esophageal erosion. Laparoscopic primary crural
repair with biological mesh buttress is very safe, with excellent short-term results. How-
ever, 5-year follow-up of these patients shows a disappointingly high rate of radiologically
apparent hiatal hernia, although the symptomatic results continue to be excellent. It is pos-
sible that the long-term results will improve with newer biologic or hybrid mesh materials.
Complete reduction of the hernia sac is a critical step in paraesophageal hernia repair.
Excision of the sac is generally recommended to the extent that it does not risk vagal nerve
injury. The crura nearly always can be reapproximated adequately to close the hernia de-
fect, although a relaxing incision adjacent to the crura may occasionally be necessary. A
crural repair using mesh interposition is to be avoided because of concerns of esophageal
erosion. An onlay buttressing of the crural repair using a horseshoe-shaped biologic mesh
appears to be the best option currently available.
Because the majority of these patients have gastroesophageal reflux and the creation of
a 360° fundoplication helps maintain the stomach in the abdominal compartment below
the crural repair, a Nissen fundoplication is typically performed for all of these patients.
A transthoracic approach is rarely necessary to properly repair even the largest of para-
esophageal hernias.
References
I. Oelschlager BK, Pellegrini CA, Hunter JG, et al. Biologic prosthesis to prevent recur-
rence after laparoscopic paraesophageal hernia repair: long-term follow-up from a
multicenter, prospective, randomized trial [published correction appears in J Am Coll
Surg. 2011;213(6):815]. J Am Coll Surg. 2011;213(4):461--468.
2. Kubasiak J, Hood KC, Daly S, et al. Improved patient outcomes in paraesophageal
hernia repair using a laparoscopic approach: a study of the national surgical quality
improvement program data. Am Surg. 2014;80(9):884--889.
3. DeMeester SR. Laparoscopic paraesophageal hernia repair: critical steps and
adjunct techniques to minimize recurrence. Surg Laparosc Endosc Percutan Tech.
20 l 3;23(5):429--435.
4. Zehetner J, Demeester SR, Ayazi S, et al. Laparoscopic versus open repair of para-
esophageal hernia: the second decade. J Am Coll Surg. 2011 ;212(5):813-820.
5. Antoniou SA, Antoniou GA, Koch 00, Pointner R, Granderath FA. Lower recur-
rence rates after mesh-reinforced versus simple hiatal hernia repair: a meta-analysis of
randomized trials. Surg Laparosc Endosc Percutan Tech. 2012;22(6):498-502.
6. Stadlhuber RJ, Sherif AE, Mittal SK, et al. Mesh complications after prosthetic rein-
forcement ofhiatal closure: a 28-case series. Surg Endosc. 2009;23(6):1219-1226.
144 CATEGORY 3 - PART II
CRITIQUE34
Esophageal perforations and anastomotic leaks are life-threatening complications and at-
tempt at repair has an operative mortality rate of 12-50%. Morbidity and mortality increase
significantly with delay in diagnosis and treatment beyond 12-24 hours. Location of per-
foration and degree of ongoing contamination determine the mode of treatment. The devel-
opment of removable covered esophageal stents has helped reduce morbidity and mortality
in treating these challenging complications.
Principles in management of esophageal perforation routinely include intravenous an-
tibiotics, nothing by mouth (NPO), securing the area of leak, wide drainage if ongoing
contamination is evident, and providing nutrition. Drainage techniques may include per-
cutaneous, thoracoscopic, or open interventions. Limited cervical esophageal perforations
frequently are treated successfully using conservative management with NPO and intrave-
nous antibiotics only. Blind placement of a nasogastric tube is discouraged.
This elderly patient had a rapid diagnosis and evaluation with a Gastrografin esophago-
gram revealing a contained leak in the mid esophagus. No drainage procedure is warranted
in this hemodynamically stable patient without signs of sepsis or mediastinal contamina-
tion. Placement of a removable stent for esophageal perforation has a reported technical
success rate of 90%. Stent placement combined with bowel rest, intravenous antibiotics,
and nutrition carries the lowest morbidity and mortality of the choices offered. Enteral
nutrition is preferred via a nasoenteric or gastric tube. If this is not possible, parenteral
nutrition can be provided. Stent removal typically occurs 8 weeks after placement.
References
1. Dasari BV, Neely D, Kennedy A, et al. The role of esophageal stents in the manage-
ment of esophageal anastomotic leaks and benign esophageal perforations. Ann Surg.
20 l 4;259(5):852-860.
2. Salminen P, Gullichsen R, Laine S. Use of self-expandable metal stents for the treatment
of esophageal perforations and anastomotic leaks. Surg Endosc. 2009;23(7): 1526-1530.
CRITIQUE35
The exact definition of a "slipped Nissen fundoplication" is elusive. However, this term
is used by surgeons and gastroenterologists to describe a fundoplication that has migrated
proximally onto the esophagus or more commonly distally onto the body of the stomach,
such that a gastric pouch exists above the fundoplication. The cardinal symptom is severe
dysphagia. If the gastric body rather than the fundus was used to construct the original
fundoplication, this will also give the appearance of a slipped fundoplication on barium
study. Esophageal dilation in the early postoperative period may increase the incidence
of this complication. Many times, the slipped fundoplication is not appreciated on upper
gastrointestinal endoscopy, and the fluoroscopic findings can be subtle.
Esophageal manometry after successful fundoplication will reveal mildly elevated rest-
ing and basal lower esophageal sphincter pressures; however, the sphincter will relax ad-
equately with the swallow. This patient has increased sphincter length and pressure, which
does not relax properly because of the fixed constriction of the slipped wrap. The pH study
is abnormal because of acid production in the stomach above the wrap. The barium image
shows a segment of dilated stomach above the fundoplication, although rugal folds are not
obvious (figure 35.2). The x-ray does not reveal a paraesophageal hernia as a cause for the
marked dysphagia in this patient. The x-ray also demonstrates an intact fundoplication. The
manometry results and barium image are not consistent with achalasia or pseudoachalasia,
both of which would typically demonstrate a presbyesophagus with poor or absent peristal-
sis within the esophageal body.
CATEGORY 3 - PART II 145
References
1. Wo JM, Trus TL, Richardson WS, et al. Evaluation and management of postfundoplica-
tion dysphagia. Am J Gastroentero/.1996;9 l (11 ):2318-2322.
2. Hoshino M, Srinivasan A, Mittal SK. High-resolution manometry patterns of lower
esophageal sphincter complex in symptomatic post~fundoplication patients. J Gastroin-
test Surg. 2012;16(4):705-714.
3. Carbo AI, Kim RH, Gates T, D'Agostino HR. Imaging findings of successful and failed .., :;.I".!
fundoplication. Radiographies. 20l4;34(7):1873-1884.
CRITIQUE36
-
"..,~ lrO""
.......,
...,
"'
Gastric carcinoid tumors develop from the enterochromaffin-like cells of the gastric fundus
mucosa. They are rare lesions, accounting for less than 1Oo/o of all gastrointestinal carci-
noids. Typically, they are classified into 3 subgroups. Type I is the most common and found
in the setting of chronic hypergastrinemia as a result of chronic atrophic gastritis or perni-
cious anemia. Type I lesions are rarely malignant, occur in women in 60-70% of cases,
and are usually small (<l cm) and multicentric. Endoscopic polypectomy is the surgical
treatment of choice for type I gastric carcinoids.
Type II gastric carcinoids are associated with gastrin producing neoplasms as seen in
Zollinger-Ellison syndrome. Type III lesions are composed of poorly differentiated neuro-
endocrine cells and carry a high rate of malignancy. Gastric resection is typically indicated
for these 2 subtypes.
References
I. Borch K, Ahren B, Ahlman H, Falkmer S, Granerus G, Grimelius L. Gastric carcinoids:
biologic behavior and prognosis after differentiated treatment in relation to type. Ann
Surg. 2005;242(1):64-73.
2. Dakin GF, Warner RR, Pomp A, Salk-y B, Inabnet WB. Presentation, treatment, and
outcome of type I gastric carcinoid tumors. J Surg Oneal. 2006;93(5):368-372.
CRITIQUE37
The management of enterocutaneous fistula (ECF) from Crohn disease is challenging due
to the puzzling disease process, frequent use of immunosuppressant therapy, and associ-
ated malnutrition. Approximately 20-30% of all ECFs result from Crohn disease. ECFs
in Crohn disease can arise spontaneously, as in this patient, or can occur after intestinal
surgery. This patient has a type I fistula that is not associated with active disease. A type 2
fistula is more complex and associated with intra-abdominal abscesses. A type I fistula is
more likely to close with conservative management than a type 2 fistula.
Infliximab, Omega-3 fatty acids, and octreotide do not decrease the need for operation
in patients with Crohn disease and ECF.
Complete resection of the ECF as well as the diseased bowel is necessary to prevent
recurrence. Wedge resection and oversewing the fistula should not be done unless the pa-
tient already has short bowel syndrome and cannot afford to lose more functional intestine.
In this patient without active disease and with poor nutritional parameters, a trial of
parenteral nutrition and bowel rest for up to 6 weeks will allow the best opportunity for
spontaneous closure of the ECF.
References
I. Schecter WP, Hirshberg A, Chang DS, et al. Enteric fistulas: principles of management.
J Am Coll Surg. 2009;209(4):484-491.
2. Mcintyre PB, Ritchie JK, Hawley PR, Bartram CI, Lennard-Jones JE. Management of
enterocutaneous fistulas: a review of 132 cases. Br J Surg. 1984;71 (4):293-296.
CRITIQUE38
Patients with either a visible vessel or an adherent clot represent a patient group with a high
risk for rebleeding. A visible vessel is a discrete vessel usually 4 mm or less in diameter as-
sociated with a re bleeding rate of 50% without therapy. An adherent clot is red in appearance,
at least 6 mm in diameter, and associated with a 35% rate ofrebleeding without therapy.
High-dose proton pump inhibitors (PP!s) are considered the standard medical treatment
for a patient with acute upper gastrointestinal bleeding. Continuous intravenous PPls are
given as a bolus infusion, followed by a continuous infusion for 72 hours. A recent com-
parison of continuous and intermittent PPI use in patients with a visible vessel or adherent
clot treated with endoscopic therapy found rebleeding rates to be no different whether
the PPis were continuously infused or given intermittently. These authors suggested that
intermittent PP! therapy would be less costly. Hz blockers are associated with pharmaco-
logic tolerance, which reduces their acid-suppressing ability. H2 blockers are not currently
recommended in these patients. Combining PPis and H2 blockers would not be required
based on current recommendations. Misoprostol is most effective for acute gastric injury
rather than peptic ulcer bleeding.
When endoscopic therapy is combined with PP ls, the rates of rebleeding are much low-
er. In a randomized trial of medical or endoscopic therapy to prevent recurrent ulcer hemor-
rhage in high-risk patients, rebleeding occurred in 35o/o of patients treated with PPis and in
CATEGORY 3 - PART II 147
References
I. Leontiadis GI, Shanna VK. Howden CW. Systematic review and meta-analysis of
proton pump inhibitor therapy in peptic ulcer bleeding. BMJ. 2005;330(7491):568.
2. Jensen DM, Kovacs TO, Jutabha R, et al. Randomized trial of medical or endoscopic
therapy to prevent recurrent ulcer hemorrhage in patients with adherent clots. Gastroen-
terology. 2002;123(2):407-413.
CRITIQUE39
Marginal ulcers occur in approximately 30% of patients after gastric bypass. Symptoms
most commonly seen with marginal ulcers include pain, dysphagia, nausea, and vomiting.
Eighty percent of patients will present with one or all of these symptoms. Bleeding is seen
in approximately 3-15% of patients. Almost half of all patients who are identified with
marginal ulcers are now diagnosed more than 12 months after their operation. This finding
contrasts with earlier studies when early endoscopy was more common, and identified ul-
cers may actually have been part of the normal healing process of the operation. Addition-
ally, most patients are currently maintained on proton pump inhibitor therapy (PPl) around
their procedure, which may contribute to the lower early ulcer rate.
Ulcer etiology can be related to anatomy or to exposure. Anatomy may include ischemia
generated from the operation, tension on the anastomosis, or gastric outlet restriction. A
larger gastric pouch (more than 6 cm long and 5 cm wide) is also associated with an in-
creased marginal ulcer rate. The type of suture used at the time of surgery is not related to
the development of marginal ulcers.
Ulcers diagnosed later tend to be related to tobacco use and exposure of the pouch
to caustic agents like gastric acid, nonsteroidal anti-inflammatory drugs (NSAIDs), and C"l
alcohol. Gastric acid exposure can occur with the development of a gastrogastric fistula, .... ::;.
and, although rare (2o/o incidence), it does need to be considered. Tobacco use is strongly .,... """
.... ..,
........
~o
associated not only with marginal ulcers (50%) but also with recalcitrant marginal ulcers
(60%). In fact, ongoing tobacco use is significantly associated with recalcitrant ulcers. He-
licobacter pylori is seen with marginal ulcers but not at the level seen with standard peptic "'
ulcer disease. Reported rates are approximately 15%.
Treatment of marginal ulcers includes ruling out anatomic causes for ulceration. Patients
should undergo an esophagogastroduodenoscopy with assessment of the gastric outlet and
to evaluate for a gastrogastric fistula. Ulcers should be biopsied to rule out cancer and to
assess for H. pylori. If patients are smoking or drinking alcohol, they need to quit. NSA!Ds
or steroids need to be discontinued, if possible. If H. pylori is present, treatment is directed
at eradication. All patients should be initiated on a PPL The vast majority of patients will
have resolution of their symptoms with this program. Ifthere are still symptoms after 8-12
weeks, risk factors need to be readdressed, and repeat endoscopy should be considered.
References
I. El-Hayek K, Timratana P, Shimizu H, Chand B. Marginal ulcer after Roux-en-Y gastric
bypass: what have we really learned? Surg Endosc. 2012;26(10):2789-2796.
148 CATEGORY 3 - PART ll
2. Ying VW, Kim SH, Khan KJ, et al. Prophylactic PP! help reduce marginal ulcers after
gastric bypass surgery: a systematic review and meta-analysis of cohort studies. Surg
Endosc. 2015;29(5):1018-1023.
3. Coblijn UK, Goucham AB, Lagarde SM, Kuiken SD, van Wagensveld BA. Develop-
ment of ulcer disease after Roux-en-Y gastric bypass, incidence, risk factors, and patient
presentation: a systematic review. Obes Surg. 2014;24(2):299-309.
CRITIQUE40
Mucosa-associated lymphoid tissue (MALT) lymphoma is a B-cell lymphoma that most
commonly presents as an indolent disease with vague dyspepsia or mild epigastric pain.
The classic B symptoms of lymphoma (fever, night sweats, and weight loss) are extremely
rare. Bone marrow involvement occurs in only 15o/o of cases. Eradication of Helicobacter
pylori results in complete remission of lymphoma in most patients. Surgical resection is
rarely required.
References
1. Zullo A, Hassan C, Ridola L, Repici A, Manta R, Andriani A. Gastric MALT lympho-
ma: old and new insights. Ann Gastroenterol. 2014;27(1):27-33.
2. Nakamura S, Sugiyama T, Matsumoto T, et al; JAPAN GAST Study Group. Long-term
clinical outcome of gastric MALT lymphoma after eradication ofHelicobacter pylori: a
multicentre cohort follow-up study of 420 patients in Japan. Gut. 2012;61(4):507-513.
Category 3 - Part III
Alimentary Tract
Items 1-39
DIRECTIONS: Each of the questions or incomplete statements is followed by 5 suggested
answers or completions. Select the answer that is the BEST in each case and fill in the
space containing the corresponding letter on the answer sheet.
1. A 62-year-old man with head and neck cancer undergoes an uneventful pretreatment
percutaneous endoscopic gastrostomy (PEG) placement. Postprocedure day 4, the
PEG is inadvertently removed. You are notified 12 hours later. The patient is comfort-
able, without evidence for peritoneal irritation, fever, or discomfort. The best method
for replacement of the PEG is to replace it
(A) endoscopically through the original site.
(B) laparoscopically through a new site.
(C) at the bedside without endoscopy
(D) with a nasogastric tube and begin broad-spectrum antibiotics with a plan for
repeat PEG placement in 7-10 days.
(E) via laparotomy.
2. A 56-year-old man who is other.vise fit is seen 7 years after abdominoperineal resec-
tion for a low rectal cancer with a symptomatic and enlarging left lower quadrant
parastomal hernia. In the past 12 months, he was hospitalized 3 times for small bowel
obstruction related to his hernia. Each time, the obstruction has resolved with naso-
gastric tube decompression and bowel rest. There is no evidence of recurrent disease.
Which of the following is the best method of management for this problem?
(A) Relocation of the stoma to the right lower quadrant
(B) Primary suture repair of the hernia
(C) Surgical repair with PTFE
(D) Surgical repair with polypropylene
(E) Continued observation
3, A 25-year-old man presents for follow-up after a second episode of bleeding from a
duodenal bulb ulcer that was successfully managed by endoscopic injection and cauter-
ization 1 month ago. He has symptoms of nausea, nonbilious vomiting, and weight loss.
You perform upper endoscopy and note evidence of a stricture at the duodenal bulb.
Which of the following is the most appropriate next step in management of this patient?
(A) Gastric emptying study
(B) CT scan to evaluate for gastrinoma
(C) Roux-en-Y gastrojejunostomy
(D) Endoscopic balloon dilation
(E) Endoscopic stent placement
7. A 40-year-old woman presents with anemia. Upper endoscopy shows a smooth 4-cm
submucosal mass in the antrum of the stomach. Endoscopic biopsy reveals gastric
mucosa. An abdominal CT scan is shown in figure 7.1. No other abnormalities are
noted on the imaging other than the gastric mass. The next most appropriate step in
management is
(A) neoadjuvant 5-fiuorouracil-based chemotherapy.
(B) radiation.
(C) imatinib.
(D) endoscopic resection.
(E) excision of the primary tumor.
Figure 7.1
152 CATEGORY 3 - PART lll
9. A 19-year-old man presented 48 hours ago with complaints of bright red blood per rec-
tum. An esophagogastroduodenoscopy demonstrated a bleeding ulcer in the posterior
duodenal bulb with an overlying clot, which was controlled via endoscopic cauteriza-
tion. He remains hemodynamically normal, with no evidence of ongoing bleeding. He
has no prior medical history and takes no medications. He remains on a proton pump
inhibitor started on admission to the hospital. Initial Helicobacter pylori rapid urease
test was negative. Which of the following is most appropriate next step in management
of this patient?
(A) BC-urea breath test
(B) I-month course of medical acid suppression
(C) Elective acid suppression surgery
(D) Gastrin level
(E) Second-look endoscopy
10. A 50-year-old woman presents with nausea, vomiting, abdominal pain, and bloating. She
has a history of an open appendectomy and is otherwise healthy with no current medi-
cal problems. Abdominal exam reveals a distended abdomen with absent bowel sounds
and mild diffuse tenderness without peritonitis. A CT scan of the abdomen demonstrates
evidence of a small bowel obstruction with a transition point and fecalization in the small
bowel proximal to the transition. Her last bowel activity was 24 hours ago. Which of the
following statements is true regarding the management of this patient?
(A) Nonoperative therapy should be attempted for no more than 12 hours.
(B) Laparoscopic exploration is contraindicated.
(C) Capsule endoscopy is indicated to identify the source of obstruction.
(D) A water-soluble contrast study is useful after initial decompression.
(E) A benign small bowel tumor is a common underlying cause.
13. Which of the following statements is true regarding narcotic bowel syndrome (NBS)?
(A) NBS is a dose-related dysmotility disorder.
(B) NBS patients typically have constipated stools.
(C) NBS is not associated with abdominal pain.
(D) Continued use of high-dose narcotics exacerbates NBS.
(E) Less than 20% of patients detoxified from narcotics in a well-structured medical
program are back on narcotics by 3 months.
14. An obese 36-year-old man presents with a rectal foreign body. The patient has no peri-
toneal signs. On plain x-ray imaging, the object appears to be a glass jar (figure 14. l ).
An attempt at removal after administering fentanyl and midazolam is unsuccessful.
The patient relates no prior surgery and is otherwise healthy. Vital signs are normal
and the patient is afebrile. On examination, the jar is palpable high in the rectum on
rectal examination. There is no free air on plain x-rays. Which of the following is the
next best step?
(A) Extraction under deep sedation and perianal local anesthesia
(B) Air insuftlation to expel the foreign body
(C) Laparotomy with extraction via pushing from above and pulling through an
anoscope
(D) Repeated enemas
(E) Removal with obstetric forceps
Figure 14.1
Copyright ©2016. Case courtesy of Frank Gaillard,
Radiopaedia.org. rID: 3284 (http://radiopaedia.org).
""'1
154 CATEGORY 3 - PART Ill
15. A 54-year-old woman presents with 4 days of abdominal pain, nausea, and vomiting.
On exam, she has peritonitis. Plain abdominal x-ray is consistent with a small bowel
obstruction. At laparotomy, no bowel ischemia is present. A 1.5-cm ilea! mass with
fibrosis of the mesentery is causing a partial bowel obstruction. The next step in the
patient's management should be
(A) enucleation of the tumor.
(B) en bloc resection including mesentery.
(C) closure of the abdomen and medical treatment with somatostatin.
(D) biopsy of the tumor and placement of distal feeding access.
(E) small bowel resection including the mass.
17. A 65-year-old woman with persistent heartburn and iron deficiency anemia, who is on
maximal proton pump inhibitor therapy and iron supplementation, undergoes upper
endoscopy for gastrointestinal bleeding that shows gastric linear mucosal erosions
(Cameron lesions). Upper gastrointestinal contrast study is seen in figure 17.1. Ap-
propriate treatment includes
(A) eradication of Helicobacter pylori and observation.
(B) endoscopic balloon dilation.
(C) per oral endoscopic myotomy (POEM).
(D) laparoscopic paraesophageal hernia repair.
(E) reduction of the hernia and gastrostomy tube.
Figure 17.1
CATEGORY 3 - PART lll 155
18. A 37-year-old man presents with a 2-week history oflower left quadrant pain that pro-
gressively worsened over the past few days. He appears comfortable, his heart rate is
80 beats per minute. his blood pressure is 128/70 mm Hg. he is afebrile. and his white
blood cell count is 13.000/mml (3600-11,200/mml). His abdomen is soft with focal
lower left quadrant tenderness and a palpable mass. A CT scan is shown in figure 18.1.
Along with fluid resuscitation and antibiotics, which of the following is the next best
step in his management?
(A) Percutaneous drainage of abscess
(B) Sigmoid colectomy and end colostomy
(C) Sigmoid colectomy, primary anastomosis, and diverting loop ileostomy
(D) Laparoscopic lavage with drain placement
(E) Repair of sigmoid perforation
Figure 18.1
19. In the adult patient with small bowel obstruction with the CT scan shown in figure
19.1, the appropriate next step is
(A) barium enema.
(B) Gastrografin upper gastrointestinal series
(C) small bowel resection.
(DJ operative reduction.
(E) adhesiolysis.
Figure 19.1
·-·-----
156 CATEGORY 3 - PART III
20. The most common anatomic site of a neuroendocrine tumor causing carcinoid syn-
drome is the
(A) stomach.
(B) small intestine.
(C) colon.
(D) rectum.
(E) pancreas.
21. The preferred procedure for a 45-year-old patient with familial adenomatous polyposis
and rectal dysplasia is
(A) total abdominal colectomy and ileorectal anastomosis.
(B) proctectomy with coloanal anastomosis.
(C) total proctocolectomy with ilea! pouch anal anastomosis.
(D) endoscopic mucosa! resection.
(E) total abdominal colectomy and end ileostomy.
22. An 82-year-old man presents with a 3-day history of constipation and a distended
abdomen. His vital signs and laboratory studies are unremarkable, but imaging reveals
a sigmoid volvulus. The patient undergoes successful endoscopic decompression of
the sigmoid volvulus. On the second hospital day, the patient has a fever and a return
of abdominal distention; a repeat plain abdominal film is obtained (figure 22.1 ). The
patient should now undergo
(A) exploration and sigmoidectomy with descending end colostomy
(Hartmann procedure).
(B) exploration with detorsion of the sigmoid and sigmoidopexy.
(C) subtotal colectomy and primary anastomosis.
(D) laparoscopic loop sigmoid colostomy.
(E) laparoscopic detorsion of the sigmoid with sigmoidopexy.
Figure 22.1
CATEGORY 3 - PART lll 157
23. A 76-year-old woman is postoperative day 6 after a dynamic screw fixation of a hip
fracture. The patient's past medical history is remarkable only for hypertension, mild
anemia, and depression. Her abdomen is distended but nontender. Her laboratory stud-
ies are unremarkable. Her pain management has been excellent on opioids. Abdominal
x-ray is shown in figure 23 .1. The next step in her management is
(A) urgent exploratory laparotomy.
(B) loop sigmoidostomy.
(C) tube cecostomy.
(D) intravenous neostigmine.
(E) polyethylene glycol electrolyte solution.
Figure 23.1
24. \Vhich of the following statements regarding the management of a patient who pres-
ents with bleeding peptic ulcer disease is true?
(A) The risk ofrebleeding is highest when an adherent clot is present within
the ulcer.
(B) Eradication of Helicobacter pylori decreases the risk of recurrent bleeding.
(C) Epinephrine injection into the ulcer base achieves durable hemostasis.
(D) Bleeding after a successful endoscopic intervention is best treated by surgery.
(E) Oral proton pump inhibitors should be given before endoscopy.
25. \Vhich of the following statements is true regarding quality metrics used to assess
colon cancer screening?
(A) Adenoma detection rate is 35% for women.
(B) A single negative fecal immunohistochemistry test should be repeated in 10 years.
(C) Excellent bowel preparation should be found in at least 65%.
(D) Withdrawal times should be greater than 10 minutes.
(E) Completion rates correlate inversely with missed colorectal cancers .
. '
158 CATEGORY 3 - PART III
26. A 69-year-old homeless man with chronic alcohol abuse presents with recent he-
matemesis. a heart rate of 110 beats per minute. a systolic blood pressure of 100 mm
Hg, and a hemoglobin of 9 g/dL (13.5-17.5 g/dL). Besides hemodynamic resuscita-
tion, further management should include which of the following?
(A) Gastric lavage with iced saline
(B) Admission for high-dose, intravenous proton pump inhibitor (PP!) therapy
(C) Discharge on PP!s ifhe responds to fluid resuscitation
(D) Admission and high-dose PP!s and transfusion of hemoglobin to 10 g/dL
(E) Admission, antibiotics, and endoscopic intervention
27. Gastroparesis refers to a symptomatic individual with delayed gastric emptying. Gas-
troparesis is most commonly
(A) associated with postviral syndromes.
(B) caused by hypothyroidism.
(C) a postsurgical complication.
(D) diagnosed by response to intrapyloric botulinum toxin A injection.
(E) idiopathic.
28. A 40-year-old man with a history of celiac disease presents with a 9-kg weight loss
and symptoms of gastric outlet obstruction. CT scan shows a mass in the postpyloric
duodenum (figure 28.1). Biopsy of the mass shows adenocarcinoma. CT shows inva-
sion of the head of the pancreas and a single, locally involved lymph node. Which of
the following statements regarding his management is true?
(A) Management should be a gastrostomy tube and palliative chemotherapy.
(B) Staging necessitates a PET scan.
(C) Radical resection is contraindicated in patients with node-positive disease.
(D) RO resection is associated with a survival advantage in stage 3 patients.
(E) Tumor invasion (T4) is a more important prognostic finding than the malignant
lymph node involvement (NI).
29. A 75-year-old man with well-controlled chronic obstructive pulmonary disease and re-
flux symptoms undergoes an upper endoscopy. He is found to have a I-cm flat plaque
in which adenocarcinoma with high grade dysplasia is identified within a 4-cm seg-
ment of Barrett esophagus. An endoscopic ultrasound shows the tumor to be confined
to the mucosa, and there is no lymphadenopathy. Which of the following is the most
appropriate therapeutic intervention?
(A) Distal esophagectomy
(B) Total esophagectomy
(C) Photodynamic therapy
(D) Endomucosal resection and ablation of the Barrett
(E) External beam radiation therapy using stereotactic techniques
31. Regarding inherited colorectal cancer syndromes, which of the following statements
is true?
(A) Approximately 20% of cases are associated with germ-line mutations.
(B) Lynch syndrome screening involves testing tumors for DNA mismatch repair
only in patients with a family history of colon cancer.
(C) Diagnosis of heritable syndromes influences clinical management.
(D) European and North American management recommendations are discordant.
(E) Only patients with more than 5 of the 9 Bethesda criteria should undergo
screening.
32. A 60-year-old man undergoes upper endoscopy for vague abdominal pain and occult
bleeding. He is found to have a 2-cm duodenal mass. There is no active bleeding. The
mass described is found to be follicular non-mucosa-associated lymphoid tissue B-cell
lymphoma. Initial treatment should consist of
(A) endoscopic ablation.
(B) chemotherapy.
(C) Helicobacter pylori therapy.
(D) pancreaticoduodenectomy.
(E) endoscopic ablation followed by radiation therapy.
33. Compared with patients treated with surgery alone for esophageal cancer, those treated
with neoadjuvant chemoradiotherapy are more likely to have increased
(A) anastomotic leak rates.
(B) pulmonary complications.
(C) chylothorax.
(D) mortality.
(E) thromboembolic rates.
34. A healthy 78-year-old woman had a chest x-ray done as part of a routine wellness check.
The lateral chest x-ray showed a possible paraesophageal hernia (PEH) confirmed by CT
scan as a type III PEH (figure 34.1 ). She denies reflux, dysphagia, pain, or shortness of
breath. Her hematocrit is normal. Which of the following would you recommend?
(A) Esophagogastroduodenoscopy, esophagram, and manometry
(B) Repair via left thoracotomy
(C) Laparoscopic repair
(D) Endoscopic reduction and fixation with percutaneous endoscopic gastrostomy
(E) Observation
CATEGORY 3 - PART III 161
35. A young male patient underwent a laparoscopic Heller myotomy with fundoplication
for achalasia. Three months postoperatively, he develops progressive, significant dys-
phagia. The best test to evaluate for incomplete myotomy would be
(A) barium esophagram.
(B) esophagogastroduodenoscopy.
(C) 24-hour pH probe.
(D) esophageal manometry.
(E) endoscopic ultrasound.
36. A 55-year-old man is found to have esophageal squamous cell carcinoma 4 cm distal
to the upper esophageal sphincter. Endoscopic ultrasound demonstrates tumor inva-
sion into, but not through, the muscularis propria. PET scan confirms no evidence of
metastatic disease. Bronchoscopy shows no tracheal involvement. Which of the fol-
lowing is the next best step in his management?
(A) Definitive chemoradiation
(B) Endoscopic mucosal resection
(C) Preoperative chemoradiation followed by esophagectomy
(D) Pharyngolaryngoesophagectomy with gastric pullup
(E) Transhiatal esophagectomy
162 CATEGORY 3 - PART Ill
38. A 57-year-old man was diagnosed with idiopathic pancreatitis 12 weeks ago. Since his
diagnosis, he has had anorexia and increasing intolerance to food. He has progressed
to the point where he has emesis anytime he tries to eat. He is hemodynamically nor-
mal and has a normal white blood cell count. A CT scan was obtained and is shown
in figure 38.1. Treatment for this finding with the best outcome and lowest associated
mortality would be
(A) percutaneous drainage.
(B) video-assisted retroperitoneal debridement.
(C) endosonographic-guided cystgastrostomy.
(D) Roux-en-Y cystojejunostomy.
(E) transduodenal sphincteroplasty and transampullary septectomy.
Figure 38.1
CATEGORY 3 - PART HI 163
39. In patients with portal hypertension ascites, which of the following interventions is
most likely to reduce the hepatic venous pressure gradient?
(A) Spironolactone at 400 mg/day
(B) Large-volume paracentesis with albumin replacement
(C) Peritoneal venous shunt
(D) Reduction in sodium intake to less than 2 g/day
(E) Transjugular intrahepatic portosystemic shunt
164 CATEGORY 3 ~PART III
Critiques 1-39
CRITIQUE 1
Many complications can follow placement of a percutaneous endoscopic gastrostomy
(PEG) tube. The tube can be inadvertently removed, which is reported to occur in l .~.4%
of cases. The timing of this removal dictates management, because the PEG tract is con-
sidered mature after 7 days. However, this timing can be affected by malnutrition, ascites
(a relative contraindication), and drugs such as steroids. A PEG tube that is dislodged early
must be assessed for the possibility of separation of the gastric hole from the anterior ab-
dominal wall. The best outcome is achieved when the tube can be replaced endoscopically
using the original site. Given the clinical picture of this patient, the stomach has probably
not fallen away from the abdominal wall, making endoscopic replacement possible.
If recognition/diagnosis is delayed and the tract is not accessible, some practitioners ad-
vocate nasogastric suction, broad-spectrum antibiotics, and repeat PEG placement in a few
days. Laparoscopic exploration might be considered but is usually not necessary as long
as the patient remains asymptomatic. Exploration and open gastrostomy tube placement
would be indicated in the presence of clinical deterioration or peritonitis.
A dislodged tube from a mature tract should also be replaced as soon as the event oc-
curs. This can usually be done at the bedside without endoscopy. At times, the tract will
need to be dilated to advance a new tube. It is usually wise to obtain a contrast study to
ensure proper placement in the stomach after tube reinsertion, regardless of the age of the
tract.
References
1. Lynch CR, Fang JC. Prevention and management of complications of percutaneous
endoscopic gastrostomy tubes. Practical Gastroenterol. 2004;22:66-76.
2. Lockett MA, Templeton ML, Byrne TK, Norcross ED. Percutaneous endoscopic
gastrostomy complications in a tertiary-care center. Am Surg. 2002;68(2):117-120.
CRITIQUE2
Parastomal herniation is the most frequent complication of stoma formation. Up to 50%
of all patients with a stoma develop a symptomatic parastomal hernia over time. Patients
with colostomy appear to be at increased risk compared with other types of stomas, with
predisposing factors including aperture size greater than 35 mm, age over 70 years, meta-
static malignancy, BMI greater than 25, diabetes mellitus, and chronic elevation of intra-
abdominal pressure. Patients with parastomal hernia experience a significant reduction in
quality of life, with symptoms including abdominal discomfort, severe abdominal pain,
and poor appliance fit with leakage of stool and resultant skin damage. Obstruction can
lead to strangulation of intestine and its sequelae. In this otherwise healthy patient with
multiple admissions for small bowel obstruction related to his hernia, an expectant ap-
proach is unreasonable; operative repair is indicated.
Repair of parastomal hernia is notoriously difficult. Several techniques of historical sig-
nificance were abandoned. Primary suture repair was popular in the 1970s but had recur-
rence rates exceeding 70%. Relocation of the stoma carries a recurrence rate of30% at the
new site with an additional incisional hernia formation rate of 20-30o/o at the old ostomy
site or the laparotomy scar.
CATEGORY 3 - PART 111
165
Several large reviews of the treatment ofparastomal hernia revealed that the recurrence
rates after synthetic mesh repair range from 6.9o/o to 17%, with no significant differences in
the technique of mesh insertion used (modified Sugarbaker or "keyhole" technique, open,
or laparoscopic). The overall rate of mesh infection in both open and laparoscopic repair
is low (3%).
Biologic mesh repair has gained popularity, primarily because of the theoretical ad-
vantage that a biologic mesh can be inserted in a contaminated area. Overall complication
rate is 26.2%, largely from seroma formation. Early studies suggested a similar short-term
recurrence rate compared with synthetic mesh. However, such materials are expensive and
lose their mechanical strength rapidly. Longer-term studies are needed to assess durability
of repair.
PTFE, once the mesh of choice for laparoscopic repair, has fallen into disfavor due to
its observed shrinkage by almost 50%, potential to cause long-term septic complications,
and recurrence rates of up to 46%.
References
I. Hotouras A, Murphy J, Thaha M, Chan CL. The persistent challenge of parasto-
mal herniation: a review of the literature and future developments. Colorectal Dis.
2013; 15(5):e202-e214.
2. lsraelsson LA. Preventing and treating parastomal hernia. World J Surg. 2005;29(8):
1086-1089.
3. Carne PW, Robertson GM, Frizelle FA. Parastomal hernia. Br J Surg. 2003;90(7):
784-793.
4. Hansson BM. Parastomal hernia: treatment and prevention 2013; where do we go from
here? Colorectal Dis. 2013;15(12):1467-1470.
CRITIQUE3
Peptic ulcer disease presents as gastric outlet obstruction in 5-8% of patients. The diagno-
sis is established by esophagogastroduodenoscopy or gastroduodenography, which can de-
termine the site, cause, and degree of obstruction. Gastric outlet obstruction is currently the
least common indication for peptic ulcer associated operations. Operations for obstruction
include vagotomy and antrectomy, vagotomy and drainage, or highly selective vagotomy
and gastrojejunostomy. When considering operative therapy. a Roux-en-Y gastrojejunos-
tomy should be avoided unless more than half the stomach has been removed, because
marginal ulceration or gastric stasis is a frequent complication. Endoscopic management
has been touted over the past several years as an alternative or at least a bridge to surgical
therapy. Several studies demonstrate good long-term outcomes with endoscopic balloon
dilation therapy for benign gastric outlet obstruction. Endoscopic stent placement is not
commonly used or recommended for benign gastric outlet obstruction management.
References
1. Yusuf TE, Brugge WR. Endoscopic therapy of benign pyloric stenosis and gastric outlet
obstruction. Curr Opin Gastroenterol. 2006;22(5):570-573.
2. Dempsey DT. Stomach. In: Brunicardi F, Andersen DK, Billiar TR, et al, eds. Schwartz's
Principles ofSurgery: 9th ed. New York, NY: McGraw-Hill;2010:889-948.
3. Lagoo J, Pappas TN, Perez A. A relic or still relevant: the narrowing role for vagotomy
in the treatment of peptic ulcer disease. Am J Surg. 2014;207(1 ):120-126.
166 CATEGORY 3 - PART Ill
CRITIQUE4
Upper gastrointestinal (UGI) bleeding can be caused by peptic ulcer disease, Mallory-
Weiss tears, gastritis, gastric neoplasms, and esophageal or gastric varices. Peptic ulcer
disease is the most common cause of upper gastrointestinal bleeding.
Placement ofa nasogastric tube is one of the first steps in managing a patient with UGI
bleeding. A bloody nasogastric aspirate and not coffee ground material is significantly as-
sociated with high-risk lesions such as actual bleeding site, visible vessel, or adherent clot,
which can be identified at subsequent endoscopy.
Esophagogastroduodenoscopy (EGD) is the standard test to diagnose and manage pa-
tients with upper gastrointestinal bleeding. The Blatchford score helps identify the need
for endoscopic intervention (table 4.1). This score is derived from systolic blood pres-
sure, pulse, presence ofmelena, syncopal episode, presence of hepatic disease, presence of
heart failure, and values for hemoglobin, and blood urea nitrogen The score ranges from
0-23 with a score of 6 or more indicating a 50% risk of needing an endoscopic interven-
tion. Endoscopy should be performed ideally within 24 hours of presentation and should
not require complete resuscitation. A prospective randomized trial of endoscopy within 12
hours to greater than 12 hours for peptic ulcer bleeding demonstrated that early endoscopy
shortened hospital stay and reduced blood transfusions while not increasing recurrences,
need for surgery, or mortality.
Most upper gastrointestinal bleeding will stop spontaneously or with endoscopic inter-
ventions. Once the bleeding has stopped, management is dictated by the source of bleed-
ing. Octreotide infusions are useful for portal hypertensive upper gastrointestinal bleeding,
but should not be used routinely for patients with upper gastrointestinal bleeding.
Answer: (B) The presence of blood within a nasogastric aspirate predicts high-risk find-
ings at endoscopy.
References
1. Park T, Wassef W. Nonvariceal upper gastrointestinal bleeding. Curr Opin Gastroen-
terol. 2014;30(6):603-608.
2. Leontiadis GI, Sreedharan A, Dorward S, et al. Systematic reviews of the clinical effec-
tiveness and cost-effectiveness of proton pump inhibitors in acute upper gastrointestinal
bleeding. Health Technol Assess. 2007;11(5l):iii-iv, 1-164.
3. Aljebreen AM, Fallone CA, Barkun AN. Nasogastric aspirate predicts high-risk
endoscopic lesions in patients with acute upper-GI bleeding. Gastrointest Endosc.
2004;59(2): 172-178.
CRITIQUES
The role oftruncal or parietal cell vagotomy in the treatment of peptic ulcer disease is dimin-
ishing. Currently, it is reserved for those patients who cannot take proton pump inhibitors,
those who persist with cigarette smoking, or those who use nonsteroidal anti-inflammatory
drugs (NSA!Ds). Bleeding or perforated gastric ulcers do not require acid reduction for
their management, thus a vagotomy is not needed and exposes the patient to unnecessary
postvagotomy complications. Patients with bleeding or perforated duodenal ulcers require
eradication of Helicobacter pylori as the common pathogenic agent in this disease.
Answer: (C) Treatment of complicated peptic ulcer disease in patients with histories of
cigarette smoking, nonsteroidal anti-inflammatory drug (NSAID) use, or adverse reactions
to proton pump inhibitors
CATEGORY 3 - PART !II
167
Score
Component
Admission Risk Marker Vaine
Blood urea nitrogen (mg/dL)
2:18.2 <22.4 mg/dL 2
2:22.4 <28 mg/dL 3
2:28 <70 mg/dL 4
2:70 mg/dL 6
Hemoglobin men (g/dL)
2:12 <13 g/dL I
2:10 <12 g/dL 3
<IO g/dL 6
Hemoglobin women (g/dL)
2:IO <12 g/dL 1
<IO g/dL 6
Systolic blood pressure
2:IOO <I09 mm Hg
2:90 <99 mm Hg 2
<90mmHg 3
Other markers
Pulse 2:100/min 1
Presentation with melena 1
Presentation with syncope 2
Hepatic disease 2
Heart failure 2
Pre-endoscopic Rockall score
Age,y
<60 0
60-79 1
2:80 2
Shock
No shock: SBP 2:100, pulse <100 0
Tachycardia: SBP 2:100, pulse 2:100 1
Hypotension: SBP <100 2
Comorbidity
No major comorbidity 0
Cardiac failure, ischemic heart 2 .,, Q
disease, any major comorbidity
Renal failure, liver failure, 3
"~
=
-
.... ...," (
""(
-0
disseminated malignanc}'.
"'
Table 4.1 Glasgow-Blatchford Score, Pre-Endoscopic Rockall Score, Modified Early
Warning Score.
Reprinted from Scandinavian Journal of Trauma, Resuscitation and Emergency Medicine, 23, S Bo7.kurt.A
KOse, ED Arslan. et al. Validity of Modified Early Warning. Glasgow Blatchford, and Pre~Endoscopic Rockall
Scores in Predicting Prognosis of Patients Presenting to Emergency Department With Upper Gastrointestinal
Bleeding, p. 109. © Bozkurt et al. 2015. Creative Commons Attribution 4.0 International License
(http://creativecommons.org/licenses/by/4.0!),
168 CATEGORY 3 - PART Ill
References
1. Lagoo J, Pappas TN, Perez A. A relic or still relevant: the narrowing role for vagotomy
in the treatment of peptic ulcer disease. Am J Surg. 2014;207(1): 120-126.
2. Reuben BC, Stoddard G, Glasgow R, Neumayer LA. Trends and predictors for vagot-
omy when performing oversew of acute bleeding duodenal ulcer in the United States.
J Gastrointest Surg. 2007;11(1):22-28.
3. Schroder VT, Pappas TN, Vaslef SN, De La Fuente SG, Scarborough JE. Vagotomy/
drainage is superior to local oversew in patients who require emergency surgery for
bleeding peptic ulcers. Ann Surg. 2014;259(6): 1111-1118.
CRITIQUE6
According to data from the Surveillance, Epidemiology, and End Results Program, there
has been a steady rise in the incidence ofadenocarcinoma of the proximal stomach and the
gastroesophageal junction, whereas the incidence of distal gastric cancers has remained
largely unchanged or has decreased slightly. The increased rate is actually the highest of
any cancer, including melanoma and lung. These findings suggest that proximal gastric
cancers have a different etiology than distal gastric cancers.
There are also epidemiologic differences between the two sites of gastric cancer. Cancer
of the gastroesophageal junction versus noncardiac gastric cancer has a male-to-female
ratio of approximately 2:1. Noncardiac cancers are significantly higher among blacks and
lower socioeconomic groups, and incidence rises progressively with age, with a peak inci-
dence between 50 and 70 years.
Other associations for proximal gastric cancers include obesity, gastroesophageal re-
flux, and an increase in the incidence of Barrett esophagus. The incidence of cancer in
patients with Barrett esophagus is estimated to be 0.8% per year. Although Helicobacter
pylori infection is associated with the risk of noncardiac gastric cancer, it appears to be
protective, or risk lowering, for gastroesophageal cancer.
References
I. Buas MF, Vaughan TL. Epidemiology and risk factors for gastroesophageal junc-
tion tumors: understanding the rising incidence of this disease. Semin Radiat Oneal.
2013;23(!):3-9.
2. Crew KD, Neugut Al. Epidemiology of gastric cancer. World J Gastroenterol.
2006; 12(3):354-362.
CRITIQUE7
Gastrointestinal stromal tumors (GISTs) can occur throughout the intestinal tract, but 50%
are found in the stomach. Vague abdominal pain, weight loss, and anemia are common
presentations. Surgical resection, with complete excision of the tumor, is the primary treat-
ment. Most patients can be spared extended anatomic resections. Lymph node metastases
are rare. Endoscopic resection can be used for smaller tumors, as long as an RO resection
can be achieved. A 4-cm tumor is not amenable to endoscopic resection.
Neoadjuvant 5-:fluorouracil-based chemotherapy may be used for gastric adenocarci-
noma but not GISTs. More than 95% of GISTs express a mutation in c-kit protein, a tyro-
sine kinase that promotes cell proliferation. Imatinib is a selective tyrosine-kinase inhibitor
that binds to the c-kit protein, blocks signaling, and leads to arrest of cellular proliferation.
Imatinib is used for unresectable or metastatic GISTs or as an adjuvant therapy in those at
high risk for recurrence, as stratified by size of the tumor and number of mitoses per high-
powered field.
CATEGORY 3 - PART III 169
References
1. Turner DJ, Jain A, Bass BL. Small-intestinal neoplasms. In: Mulholland MW, Lillemoe
KD, Doherty GM, Maier RV, Simeone DM, Upchurch GR Jr. Greenfield"s Surgery:
Scientific Principles and Practice. 5th ed. Philadelphia, PA: Lippincott Williams &
Wilkins;2011 :784-797.
2. National Comprehensive Cancer Network. NCCN Guidelines Soft Tissue Sarcoma.
Version 1.2015. Fort Washington, PA: National Comprehensive Cancer Network.
CRITIQUES
Small bowel adenocarcinomas are most commonly located in the duodenum. They can
metastasize to regional lymph nodes. Patients with Crohn disease have an increased risk
of small bowel adenocarcinomas. Surgical resection of the primary tumor and regional
lymph nodes is the only curative option. Chemoradiation is reserved for patients with more
advanced disease that is unresectable.
c-KIT mutation is associated with gastrointestinal stromal tumors, not small bowel ad-
enocarc1nomas.
References
1. Turner DJ, Jain A, Bass BL. Small-intestinal neoplasms. In: Mulholland MW, Lillemoe
KD, Doherty GM, Maier RV. Simeone DM, Upchurch GR Jr. Greenfield's Surgery:
Scientific Principles and Practice. 5th ed. Philadelphia, PA: Lippincott Williams &
Wilkins;2011 :784-797.
2. Reynolds I, Healy P, McNamara DA. Malignant tumours of the small intestine. Surgeon.
2014; 12(5):263-270.
CRITIQUE9
Peptic ulcers remain the most common cause of upper gastrointestinal bleeding. Ap~
propriate diagnosis, management, and long-term therapy have changed over the past
decades, and the general surgeon's role has evolved. The majority of patients can now
be managed medically for peptic ulcer disease. Helicobacter pylori eradication is sig-
nificantly more effective in decreasing rebleeding from peptic ulcer than proton pump
inhibitor (PPI) use alone. Studies demonstrate that diagnostic tests, including serology,
histology, urea breath test, rapid urease test, stool antigen, and culture, have high positive
predictive values (0.85-0.99) but low negative predictive values (0.45-0.75) in the acute
setting. Although biopsy-based methods of assessment have a low sensitivity during
acute upper gastrointestinal bleeding, 13C-urea breath testing is the most accurate with
high sensitivity (90-95%) and specificity (87-96%) in this setting. Serologic tests have
the added drawback of remaining positive for up to 6-12 months after eradication, thus
leading to significant overtreatment. Therefore, it is important to repeat testing if initial
tests are negative in the acute setting. If bleeding stops during the index hospitalization,
lifelong PPI use is recommended.
Surgical therapy should be considered for patients with intractable or nonhealing peptic
ulcer disease who have multiple recurrences; large ulcers (>2 cm); suspected malignancy;
or complications such as perforation, obstruction, or ongoing bleeding. Gastrinoma and
Zollinger-Ellison syndrome are important causes of peptic ulcer disease that should be
considered, and gastrin levels should be evaluated. However, acid suppression medications
170 CATEGORY 3 - PART III
should be held for several days before serum gastrin measurement, because erroneously
elevated gastrin levels may result. A second-look endoscopy refers to performance of a
preplanned endoscopy within 16-24 hours after the initial endoscopy to assess the need for
further endoscopic therapy. Although high-risk populations in which primary hemostasis
is uncertain or initial evaluation is incomplete may benefit, current consensus guidelines
recommend against routine second-look endoscopy.
References
1. Dempsey DT. Stomach. In: Brunicardi F,Andersen DK, Billiar TR, et al, eds. Schwartz s
Principles ofSurgery. 9th ed. New York, NY: McGraw-Hill;2010:889-948.
2. Lu Y, Chen YI, Barkun A. Endoscopic management of acute peptic ulcer bleeding.
Gastroenterol Clin North Am. 20!4;43(4):677-705.
3. Gisbert JP, Abraira V. Accuracy ofHelicobacter pylori diagnostic tests in patients with
bleeding peptic ulcer: a systematic review and meta-analysis. Am J Gastroenterol.
2006;101(4):848-863.
CRITIQUE 10
This patient is most likely suffering from an adhesive small bowel obstruction, and non-
operative therapy should be initially attempted, because she is not demonstrating signs or
symptoms of bowel ischemia or perforation. Intra-abdominal adhesions related to prior
abdominal surgery are the cause of small bowel obstruction in up to 75% of cases. Most
cases resolve with nonoperative therapy within 24-72 hours, after which the likelihood
of resolution diminishes substantially. Furthermore, a 48-hour limit of watchful waiting
before surgical intervention is not associated with an increase in postoperative morbidity
or mortality, although inpatient costs are higher.
The administration of hypertonic water-soluble contrast agents is both diagnostic and
therapeutic, reducing the need for surgery, time to resolution, and length of hospital stay.
Radiologic presence of contrast material in the colon within 24 hours of administration
predicts resolution. Laparoscopic treatment of small bowel obstruction is safe and is as-
sociated with a shorter hospital stay. Capsule endoscopy is contraindicated in the setting of
small bowel obstruction.
References
1. Tavakkoli A, Ashley SW, Zinner MJ. Small Intestine. In: Brunicardi F, Andersen
DK, Billiar TR, et al, eds. Schwartz's Principles of Surgery. 10th ed. New York, NY:
McGraw-Hill; 2014: 1137-1174.
2. Lombardo S, Baum K, Filho JD, Nirula R. Should adhesive small bowel obstruction be
managed laparoscopically? A National Surgical Quality Improvement Program propen-
sity score analysis. J Trauma Acute Care Surg. 2014;76(3):696-703.
3. Di Saverio S, Coccolini F, Galati M, et al. Bologna guidelines for diagnosis and manage-
ment of adhesive small bowel obstruction (ASBO): 2013 update of the evidence-based
guidelines from the world society of emergency surgery ASBO working group. World J
Emerg Surg. 2013;8(1):42.
CATEGORY 3 ~PART Ill 171
CRITIQUE 11
A dramatic increase in the survival of children with cystic fibrosis into adulthood has led
to the increased presentation of diseases associated with the decreased chloride and fluid
secretion into the small ainvays and the gastrointestinal tract secondary to defective cystic
fibrosis transmembrane conductance regulator. The incidence of distal intestinal obstruc-
tive syndrome (DIOS), characterized by acute fecal obstruction of the ileocecum, increases
with age. It occurs more commonly in patients with a history of meconium ileus as infants.
DIOS is associated with but not caused by pancreatic insufficiency. Most DIOS episodes
can be treated with intensive gastrointestinal emptying with laxatives and enemas; opera-
tive intervention is rarely required.
References
I. Colombo C, Ellemunter H, Houwen R, Munck A, Taylor C, Wilschanski M; ECFS.
Guidelines for the diagnosis and management of distal intestinal obstruction syndrome
in cystic fibrosis patients. J Cyst Fibros. 2011 ;10 Suppl 2:S24-S28.
2. van der Doef HP, Kokke FT, van der Ent CK, Houwen RH. Intestinal obstruction
syndromes in cystic fibrosis: meconium ileus, distal intestinal obstruction syndrome,
and constipation. Curr Gastroenterol Rep. 2011; 13(3):265-270.
CRITIQUE 12
The incidence of diverticulitis is increasing in North America and Europe over the last 3
decades, with up to 25% of patients with diverticulosis presenting to hospitals with di-
verticulitis during their lifetimes. The increased incidence may be due to improvement
in the accuracy of diagnosis of acute diverticulitis with cross-sectional imaging. Studies
have found a 37% increase in the incidence of diverticulitis with cross-sectional imaging
compared with clinical diagnosis. Colonic malignancies are noted more commonly in pa-
tients with complicated diverticulitis, and malignancies are found in up to 3o/o of patients
with colonoscopy after an episode of acute diverticulitis. Although colonoscopy offers the
benefit of histologic sampling of lesions, CT colonography is as sensitive in diagnosing
colorectal cancer. A study ofa large cohort of patients with the diagnosis of uncomplicated
acute diverticulitis suggested that the yield of advanced colonic neoplastic lesions was
equivalent to that of screening asymptomatic, average-risk individuals.
Answer: (B) After an episode of acute diverticulitis. the incidence of finding a malig-
nancy on colonoscopy or CT colonography is 1-3%.
References
I. Sharma PV, Eglinton T, Hider P, Frizelle F. Systematic review and meta-analysis of the
role of routine colonic evaluation after radiologically confirmed acute diverticulitis. Ann
Surg. 2014;259(2):263-272.
2. Westwood DA, Eglinton TW, Frizelle FA. Routine colonoscopy following acute uncom-
plicated diverticulitis. Br J Surg. 2011 ;98(11):1630-1634.
CRITIQUE 13
Patients with narcotic bowel syndrome (NBS) have increased frequency, duration, and
intensity of abdominal pain with continuing or increasing doses of narcotics. NBS is less
common but sometimes associated with opioid bowel disorder, a dose-related dysmotil-
ity disorder leading to constipation, gastroparesis, or ileus. NBS patients do not typically
172 CATEGORY 3 ~PART Ill
have constipation, and some may present with diarrhea. A narcotic detoxification program
incorporating fixed reductions in narcotics until a patient is weaned completely can lead to
a reduction in pain scores. The majority of patients restart narcotics during the 3 months
after detoxification.
References
1. Drossman DA, Morris CB, Edwards H, et al. Diagnosis, characterization, and 3-month
outcome after detoxification of39 patients with narcotic bowel syndrome. Am J Gastro-
enterol. 2012;107(9):1426-1440.
2. Grover CA, Wiele ED, Close RJ. Narcotic bowel syndrome. J Emerg Med.
2012;43( 6):992-995.
CRITIQUE 14
Rectal foreign bodies are a common presenting complaint in the emergency department,
and most Can be removed after appropriate imaging, especially if suitable sedation is ac-
companied by local anesthesia to allow relaxation of the anal sphincters. If the object is
palpable in the abdomen, a second operator gently pushing on the object in the suprapubic
area or left iliac fossa may help stabilize the object and allow it to be grasped transanally.
Passing a Foley catheter or a Minnesota tube above the object and then inflating the balloon
may break any suction effect and allow traction. Creativity may be required to identify or
modify a clamp or device to grasp unbreakable objects. Removal with obstetric forceps is
contraindicated for glass objects. Repeated enemas are usually not successful.
Objects that cannot be transanally extracted are usually impacted high in the rectum and
cannot be easily turned to follow the sacral curve. Laparotomy may be required in such
cases, although it may not be necessary to open the colon if the object can be manipulated
through the sacral curve from above. Flexible colonoscopy or rigid sigmoidoscopy are
performed after removal of the object to look for evidence of bowel injury, along with
review of imaging for free air. Discovery of a transmural injury into the extraperitoneal
rectal space requires diversion of the fecal stream, in the same manner that penetrating
extraperitoneal rectal injuries are managed. Patients seen with rectal foreign bodies should
be screened for intimate partner violence.
Answer: (A) Extraction under deep sedation and perianal local anesthesia
References
1. Cologne KG, Ault GT. Rectal foreign bodies: what is the current standard? Clin Colon
Rectal Surg. 2012;25(4):214-218.
2. Goldberg JE, Steele SR. Rectal foreign bodies. Surg Cl in North Am. 2010;90(1):173-184.
CRITIQUE 15
The differential diagnosis of this small bowel mass includes a neuroendocrine (carcinoid) tu-
mor, gastrointestinal stromal tumor (GIST), adenocarcinoma, non-Hodgkin lymphoma, and
metastatic disease. The most common tumors that metastasize to the small bowel are melano-
ma, lung, and breast. The defining clinical feature that helps differentiate these potential eti-
ologies and determines treatment is the fibrosis of the mesentery causing the obstruction-a
classic finding in neuroendocrine tumors (figure 15.1). Forty-five percent of neuroendocrine
tumors occur in the small intestine, and one-third are multifocal. Spread to mesenteric lymph
nodes occurs in 40-80%, but only 10% of patients have hepatic involvement with carcinoid
syndrome. Operative treatment should be en bloc resection with the involved mesentery.
CATEGORY 3 - PART Ill 173
References
1. Hrabe JE, Cullen JJ. Management of small bowel tumors. In: Cameron JL, Camer-
on AM, eds. Current Surgical Therapy. I 0th ed. Philadelphia, PA: Elsevier Saun-
ders;2011 : 106-l IO.
174 CATEGORY 3-PART 111
2. Strosberg J. Neuroendocrine tumours of the small intestine. Best Pract Res Clin Gastro-
enterol. 2012;26(6):755-773.
3. Ponce! G, Faucheron JL, Walter T. Recent trends in the treatment of well-differenti-
ated endocrine carcinoma of the small bowel. World J Gastroenterol. 2010;16(14):
1696-1706.
4. Kulke MH, Shah MH, Benson AB 3rd, et al; National Comprehensive Cancer Network.
Neuroendocrine tumors, version 1.2015. J Natl Compr Cane Netw. 2015; 13(1):78-108.
CRITIQUE 16
Anal intraepithelial neoplasia (AIN) is a precursor lesion for anal squamous cell carci-
noma. AIN is classified into 3 grades: AIN I is the least dysplastic with AIN III the most
dysplastic. More recent terminology has suggested low-grade squamous intraepithelial le-
sion (LSIL) should replace AIN I and high-grade squamous intraepithelial lesion (HSIL)
should replace AIN II and AIN III. Regardless of the terminology used, intervention in this
patient for this precursor lesion is indicated.
With high rates of metachronous lesions in this patient population, careful evaluation
of the anodenn for additional AIN lesions is warranted. High-resolution anoscopy, similar
to colposcopy for cervical cancer screening, is suggested as advantageous for all patients
with AIN. The anal canal and transition zone are inspected using a high-resolution micro-
scope. Acetic acid, Lugo! iodine solution, or both are applied. These elucidate the abnormal
telangiectasias seen with HSIL. Directed biopsy or treatment of concerning areas is per-
formed rather than blind, random biopsy, which could miss involved tissue. The treatment
of choice for AIN is local ablative therapy. Patients who undergo routine surveillance with
ablation of identified lesions rarely progress to invasive cancer. Local ablative therapy
may consist of electrocautery, laser, cryotherapy, trichloroacetic acid, topical imiquimod,
or infrared coagulation.
The other listed interventions either are indicated for invasive cancer or have significant
morbidity with no significant improvement in long-term outcome. Abdominoperineal re-
section is the historical treatment for squamous cell carcinoma of the anus. The Nigro pro-
tocol is used for invasive squamous cell carcinoma in the anal canal. Wide local excision
with advancement flap coverage would eliminate all potentially affected anoderm, but it is
an invasive procedure, requires fecal diversion, and is plagued with significant morbidity.
More recent experience with close surveillance and local ablative therapies demonstrates
excellent outcomes.
References
1. Crawshaw BP, Russ AJ, Stein SL, et al. High-resolution anoscopy or expectant manage-
ment for anal intraepithelial neoplasia for the prevention of anal cancer: is there really a
difference? Dis Colon Rectum. 2015;58(1):53-59.
2. Goldstone RN, Goldstone AB, Russ J, Goldstone SE. Long-term follow-up of infrared
coagulator ablation of anal high-grade dysplasia in men who have sex with men. Dis
Colon Rectum. 2011;54(10): 1284-1292.
3. Pineda CE, Berry JM, Jay N, Palefsky JM, Welton ML. High-resolution anoscopy
targeted surgical destruction of anal high-grade squamous intraepithelial lesions: a ten-
year experience. Dis Colon Rectum. 2008;51(6):829-835; discussion 835-837.
4. Shellito PC. Tumors of the anal region. In: Cameron JL, Cameron AM, eds. Current
Surgical Therapy. 10th ed. Philadelphia, PA: Elsevier Saunders;201l:196-200.
CATEGORY 3 - PART Ill
175
CRITIQUE 17
Paraesophageal hernias occur when a portion of the gastric fund us, the entire stomach, or
the stomach and other structures herniate through the diaphragmatic hiatus (figure 17.2).
These are differentiated from sliding hiatal hernias, in which the stomach remains in its
usual anatomical longitudinal alignment. Although reflux and heartburn symptoms are
common in sliding hiatal hernias~ they also can be seen in patients with paraesophageal
hernia. Postprandial chest fullness is common in patients with paraesophageal hernias, and
iron deficiency can be seen in up to 50°/o.
Cameron lesions are linear gastric erosions in patients with paraesophageal hernias that
can be associated with severe acute or chronic occult gastrointestinal bleeding. They are
typically identified by upper endoscopy and found proximally at the level of the diaphrag-
matic impression on the stomach. Helicobacter pylori is not involved in the pathogenesis
and is isolated in a minority of patients with Cameron lesions.
Truly asymptomatic patients with a paraesophageal hernia can be safely observed with-
out specific intervention. However, all symptomatic paraesophageal hiatal hernias should
be repaired with a transthoracic or transabdominal approach. A laparoscopic transabdomi-
nal approach is preferred for repair in which the herniated contents are reduced, and cruro-
plasty and fundoplication are then performed to minimize postoperative reflux and hernia
recurrence. Hernia reduction and gastrostomy tube placement alone is associated with a
high recurrence rate. Per oral endoscopic myotomy (POEM) and endoscopic balloon dila-
tion can be used in the treatment of achalasia but have no current role in the treatment of a
paraesophageal hernia.
References
1. Maganty K, Smith RL. Cameron lesions: unusual cause of gastrointestinal bleeding and
anemia. Digestion. 2008;77(3-4):214-217.
2. Camus M, Jensen DM, Ohning GV, et al. Severe upper gastrointestinal hemorrhage from
linear gastric ulcers in large hiatal hernias: a large prospective case series of Cameron
ulcers. Endoscopy. 2013;45(5):397-400.
3. Kohn GP, Price RR, DeMeester SR, et al; SAGES Guidelines Committee. Guidelines
for the management ofhiatal hernia. Surg Endosc. 2013;27(12):4409-4428.
CRITIQUE 18
Surgery for acute diverticulitis is indicated for patients who present with sepsis and for pa-
tients who do not improve with nonoperative treatment. The patient presented in this ques-
tion is hemodynamically normal, has a leukocytosis, and CT findings showing sigmoid
diverticulitis with abscess (figure 18.2). Radiologically guided percutaneous drainage is
usually the most appropriate treatment for patients with a large diverticular abscess. This
is most often followed by complete recovery, and these patients are encouraged to undergo
elective resection.
In the absence of severe clinical signs of sepsis, studies suggest the benefit of nonopera-
tive treatment even for patients with complicated, perforated acute diverticulitis. Patients
who recover from acute diverticulitis with perforation are encouraged to undergo elective
resection with primary anastomosis.
Simple primary repair of the sigmoid perforation is not appropriate in the setting of
diverticulitis. Laparoscopic lavage with drain placement may have a role in a patient with
more diffuse inflammation, but percutaneous drainage is preferred for a contained abscess.
References
1. Rafferty J, Shellito P, Hyman NH, Buie WD; Standards Committee of American Society
of Colon and Rectal Surgeons. Practice parameters for sigmoid diverticulitis. Dis Colon
Rectum. 2006;49(7):939-944.
2. Broderick-Villa G, Burchette RJ, Collins JC, Abbas MA, Haigh PI. Hospitaliza-
tion for acute diverticulitis does not mandate routine elective colectomy. Arch Surg.
2005;140(6):576-581: discussion 581-583.
CATEGORY 3 - PART JII 177
CRITIQUE 19
Intussusception is an uncommon cause of small bowel obstruction in adults. It is diagnosed
preoperatively in about 50o/o of the cases. When diagnosed before surgery, intussusception
can be seen on CT scan (figure 19.2) or occasionally with abdominal ultrasound. In most
cases of intussusception in adults, a neoplasm is a common cause. For this reason, surgi-
cal resection, and not just reduction and relief of obstruction, is recommended in nearly
all cases of small bowel intussusception in an adult. Although surgical adhesions are the
most common cause of small bowel obstruction, adhesiolysis is not the treatment for small
bowel obstruction secondary to intussusception.
Upper gastrointestinal study with small bowel follow through using Gastrografin leads
to a shorter time to identify the need for surgical intervention and a shorter time to reso-
lution of small bowel obstruction treated nonoperatively. However, this was shown for
adhesive small bowel obstruction, and a Gastrografin contrast study has no role in intus-
susception beyond serving as an adjunct diagnostic imaging modality.
With the increasing use of computed tomography, the radiologic diagnosis of intussus-
ception is identified with increasing frequency, even though true idiopathic intussusception
in adults is extremely rare. A review of 380,999 abdominal CT reports in adults identified
intussusception in 0.04%. Of these patients, fewer than 50% had reported abdominal pain
and only 43.5% had clinical findings on abdominal exam. Thus, associated clinical or ra-
diologic findings, such as a lead point or small bowel obstruction, are often sought before
operative exploration. A study on the use of CT scan in the pediatric population also found
spontaneous resolution of intussusception in the majority of patients who did not have
clinical correlation with the radiographic findings.
References
1. Haas EM, Etter EL, Ellis S, Taylor TV. Adult intussusception. Am J Surg. 2003; 186( 1):
75-76.
2. Barussaud M, Regenet N, Briennon X, et al. Clinical spectrum and surgical approach of
adult intussusceptions: a multicentric study. Int J Colorectal Dis. 2006;21(8):834-839.
3. Galardi N, Collins J, Friend K. Use of early Gastrografin small bowel follow-through in
small bowel obstruction management. Am Surg. 2013;79(8):794-796.
4. Napora TE, Henry KE, Lovett Tl, Beeson MS. Transient adultjejunal intussusception.
J Emerg Med. 2003;24(4):395-400.
5. Strouse PJ, DiPietro MA, Saez F. Transient small-bowel intussusception in children on
CT. Pediatr Radio/. 2003;33(5):316-320.
6. Rea JD, Lockhart ME, Yarbrough DE, Leeth RR, Bledsoe SE, Clements RH. Approach
to management of intussusception in adults: a new paradigm in the computed tomogra-
phy era. Am Surg. 2007;73(11):1098-1l05.
7. Marinis A, Yiallourou A, Samanides L, et al. Intussusception of the bowel in adults: a
review. World J Gastroenterol. 2009; 15(4):407-411.
CRITIQUE20
Neuroendocrine, or carcinoid, tumors arise most commonly from neuroendocrine cells of
the gastrointestinal tract. Approximately 25% arise from the Kulchitsk)' cells of the bron-
chopulmonary system. Most of the carcinoid tumors of the gastrointestinal tract occur in
the small intestine, with the ileum representing the majority. The appendix, rectum, and
cecum are also common sites in the gastrointestinal tract
In a minority of patients, neuroendocrine tumors secrete bioactive agents that can cause
the carcinoid syndrome. Carcinoid syndrome-thought to be secondary to a serotonin-
secreting midgut tumor-is characterized by severe flushing, diarrhea, bronchoconstric-
tion, and fibrotic valve disease. Pancreatic neuroendocrine tumors represent less than 1o/o
of total neuroendocrine tumors and are a rare cause of the carcinoid syndrome.
The 5-year survival rates of neuroendocrine tumors are better than for an adenocarci-
noma of the same site and correlate with pathologic stage. Factors influencing the progno-
sis of carcinoid tumors are complex and include anatomical location and the presence of
symptoms. In several reports, improved survival was observed in hindgut tumors, relative
to midgut and foregut tumors. However, in a review of multiple reports that included thou-
sands of patients, appendiceal or rectal carcinoids had a better prognosis than small intes-
tinal or colonic carcinoid tumors. Among patients with metastatic disease, midgut tumors
are associated with an improved survival. It is unclear whether this represents a reflection
of biology or a reporting bias.
References
1. Akerstrom G, Hellman P, Hessman 0, Osmak L. Management of midgut carcinoids.
J Surg Onco/. 2005;89(3):161-169.
2. Modlin IM, Lye KD, Kidd M. A 5-decade analysis of 13,715 carcinoid tumors. Cancer.
2003;97(4):934-959.
3. Van Gompel JJ, Sippel RS, Warner TF, Chen H. Gastrointestinal carcinoid tumors:
factors that predict outcome. World J Surg. 2004;28(4):387-392.
4. Rorstad 0. Prognostic indicators for carcinoid neuroendocrine tumors of the gastroin-
testinal tract. J Surg Oneal. 2005;89(3):151-160.
5. Onaitis MW, Kirshbom PM, Hayward TZ, et al. Gastrointestinal carcinoids: character-
ization by site of origin and hormone production. Ann Surg. 2000;232(4):549-556.
CATEGORY 3-PART Ill 179
CRITIQUE 21
Familial adenomatous polyposis (FAP) is an autosomal dominant disorder caused by a
mutation in the adenomatous polyposis coli (APC) gene on chromosome 5q2 l. Patients
with FAP typically form many polyps throughout the colon and rectum that are at a I 00%
lifetime risk of malignant transformation. For this reason, prophylactic proctocolectomy
is usually the treatment of choice. In addition, FAP patients require lifetime surveillance
for extraintestinal disorders, including foregut and thyroid tumors, desmoids tumors
(Gardner syndrome), osteomas and fibromas, and brain glioblastoma and medulloblas-
toma (Turco! syndrome).
A person with a family history of FAP should undergo genetic testing in the preteen
years. Positive testing then warrants annual surveillance colonoscopy starting at age 10-15
years, until age 50. Patients presenting with polyposis should undergo resection. The pre-
ferred treatment is proctocolectomy to remove all colon and rectal mucosa and reconstruc-
tion with ilea! pouch anal anastomosis (!PAA). Total abdominal colectomy with ileorectal
anastomosis is also a possibility but requires close, life-long surveillance of the rectum,
and thus is offered more commonly to older patients and those with high compliance with
follow-up plans. However, in the setting of rectal dysplasia, leaving the rectal mucosa is
contraindicated. Endoscopic mucosa! resection alone is inadequate treatment for patients
with FAP, especially when dysplasia is already identified. Total proctocolectomy with end-
ileostomy is an appropriate operation to offer FAP patients with rectal dysplasia as it re-
moves all affected mucosa. Life-long management of an ileostomy can be challenging to
many patients.
References
1. Aihara H, Kumar N, Thompson CC. Diagnosis, surveillance, and treatment strategies
for familial adenomatous polyposis: rationale and update. Eur J Gastroenterol Hepotol.
2014;26(3):255-262.
2. Galiatsatos P, Foulkes WD. Familial adenomatous polyposis. Am J Gastroenterol.
2006;101(2):385-398.
CRITIQUE22
The plain abdominal x-ray reveals a recurrent sigmoid volvulus (figure 22.2). After suc-
cessful endoscopic decompression, a 40-50% recurrence rate is expected. Often a decom-
pressive tube is left in place and the bowel is prepped expecting a recurrence that would
require surgical resection.
When sigmoid volvulus recurs. operative intervention is the treatment of choice. A Hart-
mann procedure with end colostomy is a safe approach for a high-risk patient, as described.
Sigmoidopexy is possible but associated with high recurrence rates. Sigmoid resection
with primary anastomosis is possible if no bowel ischemia or perforation is present. This
is best performed in good risk patients and after an attempt at bowel preparation using a
decompressive tube. There are insufficient studies comparing the outcomes of laparotomy
versus any laparoscopic procedures for volvulus.
Answer: (A) exploration and sigmoidectomy with descending end colostomy (Hart-
mann procedure).
----·---·---------------·---------------------------·-··----------------
180 CATEGORY 3 - PART Ill
References
I. Halabi WJ, Jafari MD, Kang CY, et al. Colonic volvulus in the United States: trends,
outcomes, and predictors of mortality. Ann Surg. 20 l 4;259(2):293-30 l.
2. Grossmann EM, Longo WE, Stratton MD, Virgo KS, Johnson FE. Sigmoid volvulus in
Department of Veterans Affairs medical centers. Dis Colon Rectum. 2000;43(3):414-418.
3. Oren D, Atamanalp SS, Aydinli B, et al. An algorithm for the management of sigmoid
colon volvulus and the safety of primary resection: experience with 827 cases. Dis
Colon Rectum. 2007;50(4):489-497.
CRITIQUE23
The abdominal plain film shows markedly dilated ascending and transverse colon (figure
23.2) with no gas in the distal colon. Given the clinical scenario, a likely diagnosis is co-
lonic pseudo obstruction or (Ogilvie syndrome).
Colonic pseudo obstruction is associated with narcotic use for pain, anticholinergic
drugs, calcium channel blockers, and clonidine; prolonged bedrest; and electrolyte abnor-
malities. Surgery is not usually required. Short rectal tubes are ineffective, because the
adynamic colon is usually too proximal.
Treatment is aimed at first correcting any electrolyte abnormalities, stopping pain medi-
cations and other possible causative drugs if possible, and administering neostigmine in-
travenously. Neostigmine induces colonic motility, relieving the pseudo obstruction. The
use ofneostigmine should be given only if distal obstruction has been ruled out with either
a contrast study or endoscopy. If colonoscopy is necessary to exclude distal obstruction, it
can also be used to decompress the distended colon.
CATEGORY 3 - PART JI! 181
Tube cecostomy is possible, but usually not recommended and is associated with a high
complication rate. Loop sigmoidostomy is also unnecessary. Polyethylene glycol electro-
lyte (PEG) laxative solutions given after resolution of colonic dilation may reduce recur-
rence rates.
References
I. Ponec RJ, Saunders MD, Kimmey MB. Neostigmine for the treatment of acute colonic
pseudo-obstruction. N Engl J Med. 1999;341(3):137-141.
2. ASGE Standards of Practice Committee, Harrison ME, Anderson MA, Appalaneni V,
et al. The role of endoscopy in the management of patients with known and suspected
colonic obstruction and pseudo-obstruction. Gastrointest Endosc. 2010;71 (4):669-679.
3. Sgouros SN, Vlachogiannakos J, Vassiliadis K, et al. Effect of polyethylene glycol
electrolyte balanced solution on patients with acute colonic pseudo obstruction after
resolution of colonic dilation: a prospective, randomised, placebo controlled trial. Gut.
2006;55(5):638-642.
182 CATEGORY 3 - PART Ill
CRITIQUE24
Peptic ulcer disease remains the most common cause of upper gastrointestinal bleeding,
although the overall number of patients with a bleeding peptic ulcer has declined. This
decrease is possibly related to the use of proton pump inhibitors (PP!s) and Helicobacter
pylori therapy.
Intravenous PPI therapy should be given to patients with upper gastrointestinal bleed-
ing from ulcer disease and continued for 72 hours after endoscopic therapy. This treatment
reduces the risk of further bleeding, surgery, and mortality. Oral PP!s, somatostatin, and
octreotide are not routinely recommended for bleeding related to peptic ulcer disease.
The risk of bleeding is estimated by the presence of stigmata of recent hemorrhage, with
the risk for a visible vessel being higher than adherent clot. Rebleeding rates can be reduced
by endoscopic interventions such as epinephrine injection, heater probe coagulation, and
clipping of bleeding vessels. Combination therapy with these intervention is recommended
rather than monotherapy with injection. Recurrent bleeding after initial endoscopic con-
trol of bleeding is treated with another endoscopy with appropriate interventions. Repeat
endoscopic control reduces the need for surgery without increasing the risk of death and is
associated with fewer complications than surgery.
Testing for H. pylori should be done in all patients with a peptic ulcer and treated if
positive. Treatment protocols include acid suppression, coating agents, and antibiotics. The
recurrence rate of ulcer and bleeding is higher with persistent H. pylori infection.
Answer: (B) Eradication of Helicobacter pylori decreases the risk ofrecurrent bleeding.
References
1. Park T, Wassef W. Nonvariceal upper gastrointestinal bleeding. Curr Opin Gastroen-
terol. 2014;30( 6):603-008.
2. Lau JY, Sung JJ, Lam YH, et al. Endoscopic retreatment compared with surgery in
patients with recurrent bleeding after initial endoscopic control of bleeding ulcers.
N Engl J Med. 1999;340(10):751-756.
3. Sachar H, Vaidya K, Laine L. Intermittent vs continuous proton pump inhibitor therapy
for high-risk bleeding ulcers: a systematic review and meta-analysis. JAMA Intern Med.
2014; 174(11):1755-1762.
CRITIQUE25
Several measures reflect the completeness and quality of a screening colonoscopy. The
recommended measures for the procedure include adequacy of preparation, withdrawal
times, postprocedure discomfort, and adenoma detection rate. The frequency with which
these measures are achieved can be used as an objective measure of the quality of the
work. The adequacy of the preparation should be greater than 90%, withdrawal times
should be at least 6 minutes, and postprocedure discomfort should be seen in fewer than
10% of patients.
Screening colonoscopy is the prototype for cost-effective medical intervention in the
United States. When performed well, it serves as both a cancer detection and prevention tool.
However, several essential elements must be achieved for a screening colonoscopy to be
considered complete and of high value. A high rate of incomplete colonoscopies is associated
with a higher rate of subsequent colon cancers. In a large study, individuals who underwent
screening by an endoscopist with a high completion rate were less likely to have a post-
colonoscopy colorectal cancer (PCCRC). Further, patients with proximal cancers undergoing
colonoscopy by endoscopists who performed polypectomies at high rates had a lower risk of
PCCRC. In this study, endoscopist volume of procedures was not associated with PCCRC.
The logic for adenoma detection rate as a quality measure is based on the known natural
frequency of adenomas in the population. Screened population adenoma detection rates
CATEGORY 3 - PART HJ
183
should reflect this frequency over a large number of studies performed by an endoscopist.
An adenoma detection rate range is used: 20-30% for men, 15-25o/o for women.
Answer: (E) Completion rates correlate inversely with missed colorectal cancers.
References
I. Levin B, Lieberman DA, McFarland B, et al; American Cancer Society Colorectal
Cancer Advisory Group; US Multi-Society Task Force; American College of Radiol-
ogy Colon Cancer Committee. Screening and surveillance for the early detection of
colorectal cancer and adenomatous polyps, 2008: a joint guideline from the American
Cancer Society, the US Multi-Society Task Force on Colorectal Cancer, and the Ameri-
can College of Radiology. Gastroenterology. 2008;134(5):1570-1595.
2. Baxter NN, Sutradhar R, Forbes SS, Paszat LF, Saskin R, Rabeneck L. Analysis of
administrative data finds endoscopist quality measures associated with postcolonoscopy
colorectal cancer. Gastroenterology. 2011; 140(1 ):65-72.
3. Corley DA, Levin TR, Doubeni CA. Adenoma detection rate and risk of colorectal
cancer and death. N Engl J Med. 2014;370(26):2541.
4. Bourikas LA, Tsiamoulos ZP, Haycock A, Thomas-Gibson S, Saunders BP. How we can
measure quality in colonoscopy? World J Gastrointest Endosc. 2013;5(10):468-475.
CRITIQUE26
This patient presents with a challenging social history and stigmata of potentially life-
threatening upper gastrointestinal bleeding from an unknown source. The prompt estab-
lishment of a diagnosis is critical, but attention to, and correction of, his physiological
distress is essential before definitive diagnostic testing and therapeutic intervention, which
will be upper endoscopy. In a study of the outcomes of endoscopic intervention in relation
to the timing, no correlation was found between transfusion volumes, mortality, or need for
surgery between those treated in less than versus within 24 hours. Therefore, resuscitation
is the initial priority. Patients with chronic liver disease and upper gastrointestinal bleeding
are at a high risk for the development of bacterial infections. Prophylactic antimicrobial
therapy, usually with a fluoroquinolone, reduces the rates of infection, early rebleeding,
and mortality.
Gastric lavage with iced saline is of historical interest only, because it is ineffective at
achieving hemostasis and is associated with an unnecessary risk of aspiration. The empiric
use of either proton pump inhibitors or octreotide as definitive therapy is flawed because
the diagnosis has not been established. Not all patients with hematemesis who present to
the emergency department require inpatient management. Those with normal hemoglobin,
no physiological compromise, and who are able to follow up reliably can be dismissed for
outpatient follow-up. The Glasgow-Blatchford score is a screening tool that stratifies the
risk of a patient with an acute upper gastrointestinal bleed to require a blood transfusion
or endoscopic intervention. The score includes blood urea nitrogen (BUN <6.5 mmol/L;
normal range 7-20 mg/dL), hemoglobin(> 13.0 g/dL; normal range 13.5-17.5 g/dL), sys-
tolic blood pressure (> 110 mm Hg), pulse (<I 00 beats per minute), melena, syncope, liver
disease, and cardiac failure. This patient's tachycardia and anemia suggest that inpatient
management is indicated.
A recent study included cirrhotic patients with upper gastrointestinal bleeding. A re-
strictive policy using hemoglobin level of 7 g/dL versus a liberal policy of 9 g/dL was
compared. Outcomes were better for the restrictive policy in Child A and B patients and
equivalent for Child C patients. Thus, transfusion to a hemoglobin of I 0 g/dL would not
be necessary.
---
· · - - - ..
184 CATEGORY 3 - PART 111
References
I. Villanueva C, Coloma A, Bosch A, et al. Transfusion strategies for acute upper gastro-
intestinal bleeding [published correction appears in N Engl J Med. 2013;368(24):2341].
N Engl J Med. 2013;368(1):11-21.
2. Sarin N, Monga N, Adams PC. Time to endoscopy and outcomes in upper gastrointesti-
nal bleeding. Can J Gastroenterol. 2009;23(7):489--493.
3. Laine L, Jensen DM. Management of patients with ulcer bleeding. Am J Gastroenterol.
2012; I 07(3):345-360.
4. Blatchford 0, Murray WR, Blatchford M. A risk score to predict need for treatment for
upper-gastrointestinal haemorrhage. Lancet. 2000;356(9238): 1318-1321.
5. Cremers I, Ribeiro S. Management of variceal and nonvariceal upper gastrointestinal
bleeding in patients with cirrhosis. Therap Adv Gastroenterol. 20 l 4;7(5):206-216.
CRITIQUE27
The diagnosis of gastroparesis is defined by nonmechanical delayed emptying time seen in
the context of a spectrum of symptoms that includes epigastric pain, early satiety, nausea,
vomiting, and weight loss. Approximately 25% of cases are associated with diabetes. Other
disorders also carry an association with symptomatic delayed gastric emptying, including
connective tissue disorders, hypothyroidism, medications, and postsurgical complications.
Transient gastroparesis may be seen in association with viral syndromes.
The physiological abnormalities seen in individuals with gastroparesis are diverse and
can involve failure in tone or coordination of contraction in all parts of the stomach. Not
surprisingly, 50o/o of cases are deemed to be idiopathic. Gastric emptying scintigraphy is
the key diagnostic test. If abnormal gastric emptying at 4 hours is confirmed, therapeutic
attention should be directed to optimization of medically indicated targets. Prokinetic and
anti emetic agents are the mainstays of first-line therapy.
Intrapyloric botulinum toxin A injection is used to treat gastroparesis-associated spastic
etiologies, which may be more common in children. In a small, randomized, crossover
study, botulinum toxin A injection was not superior to placebo. Intrapyloric botulinum
toxin A injection is not a part of the diagnostic criteria.
If medical approaches fail, patients may be referred for surgical procedures, such as gas-
tric stimulator implantation, gastric resection, or pyloroplasty. Surgical procedures should
be reserved for those circumstances in which there is clear failure of intensive and expert
medical intervention. Further nutritional reconstitution should be addressed before pro-
ceeding to surgical intervention.
References
1. Camilleri M, Parkman HP, Shafi MA, Abell TL, Gerson L; American College ofGastro-
enterology. Clinical guideline: management of gastroparesis. Am J Gastroenterol.
2013;108(1): 18-37.
2. Parkman HP, Yates K, Hasler WL, et al; National Institute of Diabetes and Digestive and
Kidney Diseases Gastroparesis Clinical Research Consortium. Clinical features of idio-
pathic gastroparesis vary with sex, body mass, symptom onset, delay in gastric empty-
ing, and gastroparesis severity. Gastroenterology. 2011; 140(1):I01-115.
3. Stein B, Everhart KK, Lacy BE. Gastroparesis: a review of current diagnosis and treat-
ment options.JC/in Gastroenterol. 2015;49(7):550-558.
4. Sarosiek I, Davis B, Eichler E, McCallum RW. Surgical approaches to treatment of
gastroparesis: gastric electrical stimulation, pyloroplasty, total gastrectomy and enteral
feeding tubes. Gastroenterol Clin North Am. 2015;44(1):151-167.
CATEGORY 3 - PART III
185
CRITIQUE28
Therapeutic decisions in cancer care start with a thorough understanding of the clinical
stage. The complexities of staging are significant, so reference to American Joint Commit-
tee on Cancer Staging Guidelines and National Comprehensive Cancer Network Guide-
lines assists all clinicians caring for cancer patients.
This patient is presenting with a T4 (invasion into an adjacent organ), NI (metastasis to
1-3 regional lymph nodes), MO (no distant metastases) small bowel cancer-clinical stage
3 duodenal carcinoma (figure 28.2). Although the operation at hand, a pancreaticoduode-
nectomy, carries a higher morbidity risk than most small bowel resections, the potential
oncological benefits are similar. Diffuse lymphadenopathy (N2) or metastatic disease (Ml)
would be contraindications to radical resection. Local invasion that precludes RO resection
(no residual disease postresection) is also a contraindication to resection. This patient's
significant weight loss should prompt the surgeon to consider a nutritional evaluation and
attempt at optimization before surgery.
Palliation of malignant gastric outlet obstruction is not indicated in this patient, as radi-
cal resection is feasible and appropriate. In patients with contraindications to resection, pal-
liation is a challenge. The chronic obstruction that often precedes the presentation leaves
the stomach dysfunctional, and loop gastrojejunostomy may not function immediately. En-
doscopic stenting of the duodenum is preferred to gastrostomy or surgical bypass, if it is
technically feasible.
The patient's celiac disease is the principle risk factor. Other risk factors for duodenal
adenocarcinoma include familial adenomatous polyposis, Gardner, Lynch, and juvenile
polyposis, and Peutz-Jeghers syndromes, as well as Crohn disease. A tissue diagnosis can
be obtained with endoscopy. CT scan will evaluate the local extent of the tumor, regional
lymphadenopathy, and distant metastases. Endoscopic ultrasound aids in the diagnosis of
malignant lymphadenopathy and documents metastases outside the field of resection. PET
scan has not been validated in this disease process and, therefore, has limited utility.
References
1. National Cancer Institute. PDQ® Small Intestine Cancer Treatment. Bethesda, MD:
National Cancer Institute. Updated: Apr 28, 2015. Accessed: Dec 29, 2015.
2. Cloyd JM, Norton JA, Visser BC, Poultsides GA. Does the extent of resection impact
survival for duodenal adenocarcinoma? Analysis of 1,611 cases. Ann Surg Oneal.
2015;22(2):573-580.
3. Bowdle PD, Jalal PK, Holmes GK, Houlston RS. Primary small-bowel malignancy in
the UK and its association with coeliac disease. QJM. 2003;96(5):345-353.
4. Kawahira H, Miura F, Saigo K, et al. Survival predictors of patients with primary duode-
nal adenocarcinoma. Int Surg. 2011 ;96(2):111-116.
CRITIQUE29
This patient's clinical stage is 0 (Tis, NO, MO). He presents with advanced age and at least
one important medical comorbidity. The advent of endoscopic techniques to address local
neoplastic changes in the distal esophagus has dramatically changed the therapeutic options.
The recommendation for local therapy in stage 0 disease is rather straightforward. The com-
bination of endomucosal resection with ablation of the Barrett esophagus, the fertile ground
in which further carcinomatous changes could arise, is now a standard recommendation and
has replaced photodynamic therapy, regardless of age and comorbidities. There could be
stage migration on analysis of the endomucosal resection specimen (approximately 50o/o of
cases) in that an invasive component or poorly differentiated histology may be identified.
Such findings would alter the therapeutic options and possible recommendations.
Radical resection is reserved for those patients medically fit with locoregional inva-
sive adenocarcinoma. External beam radiation alone is rarely indicated and never for
stage 0 disease. Radiation is most often used in combination with chemotherapy either
as a neoadjuvant or as definitive palliative therapy for patients with locally advanced,
symptomatic disease.
The term carcinoma in situ has been abandoned in favor of high-grade dysplasia, which
includes all noninvasive neoplastic epithelia. Carcinoma in situ is no longer used for co-
lumnar mucosae anywhere in the gastrointestinal tract.
References
I. Sgourakis G, Gockel I, Lang H. Endoscopic and surgical resection ofTJa/Tlb esopha-
geal neoplasms: a systematic review. World J Gastroenterol. 2013; 19(9): 1424-1437.
2. National Comprehensive Cancer Network. NCCN Guidelines Esophageal and Esopha-
gogastric Junction Cancers. Version 3.2015. Fort Washington, PA: National Compre-
hensive Cancer Network.
CRITIQUE30
The surgical approaches to resection of the third and fourth portions of the duodenum
are challenging. Although the duodenum is biologically small bowel, the derivation of its
blood supply; its lymphatic drainage; and the close association to the pancreas, root of the
mesentery, and great vessels make resection challenging.
When treating cancer of the third and fourth portions of the duodenum, surgeons should
attend to the same principles as with other small bowel cancers. En bloc resection of ad-
jacent organs is necessary only for T4 tumors with direct extension. Lymph node basin
retrieval follows the blood supply and does not include either the pancreas or spleen. RO re-
section (no residual disease post resection) can be accomplished with a segmental approach
CATEGORY 3 - PART !II
187
in this patient. Reconstruction of the jejunum to the second portion of the duodenum is an
anatomically satisfying solution that maintains a secure blood supply to the anastomosis
and minimizes complexity. The surgeon must be aware of and preserve the Ampulla of
Yater.
The first jejuna! branch of the superior mesenteric vein drains the proximal jejunum. It
is a consideration when widely kocherizing the head of the pancreas and dissecting the in-
frapancreatic superior mesenteric vein for pancreaticoduodenectomy and when reflecting
the small bowel mesentery in exposing the third portion of the duodenum.
Patients with locally advanced disease may benefit from a neoadjuvant approach. but
this approach has not been established as a standard. Small bowel and duodenal carcinoma
are too rare and therefore incompletely studied to have a sufficient understanding of the
most effective chemotherapy and chemoradiotherapy protocols.
References
I. National Comprehensive Cancer Network. NCCN Guidelines Colon Cancer. Version
2.2016. Fort Washington. PA: National Comprehensive Cancer Network.
2. Chung WC, Paik CN, Jung SH, et al. Prognostic factors associated with survival in patients
with primary duodenal adenocarcinoma. Korean J Intern Med 2011 ;26(1):34-40.
3. Sista F, Santis GD, Giuliani A, et al. Adenocarcinoma of the third duodenal portion: case
report and review ofliterature. World J Gastrointest Surg. 2012;4(1 ):23-26.
4. Papavasiliou P, Arrangoiz R, Zhu F, Chun YS, Edwards K, Hoffinan JP. The anatomic
course of the firstjejunal branch of the superior mesenteric vein in relation to the supe-
rior mesenteric artery. Int J Surg Oncol. 2012;2012:538769.
CRITIQUE 31
The spectrum of inherited colorectal cancer syndromes is largely underappreciated and
therefore underdiagnosed. The ability to identify patients and families with these genetic
disorders can have a profound impact not only on the natural history of cancer in the family
but also the immediate-term clinical decisions for the patient being treated. For instance,
the presence of microsatellite instability tends to predict resistance to fluoropyrimidine
(e.g., 5-FU) based protocols. The resistance to fluoropyrimidine (e.g., 5-FU) may be due to
incorporation of5-FU metabolites into DNA, overexpression ofthymidylate synthetase, or
dihydropyrimidine dehydrogenase in deficient mismatch repair tumors. Further, the failure
to identify high-risk cohorts can result in missed opportunities for prevention, delayed
cancer diagnoses, or, conversely, inappropriate screening for those who are not high risk.
Most guidelines recommend that all resected tumors undergo mutation screening, and the
approach is cost-effective.
Lynch syndrome, once called hereditary nonpolyposis colorectal cancer (HNPCC), is an
inherited disorder caused by mutations in the MLH I, MSH2, MSH6, PMS2 (mismatch re-
pair (MMR) genes) or EPCAM (a gene adjacent to MSH2). Immunohistochemistry (IHC)
and microsatellite instability (MS!) analyses, either by themselves or in conjunction, are
done on tissue to identify individuals at risk for Lynch syndrome. Those with a germline
mutation are, by definition, Lynch syndrome patients.
MSI-H (microsatellite instability-high) in tumors refers to changes in 2 or more of the
5 microsatellite markers. More than 90% of Lynch syndrome tumors are MSI-H or lack
expression of at least one of the MMR proteins by IHC. Ten to fifteen percent of sporadic
colon cancers also exhibit abnormal IHC. Therefore, the presence of an abnormal MLHl
IHC test increases the possibility of Lynch syndrome but does not make a definitive di-
agnosis. When there is loss of expression of MLHl (abnormal MLH IHC test), further
testing for BRAF mutation and MLHl hypermethylation testing is recommended. If these
188 CATEGORY 3 - PART 111
are positive, then genetic testing can stop and a sporadic cases of colon cancer is present.
If these tests are negative, they are followed by further MS! testing for MSH2, MSH6, or
PMS2 genes. An inherited syndrome is likely, and further germline genetic analysis of the
appropriate gene is recommended.
Amsterdam or Bethesda criteria are clinical and pathological criteria used to alert cli-
nicians to the possibility of a heritable pattern. Any one of these criteria is sufficient to
proceed with tumor screening. Five to ten percent of cases are due to recognized germ-
line mutations. American Society of Clinical Oncology has endorsed European Society of
Medical Oncology clinical practice guidelines.
For Amsterdam criteria, each of the following criteria must be fulfilled:
• 3 or more relatives with an associated cancer (colorectal cancer, or cancer of the en-
dometrium, small intestine, ureter or renal pelvis)
• 2 or more successive generations affected
• 1 or more relatives diagnosed before the age of 50
• I should be a first-degree relative of the other 2
• Familial adenomatous polyposis (PAP) should be excluded in cases of colorectal car-
cinoma
• Tumors should be verified by pathologic examination
The Revised Bethesda guidelines are as follows:
• Colorectal carcinoma (CRC) diagnosed in a patient who is younger than 50 years old
• Presence of synchronous or metachronous CRC or other Lynch syndrome-associated
tumors, regardless of age
• CRC with high microsatellite instability histology diagnosed in a patient younger
than 60
• CRC diagnosed in 1 or more first-degree relatives with a Lynch syndrome-associated
tumor, with 1 of the cancers being diagnosed at less than 50 years of age
• CRC diagnosed in 2 or more first-degree or second-degree relatives with Lynch syn-
drome-associated tumors~ regardless of age
References
1. Stoffel EM, Mangu PB, Gruber SB, et al: American Society of Clinical Oncology; Euro-
pean Society of Clinical Oncology. Hereditary colorectal cancer syndromes: American
Society of Clinical Oncology Clinical Practice Guideline endorsement of the famil-
ial risk-colorectal cancer: European Society for Medical Oncology Clinical Practice
Guidelines. J Clin Oneal. 2015;33(2):209-217.
2. Balmai'ia J, Balaguer F, Cervantes A, Arnold D; ESMO Guidelines Working Group.
Familial risk-colorectal cancer: ESMO Clinical Practice Guidelines. Ann Oneal.
2013;24 (suppl 6):vi73-vi80.
CRITIQUE32
In the absence of symptoms, most patients with small bowel B-cell lymphomas are treated
with chemotherapy with or without radiation therapy. Surgery is reserved for those who fail
treatment or who suffer from bleeding, perforation, or obstruction. T-cell lymphomas are
traditionally chemoresistant and will progress to symptoms of obstruction or perforation if
not resected. Resection is indicated at any onset of symptoms, because progression to life-
threatening hemorrhage or perforation portends a dismal prognosis~ regardless of cell type.
CATEGORY 3 - PART III 189
Early stage B-cell lymphoma of mucosa-associated lymphoid tissue (MALT) type tumors
can be successfully treated with He/icobacter pylori antibiotic therapy. Neither endoscopic
ablation nor radical surgery (pancreaticoduodenectomy) is indicated, because the patient de-
scribed is minimally symptomatic and is likely to respond to noninvasive treatment.
References
1. Bilimoria KY, Bentrem DJ, Wayne JD, Ko CY, Bennett CL, Talamonti MS. Small bowel
cancer in the United States: changes in epidemiology, treatment, and survival over the
last 20 years. Ann Surg. 2009;249(1):63-71.
2. Kunitake H, Hodin R. The management of small bowel tumors. In: Cameron JK,
Cameron AM, eds. Current Surgical Therapy. l l th ed. Philadelphia, PA: Elsevier Saun-
ders;2014: 122-128.
3. McKenzie S, Evers BM. Small intestine. In: Townsend CM, Beauchamp RD, Evers
BM, Mattox KL, eds. Sabiston Textbook of Surgery: The Biological Basis of Modern
Surgical Practice. 19th ed. Philadelphia, PA: Elsevier Saunders;2012:1227-1279.
CRITIQUE33
There are many potential complications after esophagectomy, including anastomotic leak,
chylothorax, cardiovascular events, thromboembolic disease, and deep and superficial sur-
gical site infections. These complications can be devastating, because patients may have
limited ability to overcome them due to the nature of the original disease and frequent as-
sociated comorbidities. Described risk factors for anastomotic leak include malnutrition.
cardiovascular disease, tobacco use, steroids, renal failure, and tumor characteristics (e.g.,
location, margins).
The added risk of neoadjuvant chemotherapy with or without radiation has varied in
small, underpowered trials. A European report of nearly 3000 patients looked specifically
at this question. No difference in mortality, anastomotic leak, or pulmonary complica-
tions was observed in patients who had esophagectomy compared with those who had
esophagectomy with neoadjuvant chemoradiotherapy. There was a reported trend to more
thromboembolic and cardiovascular complications in the chemoradiotherapy and surgery
patients. Chylothorax occurred significantly more often, presumably due to the resulting
injury and inflammation of the radiation therapy.
References
I. Gronnier C, Trechot B, Duhamel A, et al. Impact ofneoadjuvant chemoradiotherapy on
postoperative outcomes after esophageal cancer resection: results of a European multi-
center study. Ann Surg. 2014;260(5):764-770; discussion 770-771.
2. Ronellenfitsch U, Schwarzbach M, Hofheinz R, et al; GE Adenocarcinoma Meta-
analysis Group. Perioperative chemo(radio)therapy versus primary surgery for resect-
able adenocarcinoma of the stomach, gastroesophageal junction, and lower esophagus.
Cochrane Database Syst Rev. 2013;5:CD008107.
CRITIQUE34
The surgical dictum that all paraesophageal hernias (PEH), even those that are asymp-
tomatic, should be repaired electively was challenged by several studies. The rationale for
elective repair of all PEHs was the prevention of potential life-threatening complications
such as gastric torsion leading to obstruction, strangulation, and emergent operation. Fur-
190 CATEGORY 3 - PART lll
thermore, the perception that minimally invasive techniques offer a less morbid approach
to surgical correction of PEH has traditionally added support for elective PEH repair.
Recent studies suggest that the annual risk of a complication related to an asymptomatic
PEH is less than 1.5%. Based on this information, fewer than I in 5 asymptomatic patients
65 years and older, and fewer than I in 10 patients at age 85 years and older, benefit from
elective PEH repair.
It is true that most large PEH can be repaired using minimally invasive surgery. The no-
tion that PEHs, like the one shown in this patient's CT, would require a thoracotomy or lapa-
rotomy has fallen out of favor (figure 34.2). Still, elective PEH repair, even when performed
using minimally invasive techniques~ is not benign. Long-term complications such as vagal
nerve injury and hernia recurrence are significant and lead many experienced laparoscopic
surgeons to avoid elective PEH repair, particularly in asymptomatic elderly patients.
In a patient with a symptomatic PEH (e.g., severe reflux, postprandial pain, anemia),
additional workup may include esophagogastroduodenoscopy, esophagrarn, or manornetry.
However, when a patient lacks any symptoms related to their PEH he or she would not
require additional work-up.
Some studies suggest that endoscopic reduction and fixation via percutaneous endoscopic
gastrostomy is useful for fragile, elderly patients with symptomatic PEH who may not toler-
ate operative repair. This strategy would not be beneficial for an asymptomatic patient.
References
I. Stylopoulos N, Gazelle GS, Rattner DW. Paraesophageal hernias: operation or observa-
tion? Ann Surg. 2002;236(4):492-500; discussion 500-501.
2. Davis SS Jr. Current controversies in paraesophageal hernia repair. Surg Cl in North Am.
2008;88(5):959-978.
CATEGORY 3 - PART Ill 191
3. Kercher KW, Matthews BD, Ponsky JL, et al. Minimally invasive management of
paraesophageal herniation in the high-risk surgical patient. Am J Surg. 2001;182(5):
510-514.
CRITIQUE35
Postprocedure follow-up is important after Heller myotomy for achalasia. A timed barium
esophagram is the best imaging study to assess the effectiveness of the myotomy. Patients
with concordance of symptom improvement and minimal contrast retention on barium
esophagram have good long-tenn improvement, whereas patients with discordance of im-
proved symptoms but poor contrast emptying have a worse long-term prognosis and are
more prone to return with symptoms requiring reintervention.
Although timed barium esophagram is a relatively sensitive test to identify patients with
a poor result from myotomy, it is also nonspecific. Other studies need to be performed to
identify the reason for persistent dysphagia after Heller myotomy. The 2 most common
reasons for persistent dysphagia are (1) incomplete myotomy, and (2) obstruction due to
the wrap being too tight. The best way to distinguish between these 2 entities is to perform
esophageal manometry with amyl nitrite challenge. An incomplete myotomy will relax
with amyl nitrite, whereas a fixed obstruction related to the wrap will not change.
Esophagogastroduodenoscopy (EGD) is a useful study to perform ifthe persistent dys-
phagia is related to a fixed obstruction at the wrap. EGD can determine whether the nature
of the obstruction mandates surgical reintervention (e.g., slipped fundoplication) rather
than an attempt at pneumatic dilation. Infrequently, the symptoms of achalasia may be con-
fused with those of gastroesophageal reflux disease (GERD). However, a careful history
should be able to differentiate between the two conditions. A 24-hour pH probe is useful for
objectifying GERD but is not useful in evaluating patients with achalasia. Endoscopic ul-
trasound is very sensitive at characterizing the encroachment of tumor into the esophageal
musculature, but it does not play a significant role in the evaluation of achalasia, which is
a functional disturbance of the esophageal musculature.
References
1. Pandolfi.no JE, Kahrilas PJ. Presentation, diagnosis, and management of achalasia. Clin
Gastroenterol Hepatol. 2013; 11 (8):887-897.
2. Paterson WG, Goyal RK, Habib FI. Esophageal motility disorders. GI Motility Online.
2006. Available at: http://www.nature.com/gimo/contents/ptl/full/gimo20.html
CRITIQUE 36
Recommended treatment for cervical esophageal cancer has changed. The National
Comprehensive Cancer Network currently recommends definitive chemoradiation for all
esophageal cancers located within 5 cm of the upper esophageal sphincter (UES). His-
torically, pharyngolaryngoesophagectomy (PLE) with or without adjuvant radiotherapy
was recommended for esophageal cancer within 5 cm of the UES. Unfortunately, PLE
resulted in significant morbidity and mortality and left the patient with a permanent ter-
minal tracheostomy.
Several studies have shown that patients with proximal esophageal squamous cell carci-
noma treated with high-dose radiation therapy (60--68 Gy) and concomitant chemotherapy
had similar survival rates to those treated with radical surgery while sparing the morbidity
of a laryngectomy. In one study, the overall median survival for patients treated with PLE
was 19.9 months compared with the chemoradiation group patients, who had a median
192 CATEGORY 3 - PART lll
survival of24.9 months. The 2-year survivals of the 2 groups were 37.6% and 46.9%, re-
spectively. Because of the very limited survival advantage, and the extensive morbidity of
PLE, surgery is no longer recommended for primary therapy.
Endoscopic mucosa! resection (EMR) may be suitable for patients with high-grade
dysplasia (Tis), tumor-invading lamina propria/muscularis mucosae (Tia), and possibly
tumor-invading the submucosa (Tl b). For superficial lesions, multiple studies have shown
EMR to have similar oncologic effectiveness as esophagectomy. EMR is not considered
adequate once the tumor progresses deeper into the muscularis propria (T2).
Transhiatal esophagectomy would not be appropriate for tumors of the cervical esopha-
gus, because laryngectomy would be required to achieve complete resection with appropri-
ate margins.
References
1. Burmeister BH, Dickie G, Smithers BM, Hodge R, Morton K. Thirty-four patients with
carcinoma of the cervical esophagus treated with chemoradiation therapy. Arch Otolar-
yngol Head Neck Surg. 2000; 126(2):205-208.
2. National Comprehensive Cancer Network. NCCN Guidelines Esophageal and Esopha-
gogastric Junction Cancers. Version 3.2015. Fort Washington, PA: National Compre-
hensive Cancer Network.
CRITIQUE37
Optimal surgical management of esophagogastric junction (EGJ) adenocarcinoma depends
on the extensiveness of the tumor and its anatomic relationship to the gastroesophageal
junction.
Siewert classified EGJ adenocarcinoma into 3 types based on the anatomic location of
the epicenter of the tumor and the extent of the mass (figure 37.1). Siewert type I tumors
are defined as adenocarcinoma of the distal esophagus with the tumor center located within
1-5 cm above the EGJ. These tumors are primarily esophageal cancers, although they may
infiltrate the EGJ from above. Siewert type II tumors are defined as the true carcinoma of
the cardia, arising immediately at the EGJ with the tumor center within 1 cm above and 2
cm below the EGJ. Siewert type III tumors are defined as primarily gastric carcinoma with
the tumor center between 2-5 cm distal to the EGJ, although these tumors may cross the
EGJ and encroach upon the distal esophagus from below. The patient described in this item
has a Siewert type III adenocarcinoma.
The surgical approach to all types of EGJ cancer has the common goal of achieving
complete macroscopic and microscopic tumor resection. In general, Siewert type III tu-
mors with extensive gastric involvement require total gastrectomy with transhiatal resec-
tion of the distal esophagus. Esophagectomy (whether transhiatal or Ivor Lewis) would not
achieve a sufficient distal margin for these types of tumors. Proximal gastrectomy would
not achieve proximal or distal margins. In this particular patient, with tumor extending
proximal to the EGJ, a distal esophagectomy would need to be performed en bloc with total
gastrectomy to achieve a sufficient proximal and distal margins.
Answer: (B) Distal esophagectomy and total gastrectomy with Roux-en-Y esophagoje-
junostomy
CATEGORY 3 - PART rn 193
References
1. Kim JH, Park SS, Kim J, et al. Surgical outcomes for gastric cancer in the upper third of
the stomach. WorldJ Surg. 2006;30(10):1870-1876; discussion 1877- 1878.
2. Levy RM, Trivedi D, Luketich JD. Minimally invasive esophagectomy. Surg Clin North
Am. 2012;92(5):1265- 1285.
3. Rudiger Siewert J, Feith M, Werner M, Stein HJ. Adenocarcinoma of the esophagogas-
tric junction: results of surgical therapy based on anatomical/topographic classification
in 1,002 consecutive patients. Ann Surg. 2000;232(3):353- 361.
4. Ketner EP, Chu QD, Karpeh MS Jr., Khusbalani NJ. Gastric cancer. In: Chu QD, Gibbs
JF, Zibari GB, eds. Surgical Oncology: A Practical and Comprehensive Approach. New
York, NY: Springer Science+Business Media;2015: I 95-217.
CRITIQUE 38
The 2012 Revised Atlanta Classification System for pancreatitis is listed in table 38. 1.
Based on the classification, the location of the fluid collection, and the timing of the inter-
vention, different types of procedures can be recommended,
Percutaneous drainage is indicated for fluid collections more than 4 weeks in dura-
tion and that are separate from the pancreas itself. Any other fluid collection could be in
continuity with the pancreatic ductal system, and percutaneous drainage could lead to the
development of a pancreatic-cutaneous fistula. Percutaneous drainage can also be used
as the first step in managing infected necrosis if the surgeon is planning a video-assisted
retroperitoneal debridement (YARD). YARD is a less invasive way to manage infected
pancreatic necrosis. The procedure is initiated by first placing percutaneous drains into
the infected area and then surgically dilating the tracts and using minimally invasive tech-
niques to debride the retroperitoneum and establish more complete drainage. YARD should
be limited to patients with infected pancreatic necrosis.
194 CATEGORY 3 - PART Ill
Symptomatic pseudocysts and walled off necrosis adjacent to the pancreas can be man-
aged with either surgical or endoscopic techniques) based on the location of the fluid col-
lection with respect to the stomach. Fluid collections that do not abut the stomach should
be managed with a cystojejunostomy. This can be performed with either a laparoscopic or
an open technique.
Type of Fluid Time CT Appearance
Classification Collection {weeks) Necrosis of Fluid
No fluid NIA No NIA
Fluid collections that abut the stomach were traditionally managed with a surgically cre-
ated cystgastrostomy and do not require a cystojejunostomy (figure 38.2). The traditional
approach has been via laparotomy, an anterior gastrotomy, and creation of a cystgastrostomy.
The cyst is usually identified by aspiration, or more recently ultrasound, and then an incision
is made in the posterior wall of the stomach through the cyst wall. Cyst contents are evacu-
ated and the wall of the cyst and stomach are sutured, maintaining the cystgastrostomy.
Figure 38.2
CATEGORY 3 - PART III 195
References
1. Dumonceau JM, Macias-Gomez C. Endoscopic management of complications of
chronic pancreatitis. World J Gastroenterol. 2013; 19(42):7308-7315.
2. Baron TH. Endoscopic pancreatic necrosectomy. Gastroenterol Hepatol (N Y). 2008;
4(9):617--020.
3. Varadarajulu S. Bang JY, Sutton BS, Trevino JM, Christein JD, Wilcox CM. Equal effi-
cacy of endoscopic and surgical cystogastrostomy for pancreatic pseudocyst drainage in
a randomized trial. Gastroenterology. 2013; 145(3):583.el-590.el.
4. Bakker OJ, van Santvoort HC, van Brunschot S, et al; Dutch Pancreatitis Study Group.
Endoscopic transgastric vs surgical necrosectomy for infected necrotizing pancreatitis:
a randomized trial. JAMA. 2012;307(10): 1053-1061.
5. Thoeni RF. The revised Atlanta classification of acute pancreatitis: its importance for
the radiologist and its effect on treatment. Radiology. 20 l 2;262(3):751-764.
CRITIQUE39
A threshold sinusoidal pressure of 12 mm Hg is necessary for ascites to develop. Indirect
confirmation of this threshold comes from studies of the serum~ascites albumin gradient,
which correlates with the hepatic venous pressure gradient (HYPG) in the differential di-
agnosis of ascites. The serum-ascites albumin gradient cutoff level for cirrhotic ascites is
greater than 11 g/L, which corresponds to an HVPG of 11-12 mm Hg.
Sodium retention is another mechanism in cirrhotic patients with portal hypertension.
Sodium is retained secondary to splanchnic and systemic vasodilation, which is common
in patients with cirrhosis. The vasodilation reduces effective arterial blood volume, which
will be the renin-angiotensin-aldosterone system, resulting in sodium retention.
Sodium restriction and diuretics are initial therapy for cirrhotic ascites attempting to
achieve a negative sodium balance. These interventions fail in 10-20% of cirrhotic patients.
Large~volume paracentesis is the standard treatment for refractory ascites. It removes
ascetic fluid but does not modify any of the mechanisms that lead to ascites formation.
Recurrence of ascites is almost universal after large-volume paracentesis, unless there is
an improvement in liver disease. Large-volume paracentesis may also negatively affect the
hemodynamic status of cirrhotic patients. A syndrome called "postparacentesis circulatory
dysfunction" (PCD) is defined by an increase in plasma renin activity evident 6 days after
large-volume paracentesis. This condition is associated with a more rapid reaccumulation
of ascites, a decrease in renal function, and a higher mortality rate compared with patients
who do not develop this condition. PCD appears to be reduced to approximately 15% when
albumin is used to replace some of the ascetic fluid.
196 CATEGORY 3 - PART III
References
!. Bloom S, Kemp W, Lube! J. Portal hypertension: pathophysiology, diagnosis and
management. Intern Med J 2015;45(! ): 16-26.
2. Rosemurgy AS, Zervos EE, Clark WC, et al. TIPS versus peritoneovenous shunt in the
treatment of medically intractable ascites: a prospective randomized trial. Ann Surg.
2004;239(6):883-889; discussion 889-891. PM!
Category 3 - Part IV
Alimentary Tract
Items 1-40
DIRECTIONS: Each of the questions or incomplete statements is followed by 5 suggested
answers or completions. Select the answer that is the BEST in each case and fill in the
space containing the corresponding letter on the answer sheet.
Figure 1.1
3. Which of the following principles guides the initial surgical therapy for noninvasive
Paget disease of the anal canal?
(A) Mucosal stripping guided by iodine staining
(B) Local excision with margins 5-mm from the skin abnormalities
(C) Wide local excision with margins 2-cm from the skin abnormalities
(D) Mapping biopsies followed by wide local excision with 1-cm microscopic margins
(E) Complete excision with sentinel lymph node biopsy
CATEGORY 3 - PART IV 199
4. An otherwise healthy. 32-year-old woman with a history of Crohn disease has been
asymptomatic on infliximab for 8 years. Four months ago, she developed intermittent
crampy abdominal pain that has gradually worsened over the last 3 months. Upper
gastrointestinal series with small bowel follow-through revealed 4 partially obstruct-
ing strictures, each approximately 10 cm apart, beginning 50 cm from her ileocecal
valve. Her gastroenterologist added prednisone 30 mg/day to her regimen, which re-
sulted in mild improvement in her symptoms. However, 1 week after her most re-
cent infliximab injection, she developed nausea, vomiting, and obstipation. You are
consulted regarding potential surgical intervention. In addition to nutritional support,
which of the following would you recommend?
(A) Discontinue current immunosuppressive regimen, perform surgery in 6 weeks
(B) Discontinue biologics, continue steroids, perform surgery in 6 weeks
(C) Continue current immunosuppressive regimen, perform immediate surgery
(D) Continue biologics. taper steroids. perform surgery within 1 week
(E) Increase steroid dose
5. Which of the following statements regarding the diagnosis and management of acute
appendicitis is true?
(A) Wound infection rates are equivalent for laparoscopic or open appendectomy for
uncomplicated appendicitis.
(B) Intra-abdominal abscess formation is higher for laparoscopic compared with
open appendectomy for patients with complicated appendicitis.
(C) Use of oral and intravenous contrast enhances the accuracy of CT scan
diagnosis.
(D) The incidence of perforation correlates with elapsed time between presentation
and operative intervention.
(E) The cumulative risk ofrecurrence with perforation after nonoperative
management of acute appendicitis is 1Oo/o.
6. Which of the following screening modalities has the highest sensitivity and specificity
for detecting early colon cancer?
(A) Multi-target DNA stool testing
(BJ Fecal immunochemical test
(C) Colonoscopy
(D) Serum micro-RNA screening
(E) Double contrast barium enema
7. In addition to endoscopy, which of the following options delineates the standard evalu-
ation of gastroesophageal reflux disease to be undertaken before antireflux surgery?
(A) Manometry, esophageal pH monitoring, and oropharyngeal pH monitoring
(B) Manometry, esophageal pH monitoring, and gastric emptying study
(C) Manometry, esophagography, and gastric emptying study
(D) Manometry, esophageal pH monitoring, and esophagography
(E) Oropharyngeal pH monitoring, esophagography. and gastric emptying study
200 CATEGORY 3 - PART IV
9. A 52-year-old white man has a significant past medical history for having a prophylac-
tic total colectomy for familial adenomatous polyposis. On a routine screening upper
endoscopic examination, multiple polyps ( 15) were noted in the duodenal bulb and
ampulla (figure 9.1); the largest measures 9 mm. Biopsies reveal tubular adenomas
with mild dysplasia. Which of the following statements is true for this patient?
(A) Pancreaticoduodenectomy (Whipple procedure) is the treatment of choice.
(B) Duodenal cancer is a major cause of mortality.
(C) Desmoid tumor is unlikely to be a major cause of death.
(D) The major genetic abnormality is a defect in the mismatch repair gene.
(E) Nonsteroidal anti-inflammatory drugs have significantly decreased the size and
number of duodenal polyps.
10. Which of the following statements is true regarding treating esophageal perforation
with stenting?
(A) Plastic and metal stents have similar rates of migration.
(B) The mortality rate with endoscopic management is 5-10%.
(C) Stent-associated perforation occurs in less than 5% of cases.
(D) Stenting is indicated for esophageal leaks independent of luminal diameter
(E) Stenting is associated with a 25% long-tenn stricture rate
CATEGORY 3 ~PART IV 201
11. A 67-year-old woman with a longstanding history of peptic ulcer disease presents with
an acute perforation. Which of the following statements is true regarding the manage-
ment of this patient's condition?
(A) Mortality is higher in obese patients undergoing surgery for perforated peptic
ulcer.
(BJ Treatment delays ofup to 10 hours do not affect survival.
(CJ Thirty-day mortality rates are greater than 20%.
(DJ Laparoscopic repairs have a higher risk of complications compared with open
repairs.
(E) The risk of recurrence is reduced to 25% with Helicobacter pylori eradication.
Items 12-13
A 33-year-old man attempted to swallow a large piece of meat. He now presents with
drooling and dysphagia but no strider. He has had several previous episodes of food '"get-
ting stuck," which all resolved spontaneously. He is otherwise healthy and does not smoke
or drink alcohol.
12. Which of the following is the most likely underlying esophageal pathology?
(A) Schatzki ring
(BJ Zenker diverticulum
(C) Nutcracker esophagus
(DJ Esophageal carcinoma
(E) Eosinophilic esophagitis
14. A 62-year-old woman was recently managed with a course of oral antibiotics for lower-
extremity cellulitis. She now presents with complaints of multiple watery bowel
movements. She is lethargic and confused with a distended abdomen. Her blood pres-
sure is 100150 mg Hg, and her heart rate is l l 0 beats per minute. She has a white blood
cell count of 32,000/mm3 (3600-l l200/mm3), her lactate is 0.28 mmol/dL (0.5-2.2
mmol/L). her creatinine is 2.l mg/dL (0.40-l.30 mg/dL), and her base excess is -6
mEq/L (-2 to +2 mEq/L). Her stool is positive for C/ostridium difficile by polymerase
chain reaction. She is empirically started on vancomycin per a nasogastric tube and
receives crystalloid resuscitation with minimal improvement over the next 2 hours
despite the addition of norepinephrine for a persistently low systolic blood pressure.
What is the next best step?
(A) Addition of intravenous metronidazole
(BJ Vancomycin enema
(CJ Operative intervention
(DJ Addition of epinephrine
(E) Steroid bolus
202 CATEGORY 3 - PART IV
15. A 32-year-old man known to be HIV-positive presents with a !-week history of in-
tractable diarrhea and mild abdominal pain and bloating. He reports the occasional
passage of some blood-streaked stools. He denies being out of the country, changing
his usual diet, or drinking contaminated water. He admits that he has been irregularly
compliant with his antiretroviral therapy. He is afebrile. Other than some mild ab-
dominal distention, generalized abdominal discomfort to palpation without rebound,
diminished bowel sounds, and the presence of a trace amount of blood on the examina-
tion finger after digital rectal examination, his physical exam is unremarkable. All his
laboratory studies are normal, other than a CD4 count of 50 cells/µL (>500 cells/µL).
The most likely etiology of his condition is
(A) C/ostridium difficile
(B) Cryptosporidium
(C) Cytomegalovirus
(D) Coccidiomycosis
(E) Mycobacterium avium
16. A 19-year-old male college student presents to the student health service with 2 weeks
of crampy abdominal pain, tenesmus, and bloody diarrhea. He reports no fever or
other constitutional symptoms. He has not had previous episodes, and he has not trav-
eled out of the country. His physical examination is unremarkable other than some
blood streaks and mucous identified by digital rectal examination. There is no other
significant medical or family history. Which of the following evaluations is most likely
to determine the diagnosis?
(A) Stool cultures
(B) Colonoscopy
(C) Stool DNA analysis
(D) Small bowel follow-through (contrast enterography)
(E) Triple contrast CT
17. An 84-year-old man with dementia and multiple comorbidities undergoes endoscopic
retrograde cholangiopancreatography with sphincterotomy (ERCP/ES) as treatment
for gallstone pancreatitis. Which of the following statements is true regarding gall-
stone pancreatitis?
(A) ERCP/ES as sole treatment is associated with a recurrent pancreatitis rate of 5%
within the first year.
(B) Patients who undergo ERCP/ES as sole treatment have a readmission rate of
20% within the first month.
(C) Patients undergoing no intervention have a mortality rate of25% within the first
year.
(D) Patients undergoing no intervention have a 75o/o risk of recurrent pancreatitis.
(E) Delayed cholecystectomy is associated with a risk ofrecurrent gallstone
pancreatitis less than 5o/o.
19. A 66-year-old woman presents with recently diagnosed rectal cancer. Which of the
following interventions is most likely to detect metastatic mesorectal lymph nodes on
staging evaluation?
(A) PET scan
(B) Transabdominal ultrasound
(C) Multiphase contrast CT
(D) Pelvic MRI
(E) Digital anorectal exam
21. Which of the following imaging characteristics of appendicitis is appropriate for non-
operative management with antibiotics alone?
(A) Presence of a fecalith
(B) Perforated appendicitis
(C) Right lower quadrant abscess
(D) Uncomplicated appendicitis
(E) Distended appendix with calcifications
22. A 42-year-old man presents with a history of hoarseness and chest pain. Endoscopy
reveals esophagitis, and 24-hour pH monitoring is positive for reflux. Manometry re-
veals normal peristalsis. He undergoes an unremarkable Iaparoscopic total fundoplica-
tion. Compared with a partial fundoplication, he will have a higher rate of which of the
following over the next 5 years?
(A) Dysphagia
(B) Satisfaction score
(C) Reoperation rate
(D) Esophagitis
(E) Use of proton pump inhibitors
25. A 67-year-old patient with a bleeding posterior duodenal ulcer fails endoscopic treat-
ment. He has received 4 units of packed red blood cells and continues to have bright
red blood through his nasogastric tube. His hematocrit measured 27% (43-52%). and
he remains hemodynamically normal. The next step in his management should be
(A) truncal vagotomy. antrectomy, and Billroth 2 reconstruction.
(B) laparoscopic highly selective vagotomy.
(C) nonselective beta-blockade.
(D) angiography and transarterial embolization.
(E) switching from omeprazole to pantoprazole.
26. A 58-year-old woman with vague abdominal pain, mild right lower quadrant tenderness,
and normal laboratory data, undergoes CT examination of the abdomen and pelvis. Rep-
resentative views are shown in figures 26.1 and 26.2.The patient should undergo
(A) intraperitoneal hyperthermic chemotherapy.
(B) cecotomy.
(C) appendectomy.
(D) right oophorectomy.
(E) repeat CT scan in 6 months.
Figure 26.I
CATEGORY 3 - PART IV 205
Figure 26.2
27. A 28-year-old woman was diagnosed with ulcerative colitis at age 24. Her initial bout
was treated with sulfasalazine and a tapering course of steroids, with an excellent re-
sponse. She has had minimal difficulty since and is maintained on sulfasalazine. She
presented 3 weeks ago with bloody diarrhea and cramping pain. After a repeat colo-
noscopy, which confirmed a relapse, she was started on a similar course of steroids but
has had little response to therapy. The best therapeutic option at this point is
(A) cyclosporine.
(B) steroid enemas.
(C) increase the steroid dose.
(D) total proctocolectomy.
(E) infliximab.
28. In which of the following situations should an oncologic large bowel resection be
performed?
(A) Dysplastic polyp
(B) Submucosal invasion of carcinoma in polyp neck
(C) A sessile polyp with upper third submucosal adenocarcinoma invasion
(D) Sessile lesion with adenocarcinoma removed piecemeal during endoscopy
(E) Well-differentiated carcinoma involving the stalk of the polyp resected with at
least 2 mm margin
29. A 52-year-old man with chronic kidney disease and poorly controlled diabetes pres-
ents to the emergency department with crampy abdominal pain and obstipation. He
has had no prior abdominal surgery and has not undergone prior colonoscopy. CT scan
imaging is obtained and confirms distal small bowel obstruction due to long segment
ileocolonic intussusception. The obstruction is complicated by bowel wall edema and
adjacent fluid. There is no pneumatosis or free peritoneal air to suggest perforation is
present. The next step in management should be
(A) hydrostatic reduction of intussusception.
(B) endoscopic reduction of intussusception.
(C) laparoscopy and reduction ofintussusception.
(D) laparotomy and resection ofintussusception.
(E) laparotomy and diverting loop ileostomy.
---------- ---------·-----
Figure 30.l
Figure 30.2
CATEGORY 3 - PART IV
207
31. A 50-year-old man with no medical history and excellent performance status presents
with nausea, vomiting, and hematemesis. He has developed progressive anemia due
to ongoing blood loss and weight loss because of an inability to eat. CT scan imaging
confirms a large gastric mass in the antrum of the stomach with high-grade gastric
outlet obstruction, perigastric lymphadenopathy, and a single liver metastasis in the
peripheral left lobe (T3N1Ml, stage IV). Esophagogastroduodenoscopy with biopsy
confirms gastric adenocarcinoma. Diagnostic laparoscopy confirms no carcinomato-
sis. The next step in management should be
(A) chemotherapy.
(B) percutaneous endoscopic gastrostomy.
(C) balloon dilation of the antral stricture.
(D) gastrectomy and chemotherapy.
(E) chemotherapy, gastrectomy, and liver resection.
32. A 45-year-old man with a history of chronic intestinal dysmotility presents to the
emergency department with acute onset of severe midabdominal pain and fever. His
white blood cell count is l 8,000/mm3 (3600-l l ,200/mm3), and his arterial lactate is
normal. Abdominal exam reveals diffuse tenderness in the midabdomen with no evi-
dence of peritonitis. Abdominal CT scan images are shown in figures 32.1 and 32.2.
The most appropriate therapy for this patient would be
(A) intravenous heparin infusion.
(B) intravenous tissue plasminogen activator infusion.
(C) colonoscopy with tube decompression.
(D) angiography with vasodilator infusion.
(E) emergency laparotomy.
Figure 32.1
208 CATEGORY 3 - PART IV
Figure 32.2
33. A 54-year-old man has a history of ulcerative colitis diagnosed when he was 20 years
old. His disease is well controlled on a 5-aminosalicylic acid compound. On surveil-
lance colonoscopy, there is no evidence of active colitis. In the proximal transverse
colon, a 2-cm sessile polypoid lesion is seen. There are no surrounding inflammatory
changes. A saline lift is performed, and the lesion is completely resected piecemeal.
Biopsies immediately adjacent to the lesion are normal. The final pathology is con-
sistent with a tubulovillous adenoma. Other random biopsies throughout the colon are
normal without evidence of dysplasia or active inflammation. Which of the following
is the next best step in management?
(A) Colonoscopy in 6 months
(B) Colonoscopy in 5 years
(C) Extended right colectomy with primary anastomosis
(D) Total colectomy with ileorectal anastomosis
(E) Total proctocolectomy with ilea! pouch anal anastomosis
34. Which of the following is true regarding blood transfusion and risk of colorectal can-
cer recurrence?
(A) Increased risk is associated with perioperative transfusion.
(B) Autologous and allogeneic blood transfusions carry similar risks.
(C) Increased risk is associated with the age of the stored blood.
(D) Increased risk is associated with the plasma component of blood.
(E) Increased risk is negligible using current leukoreduction techniques.
CATEGORY 3 - PART JV
209
35. A 58-year-old woman was involved in a motor vehicle collision. During her workup,
a CT of the abdomen was obtained. No injuries were found, but an incidental finding
of a 2-cm submucosal mass was noted on the greater curvature of the stomach. An
outpatient esophagogastroduodenoscopy (EGDJ revealed no mucosa] abnormalities.
A fine needle aspiration done with endoscopic ultrasound tested positive for CD 117.
What is the next appropriate step in her treatment?
(AJ Observation with repeat EGD in 1 year
(BJ Laparoscopic wedge resection
(CJ Neoadjuvant imatinib followed by surgical resection
(D) Subtotal gastrectomy with regional lymph node dissection
(EJ Endoscopic submucosal resection
36. In patients with portal hypertension. which of the following treatment strategies is
most likely to cause encephalopathy?
(AJ Nonselective beta-blockers
(BJ Nitrates
(CJ Repeated endoscopic banding
(DJ Transjugular intrahepatic portosystemic shunt
(E) Long-acting octreotide
37. A 39-year-old man who smokes and has been treated for recurrent duodenal ulcers
sustains a duodenal bulb perforation after an attempted balloon dilation of a duodenal
bulb stricture due to his recurrent peptic ulcer disease. He is now 36 hours after initial
attempted endoscopic dilation. He remains hemodynamically normal. Which of the
following is the best surgical management of this patient?
(AJ Laparoscopic Graham patch
(B) Highly selective vagotomy and pyloroplasty
(CJ Covered stent
(DJ Highly selective vagotomy and gastrojejunostomy
(EJ Truncal vagotomy and antrectomy
38. A 70-year-old man has a squamous cell esophageal carcinoma 24 cm from the inci-
sors on workup for hoarseness. Preoperative workup is consistent with an uT3Nl le-
sion. After undergoing chemoradiation for 6 weeks, the patient is restaged. Multiple
endoscopic biopsies at the tumor site show fibrous tissue. Repeat PET scan shows
diminution of activity in the esophageal lesion to that of background and no evidence
of metastatic disease. Which of the following is the best option for his management?
(AJ Continued chemotherapy
(BJ Radiation
(C) Ivor Lewis esophagectomy
(D) Minimally invasive esophagectomy
(EJ Observation
21 Q CATEGORY 3 - PART IV
39. A 55-year-old man with a history of chronic kidney disease, hepatitis C, and hyperten-
sion presents to the emergency department with 12 hours of abdominal pain, swelling,
and emesis. He denies any previous abdominal masses, but exam shows a tender mass
midabdomen with no peritoneal signs. He has had no previous operations. The image
in figure 39.1 is representative of his imaging studies. The next step in management
should be
(A) exploration and definitive repair.
(B) liver biopsy and portal pressure management.
(C) manual reduction.
(D) colonoscopy with decompression.
(E) incisional biopsy.
Figure 39.1
40. Which of the following is the most common site of intestinal Crohn disease?
(A) Jejuna!
(B) Ileocolic
(C) Perianal
(D) Appendix
(E) Left colon
CATEGORY 3 - PART JV
211
Critiques 1-40
CRITIQUE 1
Neutropenic enterocolitis. also known as necrotizing enterocolitis or typhlitis (neutropenic
enterocolitis affecting the "typhlon" [Greek for cecum]), is a poorly understood disease
process, Impaired host defenses combined with mucosa! breakdown lead to necrosis of
various layers of the bowel wall. Most commonly seen in the ileocecal region. necrotizing
enterocolitis can occur in any aspect of the gastrointestinal tract. Although the true inci-
dence is not known, it does appear to be increasing with the increased use of cytotoxic and
other immunomodulating agents.
Patients typically present with the nonspecific symptoms of fever and abdominal pain.
There may also be distention, nausea, vomiting, and diarrhea. Some patients will have hema-
tochezia. The white blood cell count will typically be low and the absolute neutrophil count
will be below 500 cells/mm3. In patients with this constellation of findings, a CT scan is used
to make the diagnosis. Findings of inflammation of the bowel and associated perivisceral
edema are consistent with the diagnosis (figure 1.2). Differential diagnosis includes graft
versus host disease, ischemic colitis, Cytomegalovirus colitis, and norovirus infection.
References
1. Bojic D, Markovic S. Terminal ileitis is not always Crohn's disease. Ann Gastroenterol.
2011 ;24(4):271-275.
2. Ullery BW, Pieracci FM, Rodney JR, Barie PS. Neutropenic enterocolitis. Surg Infect
(Larchmt). 2009;10(3):307-314.
3. Davila ML. Neutropenic enterocolitis. Curr Opin Gastroenterol. 2006;22(1):44--47.
CRITIQUE2
Historically, gastric cancer was treated with standard cytotoxic chemotherapy. A fluoropy-
rimidine such as capecitabine was combined with a platin (cisplatin or oxaliplatin). Gastric
cancer was subsequently found to express tumor markers that could affect the type of treat-
ment offered. HER2 expression was noted, and initial trials of HER2 receptor blockers
were shown to be potentially useful.
Jn 2010, the ToGA trial showed superiority of trastuzumab plus chemotherapy ver-
sus chemotherapy alone in HER2-expressing gastric cancer. Interestingly, the level of
HER2 amplification was a factor in the level of response. Currently, the combination of
trastuzumab and cytotoxic chemotherapy is indicated for the treatment ofHER2-positive
gastric cancer. No studies have examined trastuzumab alone in gastric cancer. Radiation
therapy has a positive impact on gastric cancer, but its use is limited by organ motion and
targeting of the lesion. The routine use of radiation therapy in advanced gastric cancer is
not recommended.
References
1. Gomez-Martin C, Lopez-Rios F, Aparicio J, et al. A critical review of HER2-positive
gastric cancer evaluation and treatment: from trastuzumab, and beyond. Cancer Lett.
2014;351(1):30-40.
2. Fujitani K. Overview of adjuvant and neoadjuvant therapy for resectable gastric cancer
in the East. Dig Surg. 2013;30(2): 119-129.
CRITIQUE3
Tumors of the anus can be classified as either anal canal tumors or anal margin tumors. The
anal canal extends from the anorectal ring to the anal verge. The anal margin, or perianal
skin, lies just distal to the anal verge. Because the anal margin is skin, tumors seen in this
area are typically skin related and include basal cell carcinoma, melanoma, anal intraepi-
thelial neoplasm, and squamous cell carcinoma.
Paget disease is an intraepithelial adenocarcinoma and is most commonly seen in the
breast. Extramammaiy Paget disease occurs where apocrine sweat glands are found, which
includes the anal region. The median age of presentation is 60. Symptoms include anal
pruritus, bleeding, and discharge. Erythematous and eczematous rashes are found on exam.
Diagnosis is usually made with biopsy that shows the presence of large rounded vacuolated
cells known as Paget cells. Patients with the diagnosis of Paget disease of the anal canal
require full colonoscopy, as 50% will be found to have a colorectal neoplasm.
The first step in deciding the treatment of Paget disease of the anus is to assess the ex-
tent of the disease. Performing random mapping biopsies of the anal canal accomplishes
this. Mapping biopsies, performed before definitive resection, involve sampling the anal
canal, including the dentate line, anal verge, and the perianal skin 2 cm from the anal
CATEGORY 3 - PART IV 213
verge. Eight biopsies are taken at each location. Resection is then planned based on the
results of these biopsies.
For lesions that are locally confined and noninvasive (stage I and stage IIA disease),
wide local excision is the treatment of choice. One-centimeter margins confirmed by mi-
croscopic analysis are required. Stage IIB disease has an associated invasive component
or anorectal carcinoma and is treated with abdominoperineal resection (APR). Stage III
disease has metastasized to regional lymph nodes and requires an APR with inguinal lymph
node dissection. Stage IV disease involves distant metastasis; therefore, local palliative
resection, along with chemotherapy and radiation therapy, can be used.
Iodine and mucosal stripping are used for the identification and treatment of human
papillomavirus infection. Sentinel lymph node biopsy has not been investigated in the di-
agnosis and treatment of Paget disease of the anus.
Answer: (D) Mapping biopsies followed by wide local excision with 1-cm microscopic
margins
References
1. Kyriazanos ID, Stamos NP, Miliadis L, Noussis G, Stoidis CN. Extra-mammary Paget's
disease of the perianal region: a review of the literature emphasizing the operative
management technique. Surg Oneal. 2011 ;20(2):e61--<071.
2. Leonard D, Beddy D, Dozois EJ. Neoplasms of anal canal and perianal skin. Clin Colon
Rectal Surg. 2011;24(1 ):54-63.
CRITIQUE4
Despite advances in medical management, approximately 70% of patients with Crohn dis-
ease will need surgical intervention during their lifetime for the management of complica-
tions. These complications include chronic fistulae (perirectal or enteric), stricture with or
without obstruction, perforation, cancer, and, less frequently, intractability. This patient has
evidence of increasingly symptomatic strictures with partial obstruction that are persistent
and progressive despite appropriate medical therapy with infliximab and the addition of
steroids. Given her increasing symptoms and obstipation, it is unlikely that her symptoms
will resolve without surgical intervention.
The question as to whether to suspend immunomodulating therapy during the periop-
erative period is challenging. Studies evaluating the impact of perioperative immunomodu-
lation on postoperative outcomes are mixed and limited by relatively small numbers of
patients. Biologicals, such as the antitumor necrosis factor monoclonal antibody intlix-
imab, have an elimination half-life of7-19 days, thus requiring a 3--4 month period for
full clearance. However, the inability to measure detectable levels of the drug does not
necessarily imply that residual systemic effects will not persist. Similar clearance periods
apply to thiopurines. Corticosteroids given within 1-2 months of intestinal surgery are
reported to increase operative morbidity. Many studies, however, do not distinguish surgi-
cal (anastomotic leak, abscess, and wound failure) from medical (urinary and respiratory)
complications when discussing postoperative morbidities associated with immunomodula-
tion, making data interpretation difficult. Although the presence offistulae or abscess at the
time of operation increases postoperative morbidity, a well-controlled trial showed no dif-
ference in complication rates between patients taking biologicals less than 14 days before
operation compared with those receiving them up to 180 days before surgery.
Given the progression and severity of this patient's symptoms, delaying her operation to
decrease her immunosuppressive regimen carries no clear advantage. Proceeding will allow
rapid taper of her steroids and possible discontinuation ofher biological immunosuppression.
References
1. Ali T, Yun L, Rubin DT. Risk of post-operative complications associated with anti-TNF
therapy in inflammatory bowel disease. World J Gastroenterol. 2012; 18(3): 197-204.
2. Bafford AC, Powers S, Ha C, et al. Immunosuppressive therapy does not increase
operative morbidity in patients with Crohn's disease. J Clin Gastroenterol. 2013;47(6):
491-495.
3. Myrelid P, Marti-GallostraM,AshrafS, etal. Complications in surgery forCrohn 's disease
after preoperative anti tumour necrosis factor therapy. Br J Surg. 2014; 101 (5):539-545.
4. Waterman M, Xu W, Dinani A, et al. Preoperative biological therapy and short-term
outcomes of abdominal surgery in patients with inflammatory bowel disease. Gut.
20 l 3;62(3):387-394.
CRITIQUES
Diagnosis and management of acute appendicitis continues to evolve. Once diagnosed on
the basis of history and physical findings alone, the availability of CT scanning has be-
come a routine component of the assessment for appendicitis~ with the goal of enhancing
the sensitivity and specificity of diagnosis. CT scanning decreases the resection of normal
appendices and identifies early appendicitis that might be managed nonoperatively. CTs
also allow identification of complicated appendicitis-defined as appendiceal phlegmon
or organized abscess. The CT scan may be performed without contrast, because neither the
addition of intravenous or oral contrast enhances the accuracy of diagnosis. If nonoperative
management is elected for mild appendicitis, the cumulative risk of recurrence is estimated
to be 5-20%, with recurrence complicated by perforation estimated at only 3%. Thus, in-
terval appendectomy may not be essential. However, if malignancy is a concern for older
patients presenting with a phlegmon, colonoscopy followed by appendectomy confirms a
correct diagnosis (particularly if an appendicolith is identified).
More than 98o/o of patients presenting with acute appendicitis continue to be operatively
managed. A delay of several hours from the time of presentation to surgery does not sig-
nificantly increase the risk of perforation. Although the incidence of both superficial and
deep surgical site infection is lower with laparoscopic appendectomy for patients with
uncomplicated appendicitis, the risk of intra-abdominal abscess formation increases by
3-fold with a laparoscopic approach compared with open appendectomy for patients with
complicated appendicitis. Thus, the choice of approach may depend not only on patient
habitus, past surgical history, and ease of access but also the complexity of the acute infec-
tious process.
References
1. Drake FT, Mottey NE, Farrokhi ET, et al. Time to appendectomy and risk of perforation
in acute appendicitis. JAMA Surg. 2014;149(8):837-844.
2. Mccutcheon BA. Chang DC . Marcus LP, et al. Long-term outcomes of patients with
nonsurgically managed uncomplicated appendicitis. J Am Coll Surg. 2014;218(5):
905-913.
3. Wray CJ, Kao LS, Millas SG, Tsao K, Ko TC. Acute appendicitis: controversies in diag-
nosis and management Curr Prob! Surg. 2013;50(2):54-86.
4. Drake FT, Alfonso R, Bhargava P, et al; Writing Group for SCOAP-CERTAIN. Enteral
contrast in the computed tomography diagnosis of appendicitis: comparative effective-
ness in a prospective surgical cohort. Ann Surg. 2014;260(2):311-316.
CATEGORY 3 - PART IV 215
CRITIQUE6
Screening for early colon cancer is undergoing an evolution. Despite recommendations,
a substantial proportion of the US population is not up to date with respect to early de-
tection. Development of simple, noninvasive testing modalities with high sensitivity and
specificity for both colorectal cancer and advanced precancerous lesions would potentially
improve clinical outcomes.
For many years, fecal immunochemical testing (FIT) for the presence of blood in the
stool was used as a noninvasive, inexpensive, and easily applicable means of screening
at-risk populations. Although sensitive, it is not particularly specific as a colorectal cancer
marker and must be further evaluated by either colonoscopy or contrast imaging. Multitar-
get DNA testing of stool is a highly sensitive as a means of detecting both colorectal can-
cer (92%) and advanced precancerous lesions (42%), significantly exceeding that of FIT
screening. However, FIT is more specific than DNA testing due to its lower false-positive
rate. Blood-based biomarkers, including circulating coding and noncoding RNA (includ-
ing mRNA and micro-RNA microarrays), are being tested with a particular focus on early
stage colorectal cancer patients but have not been validated beyond proof-of-principle and
pilot stages.
Double contrast barium enema identifies large polyps or masses, but colonoscopy is
still required to confirm and biopsy worrisome lesions. Colonoscopy remains the most ef-
fective screening method currently available, with the highest specificity and sensitivity.
The attendant risks of the procedure are well recognized and include bleeding, perforation,
missed adenoma (6-12%), and missed cancer (5%). Ultimately combining less-invasive
pretesting may serve to focus colonoscopy with biopsy on those patients at most risk for
development of a malignancy.
References
I. Ganepola GA, Nizin J, Rutledge JR, Chang DH. Use of blood-based biomarkers for
early diagnosis and surveillance of colorectal cancer. World J Gastrointest Oneal.
2014:6(4):83-97.
2. Imperiale TF, Ransohoff DF, Itzkowitz SH. Multitarget stool DNA testing for colorec-
tal-cancer screening. N Engl J Med. 2014;371(2): 187-188.
CRITIQUE?
In patients who present with symptoms of gastroesophageal reflux disease (GERD), a
proper evaluation before antireflux surgery is essential. Proof of pathologic gastroesopha-
geal reflux and characterization of associated abnormalities in foregut structure and func-
tion increases the likelihood of a successful surgical outcome. Inadequate preoperative
evaluation is a major contributor to a poor outcome after surgery. Three factors are most
predictive ofa successful symptomatic outcome after antireflux surgery: (I) the presence
of typical symptoms of GERD, (2) symptomatic improvement in response to acid sup-
pression therapy before surgery, and (3) an abnormal score on ambulatory esophageal pH
monitoring. Standard preoperative diagnostic studies include flexible upper gastrointesti-
nal endoscopy, barium esophagography. stationary esophageal manometry, and ambulatory
esophageal pH monitoring. Each test has a specific role in the diagnosis and workup of
GERD. Results are combined to document the presence and severity of disease, which fac-
tors into planning the operative approach.
Testing for esophageal structural abnormalities is addressed by upper endoscopy and
barium esophagography. Esophageal mucosa! injury, such as occurs in esophagitis and
Barrett esophagus, can be identified and biopsied via endoscopy. Areas of suspected malig-
nancy can also be biopsied in this manner. Barium esophagram is ideally a video-recorded
216 CATEGORY 3 ~ PART IV
examination to evaluate the dynamic changes of the esophagus with respect to esophageal
peristalsis, bolus transport, and reducibility ofhiatal hernia.
Testing for esophageal physiologic abnormalities is addressed by esophageal manom-
etry and esophageal pH testing. Esophageal manometry is the most reliable method to
assess the function of the lower esophageal sphincter (LES) and the esophageal body. The
primary purposes of performing esophageal manometry before antireflux surgery are to
exclude achalasia, to assess peristaltic coordination and contractile force of the esopha-
geal body, and to measure the exact location of the gastroesophageal junction for accurate
pH probe placement Ambulatory esophageal pH testing is the gold standard to determine
whether there is pathological GERD. This can be done with a transnasal catheter for 24
hours or a wireless pH system, which collects 48 hours of data.
The gastric emptying study is not a routine component of the preoperative workup be-
fore antireflux surgery. This test should be obtained selectively in the preoperative evalua-
tion of patients with significant nausea, vomiting, and bloating or those with retained food
in the stomach on endoscopy after an overnight fast.
Laryngopharyngeal reflux (LPR) is common. However, these symptoms can also be
associated with factors other than GERD, such as tobacco and alcohol abuse, allergies,
postnasal drip, and chronic sinusitis, all of which irritate the hypopharynx. The outcomes
of antireflux surgery when performed for LPR symptoms are less favorable than when sur-
gery is performed in patients with typical GERD symptoms. The oropharyngeal pH cath-
eter measures oropharyngeal acid reflux and attempts to establish the temporal relationship
between extraesophageal reflux symptoms and LPR acid events. No data support the deci-
sion to proceed with antireflux surgery based on the results of oropharyngeal pH testing.
References
I. Jobe BA, Richter JE, Hoppo T, et aL Preoperative diagnostic workup before antireflux
surgery: an evidence and experience-based consensus of the Esophageal Diagnostic
Advisory Panel. J Am Coll Surg. 2013;217(4):586-597.
2. Wilshire CL, Watson TJ. Surgical management of gastroesophageal reflux disease.
Gastroenterol C/in North Am. 2013;42(1):119-131.
CRITIQUES
Gastrointestinal stromal tumors (GISTs) are the most common sarcomatous tumors of the
gastrointestinal tract. They are a distinct type of tumor derived from the interstitial cells of
Cajal, an intestinal pacemaker cell. Thus, GISTs have smooth muscle and neuroendocrine
features. They are identifiable by immunohistochemical staining for the c-kit proto-
oncogene (CDl 17), found in more than 90% of these tumors, and for CD34, found in 80%
of these tumors. They are typically found in the stomach (40-60%), but they may also
occur in the small intestine and colon. They vary in presentation and clinical course from
small benign tumors to massive lesions with necrosis, hemorrhage, and wide metastases.
The mainstay of treatment is local surgical excision, with negative microscopic margins
and intact pseudocapsule without tumor rupture. Depending on tumor size and location,
this may entail wide local excision, enucleation, sleeve gastrectomy, or total gastrectomy,
with or without en bloc resection of adjacent organs. Because lymph node metastases are
uncommon, lymphadenectomy is generally not indicated.
Recurrence rates are in the range of 40o/o. Recurrence usually manifests as hepatic me-
tastasis, but one-third of recurrences are isolated local lesions. Recurrence can occur many
years after initial presentation, warranting long-term follow-up.
The tyrosine kinase inhibitor imatinib mesylate is selective for c-kit as well as for other
tyrosine kinases. It was originally designed to treat chronic myelogenous leukemia. How-
CATEGORY 3 - PART IV 217
ever, in randomized controlled trials, it was effective treatment for metastatic GIST or
disease that carries a high risk for recurrence, which is determined by tumor size, mitotic
index, site, rupture, and completeness of surgery. The side effects are generally mild. The
size of this GIST mandates imatinib treatment.
References
1. Sicklick JK, Lopez NE. Optimizing surgical and imatinib therapy for the treatment of
gastrointestinal stromal tumors. J Gastrointest Surg. 2013; 17(11):1997-2006.
2. Dematteo RP, Ballman KV, Antonescu CR, et al; American College of Surgeons Oncol-
ogy Group (ACOSOG) Intergroup Adjuvant GIST Study Team. Adjuvant imatinib
mesylate after resection of localised, primary gastrointestinal stromal tumour: a
randomised, double-blind, placebo-controlled trial [published correction appears in
Lancet. 2009;374(9688):450]. Lancet. 2009;373(9669):1097-1104.
CRITIQUE9
The majority of inherited colorectal cancers are accounted for by hereditary nonpolypo-
sis colorectal cancer (HNPCC) or Lynch syndrome and familial adenomatous polyposis
(FAP). HNPCC is the most common form of heritable colorectal cancer, accounting for
1-3% of all cases of colorectal cancer. FAP is the second most common form of heritable
colorectal cancer, accounting for approximately 1% of all colorectal cancer. FAP is autoso-
mal dominant with nearly 100% penetrance, which means that half of the offspring of an
affected individual parent are at risk of developing FAP, and the affected individuals will
have nearly 100% chance of developing colorectal cancer by the age of 60.
FAP is caused by mutations in the adenomatous polyposis coli gene, a tumor suppressor
gene located in the long arm of chromosome 5 (5q2 l) that is involved with regulating cell
cycle progression. Patients with FAP develop more than 100 polyps in their colon, and,
unlike HNPCC, invasive cancers tend to develop on the left colon. Extracolonic manifesta-
tions include desmoid tumors, periampullary neoplasms, osteomas, odontomas, supernu-
merary teeth, sebaceous cysts, hepatoblastomas, thyroid tumors, and congenital hypertro-
phy of the retinal pigmented epithelium, the last being pathognomonic for FAP.
Desmoid tumors and duodenal cancer are the leading cause of death for patients who
already have had a prophylactic total colectomy. Because nearly 90% of FAP patients will
develop duodenal polyps but only 4.5% will develop duodenal adenocarcinoma in their
lifetime, surveillance with esophagogastroduodenoscopy (EGD) and staging of duodenal
polyps becomes an important component of care. Surveillance with EGD should begin at
age 20 or at the time of prophylactic colectomy, whichever is earlier, and should include
biopsy of any suspicious polyps. The Spigelman classification can be used to stage the
polyps and help with decision making. This classification uses 4 parameters (polyp num-
ber, polyp size, histology, degree of dysplasia) and assigns points for each (table 9.1). The
following is the Spigelman staging system: stage 0: no polyps; stage 1: 1-4 points; stage
2: 5-6 points; stage 3: 7-8 points; stage 4: 9-12 points. Surveillance with EGD should be
performed accordingly: stage 0-1: every 5 years; stage 2: every 3 years; stage 3: every 1-2
years; stage 4: surgical intervention.
No studies demonstrate efficacy of nonsteroidal anti-inflammatory drugs to suppress
polyp formation in the duodenum. Treatment options for duodenal polyps include endo-
scopic ablation and transduodenal excision. Pancreas-preserving duodenectomy or pan-
creaticoduodenectomy should be considered in patients with severe duodenal polyposis
(Spigelman 4) or duodenal carcinoma.
The total Spigelman points for this patient is 6, which places him as stage 2. Radical
surgery such as a Whipple procedure is not indicated at this point.
218 CATEGORY 3 - PART JV
Number of Points
2 3
Number of polyps 1-4 5-20 >20
Polyp size (mm) 1-4 5-10 >10
Histology Tubulous Tubulovillous Villous
Dysplasia Mild Moderate Severe
Stage Spigelman score•
0 0
I 1-4
III 5-6
III 7-8
IV 9-12
"'Addition of points.
References
1. Bulow S, Bjork J, Christensen IJ, et al; DAF Study Group. Duodenal adenomatosis in
familial adenomatous polyposis. Gut. 2004;53(3):381-386.
2. Saurin JC, Gutknecht C, Napoleon B, et al. Surveillance of duodenal adenomas in
familial adenomatous polyposis reveals high cumulative risk of advanced disease.
JC/in Oneal. 2004;22(3):493-498.
3. Schulmann K, Hollerbach S, Kraus K, et al. Feasibility and diagnostic utility of video
capsule endoscopy for the detection of small bowel polyps in patients with hereditary
polyposis syndromes. Am J Gastroenterol. 2005;100(1):27-37.
CRITIQUE IO
The use of endoscopically placed esophageal stents to treat leaks and perforations has
expanded in clinical practice. The reported overall stent migration rate is 27% with plastic
stents and 11 % with metallic stents. Stent perforation occurs in approximately 2% of the
patients. A combined series reported only 11 postprocedure strictures in 340 patients.
If the diameter of the esophagus or conduit is too large, no stent will be large enough
to cause apposition and sealing. Endoscopic reinterventions are required in approximately
17% of patients who have previously undergone stenting. Reintervention includes place-
ment of another stent for persistent leak, stent migration, or repositioning of the stent.
Endoscopic placement of fistula plugs or resolution clips for persistent fistula may be per-
formed. The mortality rate of 13% for the endoscopic management of perforations and
leaks is lower than the mortality rates reported with surgical intervention.
References
I. Dasari BV, Neely D, Kennedy A, et al. The role of esophageal stents in the manage-
ment of esophageal anastomotic leaks and benign esophageal perforations. Ann Surg.
2014;259(5):852-860.
2. D'Cunha J, Rueth NM, Groth SS, Maddaus MA, Andrade RS. Esophageal stents
for anastomotic leaks and perforations. J Thorac Cardiovasc Surg. 2011 ;142(1 ):
39.el-46.el.
CRITIQUE 11
Improved medical management of peptic ulcer disease has virtually eradicated the need
for acid-reducing surgery but perforation as the presenting symptom still requires surgery
in the majority of cases. The elderly commonly present with a perforated peptic ulcer and
they have high mortality (up to 25%) and morbidity (up to 50%) rates. These high rates
are related to the elderly patient's comorbidities. Laparoscopic closure of perforated peptic
ulcers has lowered these mortality and morbidity rates.
Eradication of Helicobacter pylori significantly reduces the 1-year recurrent ulcer rate
to 5%. The 30-day mortality rate for peptic ulcer perforation is more than 20%, and 60-day
mortality is approximately 25% but these results are independent of patient BM!. Actually,
being underweight is associated with a more than 2-fold increased mortality risk. Being
overweight is neither protective nor an adverse prognostic factor. A surgical delay defined
as from admission to surgery exceeding 6 hours is a negative prognostic factor. For every
hour delay there is an adjusted 2.4% decreased probability of survival compared with the
previous hour.
References
!. Buck DL, M011er MH; Danish Clinical Register of Emergency Surgery. Influence
of body mass index on mortality after surgery for perforated peptic ulcer. Br J Surg.
2014;101(8):993-999.
2. S0reide K, Thorsen K, S0reide JA. Strategies to improve the outcome of emergency
surgery for perforated peptic ulcer. Br J Surg. 2014; I 0 I (I ):e5 l-e64.
3. Antoniou SA, Antoniou GA, Koch 00, Pointner R, Granderath FA. Meta-analysis of
laparoscopic versus open repair of perforated peptic ulcer. JSLS. 2013;17(1):15-22.
4. Wadaani HA. Emergent laparoscopy in treatment of perforated peptic ulcer: a local
experience from a tertiary centre in Saudi Arabia. World J Emerg Surg. 2013;8(1): I 0.
5. Sanabria AE, Morales CH, Villegas MI. Laparoscopic repair for perforated peptic ulcer
disease [published update appears in Cochrane Database Syst Rev. 2013;2:CD004778].
Cochrane Database Syst Rev. 2005;(4):CD004778.
CRITIQUE 12
Food bolus impaction in otherwise healthy adults was historically associated with peptic
strictures or mucosal (i.e .. Schatzki) rings. However, recent evidence suggests that the
underlying cause in up to 50o/o of patients is eosinophilic esophagitis (figure 12.1). The
diagnosis of eosinophilic esophagitis should be suspected if multiple concentric rings or
white exudate is seen on endoscopy. Mucosa! biopsy at multiple levels along the esopha-
gus and in the stomach should be performed once any inflammation from the food bolus
impaction has resolved.
Zenker diverticulum commonly presents with regurgitation of food or particulate mat-
ter, foul breath, and a sensation of pharyngeal fullness. Nutcracker esophagus does not
commonly present with food bolus impaction but rather dysphagia and substemal chest
220 CATEGORY 3 - PART IV
pain. Esophageal carcinoma should be considered in patients with food bolus impaction,
but this would be unusual in a young, otherwise healthy patient with no known risk factors.
(• ) (b)
(c) (d)
References
1. Mahesh VN, Holloway RH, Nguyen NQ. Changing epidemiology of food bolus impac-
tion: is eosinophilic esophagitis to blame? J Gastroenterol Hepatol. 20 13;28(6):963-966.
2. Desai TK, Stecevic V, C hang CH, Goldstein NS, Badizadegan K, F uruta GT. Associa-
tion of eosinophilic infiammationwith esophageal food impaction in adults. Gastrointest
Endosc. 2005;61(7):795- 80 1.
3. Furuta GT, Liacouras CA, Collins MH, et al; First International Gastrointestinal Eosino-
phil Research Symposium (FIGERS) Subcommittees. Eosinophilic esophagitis in chil-
dren and adults: a systematic review and consensus recommendations for diagnosis and
treatment. Gastroenterology. 2007; 133(4): 1342-1 363.
4. Chauvin A, Viala J, Marteau P, Hermann P, Dray X . Management and endoscop-
ic techniques for digestive foreign body and food bolus impaction. Dig Liver Dis.
20 13;45(7):529- 542.
5. Saadah 01, Aburiziza AJ, Abu Shakra RI. Eosinophilic esophagitis in children from
western Saudi Arabia: relative frequency, clinical, pathological, endoscopic, and immu-
nological study. Gastr Res Pract. 20 12; Article ID 328253.
CATEGORY 3 - PART JV
221
CRITIQUE 13
Management of food bolus impaction typically involves urgent endoscopy. A chest x-ray
can be performed, but other radiographic workup, especially with contrast, is unnecessary
due to the risk of aspiration. Oral meat tenderizer should not be administered. because this
strategy can further injure the esophagus. Glucagon (1 mg intravenously) to pi-emote rela'X-
ation of the lower esophageal sphincter is relatively low risk; however, this therapy alone
rarely results in passage of the food bolus. Morphine and atropine are typically not used in
cases of food bolus impaction, although atropine may be used as an antisialagogue by an
anesthesia provider if general anesthesia is required. Urgent intubation is not needed in the
case described, because the patient appears to be protecting his ainvay. Surgical explora-
tion is also not indicated.
At endoscopy, the food bolus should be retrieved or very gently advanced into the stom-
ach. In most cases, retrieval will be necessary due to the pathology in the esophagus and
any resulting inflammation from the impacted food. After removal of the obstructing bolus,
the esophagus should be thoroughly examined. If the obstruction was due to a mucosal
ring with no surrounding inflammation, dilation can be performed during the index proce-
dure. However, in most cases of food bolus impaction, a subsequent endoscopic examina-
tion will be required. Patients with eosinophilic esophagitis can be managed with dietary
modification (i.e., avoiding foods to which they are allergic) or with topical corticosteroids
taken by mouth.
References
1. Mahesh VN, Holloway RH, Nguyen NQ. Changing epidemiology of food bolus impac-
tion: is eosinophilic esophagitis to blame? J Gastroenterol Hepatol. 2013;28(6):963-
966.
2. Desai TK, Stecevic V. Chang CH, Goldstein NS, Badizadegan K, Furuta GT. Associa-
tion of eosinophilic inflammation with esophageal food impaction in adults. Gastroin-
test Endosc. 2005:61(7):795-801.
3. Furuta GT. Liacouras CA. Collins MH, et al; First International Gastrointestinal Eosino-
phil Research Symposium (FI GERS) Subcommittees. Eosinophilic esophagitis in chil-
dren and adults: a systematic review and consensus recommendations for diagnosis and
treatment. Gastroenterology. 2007; 133(4): 1342-1363.
4. Chauvin A, Viala J, Marteau P, Hermann P, Dray X. Management and endoscop-
ic techniques for digestive foreign body and food bolus impaction. Dig Liver Dis.
20 I 3;45(7):529-542.
5. Saadah OJ, Aburiziza AJ, Abu Shakra RI. Eosinophilic esophagitis in children from
western Saudi Arabia: relative frequency, clinical, pathological, endoscopic, and immu-
nological study. Gastr Res Pract. 2012; Article ID 328253.
CRITIQUE 14
This patient has fulminant Clostridium difficile colitis. Determining the need for and tim-
ing of operative intervention in these cases can be very challenging, because some pa-
tients improve with antibiotic therapy and resuscitation alone, whereas others continue
a downward spiral into multiorgan failure and death. This patient is manifesting signs of
hypoperfusion or shock and has minimal improvement with current medical management.
Consequently, urgent operative intervention is indicated.
The gold standard operation in this case remains a total abdominal colectomy. However,
diverting loop ileostomy with antegrade colonic lavage was successful in a case series
from a single institution. Adding metronidazole or changing the route of vancomycin ad-
ministration would not likely alter the clinical course of this patient. A bolus of systemic
222 CATEGORY 3 - PART IV
steroids would increase this patient's likelihood of developing other nosocomial infections
and would be indicated only if she progressed to shock refractory to multiple vasopressors.
Her lack of response to norepinephrine is another indication that she should be treated
with operative intervention. Ideally, surgical intervention in a patient with fulminant C.
difficile should be performed before multiple vasopressors are required, because the need
for preoperative vasopressors in these patients is associated with increased mortality. She
might need further presser support, but that is best provided by adding vasopressin and not
epinephrine.
References
I. Ferrada P, Velopulos CG, Sultan S, et al. Timing and type of surgical treatment of Clos-
tridium dif.Jici/e-associated disease: a practice management guideline from the Eastern
Association for the Surgery of Trauma [published correction appears in J Trauma Acute
Care Surg. 2014;77(1):186]. J Trauma Acute Care Surg. 2014;76(6): 1484-1493.
2. Lee DY, Chung EL, Guend H, Whelan RL, Wedderburn RV, Rose KM. Predictors of
mortality after emergency colectomy for Clostridium difficile colitis: an analysis of
ACS-NSQIP. Ann Surg. 2014;259(1):148-156.
3. Khanna S, Pardi DS. Clostridium dif.Jicile infection: new insights into management.
Mayo Clin Proc. 2012;87(11):1106-1117.
CRITIQUE 15
The diarrheal disease accompanied by bloody stools and abdominal pain/discomfort
prompts frequent surgical consultation to exclude the possibility of surgical conditions.
Although the majority of these patients do not have a condition that requires surgical inter-
vention, at least acutely, it is prudent for the surgeon to have an understanding of the causes
of diarrhea in these immunocompromised individuals. The differential diagnosis includes
infectious etiologies, as well as conditions associated with HIV infection itself (e.g., HIV-
enteropathy). The change in the bacteriology of diarrhea in HIV-infected individuals is as-
cribed to the availability of highly active antiretroviral therapy (HAART) in economically
advantaged societies.
Cryptosporidiosis was once an AIDS-defining condition and was associated with a
greater mortality than all the others (e.g., Kaposi sarcoma, Pneumocystis pneumonitis,
Burkett lymphoma). Before aggressive antiretroviral therapy, opportunistic infectious or-
ganisms were thought to be the most common causes of diarrhea in the HIV population.
The use ofHAART is credited for the decline in the incidence of this infectious disease as
well as enteritis due to Cytomegalovirus, Coccidiomycosis, and Mycobacterium species,
also once considered AIDS-defining conditions. On the contrary, some suggest that the
difficulty in the laboratory identification of Clostridium difficile before 2002 may have
obscured the prevalence of this organism.
Whatever the cause, the incidence of infectious enteritis due to C. difficile has increased
and is now the most common pathogen isolated in immunocompromised individuals,
whether due to HIV infection or therapeutic immunosuppression after transplantation.
Unlike most C. difficile colitis in individuals with intact immune systems, the condi-
tion is not always associated with prior antibiotic therapy or institutionalization. This is
associated with individuals with extremely low CD4 counts (50 or less) and the increasing
virulence of newer strains of C. difficile prevalent in the community. C. difficile is the most
common of the Clostridium species isolated (98%) in these patients, representing nearly
60% of all the pathogens identified.
References
1. Krones E, HOgenauer C. Diarrhea in the immunocompromised patient. Gastroenterol
Clin North Am. 2012;41(3):677-701.
2. Haines CF, Moore RD. Bartlett JG, et al. Clostridium difficile in a HIV-infected cohort:
incidence, risk factors, and clinical outcomes. AIDS. 2013;27(17):2799-2807.
3. Sanchez TH, Brooks JT, Sullivan PS, et al; Adult/Adolescent Spectrum of HIV Disease
Study Group. Bacterial diarrhea in persons with HIV infection, United States, 1992-
2002. Clin Infect Dis. 2005;41(11):1621-1627.
4. Centers for Disease Control and Prevention (CDC). Cryptosporidiosis outbreaks asso-
ciated with recreational water use-five states, 2006. MMWR Morb Mortal Wkly Rep.
2007;56(29):729-732.
CRITIQUE 16
Stool cultures may be the first diagnostic test performed by many primary care physicians,
but cultures should not delay endoscopic evaluation and, if negative for pathogens, do not
obviate the need for endoscopic evaluation. Most gastroenterologists would recommend
colonoscopy. with biopsy if indicated, to confirm the diagnosis of suspected inflammatory
bowel disease in cases where the disease is highly likely. Some reports suggest that culture
of biopsy material may have more utility than simple fecal cultures.
Imaging studies may be helpful in determining the extent of Crohn disease) because
endoscopic evaluation of the terminal ileum is often difficult. Stool analysis for DNA and
specific proteins has found utility in the past decade for screening for colorectal cancer.
Calprotectin, a calcium binding protein and heterodimer of S 1OOA8/A9 that accounts for
60% of the cytosolic protein content of neutrophils, is released into the fecal stream from
the site of inflammation. Its stability and resistance to bacterial degradation makes it a good
marker for distinguishing inflammatory bowel disease from irritable bowel syndrome.
Fecal Sl00Al2 (calgranulin C), another of the S!OO-family of proteins, is also easily
measured and recently is found to be superior to S 1OOA8/A9 in distinguishing between
the conditions. These tests are not substitutes for endoscopic confirmation of suspected
inflammatory bowel disease, patticularly in protracted cases of bloody diarrhea, and the
use of these tests can delay definitive diagnosis.
References
1. Abreu MT, Harpaz N. Diagnosis of colitis: making the initial diagnosis. Clin Gastroen-
terol Hepatol. 2007;5(3):295-301.
2. Fry RD, Mahmond NN, Maron DJ, Bleier JIS. Colon and rectum. In: Townsend CM,
Beauchamp RD, Evers BM, Mattox KL, eds. Sabiston Textbook a/Surgery: The Biolog-
ical Basis of Modern Surgical Practice. 19th ed. Philadelphia, PA: Elsevier Saun-
ders;2012: 1259-1381.
3. Katz JA. Management ofinflammatory bowel disease in adults. J Dig Dis. 2007;8(2):65-71.
4. Adams SM, Bornemann PH. Ulcerative colitis. Am Fam Physician. 2013;87(10):
699-705.
5. Dabritz J, Musci J, Foell D. Diagnostic utility of faecal biomarkers in patients with
irritable bowel syndrome. World J Gastroenterol. 2014:20(2):363-375.
6. Sherwood RA. Faecal markers of gastrointestinal inflammation. J Clin Pathol.
20 l 2;65(11 ):981-985.
224 CATEGORY 3 - PART IV
CRITIQUE 17
Ideally, patients with gallstone pancreatitis should undergo Iaparoscopic cholecystectomy
before discharge from their initial episode of pancreatitis. This approach results in a rate of
recurrent gallstone pancreatitis of less than 2%. However, mitigating factors may lead to a
recommendation to delay or even avoid cholecystectomy. Because the initiating factor is
passage of a gallstone, there is interest in endoscopic retrograde cholangiopancreatography
with sphincterotomy (ERCP/ES) as treatment for gallstone pancreatitis. In theory, this ap-
proach would allow relatively unimpeded passage of stones through the sphincter of Oddi
without obstruction and subsequent recurrence of pancreatitis. This would be a preferred
approach in patients with significant comorbidities or limited lifespan.
Recurrent pancreatitis rate is 5% in the first year after ERCP/ES as sole treatment for
gallstone pancreatitis. The readmission rate is 13%. Mortality (all-cause) in patients un-
dergoing no intervention is 4o/o. The rate of recurrent pancreatitis (at I year) in patients
undergoing no intervention is 11-13o/o. Delayed cholecystectomy is associated with a risk
of recurrent gallstone pancreatitis of 7%.
Answer: (A) ERCP/ES as sole treatment is associated with a recurrent pancreatitis rate
of 5% within the first year.
References
I. Hwang SS, Li BH, Haigh PI. Gallstone pancreatitis without cho!ecystectomy. JAMA
Surg. 2013:148(9):867-872.
2. El-Dhuwaib Y, DeakinM, David GG, Durkin D, Corless DJ, Slavin JP. Definitive manage-
ment of gallstone pancreatitis in England. Ann R Coll Surg Engl. 2012;94( 6):402-406.
CRITIQUE 18
Treatment of perforated diverticulitis is a scenario commonly encountered by general sur-
geons. In selected cases (Hinchey I and II), nonoperative management is appropriate. In
cases of feculent peritonitis (Hinchey IV), operative management is indicated. Although
antibiotics are one component of the management of these patients, nonoperative manage-
ment is inadequate. Similarly, given the diffuse nature of the process, percutaneous drain-
age is not feasible or effective. The goals of operative treatment are source control and
removal of peritoneal contamination. Proximal diverting colostomy does not optimally
control the source of contamination.
Trials are under way evaluating laparoscopic irrigation and drainage; however, feculent
peritonitis is not considered an appropriate presentation for this procedure. Laparoscopic
irrigation and drainage may have a role in Hinchey III diverticulitis (purulent peritonitis).
Laparotomy with bowel resection is the most appropriate approach. This approach con-
trols the source of contamination and allows for peritoneal lavage. Many patients undergo
end colostomy formation (Hartmann procedure). In selected cases, primary anastomosis is
possible. One randomized trial showed a reduction in the rate of postoperative peritonitis,
the need for early reoperation, and shorter length of stay in patients undergoing sigmoid
resection at the time of their initial exploration for perforated sigmoid diverticulitis.
References
I. Feingold D, Steele SR, Lee S, et al. Practice parameters for the treatment of sigmoid
diverticulitis. Dis Colon Rectum. 2014;57(3):284-294.
2. Zeitoun G, Laurent A, Rouffet F, et al. Multicentre, randomized clinical trial of primary
versus secondary sigmoid resection in generalized peritonitis complicating sigmoid
diverticulitis. Br J Surg. 2000;87(10): 1366-1374.
CATEGORY 3 - PART JV 225
CRITIQUE 19
Accurate staging of rectal cancer is paramount for developing the most appropriate treat-
ment plan. This includes accurately staging regional (mesorectal) lymph nodes. Because of
the inability of physical examination (i.e., digital anorectal exam) to accurately stage these
nodes, the surgeon must rely on imaging. Pelvic MRI is the most accurate of the listed
imaging modalities, with sensitivities of 58-85% and specificity of 88-97o/o.
PET scan does not achieve this level of accuracy for detecting nodal disease (sensitiv-
ity 29% ). Transabdominal ultrasound will not be able to adequately image the mesorectal
lymph nodes due to the confines of the pelvis, including pelvic bones and air-filled rectum.
Endorectal ultrasound is an alternative imaging modality with sensitivity equal to that of
MRI. Multiphase CT has a sensitivity of only 55% and specificity of 74% for detecting
mesorectal lymph nodes.
References
I. Yimei J, Ren Z, Lu X, Huan Z. A comparison between the reference values of MRI and
EUS and their usefulness to surgeons in rectal cancer. Eur Rev Med Pharmacol Sci.
2012; 16( 15):2069-2077.
2. Park JS, Jang YJ, Choi GS, et al. Accuracy of preoperative MRI in predicting pathol-
ogy stage in rectal cancers: node-for-node matched histopathology validation of MRI
features. Dis Colon Rectum. 2014;57(1):32-38.
3. Liang TY, Anil G, Ang BW. Imaging paradigms in assessment of rectal carcinoma: loco-
regional and distant staging. Cancer Imaging. 2012;12:290-303.
CRITIQUE20
Esophageal cancer has a rising incidence, particularly of adenocarcinoma compared with
squamous cell carcinoma. Nearly half a million cases are diagnosed annually with nearly
that many deaths. Fewer than half of patients survive longer than 5 years. Treatment has
primarily been surgical.
A prospective, randomized trial compared patients with resectable esophageal or
esophagogastric-junction tumors to surgery alone or chemotherapy and radiation followed
by surgery. Patients in the chemoradiotherapy group had longer overall survival, and more
patients underwent complete tumor-free resection. No difference in morbidity or early
mortality was observed. Interestingly, complete pathologic response was achieved in 29%
of patients.
Comparing the survival rates of patients who receive radiation and chemotherapy with
published survival rates of patients receiving preoperative chemotherapy alone suggests
that there is benefit to the addition of radiation. Radiation therapy followed by surgery is
not recommended by National Comprehensive Cancer Network guidelines.
References
1. van Hagen P, HulshofMC, van Lanschot JJ, et al; CROSS Group. Preoperative chemo-
radiotherapy for esophageal or junctional cancer. N Engl J Med. 2012;366(22):2074-
2084.
2. Ronellenfitsch U, Schwarzbach M, Hofheinz R, et al; GE Adenocarcinoma Meta-
analysis Group. Perioperative chemo(radio)therapy versus primary surgery for resect-
able adenocarcinoma of the stomach, gastroesophageal junction, and lower esophagus.
Cochrane Database Syst Rev. 2013;5:CD008107.
226 CATEGORY 3 - PART !V
CRITIQUE21
Since McBumey's first report in 1889, appendectomy has been the first-line therapy for
treatment of appendicitis. In the United States alone, more than 300,000 appendectomies
are performed annually. Although definitive and potentially lifesaving, these procedures
are not without short- and long-term risks, including wound infections, pneumonia, small
bowel obstruction, female infertility, and death. In fact, a Swedish population-based, case
control study demonstrated that appendicitis was responsible for only a small portion of
deaths after appendectomy and that mortality was strongly associated with comorbidity
status and negative appendectomy. In addition, the volume of procedures adds to the al-
ready high healthcare costs in the United States. Furthermore, the progression of acute
uncomplicated appendicitis might not lead inevitably to complicated disease (20% of pa-
tients presenting), but it may instead spontaneously resolve. Consequently, nonoperative
management of acute uncomplicated appendicitis with antibiotic therapy has gained in
popularity with several reported randomized controlled studies comparing this approach
to appendectomy. Such work demonstrates that nonoperative antibiotic therapy results in
lower complication rates and total sick leave time with similar mortality and hospital costs
compared with appendectomy. Treatment failure rates, however, are higher (27.3o/o com-
pared with 0% in the APPAC randomized controlled trial from Finland) as well as recur-
rence rates. Hospital stays may also be longer.
Thus, initial nonoperative therapy with antibiotics alone may be a reasonable approach
for patients presenting with acute uncomplicated appendicitis. In patients with complicated
appendicitis, other interventions are required. Because the presence of a fecalith on imag-
ing is associated with a high risk of perforation, such patients should undergo appendec-
tomy. Perforated appendicitis is a form of complicated appendicitis requiring intervention.
The presence of a right lower quadrant abscess should prompt percutaneous or operative
drainage. A dilated cystic appearing appendix with calcification is an indication for appen-
dectomy to rule out cystadenocarcinoma.
References
1. McCutcheon BA, Chang DC, Marcus LP, et al. Long-term outcomes of patients with
nonsurgically managed uncomplicated appendicitis. J Am Coll Surg. 2014;218(5):
905-913.
2. Di Saverio S, Sibilio A, Giorgini E, et al. The NOTA study (Non Operative Treatment
for Acute Appendicitis): prospective study on the efficacy and safety of antibiotics
(amoxicillin and clavulanic acid) for treating patients with right lower quadrant abdomi-
nal pain and long-term follow-up of conservatively treated suspected appendicitis. Ann
Surg. 2014;260(1):109-117.
3. Varadhan KK, Neal KR, Lobo DN. Safety and efficacy of antibiotics compared with
appendicectomy for treatment of uncomplicated acute appendicitis: meta-analysis of
randomised controlled trials. BMJ. 2012;344:e2156.
4. Salminen P, Paajanen H, Rautio T, et al. Antibiotic therapy vs appendectomy for treat-
ment ofuncomplicated acute appendicitis: TheAPPAC randomized clinical trial. JAMA.
20 l 5;313(23):2340-2348.
5. Vons C, Barry C, Maitre S, et al. Amoxicillin plus clavulanic acid versus appendicec-
tomy for treatment of acute uncomplicated appendicitis: an open-label, non-inferiority,
randomised controlled trial. Lancet. 2011;377(9777):1573-1579.
CATEGORY 3 - PART JV
227
CRITIQUE22
Gastroesophageal reflux disease (GERD) is a common gastrointestinal disorder. In the
United States alone, heartburn symptoms account for up to 5 million physician office visits
per year. Daily symptoms of GERD affect up to 20% of patients, and Barrett esophagus de-
velops in up to 10%. Laparoscopic fundoplication remains the preferred surgical treatment
of GERD in which the stomach fundus is wrapped completely (i.e., totally) or partially
around the esophagus in an effort to prevent reflux of gastric contents into the esophagus.
Nissen total fundoplication remains the most commonly performed procedure.
Before operative intervention, objective documentation of GERD is mandatory. Direct
endoscopic evidence of GERD can include erythema, slough of esophageal mucosa, stric-
ture, and Barrett esophagus. In peptic stricture and Barrett esophagus, malignancy must be
ruled out by histology. In the absence of direct endoscopic evidence, 24-hour ambulatory
esophageal pH monitoring is required. In addition to the endoscopy and pH monitoring,
preoperative workup may include barium swallow to delineate anatomy and esophageal
manometiy to rule out motility disorders.
Total fundoplication involves a 360° wrap of the esophagus. Partial fundoplications
typically involve either a 90" to 180" anterior wrap or a 200" to 270° posterior wrap of the
esophagus. Meta-analyses of randomized controlled trials comparing total fundoplication
to various forms of partial fundoplication demonstrate similar 5-year rates of patient sat-
isfaction, esophagitis, and postoperative use of proton pump inhibitors. Reoperation rates
are similar for partial fundoplication compared with total fundoplication. Finally, partial
fundoplication has lower rates of postoperative dysphagia and gas-related symptoms com-
pared with total fundoplication, whereas total fundoplication may be more effective at
controlling postoperative reflux symptoms.
References
1. Broeders JA, Roks DJ, Ahmed Ali U, et al. Laparoscopic anterior 180-degree versus
Nissen fundoplication for gastroesophageal reflux disease: systematic review and meta-
analysis of randomized clinical trials. Ann Surg. 2013;257(5):850-859.
2. Ma S, Qian B, Shang L, Shi R, Zhang G. A meta-analysis comparing laparoscopic
partial versus Nissen fundoplication. ANZ J Surg. 2012 ;82( 1-2): 17-22.
3. Stefanidis D, Hope WW, Kohn GP, Reardon PR, Richardson WS, Fanelli RD; SAGES
Guidelines Committee. Guidelines for surgical treatment of gastroesophageal reflux
disease. Surg Endosc. 2010;24( 11):2647-2669.
CRITIQUE23
Gastric cancer is the fourth most common cancer worldwide, and it is often diagnosed at
an advanced stage. Stage IV presentations constitute 40o/o of all cases, and 5-year survival
rates hover at 5%. Treatment of patients with metastatic disease is noncurative and should
be directed toward palliation (i.e., interventions designed to relieve symptoms or improve
quality oflife).
Chemotherapy is a first-line intervention in the treatment of advanced gastric cancer,
having both a survival and quality-of-life benefit compared with best supportive care. Five
classes of cytotoxic agents are typically used: fluoropyrimidines, platinum-based agents,
taxanes, topoisomerase inhibitors, and anthracyclines. They are often used in combinations
of2 or 3 agents, both of which demonstrate superior responses compared with single-agent
chemotherapy. Chemoradiation is not appropriate for stage IV disease.
Several recent systematic reviews demonstrate that palliative gastrectomy leads to a
better overall survival compared with no resection in advanced stage disease and that gas-
tric resection combined with chemotherapy has a better overall survival compared with
228 CATEGORY 3 - PART IV
References
I. Sun J, Song Y, Wang Z, et al. Clinical significance of palliative gastrectomy on the
survival of patients with incurable advanced gastric cancer: a systematic review and
meta-analysis. BMC Cancer. 2013;13:577.
2. Lasithiotakis K, Antoniou SA, Antoniou GA, Kaklamanos I, Zoras 0. Gastrectomy
for stage IV gastric cancer. A systematic review and meta-analysis. Anticancer Res.
2014;34(5):2079-2085.
3. Lordick F, Allum W, Carneiro F, et al. Unmet needs and challenges in gastric cancer: the
way forward. Cancer Treat Rev. 2014;40(6):692-700.
4. Karpeh MS Jr. Palliative treatment and the role of surgical resection in gastric cancer.
Dig Surg. 2013;30(2): 174-180.
CRITIQUE24
Meckel diverticulum, first described in 1598, is the most common congenital abnormality
of the gastrointestinal tract. The reported prevalence is 2-4%. The embryologic basis for
this anomaly occurs during the eighth week of gestation when there is failure or incom-
plete closure of the vitelline (omphalomesenteric) duct leading to a variety of congenital
gastrointestinal tract abnormalities. The failure of the left vitelline artery obliteration may
lead to a persistent mesodiverticular band that tethers the diverticulum to the umbilicus and
can serve as a lead point for intestinal volvulus. The location of the diverticulum can vary
throughout the alimentary tract; however, they are typically located within I 00 cm of the
ileocecal valve and are true diverticula with the walls containing all layers of the intestinal
tract.
A histopathological review ofresected symptomatic Meckel diverticula found that 74%
of surgical specimens were abnormal, with more than 40% containing ectopic tissue, 6%
having an enterolith, and 25o/o with evidence of diverticulitis. Pancreatic heterotopia is
found in only 5-16% of cases. When located in the distal diverticulum, pancreatic hetero-
topia can serve as the intussusceptum. By far the most common heterotopic tissue found
in resected specimens is gastric mucosa (33o/o). Acid secretion from the ectopic tissue can
lead to ulceration and bleeding associated with the diverticulum. The presence of hetero-
topic gastric tissue can be of diagnostic utility in the evaluation of gastrointestinal bleeding
and suspicion for a Meckel diverticulum, especially in children. Technetium-99m pertech-
netate scintigraphy, or Meckel scan, detects the ectopic gastric mucosa by taking up the
pertechnetate anion and manifests as a small area of increased activity. In the pediatric
population, this diagnostic test is sensitive (85%) and specific (95%) but is less accurate in
adults because the prevalence of gastric heterotopia declines with age.
A retrospective review of 5 decades of clinical experience with Meckel diverticulum at
the Mayo Clinic identified 1476 patients with Meckel diverticulum at the time of surgery.
Of these patients, 84% were asymptomatic Of patients with symptomatic disease, painless
lower gastrointestinal bleeding (38%) was the most common clinical presentation requir-
ing surgical intervention in the pediatric population. This is in contrast to adults, who most
commonly present with obstruction (34%). Nearly 25% of symptomatic Meckel diver-
ticulum presents with diverticulitis, including perforated diverticulitis, and it should be a
diagnosis considered in the setting of intra-abdominal abscesses without known causation.
CATEGORY 3- PART IV 229
Answer: (A) It is the most common congenital anomaly of the gastrointestinal tract.
References
1. Levy AD, Hobbs CM. From the archives of the AFIP, Meckel diverticulum: radiologic
features with pathologic correlation. Radiographies. 2004;24(2):565-587,
2. Park JJ, Wolff BG, Tollefson MK, Walsh EE, Larson DR. Meckel diverticulum: the
Mayo Clinic experience with 1476 patients (1950-2002). Ann Surg 2005;241(3):529-
533.
3. Cullen JJ, Kelly KA, Moir CR, Hodge DO, Zinsmeister AR, Melton LJ 3rd. Surgical
management ofMeckel's diverticulum. An epidemiologic, population-based study. Ann
Surg 1994;220(4):564--568; discussion 568-569.
4, Soltero MJ, Bill AH. The natural history of Meckel's diverticulum and its relation to
incidental removal. A study of 202 cases of diseased Meckel 's diverticulum found in
King County, Washington, over a fifteen year period. Am J Surg. 1976;132(2): 168-173.
CRITIQUE25
Although surgery is a safe and effective approach to treat selected patients with evidence of
persistent ulcer bleeding or rebleeding, other options should be considered before surgery,
such as a second attempt at endoscopic hemostasis or angiography. Angiography with tran-
sarterial embolization provides a nonoperative option for patients where acute peptic ulcer
bleeding is not controlled by endoscopy. Absorbable gelatin, polyvinyl alcohol, cyanoac-
rylate glues, and coils are used to embolize bleeding lesions. The primary technical success
rates range from 52% to 58%, with recurrent bleeding requiring repeated embolization
procedures in about 10-20% of patients.
Emergency surgery for bleeding peptic ulcer disease entails oversewing of the ulcer
(to ligate the bleeding artery) and pyloroplasty (for gastric drainage) with consideration
for truncal vagotomy (to decrease acid secretion). More time-consuming procedures (e.g.,
highly selective vagotomy) can be performed either during standard laparotomy or lapa-
roscopically for nonemergency ulcer surgery. Similarly, antrectomy and Billroth 2 recon-
struction is now reserved for an elective situation when obstruction complicates peptic
ulcer disease.
All patients with bleeding peptic ulcers should receive acid-suppressive treatment with
a proton pump inhibitor (PPI). In general, a high dose intravenous PPls should be initiated
in all patients with suspected clinically significant upper gastrointestinal bleeding before
endoscopy as part of initial management. There is no difference in efficacy among the
various PPis.
230 CATEGORY 3 - PART IV
Nonselective beta-blockers lower portal pressure and reduce the risk of first bleeding
in patients with esophageal varices. For these reasons, they are the treatment of choice for
primary prophylaxis in patients at high risk for variceal bleeding but not for treatment of a
bleeding peptic ulcer.
References
I. Gralnek IM. Will surgery be a thing of the past in peptic ulcer bleeding? Gastrointest
Endosc. 2011;73(5):909-910.
2. Saltzman JR: Overview of the treatment of bleeding peptic ulcer. In: Feldman M, Travis
AC. Waltham, MA: UpToDate;2015. Updated: Sept I, 2015. Accessed: Jan 7, 2016.
CRITIQUE26
The presence of a tubular distended cystic mass with wall calcification in the right lower
quadrant adjacent to the cecum strongly suggests an appendiceal mucocele (figure 26.3).
The course and prognosis of appendiceal mucoceles relates to their histologic subtypes,
consisting of mucosal hyperplasia, simple or retention cysts, mucinous cystoadenomas,
and mucinous cystadenocarcinoma. The nonneoplastic mucoceles have a higher preva-
lence compared with the neoplastic mucoceles. Surgical resection should be pursued~ even
for a benign-appearing appendiceal mucocele, because lesions that appear to be benign on
imaging studies may harbor a cystadenocarcinoma. Appendectomy, open or laparoscopic,
should be considered ifthere is no hint of spread or apparent nodularities in the wall of the
appendix. When there is suspicion of nodal spread, involvement of the terminal ileum or
cecum, or positive margins on pathologic examination, a right hemicolectomy should be
performed. Cecotomy would not be indicated under any circumstances.
Laparoscopic exploration alone without appendectomy or observation with repeat CT
scan is not acceptable because of the risk of cystadenocarcinoma. Intraperitoneal hyper-
thermic chemotherapy is advocated after radical surgical removal and debulking in cases
of pseudomyxoma peritonei and disseminated peritoneal adenomucinosis associated with
mucin-producing adenocarcinoma Right oophorectomy alone is not indicated unless part
of a more radical operation for debulking.
References
1. Zissin R, Gayer G, Kots E, Apter S, Peri M, Shapiro-Feinberg M. Imaging ofmucocele
of the appendix with emphasis on the CT findings: a report of I 0 cases. C/in Radio!.
1999;54(12):826-832.
2. Demetrashvili Z, Chkhaidze M, Khutsishvili K, et al. Mucocele of the appendix: case
report and review ofliterature. Int Surg. 2012;97(3):266-269.
3. Shankar S, Ledakis P, El Halabi H, Gushchin V, Sardi A. Neoplasms of the appendix:
current treatment guidelines. Hematol Oneal Cl in North Am. 2012;26(6): 1261-1290.
CATEGORY 3- PART !V 231
CRITIQUE27
Relapse after initial control of symptoms of ulcerative colitis is unpredictable, with great
variability in severity among patients affected by the disease. Conventional medical
therapy with 5-aminosalicylic acid and corticosteroids is less effective once relapse has
occurred, and continuing corticosteroids is not an effective means of maintaining remis-
sion. This includes oral steroids as well as steroid enemas. Cyclosporine, a calcineurin
inhibitor, was historically used in patients who were unresponsive to the reinstitution of
conventional therapy, but this practice was abandoned because of serious side effects seen
in 15% of patients and a reported mortality of up to 3.5%. Most complications reported
with the use of cyclosporine occur in patients who are concomitantly receiving steroids
and included nephrotoxicity (24%), neurotoxicity (51 %), and infections (21 %). Although
most of these complications could be mitigated with dose adjustment, immunomodulating
agents targeted against tumor necrosis factor-a. such as infliximab and adalimumab are
now the agents of choice, particularly in refractory disease. Surgery is reserved for toxic
megacolon, massive bleeding, dysplasia, carcinoma, or medical intractability associated
with severe symptoms.
References
1. Burness CB, Keating GM. Adalimumab: a review of its use in the treatment of patients
with ulcerative colitis. BioDrugs. 2013;27(3):247-262.
2. Cohen JL, Strong SA. Hyman NH, et al; Standards Practice Task Force American Soci-
ety of Colon and Rectal Surgeons. Practice parameters for the surgical treatment of
ulcerative colitis. Dis Colon Rectum. 2005;48(11):1997-2009.
3. Danese S. Adalimumab in ulcerative colitis: ready for prime time. Dig Liver Dis.
2013;45(1):8-13.
4. Mehta SJ, Silver AR, Lindsay JO. Review article: strategies for the management of
chronic unremitting ulcerative colitis. Aliment Pharmacol Ther. 2013;38(2):77-97.
________________________ ________ ,
232 CATEGORY 3 - PART JV
5. Stemthal MB, Murphy SJ, George J, Kornbluth A, Lichtiger S, Present DH. Adverse
events associated with the use of cyclosporine in patients with inflammatory bowel
disease. Am J Gastroenterol. 2008; 103(4):937-943.
CRITIQUE28
Adenomatous polyps are by definition benign, but they do have the potential for malignant
transformation consistent with the polyp-cancer sequence. Polyps are classified by their
macroscopic structural configuration: (I) sessile, which have broad bases and do not have a
stalk, or (2) pedunculated, which have a supporting stalk. Histologic differentiation is also
possible: tubular, vil!ous, and tubulovillous. Finally, varying degrees of dysplasia or cancer
can be seen in any of the polyps.
The depth of adenocarcinoma invasion into the polyp defines the risk of lymph node
metastasis and whether endoscopic management of the polyp is possible. Haggitt et al.
described a classification system for pedunculated polyps with levels of invasion rang-
ing from 0 to 4 (figure 28.1). Level 0 indicates an intramucosal carcinoma, which are not
invasive and pose no risk of lymphatic invasion. When there is invasion of the muscularis
mucosa into the submucosa in the head of the polyp, a level 2 classification is assigned.
These lesions can be treated by polypectomy with a stalk that is clear of cancer. When the
stalk is invaded, it is a level 3 lesion. Level 4 lesions signify invasion of the adenocarci-
noma into the bowel wall below the polyp stalk and have an increased rate of lymph node
involvement. Because sessile polyps do not contain stalks they are classed as level 4 and
are considered to be equivalent to a level 4 pedunculated polyp.
The Kudo classification system addressed the recognition that the risk of lymph node
metastasis in each level 4 lesion is not the same, 3 types (figure 28.2) of invasion are de-
scribed. Submucosal invasion is classified into thirds, with Sm I lesions exhibiting invasion
into the upper third of the submucosa, Sm2 invasion into the middle third of the submu-
cosa, and Sm3 invasion into the lower third of the submucosa.
Endoscopic resection is possible ifthe risk of nodal involvement is low. The risk is less
than 1% for polyps with a Haggitt level 1, 2, or 3 that are completely removed. Endoscopic
polypectomy is also possible and safe for Haggitt level 4 lesions with favorable histology
or sessile lesions with Sm 1 or Sm2 depth without additional unfavorable histologic fea-
tures and margins bigger than 2 mm. It is believed that these lesions are low risk and can be
favorably managed with endoscopic polypectomy alone provided margins are larger than
2 mm. Endoscopic resection is not considered adequate for pedunculated polyps with Hag-
gitt level 4 lesions exhibiting unfavorable histology, sessile lesions with Sm 1 or Sm2 depth
and poor histologic features, and sessile lesions with Sm3 depth. These lesions have nodal
involvement in l 2-25o/o of cases. When a polyp is removed in a piecemeal fashion, histo-
logic interpretation is difficult and is another indication for oncologic segmental resection.
Answer: (D) Sessile lesion with adenocarcinoma removed piecemeal during endoscopy
References
1. Ramirez M, Schierling S, Papaconstantinou HT, Thomas JS. Management of the malig-
nant polyp. Clin Colon Rectal Surg. 2008;21(4):286-290.
2. Nivatvongs S. Surgical management of malignant colorectal polyps. Surg Clin North
Am. 2002;82(5):959-966.
3. Haggitt RC, Glotzbach RE, Soffer EE, Wruble LD. Prognostic factors in colorectal
carcinomas arising in adenomas: implications for lesions removed by endoscopic
polypectomy. Gastroenterology. l 985;89(2):328-336.
CATEGORY 3 - PART JV
~--~llrl:
proprli - ---
"i'~~}~"i;~:~:=·~~::~±~~::
--- - L:JJ}~~~ ~~~~~4'.~~~~
----~--...::.-~~
Figure 28.2 Depth of invasion into submucosa. Sml: invasion into upper third of
submucosa; Sm2: invasion into middle third of submucosa; Sm3: invasion into lower
third of submucosa.
Reprinted with pennission from World Journal of Surgery Surgical Management of Early Colorectal Cancer,
24. 2000, p. 1053, S Nivatvongs. Copyright© 2000 by the Socie'tc' Internationale de Chirurgie. With
permission of Springer.
CRITIQUE29
Bowel intussusception in adult patients is substantially different from pediatric intussus-
ception regarding its etiology, clinical presentation, and treatment strategies. In children,
intussusception is usually idiopathic and benign, whereas 65o/o of adult intussusceptions
are associated with malignancy.
Contrast-enhanced CT scan of the abdomen is the most sensitive diagnostic method to
confirm bowel intussusception, with a diagnostic accuracy ranging from 60 to lOOo/o. The
most characteristic signs include target or sausage-shaped lesions as a concentric hyper-
234 CATEGORY 3 - PART IV
References
1. Mrak K. Uncommon conditions in surgical oncology: acute abdomen caused by ileoco-
lic intussusception. J Gastrointest Oneal. 2014;5(4):E75-E79.
2. Zubaidi A, Al-Saif F, Silverman R. Adult intussusception: a retrospective review. Dis
Colon Rectum. 2006;49(10): 1546-1551.
3. Erkan N, Haciyanli M, Yildirim M, Sayhan H, Vardar E, Polat AF. Intussusception
in adults: an unusual and challenging condition for surgeons. Int J Colorectal Dis.
2005;20(5):452-456.
4. Marinis A, Yiallourou A, Samanides L, et al. Intussusception of the bowel in adults: a
review. World J Gastroenterol. 2009; 15(4):407-411.
CATEGORY 3 - PART IV 235
CRITIQUE30
Local recurrence of colorectal cancer has significantly decreased from 20-30o/o to 6-10%
with a combination of improved surgical therapy as well as radiation and chemotherapy.
However, in approximately 50% of patients with recurrence, locally recurrent tumor is
the only manifestation of disease, and the prognosis is poor without an attempt at surgical
resection. Predictors of recurrence include nonradical resection or incomplete total me-
sorectal excision, positive circumferential resection margin, more advanced T-stage, and
tumor-involved lymph nodes.
Local colorectal cancer recurrence develops in the posterior bony pelvis as an isolated
event in approximately half of patients with recurrence. Although radiation can palliate
sacral root pain, the disease is usually progressive.
If the patient has local recurrence in the absence of distant metastasis, surgical manage-
ment represents a viable treatment option (figure 30.3). A complete en bloc resection of the
recurrent cancer with sacral resection and preservation of nerve roots in the posterior pelvis
and the sciatic nerve can be performed to attempt curative resection. To achieve tumor-free
resection margins (RO resection), resection of involved organs must be en bloc. Central
or anterior pelvic recurrences are more likely to have RO resections than other locations,
especially lateral recurrences.
Figure 30.3
Pelvic exenteration is not warranted) because the patient does not have involvement of the
urogenital organs. Chemoradiation is not warranted in this patient, because imaging suggests
the recurrence is surgically resectable. Up to 80% of patients with recurrent tumor initially
not amenable to curative resection may be converted to resectable status after high-dose pre-
operative chemoradiation. All other options are palliative in nature (figure 30.4).
236 CATEGORY 3 - PART IV
Radical resection
outside original
plane of dissection
Figure 30.4 Algorithm for management of operable central recurrences. Notes. AR=
anterior resection: APER= abdominoperineal excision.
From Clinical Algorithms for the Surgical Management of Locally Recurrent Rectal Cancer. by AH Mimezami.
PM Sagar. D Kavanagh. et al. 2010. Diseases ofthe Colon and Rectum. 53. p. 1253. Copyright: \02010 The
ASCRS 2010 Reprinted with permission.
References
1. Mimezami AH, Sagar PM, Kavanagh D, Witherspoon P, Lee P, Winter D. Clinical
algorithms for the surgical management of locally recurrent rectal cancer. Dis Colon
Rectum. 20I0;53(9):1248- 1257.
2. Nielsen MB, Laurberg S, Holm T. Current management of locally recurrent rectal
cancer. Colo rectal Dis. 201 1; 13(7):732-742.
3. Sagar PM. Extended surgery for local recurrence and advanced rectal cancer. Colorectal
Dis. 2006;8 (suppl 3):43-46.
4. Colibaseanu DT, Dozois EJ, Mathis KL, et al. Extended sacropelvic resection for local-
ly recurrent rectal cancer: can it be done safely and with good oncologic outcomes? Dis
Colon Rectum. 20 14;57(1):47- 55.
CRITIQUE 31
Even though the incidence and mortality from gastric cancer has decreased during the past
2 decades, the proportion of stage IV gastric cancer increased significantly during the same
period and comprised more than 40% of total cases.
Two meta-analyses showed that palliative gastrectomy had a statistically significant
survival benefit on patients with incurable advanced gastric cancer, especially stage Ml
gastric cancer patients. Survival advantage was longer when chemotherapy was used. For
patients with liver metastasis, combined hepatic resection and chemotherapy may provide
better survival compared with simple palliative gastrectomy. One-year survival was greater
in patients undergoing gastrectomy versus conservativt_! (p <.000 I) or nonresectional treat-
ment (p <.0001 ). Given this patient's health, liver resection with gastrectomy and chemo-
therapy offers the best chance for survival at 1 year. Percutaneous endoscopic gastrectomy,
balloon dilation of the antral stricture, and chemotherapy offer inferior outcomes.
CATEGORY 3 - PART IV 237
References
I. Lasithiotakis K, Antoniou SA, Antoniou GA, Kaklamanos I, Zoras 0. Gastrectomy
for stage IV gastric cancer. a systematic review and meta-analysis. Anticancer Res.
20 14:34(5):2079-2085.
2. Sun J, Song Y, Wang Z, et al. Clinical significance of palliative gastrectomy on the
survival of patients with incurable advanced gastric cancer: a systematic review and
meta-analysis. BMC Cancer. 2013;13:577.
CRITIQUE32
The etiology ofpneumatosis intestinalis, or air contained within the submucosa of the intes-
tinal wall, can vary from benign to a significant life-threatening intra-abdominal pathology.
In this case, CT scan imaging shows widespread small bowel pneumatosis with numer-
ous foci of air in the mesenteric vasculature, evidence of extensive portal venous gas, and
free fluid, all features concerning for intestinal ischemia. There is no clear evidence of
thrombosis or embolus of the mesenteric vasculature.
CT imaging may also show twisted intestine, mesentery, and mesenteric vessels-
known as the "whirl or whirlpool sign"--consistent with the diagnosis of small intestinal
volvulus (figure 32.3). Midgut volvulus is a life-threatening emergency, and early diagno-
sis is key to improving the prognosis. Early surgical intervention is warranted. Intravenous
heparin or tissue plasminogen activator infusion is not indicated. because the intestinal
ischemia is related to the volvulus, not to intra-arterial mesenteric thrombus. Angiography
with vasodilator infusion is indicated when treating a vasospastic condition. Colonoscopy
with tube decompression is indicated for colonic ileus or pseudo-obstruction.
References
1. Video of the month: whirlpool sign in intestinal volvulus. Am J Gastroenterol.
2014;109(4):472.
2. Zhou H, Yan Y, Li C. The whirlpool sign: midgut volvulus. Emerg Med J. 2014:
31(12):1015.
3. Menke J. Diagnostic accuracy of multidetector CT in acute mesenteric ischemia:
systematic review and meta-analysis. Radiology. 2010;256(1):93-101.
CRITIQUE33
Patients with longstanding ulcerative colitis have a known increased risk of developing co-
lon cancer and require frequent endoscopic surveillance. Dysplasia in the setting of chronic
ulcerative colitis is an ominous sign and oftentimes will warrant colectorny to prevent the
development of colon cancer. The dysplasia associated with chronic ulcerative colitis is
usually, but not always, in the area of active colitis. An endoscopically visible lesion with
dysplasia in the setting of chronic ulcerative colitis is known as a dysplastic-associated
lesion or mass (DALM). In patients with chronic ulcerative colitis, the presence of a true
DALM is an indication for a proctocolectomy, because the incidence of having colon can-
cer within the flat lesion or of developing colon cancer in the near future is quite high.
These lesions are often very flat and can be very difficult to see endoscopically, making
detection troublesome.
However, patients with chronic ulcerative colitis can also develop sporadic adenomas
similar to the rest of the population. Sporadic adenomas may also have evidence of dys-
plasia, so distinguishing a sporadic adenoma from a true DALM can be a challenge. This
distinction is important, however, because the treatment for a sporadic adenoma is simple
polypectomy and does not warrant a colectomy.
In general, lesions found outside the area of inflammation are more likely to be sporadic
adenomas. Therefore, patients with chronic ulcerative colitis who have a polypectomy
performed should have extensive sampling of the rest of the colon with random biopsies
throughout the colon. In particular, the area immediately surrounding the resected polyp
should be biopsied, looking for any evidence of occult dysplasia. If dysplasia is noted on
any of these other biopsies, the resected lesion is more likely to be a DALM, and procto-
colectomy would be warranted. If the remaining biopsies are all normal, surveillance is an
acceptable treatment alternative.
In this scenario, the patient has a history of very well controlled colitis. A polyp was
successfully removed endoscopically. Other biopsies, including just adjacent to the lesion,
are all normal. Therefore, this lesion is best classified as a sporadic adenoma. Because of
its size and the fact that it was removed piecemeal, early surveillance is warranted to ensure
there is no recurrence. Ideally, the area of the polyp should also be tattooed at the time of
polypectomy for easy identification of the area.
Colonoscopy at 5 years is an appropriate interval for patients having routine surveil-
lance after a routine polypectomy, but this is too long for patients with longstanding chron-
ic ulcerative colitis and for those patients with a complex polypectomy.
For patients with true DALMs, a total proctocolectomy with or without an ilea] anal
pouch is probably the best option. However, limited resections of the affected area (i.e.,
extended right colectomy, total colectomy with ileorectal anastomosis) can be done for
those patients without active colitis who are poor candidates for a total proctocolectomy.
References
1. Odze RD. Adenomas and adenoma-like DALMs in chronic ulcerative colitis: a clinical,
pathological, and molecular review. Am J Gastroenterol. 1999;94(7): 1746-1150.
CATEGORY 3 - PART IV
239
CRITIQUE34
Recurrence after colon cancer surgery depends on multiple factors-nutrition, tumor stage,
comorbidities, and anemia. The decision to transfuse a patient in the perioperative period
should not be taken lightly. In a large meta-analysis conducted by the Cochrane Group,
the effects of perioperative blood transfusion on recurrence in a randomized study yielded
an odds ratio of l.42 (95% confidence interval, 1.20-1.67, p<.0001) against transfused
patients. This occurs regardless of whether the transfusion is performed preoperatively or
postoperatively.
Intraoperative cell salvage and autologous blood transfusion do not increase risk of
recurrence. Studies suggest that this risk is related to the red cell component, because leu-
koreduction techniques do not alter the associated risk. Blood plasma component transfu-
sions do not appear to affect the recurrence risk. Finally, there is no evidence that the age
of transfused blood plays a role in recurrence risk. Several studies have found no difference
in transfusion of older (>21 days) versus younger blood in patients with colorectal surgery
References
1. Cata JP, Wang H, Gottumukkala V, Reuben J, Sessler DI. Inflammatory response,
immunosuppression, and cancer recurrence after perioperative blood transfusions. Br J
Anaesth. 2013;110(5):690-701.
2. Amato A, Pescatori M. Perioperative blood transfusions for the recurrence of colorectal
cancer. Cochrane Database Syst Rev. 2006;(1 ):CD005033.
CRITIQUE35
This patient has a gastrointestinal stromal tumor (GIST). GISTs comprise 0.1-3% of all
gastrointestinal malignancies. They occur most commonly in the stomach (50-70% ). small
intestine (25-35%), and colon/rectum (5-10%). Most GISTs express KlT (CD! 17), a por-
tion of the tyrosine kinase receptor encoded by the protooncogene c-kit. GISTs spread via
direct local invasion, most commonly to the liver, omentum, and peritoneum. They do not
spread via lymph nodes.
Surgery remains the primary treatment option for GIST. Resection to negative margins
is considered optimal surgical therapy. For most tumors (:SS cm), this can be accomplished
via laparoscopic wedge resection. Larger tumors, or those adhered to adjacent structures,
require laparotomy.
Observation with repeat esophagogastroduodenoscopy in 1 year would be considered
appropriate for GISTs less than 1 cm, which have a very low malignant potential. Neo-
adjuvant imatinib is not indicated for a GIST amendable to resection and is indicated for
those situations in which tumor shrinkage would either simplify an operation or allow an
operation to be downstaged. Subtotal gastrectomy with regional lymph node dissection is
appropriate for gastric adenocarcinoma but would be too extensive for a GIST. Endoscopic
submucosal resection is not recommended, because the GIST involves the muscularis pro-
pria, and attempts at resection could result in perforation.
References
1. Raul CP, Pawlik TM. Gastrointestinal stromal tumors. In: Cameron JL, Cameron AM, eds.
Current Surgical Therapy. 11th ed. Philadelphia, PA: Elsevier Saunders;2014:96-103.
2. Fairweather M, Raut CP. Surgical management of GIST and intra-abdominal visceral
leiomyosarcomas. J Surg Oneal. 2015;111(5):562-569.
CRITIQUE36
When a diagnosis of cirrhosis is made, approximately 50% of patients will already have
gastroesophageal varices from portal hypertension. Portal hypertension is present when
the hepatic venous pressure gradient (HVPG) is greater than 5 mm Hg, but it is consid-
ered clinically significant when the HVPG is greater than 10 mm Hg. The more severe
the cirrhosis is, the more common varices are, with the highest rate among patients with
Child-Turcotte-Pugh (Child) class B or C disease. The 6-week mortality for each episode
of variceal hemorrhage has fallen with modem management but is still 15-200/o overall;
ranging from Oo/o among patients with Child class A disease to approximately 30o/o among
patients with Child class C disease.
Nonselective beta-adrenergic blockers lower portal flow by a reduction of cardiac out-
put (beta-! blockade) and splanchnic vasoconstriction (beta-2 blockade), making these
agents better than selective beta-blockers to treat a patient with bleeding esophageal vari-
ces. They are given orally and are helpful in the long-term treattnent of portal hypertension.
Nitrates can be used as direct vasodilators. Splanchnic vasoconstrictors such as vasopressin
and somatostatin are used in the acute setting ofvariceal bleeding to reduce portal pressure,
but none of these induces hepatic encephalopathy.
A reduction in the HVPG to less than 12 mm Hg or a reduction of more than 20°/o from
the baseline value is associated with a decreased risk ofvariceal hemorrhage and improved
survival. A shunt between the portal and systemic venous systems will lower portal pres-
sure. This can be achieved surgically or by placement of a transjugular intrahepatic porto-
systemic shunt (TIPS). After TIPS, 20% of patients develop encephalopathy.
Endoscopic banding or sclerotherapy can eradicate varices, but retreatment is often nec-
essary and encephalopathy does not usually occur from the procedures.
References
I. Bloom S. Kemp W, Lube! J. Portal hypertension: pathophysiology, diagnosis and
management. Intern Med J. 20 l 5;45(1):16-26.
2. Garcia-Tsao G, Bosch J. Management ofvarices and variceal hemorrhage in cirrhosis
[published correction appears in N Engl J Med. 2011 ;364(5):490. Dosage error in article
text.]. N Eng!J Med. 2010;362(9):823-832.
CRITIQUE37
Operative management of peptic ulcer disease and its complications is less common, be-
cause medical and endoscopic therapies are treatment modalities with excellent short- and
long-term results. However, the surgeon must remain abreast of the disease~ because surgi-
cal management continues to have a role in treatment.
The ideal operation for peptic ulcer associated gastric outlet obstruction should diag-
nose and treat the condition by relieving the obstruction and treating the underlying ulcer
disease. Graham patch closure treats the acute perforation but fails to treat the underlying
gastric outlet obstruction or peptic ulcer disease. Highly selective vagotomy with pyloro-
plasty or gastrojejunostomy are options but probably are not well suited in patients with
CATEGORY 3 - PART IV
241
intractable duodenal ulcer disease, gastric outlet obstruction, fundic peptic ulceration, and
perforated ulcer with more than 24 hours of soilage. Furthermore, success is related to the
operative experience of the surgeon., and recurrence rates are relatively high, ranging from
5 to 30o/o. Currently, endoscopic stent placement is not routinely used in benign stricture
or perforation situations.
Given its extremely low recurrence rate, truncal vagotomy and antrectomy should be
used to treat resistant or complicated peptic ulcer disease in patients with a high risk for
recurrence with less aggressive therapy. Vagotomy in the patient with a chronically ob-
structed stomach may lead to chronic gastroparesis. Although an uncommon complication,
gastroparesis after vagotomy and antrectomy can be debilitating, and medical treatment
often fails.
References
1. Lagoo J, Pappas TN, Perez A. A relic or still relevant: the narrowing role for vagotomy
in the treatment of peptic ulcer disease. Am J Surg. 2014;207(1):120-126.
2. Stomach. In: Brunicardi F, Andersen DK, Billiar TR, et al, eds. Schwartz's Principles of
Surgery. 9th ed. New York, NY: McGraw-Hill;2010:889-948.
3. Kochar R. Shah N. Enteral stents: from esophagus to colon. Gastrointest Endosc.
2013;78(6):913-918.
CRITIQUE38
Esophagectomy is an aggressive operation that should not be undertaken lightly, particu-
larly in elderly patients. Multiple studies suggest that esophageal squamous cell carcinoma
responds quite well to chemoradiation and that overall patient survival is not significantly
enhanced by the addition of esophagectomy. Moreover, a favorable response of the tumor
to chemoradiation identifies a group of patients with good prognosis, regardless of whether
surgery is performed. In these patients, routine esophagectomy is no longer recommended.
Instead, when a patient shows a favorable response to induction therapy, as demonstrated
by resolution of gross tumor observed endoscopically and reduction of metabolic activity
on PET scan, careful observation is recommended rather than additional chemotherapy,
radiotherapy, or surgery.
References
I. Stahl M, Stuschke M, Lehmann N, et al. Chemoradiation with and without surgery in
patients with locally advanced squamous cell carcinoma of the esophagus [published
correction appears in J C/in Oneal. 2006;24(3):531]. J C/in Oneal. 2005;23(10):
2310-2317.
2. Bedenne L, Michel P, Bouche 0, et al. Chemoradiation followed by surgery compared
with chemoradiation alone in squamous cancer of the esophagus: FFCD 9102. J Cl in
Oneal. 2007;25(1OJ:1160-1168.
3. Levy RM, Trivedi D, Luketich JD. Minimally invasive esophagectomy. Surg C/in North
Am. 2012;92(5):1265-1285.
242 CATEGORY 3 - PART JV
CRITIQUE39
Surgical therapy is warranted in patients with intestinal obstruction associated with pre-
sumed or confirmed ischemia, necrosis, perforation, and peritonitis or in those patients in
whom the obstruction does not resolve within 48 hours of conservative therapy. The use of
prosthetic mesh repair in the emergency management of the acutely incarcerated or stran-
gulated ventral hernia is safe. Some studies indicate that nonoperative management beyond
48 hours does not diminish the need for surgery and may even increase surgical morbidity.
Assessing liver fibrosis in the management of patients with chronic hepatitis C is a clini-
cally significant issue~ but it would not take priority in this setting. Liver biopsy has tradi-
tionally been considered the gold standard for the evaluation of tissue damage, including
fibrosis. but it is an invasive procedure with a risk of rare but potentially life-threatening
complications. Colonoscopy and incisional biopsy are clearly not indicated in this case of
small bowel obstruction due to incarcerated ventral hernia.
The patient's history, clinical findings, and CT showing free fluid in the hernia sac indi-
cate surgical intervention is the next step (figure 39.2).
References
I. Bessa SS,Abdel-RazekAH. Results of prosthetic mesh repair in the emergency manage-
ment of the acutely incarcerated and/or strangulated ventral hernias: a seven years study.
Hernia. 2013;17(1 ):59-65.
2. Perry Z, Netz U, YitzhakA, Mizrahi S. Pros and cons in the approach to an incarcerated
umbilical hernia in the pregnant woman. Am Surg. 2011 ;77(3):E43-E44.
3. Hue! T. Acute GI obstruction. Best Pract Res Clin Gastroenterol. 2013;27(5):691-707.
4. Castera L, Bedossa P. How to assess liver fibrosis in chronic hepatitis C: serum markers
or transient elastography vs. liver biopsy? Liver Int. 2011:31 (suppl l ): 13-17.
CATEGORY 3 - PART IV 243
CRITIQUE40
Crohn disease (regional ileitis) is reported to affect 1.2 million people in the United States
(3-7 individuals/! 00,000), with an equal distribution among men and women. African
Americans have an incidence similar to whites. The disease is more common in those of
Jewish heritage compared with age-matched controls, and it is more common in urban ar-
eas. It is generally a disease of young adults, with a peak incidence in the second and third
decades of life. Crohn disease is the most common primary surgical disease of the small
intestine. Crohn disease may affect any portion of the gastrointestinal tract from the mouth
to the anus. Ileocolic involvement occurs in 55-60% of patients, followed by small bowel
only (30%), and isolated colonic involvement (15-20%). The appendix is rarely the only
site of Crohn disease. The rectum is generally spared in contrast to ulcerative colitis, but
perianal involvement (commonly fistula and abscesses) is found in 33o/o percent of patients
with large bowel involvement.
References
1. Cheifetz AS. Management of active Crohn disease. JAMA. 20 l 3;309(20):2150-2158.
2. McKenzie S, Evers BM. Small intestine. In: Townsend CM, Beauchamp RD, Evers
BM, Mattox KL, eds. Sabiston Textbook of Surgery: The Biological Basis of Modern
Surgical Practice. 19th ed. Philadelphia, PA: Elsevier Saunders;2012:1227-1279.
Category 4 - Part I
Abdomen
Items 1-30
DIRECTIONS: Each set of lettered headings or illustrations is followed by a list of num-
bered words, phrases, or illustrations. For each numbered word, phrase, or illustration, fill
in the space on the answer sheet containing
(A) if the item is associated with (A) only,
(B) ifthe item is associated with (B) only,
(CJ if the item is associated with both (A) and (B),
(D) ifthe item is associated with neither (A) nor (B).
Items 1-3
(A) Laparoscopic mesh repair
(B) Open mesh repair
(C) Both
(D) Neither
1. Recurrence after initial open mesh inguinal hernia repair
Items 4-7
(A) Laparoscopic cholecystectomy
(B) Percutaneous cholecystostomy
(CJ Both
(D) Neither
4. Acute cholecystitis and septic shock on multiple pressers
5. Acalculous cholecystitis
Items 8--11
(AJ Cross-linked biologic matrix
(BJ Non-cross-linked biologic matrix
(CJ Both
(D) Neither
8. Promotes early cellular and vascular infiltration
Items 12-15
(A) Enucleation
(B) Hypertonic saline injection
(C) Observation
(D) Resection to negative margins
(E) Transarterial embolization
12. 5-cm focal nodular hyperplasia (FNH), asymptomatic
Items 16-18
(A) Chromic gut
(B) Polyglactin 910
(C) Polyglecaprone 25
(D) Polydioxanone
(E) Silk
16. Shortest time for absorption
Items 19-22
(A) Repeat colonoscopy in 10 years
(B) Repeat colonoscopy in 3 years
(C) Repeat colonoscopy in 6 months
(D) Repeat colonoscopy in I year
(E) CT colonography in 6 months
19. Presence of2 hyperplastic polyps on initial screening colonoscopy
20. Presence of 2 tubular adenomas, each less than 1 cm and without dysplasia, on initial
screening colonoscopy
21. Presence of 1.1-cm tubular adenoma with dysplasia, margin of resection negative on
initial screening colonoscopy
23. A 65-year-old woman with a BM! of 45 presents with a midline incisional hernia
defect after a total abdominal hysterectomy for fibroids. The defect is infraumbilical, 6
cm wide, and reducible. In the treatment of this ventral hernia, laparoscopic and open
repair have similar rates of which of the following?
(A) Mesh infection
(B) Wound morbidity
(C) Cost of therapy
(D) Hernia recurrence
(E) Intraoperative complications
24. Which of the following statements regarding the nonoperative management of uncom-
plicated acute appendicitis is true?
(A) No randomized trials compare nonoperative versus operative management.
(B) Most failures of nonoperative management occur within the first year after
treatment.
(C) Patients treated with surgery have fewer complications than patients treated
nonoperatively.
(D) Nonoperative management is associated with increased mortality.
(E) Failure of initial nonoperative management is associated with an increased risk
of developing complicated appendicitis.
25. Which of the following statements is true regarding the diagnosis and management of
hepatocellular carcinoma (HCC)?
(A) PET scan is the preferred surveillance imaging in high-risk populations.
(B) Alpha-fetoprotein greater than 3 times normal is diagnostic.
(C) A tissue diagnosis is required before initiating therapy.
(D) Screening reduces the overall mortality associated with HCC.
(E) Triple-phase CT is diagnostic in high-risk populations.
26. A patient with a cystic mass in the head of the pancreas undergoes cross-sectional
imaging and cystic fluid sampling. Which of the following clinical presentations of a
cystic mass in the pancreas has the highest risk of malignancy?
27. A 56-year-old man with Child-Pugh B cirrhosis with hepatitis C is found to have a 6-cm
mass in segments II and III with arterial enhancement and venous washout on triple-
phase CT. His alpha-fetoprotein level is 450 n!imL. An indocyanine green clearance
(!CG) is obtained to assess his suitability for resection. The minimal percentage clear-
ance at 15 minutes after injection ofICG needed to proceed with surgery is
(A) 50%.
(B) 60%.
(C) 70%.
(D) 80%.
(E) 90%.
28. A 58-year-old alcoholic man undergoes screening colonoscopy and is found to have a
large sessile lesion in his sigmoid colon positive for adenocarcinoma on biopsy. On CT,
he has a steatotic liver with synchronous metastases throughout his right liver lobe and
segments !Va and !Vb of his left lobe. The next step in his management should include
(A) chemotherapy alone.
(B) diverting colostomy.
(C) sigmoid colectomy.
(D) extended right hepatectomy.
(E) sigmoid colectomy with extended right hepatectomy.
29. A 63-year-old woman presents with acute onset of epigastric abdominal pain and is
found to have leukocytosis. Her abdomen is tender, but she does not have peritonitis.
Her CT shows a perforation of a duodenal diverticulum that is contained. Which of the
following statements is true?
(A) A Whipple procedure is required.
(B) Endoscopy is required.
(C) Elevated serum amylase and lipase will confirm the diagnosis.
(D) These diverticula commonly arise from the second portion of the duodenum.
(E) Life-long proton pump inhibitor therapy will be required.
30. A 55-year-old alcoholic man presents to clinic with signs and symptoms consistent
with chronic pancreatitis. CT reveals enlargement of the pancreatic head with multiple
stones and strictures in his proximal main pancreatic duct. In discussing treatment
options with him, which of the following would result in the best long-term pain
control?
(A) Endoscopic sphincterotomy and stone extraction alone
(BJ Endoscopic sphincterotomy, stone extraction, and stent placement
(C) Longitudinal pancreaticojejunostomy
(D) Longitudinal pancreaticojejunostomy with limited resection head of pancreas
(E) Pancreaticoduodenectomy with reconstruction
-----~-----··----·-~~·------·--------·---------·
.
250 CATEGORY 4- PART 1
Critiques 1-30
CRITIQUE 1-3
The choice of repair for a recurrent inguinal hernia will depend on the initial re-
pair used. Initial repairs can include primary conventional tissue repair [(Bassini)
Shouldice, McVay), primary anterior mesh repair (Lichtenstein, onlay patch, plug
and patch), or primary posterior mesh repair (Read, Rives, Stoppa, Kugel), or a
laparoscopic repair]. Recurrence rate after mesh repair differs with the type ofre-
pair and should be taken into consideration during subsequent repairs. The choice
of anterior versus posterior open approach should be guided by the initial repair.
A careful review of previous operative reports is paramount in guiding the sub-
sequent repair. If the initial repair was a tissue repair, either anterior or posterior
approaches can be used for repair of the recurrent hernia. If the initial repair was
a mesh repair, then the recurrent repair should preferably employ an approach in
the space in which the tissues planes have not been violated previously.
Reprinted from Journal of the American College of Surgeons. 209(5). Itani Ki\![, Fitzgibbons R Jr,
Awad SS. Duh QY. Ferzli GS. Management of Recurrent Inguinal Hernias. pages 654-655, Copyright
2009. with permission from Elsevier.
Selecting one approach over the other also depends on the experience and comfort of
the surgeon with that technique. Although a laparoscopic approach might be preferable in
previously repaired anterior tissue repair, the absence of a mesh with its associated scarring
and fibrosis makes a redo anterior approach with mesh more feasible and acceptable, espe-
cially iflaparoscopic expertise is not available. The same principle holds true in children;
however, mesh is rarely used in the repair of primary or recurrent inguinal hernia. In this
instance, a unilateral hernia repaired using a laparoscopic repair originally (posterior ap-
proach) is best repaired using an anterior approach without mesh.
Answers:
I. (A) Laparoscopic repair
2. (C) Both
3. (B) Open mesh repair
References
1. Itani KM, Fitzgibbons R Jr, Awad SS, Duh QY, Ferzli GS. Management of recurrent
inguinal hernias. J Am Coll Surg. 2009;209(5):653-658.
2. Wright JE. Recurrent inguinal hernia in infancy and childhood. Pediatric Surg Int.
1994; 9(3): 164-166.
3. Brandt ML. Pediatric hernias. Surg C/in North Am. 2008;88(1):27-43.
4. Simons MP, Aufenacker T, Bay-Nielsen M, et al. European Hernia Society guidelines
on the treatment of inguinal hernia in adult patients. Hernia. 2009; 13(4):343-403.
CRITIQUE 4-7
The Acute Cholecystectomy-Delayed Cholecystectomy study demonstrated that for pa-
tients who were not pregnant, not terminal, American Society of Anesthesiologists (ASA)
class I-Ill, not in septic shock, and without a perforated gallbladder, laparoscopic cholecys-
tectomy within 24 hours for acute calculous cholecystitis had significantly lower morbidity
and cost. Only 2 prospective trials have been conducted on percutaneous cholecystostomy
in high surgical risk acute calculous cholecystitis. Both showed no significant difference
CATEGORY 4 - PART I
251
in mortality and possibly reduced costs and morbidity. However, percutaneous cholecys-
tostomy can be performed safely on patients who are not othenvise operative candidates.
such as ASA class IV and V and patients with severe sepsis. Percutaneous cholecystostomy
may be the only treatment required for acalculous cholecystitis in the critically ill, with
success rates between 57o/o and 100%. However, successful laparoscopic cholecystectomy
is reported in surgical candidates.
Gallbladder polyps that are 6-9 mm on ultrasound without signs of malignancy or risk
factors for gallbladder cancer (e.g., age over 60, sessile morphology, gallstones, primary
sclerosing cholangitis) can be safely observed with interval ultrasound studies.
Answers:
4. (B) Percutaneous cholecystostomy
5. (C) Both
6. (D) Neither
7. (A) Laparoscopic cholecystectomy
References
I. Gurusamy KS, Rossi M, Davidson BR. Percutaneous cholecystostomy for high-risk
surgical patients with acute calculous cholecystitis. Cochrane Database Syst Rev.
2013;8:CD007088.
2. Chung YH, Choi ER, Kim KM, et al. Can percutaneous cholecystostomy be a definitive
management for acute acalculous cholecystitis? J Clin Gastroenterol. 2012;46(3):216-
219.
3. Gutt CN, Encke J, Koninger J, et al. Acute cholecystitis: early versus delayed chole-
cystectomy, a multicenter randomized trial (ACDC study, NCT00447304). Ann Surg.
2013;258(3):385-393.
4. Gallahan WC, Conway JD. Diagnosis and management of gallbladder polyps. Gastro-
enterol Clin North Am. 2010;39(2):359-367.
CRITIQUE 8-11
Biologic matrices provide a scaffold for tissue and vascular ingrowth. Today they are pop-
ular in abdominal wall reconstruction. Cross-linking is a chemical step that attempts to
render the collagen of the biologic matrix less prone to degradation in vivo by naturally
occurring collagenases. Cross-linking also delays cellular infiltration and neovasculariza-
tion in the short term.
A gradual remodeling of an implanted tissue graft is essential in abdominal wall repair,
because too rapid of an absorption of the matric scaffold will result in graft failure and
recurrence of the hernia. Because cross-linked matrixes are poorly incorporated by the
host, they become encapsulated with fibrous tissue and act as foreign body material instead
of serving as a scaffold for remodeling. Non-cross-linked matrices actually promote early
vascularization and cellular ingrowth.
Biologic matrix can be placed intraperitoneally, but these products perform best when
placed between 2 layers of vascularized tissue that promote fibroblast and vessel ingrowth
within the scaffold.
Biologic matrices are regulated under the US Food and Drug Administration as a medi-
cal device. The approval process is based primarily on safety but does not make any claims
with regard to efficacy. These matrices were introduced into clinical practice and were used
in contaminated fields without prior trials or US Food and Drug Administration endorse-
ment for that particular indication.
252 CATEGORY 4- PART I
Answers:
8. (B) Non-cross-linked biologic matrix
9. (C) Both
10. (C) Both
11. (A) Cross-linked biologic matrix
References
1. Kissane NA, Itani KM. A decade of ventral incisional hernia repairs with biologic acellu-
lar dermal matrix: what have we learned? [published correction appears in Plast Recon-
str Surg. 2013; 131 (2):427-428]. Plast Reconstr Surg. 2012; 130(5 suppl 2): 194S-202S.
2. NovitskyYW, Rosen MJ. The biology ofbiologics: basic science and clinical concepts.
Plast Reconstr Surg. 2012;130(5 suppl 2):9S-l 7S.
3. Rosen MJ, Denoto G, Itani KM, et al. Evaluation of surgical outcomes of retro-rectus
versus intraperitoneal reinforcement with bio-prosthetic mesh in the repair of contami-
nated ventral hernias. Hernia. 2013;17(1):31-35.
CRITIQUE 12-15
Incidentally discovered liver lesions are becoming increasingly common as medical imag-
ing continues to improve. The majority of these lesions are benign and require no further
treatment. Treatment decisions are based on the size of the lesion as well as symptoms.
Hepatic hemangiomas are the most common benign neoplasm of the liver. Heman-
giomas have no risk of malignant degeneration and a very low risk of bleeding. Lesions
smaller than 5 cm rarely cause symptoms. An asymptomatic hemangioma can be safely
observed. Hemangiomas that bleed may be embolized. Indications for surgery are symp-
toms (recurrent abdominal pain) and extremely large lesions(> 10 cm). For those requiring
surgery, resection is the treatment of choice. Enucleation is possible in many cases, because
the hemangioma tends to compress adjacent liver tissue, creating a plane of dissection.
Focal nodular hyperplasia (FNH) is the second most common benign hepatic lesion.
FNH has no potential for malignancy, spontaneous rupture, or hemorrhage. The only in-
dications for resection of FNH are symptoms (abdominal pain) and inability to rule out
malignancy. A definitive diagnosis, however, is usually possible with modern imaging
techniques. Asymptomatic FNH does not require intervention. Annual ultrasound for 2-3
years is prudent for women who wish to continue oral contraceptive (OCP) use.
Hepatic adenomas (HA) are benign liver masses that tend to be hormonally sensitive.
The classic presentation is in young women (aged 20s-40s) with a history of OCP use
and in men with a history of anabolic steroid use. Many HAs regress with discontinuation
of OCPs and steroids. Management strategies of HA are more aggressive than for other
benign liver masses, because they have a risk of hemorrhage and malignant degeneration
to hepatocellular carcinoma (HCC). The risk of hemorrhage or an occult focus of HCC
increases in HAs greater than 5 cm in size. Indications for surgery include (1) size at least
5 cm, (2) male patient, and (3) inability to rule out malignancy. Management of an acute
rupture with hemorrhage is initial angioembolization followed by elective resection. If pos-
sible, resection to negative margins should be done in case underlying HCC is identified
on final pathologic analysis.
Hypertonic saline injections are used to treat hydatid disease as a component of PAIR:
percutaneous aspiration, injection of scolicidal agent, and re-aspiration.
Answers:
12. (C) Observation
13. (D) Resection to negative margins
14. (A) Enucleation
15. (E) Transarterial embolization
CATEGORY 4- PART I
253
References
1. Marrero JA, Ahn J, Rajender Reddy K; American College of Gastroenterology. ACG
clinical guideline the diagnosis and management of focal liver lesions. Am J Gastroen-
terol. 2014;109(9):1328-1347.
2. Agrawal S, Agarwal S, Amason T, Saini S, Belghiti J. Management of hepatocellular
adenoma: recent advances. C/in Gastroenterol Hepatol. 2015; 13(7): 1221-1230.
3. Hirose K. The management of benign liver lesions. In: Cameron JL, Cameron AM, eds.
Current Surgical Therapy. 11th ed. Philadelphia, PA: Elsevier Saunders:2014:322-327.
CRITIQUE 16-18
Gut suture consists of strands of highly purified collagen processed from the submucosa
layer of sheep intestine or the serosa layer of beef intestine. Gut suture is absorbed based
on whether it is plain or chromic, the tissue it is used in, and the general health status
of the patient. Plain surgical gut suture is absorbed by a proteolytic enzymatic digestive
process and is completed by 70 days, with tensile strength maintained for 7-10 days. Plain
gut treated with a chromium salt solution is called chromic gut. This process prolongs
absorption time to more than 90 days, with tensile strength retained for 10-14 days.
Polyglactin 910 is a synthetic absorbable suture formed from a copolymer of lactide
and glycolide (from lactic and glycolic acid). The lactide slows water penetration into the
suture, which improves the duration of tensile strength for this suture. Polyglactin 910
sutures retain approximately 65% of original tensile strength 14 days after implantation.
Absorption is minimal for approximately 40 days and complete by 56-70 days.
Polyglecaprone 25 is virtually inert in time and absorbs predictably. At 7 days, 50-60%
of initial strength remains, reduced to 20-30% at 14 days, with all original strength lost at
21 days. Absorption is complete at 91-119 days.
Polydioxanone is a polyester that retains 70% of tensile strength at 14 days postimplan-
tation, 50% at 28 days, and 25% at 42 days. Absorption is minimal until approximately the
90th day postoperatively and essentially complete within 6 months.
Silk suture is braided raw silk. Surgical silk loses tensile strength when exposed to
moisture. Although silk is classified as a nonabsorbable suture, long-term studies show
that by 1 year, it has lost all of its tensile strength and usually cannot be detected in tissue
after 2 years.
Answers:
16. (B) Polyglactin 910
17. (C) Polyglecaprone 25
18. (A) Chromic gut
References
I. Ethicon, a Johnson & Johnson Company. Wound closure manual. lst ed. Somerville,
NJ: Ethicon, Inc.;1994.
2. Molea G, Schonauer F, Bifulco G, D' Angelo D. Comparative study on biocompatibility
and absorption times of three absorbable monofilament suture materials (Polydioxa-
none, Poliglecaprone 25, Glycomer 631 ). Br J Plast Surg. 2000;53(2): 137-141.
3. Tajirian AL, Goldberg DJ. A review of sutures and other skin closure materials. J Cos met
Laser Ther. 2010;12(6):296-302.
254 CATEGORY 4- PART!
CRITIQUE 19-22
Guidelines for postpolypectomy colonoscopy surveillance are established to minimize the
risks of screening colonoscopy while preventing interval development of cancers and cancer-
related mortality. Up to 17% of polyps at least IO mm in size are missed during colonoscopy.
In many situations, subsequent cancer development may be more likely related to these
missed lesions than to interval development of new lesions between colonoscopies.
Recommendations for repeat screening colonoscopy intervals are based on the size
and pathology of polyp(s) removed at the index colonoscopy. Hyperplastic polyps do
not portend any increased risk of developing colorectal malignancy and therefore require
no increased surveillance from the standard 10-year follow-up. Patients with I or 2
adenomatous polyps, less than I 0 mm in diameter, are considered low risk, and I 0-year
follow-up is recommended. Patients with 3 or more adenomatous polyps, I or more polyps
greater than 10 mm in size~ or adenomatous polyps with villous or high-grade histology
are considered high risk, and 3-year follow-up is recommended. Six-month and I-year
follow-up regimens are not routinely recommended for repeat screening colonoscopy or
colonography intervals when polyps are discovered.
Answers:
19. (A) Repeat colonoscopy in 10 years
20. (A) Repeat colonoscopy in 10 years
21. (B) Repeat colonoscopy in 3 years
22. (B) Repeat colonoscopy in 3 years
References
I. Hassan C, Quintero E, Dumonceau JM, et al; European Society of Gastrointestinal
Endoscopy. Post-polypectomy colonoscopy surveillance: European Society of Gastro-
intestinal Endoscopy (ESGE) guideline. Endoscopy. 2013;45(10):842-851.
2. Winawer SJ, Zauber AG, Fletcher RH, et al; US Multi-Society Task Force on Colorectal
Cancer; American Cancer Society. Guidelines for colonoscopy surveillance after polyp-
ectomy: a consensus update by the US Multi-Society Task Force on Colorectal Cancer
and the American Cancer Society. Gastroenterology. 2006; 130(6): 1872-1885.
3. Lieberman DA, Rex DK, Winawer SJ, Giardiello FM, Johnson DA, Levin TR; Unit-
ed States Multi-Society Task Force on Colorectal Cancer. Guidelines for colonoscopy
surveillance after screening and polypectomy: a consensus update by the US Multi-
Society Task Force on Colorectal Cancer. Gastroenterology. 2012; 143(3):844-857.
CRITIQUE23
Incisional hernia is the most frequent complication after laparotomy, resulting in approxi-
mately 200,000 repairs annually. Both laparoscopic and open approaches are successfully
used in the surgical treatment of incisional hernias. Multiple comparisons have been made,
examining each technique's advantages and disadvantages vis-a-vis the other. The lapa-
roscopic approach has a lower rate of wound complications, mesh infection~ and hospital
length of stay compared with the open approach. This advantage is offset by higher major
or severe complications, particularly intraoperative enterotomy, and hospital costs. Recur-
rence rates are similar for both approaches. The laparoscopic approach is especially recom-
mended in obese patients or elderly patients with small to medium defects because of its
minimal flap creation and soft tissue dissection.
References
I. Butler CE, Baumann DP, Janis JE, Rosen MJ. Abdominal wall reconstruction. Curr
Prob/ Surg. 2013;50(12):557-586.
2. Sauerland S, Walgenbach M, Habermalz B, Seiler CM, Miserez M. Laparoscopic versus
open surgical techniques for ventral or incisional hernia repair. Cochrane Database Syst
Rev. 2011 ;(3):CD00778 l.
3. Eker HH, Hansson BM, Buunen M, et al. Laparoscopic vs. open incisional hernia repair:
a randomized clinical trial. JAMA Surg. 2013;148(3):259-263.
4. Itani KM, Hur K, Kim LT, et al; Veterans Affairs Ventral Incisional Hernia Investiga-
tors. Comparison oflaparoscopic and open repair with mesh for the treatment of ventral
incisional hernia: a randomized trial. Arch Surg. 2010;145(4):322-328; discussion 328.
CRITIQUE 24
Appendicitis is the most frequent intra-abdominal emergency, and surgery has long been
the mainstay of therapy. Recently, multiple randomized clinical trials were conducted com-
paring nonoperative management with antibiotics to surgery in patients with uncomplicated
appendicitis. Nonoperative management, had a success rate of 44-85% in individual stud-
ies. Most recurrences in the nonoperative group occur within the first year after treatment.
Surgery is associated with a higher rate of complications compared with nonoperative
management. Mortality was similar between operative and nonoperative management. A
meta-analysis of 4 randomized trials consisting of 900 patients found a relative risk reduc-
tion in complications of 31 % in the antibiotic treatment group compared with the appen-
dectomy group. Nonoperative management is associated with shorter length of stay and de-
creased sick leave time compared with surgical management. Antibiotic therapy may delay
appendectomy in patients who are not improving, but this delay is not associated with an
increased risk of developing complicated appendicitis, including perforation or peritonitis.
Answer: (B) Most failures of nonoperative management occur within the first year after
diagnosis.
References
1. Varadhan KK. Neal KR, Lobo DN. Safety and efficacy of antibiotics compared with
appendicectomy for treatment of uncomplicated acute appendicitis: meta-analysis of
randomised controlled trials. BM!. 2012;344:e2156.
2. Hansson J, Korner U, Khorram-Manesh A, Solberg A, Lundholm K. Randomized
clinical trial of antibiotic therapy versus appendicectomy as primary treatment of
acute appendicitis in unselected patients [published correction appears in Br J Surg.
2009;96(7):830]. Br J Surg. 2009:96(5):473-481.
3. Di Saverio S, Sibilio A, Giorgini E, et al. The NOTA Study (Non Operative Treat-
ment for Acute Appendicitis): prospective study on the efficacy and safety of antibiotics
(amoxicillin and clavulanic acid) for treating patients with right lower quadrant abdomi-
nal pain and long-term follow-up of conservatively treated suspected appendicitis. Ann
Surg. 2014;260(1):109-117.
4. Salminen P, Paajanen H, Rautio T, et al. Antibiotic therapy vs appendectomy for treat-
ment of uncomplicated acute appendicitis: The APPAC Randomized Clinical Trial.
JAMA. 2015;313(23):2340-2348.
5. Livingston E, Vons C. Treating appendicitis without surgery. JAMA. 2015;313(23):
2327-2328.
256 CATEGORY 4- PART I
CRITIQUE25
Hepatocellular carcinoma (HCC) is the fifth most common cancer in men and ninth in
women in the world. It arises most commonly in the setting of chronic liver disease-
cirrhosis caused by hepatitis C virus, alcohol, and nonalcoholic steatohepatitis (NASH)-
although it is also seen before the onset of cirrhosis in the setting of hepatitis B and
sporadically in individuals older than 60 and in those with certain types of hepatic adenoma
(those with nuclear localization ofbeta-catenin).
In patients with cirrhosis, the diagnosis of HCC can be made with triple-phase CT
scan. Liver Imaging Reporting and Data System (LiRADS) criteria include the presence
of arterial-phase enhancement (hypervascularity), portal venous phase washout, and
threshold growth if a historical comparison is available. The scale runs from 1 (definitely
benign) to 5 (diagnostic of HCC). LiRADS 4 and 5 lesions can be treated without a biopsy.
A tissue diagnosis may be necessary in low-risk patients and when there is nondiagnostic
imagining. Serum alpha fetoprotein has a high positive predictive value when greater than
500 ng/mL but a low negative predictive value. Approximately 20% of patients with HCC
have a normal alpha fetoprotein. Alpha fetoprotein levels are not used as a diagnostic
criterion for HCC.
An effective screening approach that significantly reduces HCC-related mortality has
not been identified. Serial serum alpha fetoprotein screening in more than 5000 chronic
hepatitis B carriers in China identified more early-stage cancers than the unscreened popu-
lation but did not significantly reduce mortality. The authors concluded this was likely a
function of a lead-time bias--diagnosing the disease earlier but not changing the disease-
specific mortality.
PET scans are neither sensitive nor specific forthe diagnosis of HCC.
References
1. Forner A, Llovet JM, Bruix J. Hepatocellular carcinoma. Lancet. 20l2;379(9822):1245-
1255.
2. Aghoram R, Cai P, Dickinson JA. Alpha-fetoprotein and/or liver ultrasonography for
screening of hepatocellular carcinoma in patients with chronic hepatitis B. Cochrane
Database Syst Rev. 2012;9:CD002799.
3. Chen JG, Parkin DM, Chen QG, et al. Screening for liver cancer: results of a randomised
controlled trial in Qidong, China. J Med Screen. 2003;10(4):204--209.
4. Huang J, Zeng Y. Current clinical uses of the biomarkers for hepatocellular carcinoma.
Drug Discov Ther. 2014;8(2):98-99.
5. Song P, Tang W, Kokudo N. Serum biomarkers for early diagnosis of hepatocellular
carcinoma. Transl Gastrointest Cancer. 2014;3(2):103-105.
CRITIQUE26
Pancreatic cysts are more frequently recognized on cross-sectional abdominal imaging
than in the past. A small percentage of pancreatic cysts can develop into pancreatic cancer,
and these patients may benefit from surgical resection. The International Association of
Pancreatology updated its consensus guidelines for the surgical management of pancreatic
cysts in 2012. The updated guidelines attempted to be more accurate in predicting malig-
nant cysts, but they still have a relatively low positive predictive value. The use of cyst
fluid biopsy samples and other biomarkers to risk stratify patients has not been successful
in identifying patients whose cysts require surgical resection.
Determinants of the malignant potential of a pancreatic cyst include mucin in the cyst
contents, cyst size, solid tumor in the cyst, and main pancreatic duct dilation. Loculation
within the cyst does not predict malignancy. Cysts containing serous fluid are generally be-
CATEGORY 4- PART I
257
nign and include serous cystadenomas and benign epithelial cysts. Accordingly, the patient
with the largest mucin-containing cyst and greatest dilation of the main duct represents the
patient with the highest risk of associated malignancy.
Mucin-containing cysts include intraductal papillary neoplasms (IPMN) and mucinous
cystic neoplasms(MCN). MCN occur almost exclusively in women55-65 years old, whereas
IPMN occur in older men and women, with men predominating. Current recommendation
from guidelines recommend resection of all MCN, main duct IPMN, branch duct-IPMN
with a solid component, main pancreatic duct size at least 1 cm, obstructive jaundice, or
cytology suspicious or positive for cancer. Although guidelines recommend resection of
the unilocular, mucin-containing cyst in the 55-year-old woman as it is a presumed MCN,
the risk of malignant cells within the lesion is less than the 65-year-old man with the large
IPMN with main duct involvement.
Answer:
(C) 65-year-old man 5 Mucin 1.5
References
I. Lee LS. Diagnostic approach to pancreatic cysts. Curr Opin Gastroenterol.
2014;30(5):511-517.
2. Tanaka M, Fernandez-de! Castillo C, Adsay V, et al; International Association of
Pancreatology. International consensus guidelines 2012 for the management of IPMN
and MCN of the pancreas. Pancreatology. 2012; 12(3): 183-197.
3. Katz MH, Mortenson MM, Wang H, et al. Diagnosis and management of cystic neoplasms
of the pancreas: an evidence-based approach. J Am Coll Surg. 2008;207(1):106-120.
CRITIQUE27
Hepatocellular carcinoma (HCC) is the most common primary malignancy of the liver.
More than 80% of the time, it develops in the setting of cirrhosis resulting from chronic liv-
er disease due to etiologies such as chronic viral infection (i.e., hepatitis B or C), alcohol-
ism, and nonalcoholic steatohepatitis (NASH). Other risk factors include exposure to sub-
stances such as aflatoxins, azo dyes, aromatic amines, N-nitroso compounds, chlorinated
hydrocarbons, hydrosol compounds, pesticides, and radiation. In addition, anabolic steroid
and oral contraceptive use are risk factors. In patients at risk for HCC, an alpha-fetoprotein
level greater than 400 ng/mL is highly suspicious for the presence of HCC. Diagnosis is
made based on radiographic imaging criteria and clinical findings. On triphasic hepatic CT,
HCC exhibits a classic arterial enhancement with venous washout.
Although only 30% of patients with HCC may qualify for operative treatment, sur-
gery remains the best option available. Because the leading cause of death after resection
for HCC is liver failure, proper preoperative assessment of hepatic reserve is essential to
identify patients at risk. Both the Child-Pugh and Model for End-Stage Liver Disease clas-
sifications can aid in determining those patients at greatest risk. For example, Child-Pugh
C patients have a greater than 25o/o perioperative mortality. and, consequently, resection is
contraindicated.
Triphasic liver CT can be used to calculate the future liver remnant. A future liver rem-
nant less than 40% is predictive of postoperative liver failure. Finally, indocyanine green
(!CG) clearance can be used to assess the functional capacity of the liver. The dye is in-
jected into the bloodstream, and clearance from the liver is measured 15 minutes after
injection. Less than 10% of the dye should be detectable at this point in normal functioning
livers. With 15-20% retention, a 2-segment resection is possible. With 21-30% retention,
a single segment or wedge resection is indicated. Greater than 40% retention of dye at 15
minutes is predictive of postoperative liver failure regardless of resection size.
258 CATEGORY 4- PART I
In this patient requiring a 2-segment resection for treatment of his 6-cm HCC, the mini-
mum !CG clearance would be 80%, corresponding to a 20% retention rate, which would
allow performance of a 2-segment resection without the development of liver failure.
References
I. Locke JE, Cameron AM. Treatment ofhepatocellular carcinoma: resection versus trans-
plantation. In: Cameron JL, Cameron AM, eds. Current Surgical Therapy. 11th ed. Phil-
adelphia, PA: Elsevier Saunders; 2014:332-335.
2. Belghiti J, Fuks D. Liver resection and transplantation in hepatocellular carcinoma.
Liver Cancer. 2012;1(2):71-82.
CRITIQUE28
Colorectal cancer is the second leading cause of cancer deaths worldwide, and hepatic
metastases accounts for two-thirds of these deaths. Approximately 25% of patients with
colorectal cancer present with synchronous liver lesions, and 30% will develop metachro-
nous liver metastases during the course of their disease. Resection remains the standard
therapy for cure in patients with hepatic involvement, with morbidity and mortality less
than 30% and 3%, respectively.
Patients who present with synchronous colorectal and hepatic lesions present a chal-
lenge to surgeons. Options for care include colon-first, liver-first, and concomitant resec-
tions of disease in patients with resectable tumors and metastases. In a recent systematic
review of 18 papers on the topic, none of the 3 surgical strategies demonstrated inferiority
compared with the others. Hence, the approach taken should be based on patient factors.
An expert consensus came to the same conclusion, stating that resection should be based
on concern for complications at the primary tumor site, progression of liver disease, and
complexity of simultaneous resection. In patients with unresectable liver metastases, re-
section of the primary tumor is no longer advocated in the absence of complications (i.e.,
obstruction, bleeding, perforation). This evolution has occurred because of the introduc-
tion of more effective chemotherapeutic drugs, such as oxaliplatin and irinotecan, to treat
colorectal cancer. Consequently, primary chemotherapy leads to higher response and con-
trol rates, resulting in complication rates dropping to less than 1Oo/o.
In this patient with sigmoid cancer, hepatic metastases involving 80o/o of his liver, and
liver disease, chemotherapy alone is the most appropriate next step in his management. He
is asymptomatic, making a diverting colostomy or sigmoid resection unnecessary. Given
his liver disease and the extent of metastatic involvement. he would not tolerate an ex-
tended right hepatectomy. Finally, simultaneous resection is not indicated, because his liver
disease is unresectable.
References
1. Clark ME, Smith RR. Liver-directed therapies in metastatic colorectal cancer. J Gastro-
intest Oncol. 2014;5(5):374--387.
2. de Mestier L, Manceau G, Neuzillet C, et al. Primary tumor resection in colorectal
cancer with unresectable synchronous metastases: a review. World J Gastrointest Oneal.
2014;6(6): 156--169.
3. Lykoudis PM, O'Reilly D, Nastos K, Fusai G. Systematic review of surgical manage-
ment of synchronous colorectal liver metastases. Br J Surg. 2014; I 01 (6):605-612.
CATEGORY 4- PART I
259
CRITIQUE29
The duodenum is the second most common site of diverticula formation in the intestinal
tract. Duodenal diverticula are classified as either congenital or acquired. Congenital duo-
denal diverticula contain all layers of the duodenal wall. Acquired duodenal diverticula are
pulsion diverticula containing only mucosa, submucosa, and muscularis mucosa protrud-
ing through a weakness in the duodenal wall. The ampulla of Yater is an example of such an
area of weakness in the duodenal wall, accounting for the common location of diverticula
within the second portion.
Complications of duodenal diverticula, although rare, can include perforation,
inflammation, compression ofsurrounding structures, or neoplastic transformation. Surgical
treatment options for perforation, especially in the presence of generalized peritonitis or
contamination, include surgical drainage, diverticulectomy, or pancreaticoduodenectomy.
In carefully selected cases of contained perforation, management can be nonoperative with
nasogastric tube decompression, antibiotics, and bowel rest with or without endoscopically
placed drains or stents.
The patient described has acute abdominal pain but without diffuse peritonitis;
her images show a contained perforation of the duodenum. In this setting, nasogastric
decompression, intravenous antibiotics, and bowel rest are appropriate.
This is unrelated to peptic ulcer disease; therefore, proton pump inhibitor therapy is not
indicated. Although amylase and lipase may be elevated, they are not diagnostic of duode-
nal diverticula. Endoscopy and Whipple are not required in this setting.
Answer: (D) These diverticula commonly arise from the second portion of the duodenum.
References
I. Rossetti A, Christian BN, Pascal B, Stephane D, Philippe M. Perforated duodenal diver-
ticulum, a rare complication ofa common pathology: a seven-patient case series. World
J Gastrointest Surg. 20!3;5(3):47-50.
2. Costa Simoes V, Santos B, Magalhaes S, Faria G, Sousa Silva D, Davide J. Perforated
duodenal diverticulum: surgical treatment and literature review. Int J Surg Case Rep.
20 l 4;5(8):547-550.
CRITIQUE30
Chronic pancreatitis is characterized by recurring inflammation in the pancreas, which
results in progressive fibrosis and manifests clinically as pain, exocrine or endocrine dys-
function, and weight loss. Approximately 80% of chronic pancreatitis is caused by exces-
sive alcohol consumption; other etiologies include autoimmune disorders, choledocholi-
thiasis, and congenital malformations.
Phannacologic treatment of chronic pancreatitis consists of pain control and replace-
ment of exocrine and endocrine function as required. Palliative options for long-term
chronic pancreatitis-induced pain include procedures involving pancreatic head resections,
such as the Beger (figure 30.1) or Frey procedure (figure 30.2) or pancreaticoduodenec-
tomy (Whipple procedure), surgical drainage procedures, or endoscopic ductal decompres-
sion. Options for surgical drainage can include procedures such as the Puestow procedure
(figure 30.3) or modifications of this classic operation such as the Izbicki operation or
Hamburg or Berne procedure.
The Beger procedure includes duodenum-sparing resection of most of the pancreatic
head with division of the pancreatic body over the portal vein and reconstruction via a
side-to-side and side-to-end pancreaticojejunostomy to drain the remaining head and tail of
the pancreas. The Frey procedure combines a duodenal-sparing pancreatic head resection
with overlay pancreaticojejunostomy and longitudinal duct drainage without division of
the pancreas. The Puestow procedure consists of opening the duct in a longitudinal fashion
260 CATEGORY 4 - PART I
and draining it via a lateral pancreaticojejunostomy while preserving the head of the gland.
Endoscopic treatment for pain related to chronic pancreatitis can be accomplished with
sphincterotomy and stone extraction with or without ductal dilation or stent placement for
main duct decompression.
References
I. Bachmann K, Tomkoetter L, Erbes J, et al. Beger and Frey procedures for treatment
of chronic pancreatitis: comparison of outcomes at 16-year follow-up. J Am Coll Surg.
2014:219(2):208-216.
2. Cahen DL, Gouma DJ, Nio Y. et al. Endoscopic versus surgical drainage of the pancre-
atic duct in chronic pancreatitis. N Engl J Med. 2007;356(7):676--684.
3. Mihaljevic AL, Kleeff J, Friess H, Buchler MW, Beger HG. Surgical approaches to
chronic pancreatitis. Best Pract Res C/in Gastroenterol. 2008;22(1):167-181.
4. K KOninger J, Friess H, Miiller M, Biichler MW. Duodenum preserving pancreatic head
resection in the treatment of chronic pancreatitis. Rocz Akad Med Bialymst. 2004;49:53-
60.
5. Izbicki JR, Bloechle C, Broering DC, Knoefel WT, Kuechler T, Broelsch CE. Extended
drainage versus resection in surgery for chronic pancreatitis: a prospective randomized
trial comparing the longitudinal pancreaticojejunostomy combined with local pancre-
atic head excision with the pylorus-preserving pancreatoduodenectomy. Ann Surg.
I 998;228(6):771-779.
6. Bachmann K, Tomkoetter L, Kutup A, et al. Is the Whipple procedure harmful for
long-term outcome in treatment of chronic pancreatitis? 15-years follow-up comparing
the outcome after pylorus-preserving pancreatoduodenectomy and Frey procedure in
chronic pancreatitis. Ann Surg. 2013:258(5):815-820; discussion 820-821.
Category 4 - Part II
Abdomen
Items 1-28
DIRECTIONS: Each of the questions or incomplete statements is followed by 5 suggested
answers or completions. Select the answer that is the BEST in each case and fill in the
space containing the corresponding letter on the answer sheet.
4. A 75-year-old woman with familial adenomatous polyposis who has had a total
colectomy complains of the new onset of intermittent abdominal pain, which is worse
with eating. An MRI shows 2 masses in the mesentery. Which of the following is the
most likely diagnosis?
(A) Desmoid
(B) Liposarcoma
(C) Carcinoid
(D) Lymphoma
(E) Leiomyoma
CATEGORY 4- PART II 265
Figure 3.1
9. Compared with laparoscopic incisional hernia repair with synthetic mesh, open inci-
sional hernia repair with synthetic mesh is associated with
(A) similar seroma rates.
(B) increased readmission rates.
(C) increased reoperative rates.
(D) decreased total costs.
(E) decreased mortality.
10. Which of the following perioperative antibiotic regimens is associated with the lowest
risk of surgical site infection after colorectal surgery?
(A) Oral antibiotics alone
(B) Oral antibiotics and intravenous fiuoroquinolone and metronidazole
(C) Oral antibiotics and intravenous cefazolin and metronidazole
(D) Oral antibiotics and intravenous cefazolin
(E) Intravenous cefotetan alone
11. The most reliable method to detect small liver metastasis (<l cm) from colorectal
carcinoma is
(A) transabdominal ultrasound.
(B) multidetector CT scan.
(C) fiudeoxyglucose-PET scan.
(D) PET-CT scan.
(E) contrast-enhanced MRI.
12. The lowest rate of pancreatic leak after distal pancreatectomy is achieved with
(A) preoperative pancreatic stent placement.
(B) suture ligation of the duct.
(C) fibrin glue sealant applied to the transected duct.
(D) stapler transection of the pancreas.
(E) postoperative somatostatin analogue.
13. The patient shown in figure 13.1 becomes hypotensive and hypoxic during fascia! clo-
sure after a splenectomy. Which of the following is the optimal surgical management
at this time?
(A) Vacuum closure of the abdominal wall
(B) Progressive fascia! closure with closure device
(C) Bridge closure with biologic mesh
(D) Polytetrafiuoroethylene (PTFE) patch closure of the abdominal wall
(E) Component separation and primary abdominal wall closure
CATEGORY 4 - PART II
267
Figure 13.1
15. A 38-year-old woman presents with right upper quadrant pain, postprandial gas, and
bloating associated with nausea. Gallbladder ultrasound shows a distended gallbladder
with no stones. Which of the following statements is most accurate regarding chole-
cystokinin cholescintigraphy?
(A) It is useful when patients describe recurrent epigastric or right upper quadrant
pain episodes lasting 30 minutes or longer.
(B) It is useful in detennining the etiology of atypical abdominal pain.
(C) Symptom reproduction is the most useful predictor of right upper quadrant
symptom relief by cholecystectomy.
(D) H2 receptor antagonists increase gallbladder contractility and may produce a
false-negative test.
(E) An abnormal gallbladder ejection fraction is highly specific for
gallbladder disease.
16. A 22-year-old woman develops acute cholecystitis during the 32nd week of preg-
nancy. Which of the following statements regarding her management is true?
(A) Premature labor is less common than in the second trimester.
(B) Intraoperative cholangiography is associated with an adverse fetal outcome.
(C) Open cholecystectomy is safer than laparoscopic cholecystectomy.
(D) Postoperative endoscopic retrograde cholangiopancreatography is safe.
(E) Gallstone pancreatitis is associated with a fetal loss of 20%.
268 CATEGORY 4- PART II
17. A 48-year-old woman with known cirrhosis from prior alcohol abuse develops in-
creasing confusion. Her blood ammonia levels are elevated. A diagnosis of hepatic
encephalopathy is made secondary to hepatic failure. The best definitive treatment is
(A) portacaval shunt.
(B) protein restriction.
(C) splenorenal shunt.
(D) transjugular intrahepatic portosystemic shunt.
(E) liver transplantation.
18. A 34-year-old man is diagnosed with a transverse colon cancer in the setting of mul-
tiple colon polyps. There is no prior family history of colon cancer. He has a total proc-
tocolectomy with an ilea] pouch anal anastomosis. His tumor is a T3NOMO, and more
than I 00 other polyps were noted throughout his colon and rectum. Two years later, he
has a surveillance CT scan, and a 3-cm mesenteric mass is noted. A core needle biopsy
shows very bland-appearing spindle cells. Which of the following is the next step in
management of this patient?
(A) Sulindac
(B) Imatinib
(C) FOLFOX (5-fluorouracil, leucovorin, and oxaliplatin)
(D) Radiation
(E) Surgical excision
19. A 47-year-old woman presents with acute onset of abdominal pain. CT shows evi-
dence of acute portal vein thrombosis (PVT). Which of the following is true regarding
acute PVT?
(A) In most patients, only the portal vein is involved.
(B) A myeloproliferative disorder is rarely associated with PVT.
(C) Clinically significant ascites is uncommon.
(D) Recanalization occurs frequently without anticoagulation.
(E) Common initial presentation is gastrointestinal bleeding from varices.
20. A 57-year-old woman presents for acute symptoms of right upper quadrant pain. The
CT scan obtained is shown in figures 20.1 and 20.2. Which is the following is the most
appropriate treatment?
(A) Estrogen therapy
(B) Somatostatin
(C) Sclerotherapy
(D) Fenestration
(E) Hepatic resection
CATEGORY 4- PART II
269
Figure 20.1
Figure 20.2
270 CATEGORY 4- PART II
21. A 35-year-old man develops acute necrotizing pancreatitis related to a drug reaction.
The CT scan shown in figures 21.1 and 21.2 was obtained 1 month after the acute pro-
cess. Although he is afebrile, he is having trouble eating, has decreased stamina, and
has moderate discomfort to palpation of the right lower quadrant. His white blood cell
count is mildly elevated. Which of the following is the most appropriate management
in this patient?
(A) Expectant management for continued resolution
(B) Cyst gastrostomy
(C) Percutaneous aspiration
(D) Transabdominal debridement
(E) Retroperitoneal debridement
Figure 21.l
Figure 21.2
CATEGORY 4- PART II
271
22. Which of the following is a component of both the Child-Pugh and Model for End-
Stage Liver Disease scoring systems?
(A) Albumin
(B) Aspartate transaminase
(C) Creatinine
(D) Encephalopathy
(E) International normalized ratio
23. A 74-year-old woman presents with 4 days of nausea, feculent vomiting, and medial
thigh pain. Her last bowel movement was 2 days ago. She has no history of abdominal
surgery. Her abdomen is distended and mildly tender. Her white blood cell count is
14,000/mml (3600-ll,200/mm3). A CT scan is shown in figure 23.1. Which of the
following is the most appropriate next step?
(A) Nasogastric tube, intravenous fluids, and observation
(B) Preperitoneal groin exploration
(C) Femoral groin exploration
(D) Inguinal groin exploration
(E) Midline laparotomy
Figure 23.1
- - - - - - - ··-------
Figure 24.1
25. An 82-year-old man presents with a 7-day history of right upper quadrant pain with
nausea and vomiting. His medical history is remarkable for hepatitis C and cirrhosis.
His white blood cell count is 18,500/mml (3600-11,200/mml) with 13% bands.
Total bilirubin is 5.5 mg/dL (0.2-1.9 mg/dL), and alkaline phosphatase is 325 units/L
(<95 IU/L). CT scan of the abdomen and pelvis demonstrates severe cholecystitis
with pericholecystic fluid, cholelithiasis, and a normal common bile duct. The most
appropriate next step should be
(A) magnetic resonance cholangiopancreatography.
(B) hepatobiliary iminodiacetic acid scan.
(C) endoscopic retrograde cholangiopancreatography.
(D) percutaneous cholecystostomy.
(E) open cholecystectomy.
CATEGORY 4- PART II 273
26. Which of the following statements regarding colorectal liver metastases is true?
(A) Colorectal metastases represent the third most common malignant tumor of
the liver.
(B) Resectability is based on the volume of liver remaining after resection.
(C) Four or more hepatic metastases are a contraindication to liver resection.
(D) Radiofrequency ablation is appropriate for metastatic tumors larger than 5 cm.
(E) Neoadjuvant chemotherapy yields best results when used for single colorectal
metastases.
27. An 80-year-old woman is admitted to the medical intensive care unit with pneumonia
and sepsis. She is intubated and on norepinephrine and vasopressin for 5 days without
improvement. Current therapy includes piperacillin/tazobactam and vancomycin. Her
liver function tests are elevated, with a total bilirubin of 1.8 mg/dL (0.2-1.9 mg/dL),
aspartate aminotransferase of60 U/L (8-40 IU/L), alanine aminotransferase of50 U/L
(7-55 IU/L), and alkaline phosphatase of 100 U/L (<95 IU/L). An ultrasound of her
right upper quadrant demonstrates an enlarged gallbladder with pericholecystic fluid
and wall thickening, but no gallstones. The common bile duct measures 4 mm. Which
of the following is the most appropriate next step in the management of this patient?
(A) Open cholecystectomy
(B) Percutaneous cholecystostomy tube placement
(C) Endoscopic retrograde cholangiopancreatography
(D) Observation
(E) Magnetic resonance cholangiopancreatography
Critiques 1-28
CRITIQUE 1
In the United States, more than 2 million laparotomies are performed annually to treat
benign conditions. Up to 16o/o of midline incisions will eventually develop herniation,
adding to patient morbidity and increased healthcare costs. Often, the risk factors associ-
ated with wound complications and eventual ventral hernia formation (i.e., obesity, degree
of contamination) are beyond a surgeon's control. Nonetheless, the surgeon can control
the quality of the suturing technique for a fascia! closure, including the method of clo-
sure and the suture used. The optimal closure technique is still being sought to minimize
wound complications, and, hence, incisional hernia formation. The use of nonabsorbable
or slowly absorbable sutures has a lower hernia rate than quickly absorbable sutures. A
continuous suture technique leads to a more rapid and stronger wound closure than inter-
rupted sutures. Monofilament sutures result in fewer surgical site infections compared with
braided sutures. A suture length-to-wound length ratio higher than 4-to-I decreases the rate
of incisional hernia by 4-fold. Finally, the use of 2-0 monofilament sutures mounted on a
small needle (e.g., 20-mm tapered half-circle) with the placement of stitches within the
aponeurosis 5-8 mm from the wound edge and 4-5 mm apart helps to minimize the rate of
incisional hernia. Thus, for this patient, the use of a continuous, monofilament sutures with
a 4: I suture-to-wound length ratio is optimal.
References
I. Millboum D, Cengiz Y, Israelsson LA. Effect of stitch length on wound complica-
tions after closure of midline incisions: a randomized controlled trial. Arch Surg.
2009;144(1I):1056-1059.
2. Muysoms FE, Antoniou SA, Bury K, et al; European Hernia Society. European Hernia
Society guidelines on the closure of abdominal wall incisions. Hernia. 2015; 19(1):1-24.
3. Harlaar JJ, Deerenberg EB, van Ramshorst GH, et al. A multicenter randomized
controlled trial evaluating the effect of small stitches on the incidence of incisional
hernia in midline incisions. BMC Surg. 2011;1 l :20.
4. Deerenberg EB, Harlaar JJ, Steyerberg EW, et al. Small bites versus large bites for
closure of abdominal midline incisions (STITCH): a double-blind, multicentre,
randomised controlled trial. Lancet. 2015;386(10000): 1254-1260.
CRITIQUE2
Necrotizing pancreatitis is seen in approximately I in 5 patients with pancreatitis and is
associated with a mortality rate of 10-30%. Patients are at high risk for multiorgan failure
and secondary infection of the necrotic pancreatic bed. The risk of secondary infection is
25-70%, adding to the already high rate of morbidity and mortality. After initial admission
and resuscitation, patients with severe pancreatitis should undergo imaging to determine
the extent of necrosis. Contrast-enhanced axial imaging is the modality of choice. The true
extent of necrosis may not be apparent for several days. CT offers the additional benefit of
assessing for further local complications associated with severe pancreatitis.
Enteral nutrition should be started within 72 hours. The goal is to achieve an early posi-
tive nitrogen balance. If gastric feeds are not tolerated, continuous, jejuna! feeds should be
used. In the setting of a paralytic ileus, parenteral nutrition can be used until resolution of
the ileus.
CATEGORY 4- PART II 275
Sterile necrosis must be distinguished from infected necrosis, because these 2 processes
are managed differently. Antibiotics have no apparent role in the setting of sterile necrosis.
Although some early studies supported the use of antimicrobial therapy in the setting of
severe necrosis, 2 large. randomized, double-blinded, multicenter studies and a Cochrane
meta-analysis determined prophylactic antibiotics were not protective.
Infected necrosis warrants debridement. There is no indication for surgical debridement
of sterile necrosis. Recently, several reports described the benefits of using a '"step-up'' ap-
proach to debridement. This approach advocates beginning with radiologic drainage, fol-
lowed by minimally invasive retroperitoneal necrosectomy if drainage fails. This approach
seeks to avoid the high morbidity and mortality associated with open necrosectomy.
References
I. van Santvoort HC, Hesselink MG, Bakker OJ, et al; Dutch Pancreatitis Study Group.
A step-up approach or open necrosectomy for necrotizing pancreatitis. N Engl J Med.
2010;362(16): 1491-1502.
2. Zerem E. Treatment of severe acute pancreatitis and its complications. World J Gastro-
enterol. 2014;20(38): 13879-13892.
CRITIQUE3
This patient presents with an impressive mass in the retroperitoneal space on the left.
Twenty-four-hour urinary catecholamine collection is elevated. Therefore, unless proven
otherwise, this patient has a pheochromocytoma. The enlarged mass and the appearance
of other nodules in the periaortic area strongly suggest a malignant pheochromocytoma
(figure 3.2). This tumor does not arise from the kidney, and attention should be turned to the
appropriate preoperative workup and management of this patient in preparation for surgery.
Figure 3.2 The enlarged mass and the appearance of other nodules in
the periaortic area strongly suggest a malignant pheochromocytoma.
276 CATEGORY 4- PART II
Answer: (C) Obtaining plasma-free metanephrine off medication is the next appropriate test.
References
l. Lenders JW, Duh QY, Eisenhofer G, et al; Endocrine Society. Pheochromocytoma and
paraganglioma: an endocrine society clinical practice guideline. J Clin Endocrinol
Metab. 2014;99( 6): 1915-1942.
2. Prejbisz A, Lenders JW, Eisenhofer G, Januszewicz A. Cardiovascular manifestations
ofphaeochromocytoma. J Hypertens. 2011;29(11):2049-2060.
3. Strong VE, Kennedy T, Al-Ahmadie H, et al. Prognostic indicators of malignancy in
adrenal pheochromocytomas: clinical, histopathologic, and cell cycle/apoptosis gene
expression analysis. Surgery. 2008;143(6):759-768.
4. Young WF Jr. Clinical practice. The incidentally discovered adrenal mass. N Engl J
Med. 2007;356(6):601-610.
CRITIQUE4
The relative risk of developing desmoid tumors is much higher in patients with familial
adenomatous polyposis (FAP) compared with the general population. Desmoids occur in
7.5-16% ofFAP patients. Gardner syndrome, considered a variant ofFAP, includes extra-
colonic manifestations such as osteomas and desmoid tumors. Given her history of FAP
and the location of the masses, a diagnosis of desmoid is most likely.
Diagnosis of desmoids and other aggressive fibromatosis (AF) is made based on clini-
cal, radiological, and histological parameters. AF/desmoids characteristically infiltrate
deep tissue and muscles as opposed to pushing the adjacent tissue, as is seen in most sar-
comas. MRI is an excellent tool for the identification and characterization of mesenteric
masses (figure 4.1, table 4.l). On MRI, the lesions are infiltrative with an irregular or lobu-
lated contour. Homogeneous isointensity or mild hyperintensity on Tl-weighted images
and heterogeneous high signal on T2-weighted images is seen.
CATEGORY 4 ~PART II
277
Mesenteric mass
~ ! /
, . .- j
Unilocu!ar
Associated
with bowel
Multi!oculated
~ymphangioma ,/
Single lesion
Smooth margins
GIST
Multiple lesions
~Metastatic disease
Lymphoma
Mesothelioma
Duplication Neurogenic
cyst Solitary fibrous Fat containing
Findings of Surgical site tumor Liposarcoma
pancreatitis Lymphocele
Pseudocyst Infiltrating
border
Random
location
Mesothelial Ileal location
/ Contracting,
cyst Arterial distorting margins
enhancement Delayed
Carcinoid hyperenhancement
Desmoid
Sc/erasing mesenteritis
For symptomatic patients, treatment should be based on the location of the tumor
and potential morbidity of treatment. Treatment options include resection, radiation, or
systemic therapy. Radiation can be preoperative, intraoperative, or postoperative. Systemic
therapies for desmoids include anti-inflammatory drugs, hormonal (tamoxifen) agents,
biologic agents (the tyrosine kinase inhibitor imatinib), or chemotherapy (doxorubicin).
The decision making for treatment recommendations should be multidisciplinary and
should include the preferences and quality-of-life considerations of the patient.
Carcinoid tumors originate in the intestine. Frequently, the lymph nodes in the
mesentery are seen on scans; however, their appearance is different, causing a sclerosing
rather than infiltrative appearance. Carcinoids show arterial hyperenhancement and
tethering of bowel loops, whereas desmoids tend to show delayed hyperenhancement.
Lymphoma, while the most common solid neoplasm of the mesentery, is usually associated
with bulky adenopathy with preservation of fat around the mesenteric vessels, something
characterized as the "sandwich sign." Specifically, the nodes are dorsal and ventral to the
mesenteric vessels, appearing similar to "'sandwich buns" with the mesenteric vessels and
surrounding fat appearing as the "sandwich meat." A liposarcoma would show up as mostly
fat. A leiomyoma is not usually present in the mesentery. These tumors are difficult to
differentiate from GIST tumors, because both are mesenchymal tumors generally located
within the muscularis mucosa or submucosa. They usually cause obstructive symptoms or
may lead to ulceration and bleeding. Leiomyomas are not associated with FAP.
References
I. von Mehren M. Randall RL. Benjamin RS. et al; National Comprehensive Cancer
Network. Soft tissue sarcoma. version 2.2014. J Natl Compr Cane Netw. 2014;12(4):
473-483.
2. Bonvalot s. Desai A. Coppola S, et al. The treatment of desmoid tumors: a stepwise
clinical approach. Ann Oneal. 2012;23 (suppl 10):xl58-xl66.
3. Ezhapilli SR, Moreno CC, Small WC, Hanley K, Kitajima HD, Mittal PK. Mesen-
teric masses: approach to differential diagnosis at MRI with histopathologic correlation.
J Magn Reson Imaging. 2014;40(4):753-769.
CRITIQUES
Complex abdominal wall hernias represent a therapeutic dilemma for surgeons. They are
fraught with a high recurrence rate, and the best technique for their repair is not yet estab-
lished. Complex abdominal wall hernias are defined by clinical and patient factors. Clinical
factors include open abdomen management. previously infected wound, previously repaired
hernia, or hernias with loss of domain. Patient factors include obesity, active smoking, dia-
betes, or chronic obstructive pulmonary disease. These hernias have a much higher rate of
recurrence than do noncomplex hernias and, thus, require a more extensive operation.
Although the best operation for these hernias has yet to be determined, a retrorectus
procedure is commonly used. In this repair, the mesh is placed between the rectus muscle
and the posterior rectus sheath. The fascia is then closed above and below the mesh. This
approach may require a release or separation of the muscular layers so the fascia can be
closed without tension. The can be done using either an open or laparoscopic technique.
Other principles guiding the use of mesh in the repair of hernias are as follows:
1. The use of mesh seems to reduce recurrence rates and is recommended for the
repair of complex hernias.
2. The use of synthetic mesh is associated with an increased risk of infection.
3. Mesh placed in the retrorectus position has lower complication rates, including
recurrence, than does placement of mesh in the overlay or underlay positions.
4. Every attempt should be made to close the abdominal wall native tissue
regardless of the positioning of the mesh.
5. Although the use of biologic mesh may reduce the infection rate, biologic
meshes are thought to have a higher recurrence rate and are not considered
superior in overall outcomes.
6. Large-pore or lightweight mesh allows better tissue incorporation of the mesh,
which is thought to help reduce the recurrence rate and possibly infection rate.
7. Cross-linking is a feature of biologic mesh, and it is thought to prevent the
degradation of the mesh, resulting in a lower incidence of recurrence.
Answer: (A) incisional hernias that result from a surgical site infection should be con-
sidered complex.
References
I. Ventral Hernia Working Group, Breuing K, Butler CE, Ferzoco S, et al. Incisional
ventral hernias: review of the literature and recommendations regarding the grading and
technique of repair. Surgery. 201O;148(3):544--558.
2. Sanchez VM, Abi-Haidar YE, Itani KM. Mesh infection in ventral incisional
hernia repair: incidence, contributing factors, and treatment. Surg Infect (Larchmt).
2011;12(3):205-210.
3. Berrevoet F, Maes L, De Baerdemaeker L, Rogiers X, Troisi R, de Hemptinne B.
Comparable results with 3-year follow-up for large-pore versus small-pore meshes in
open incisional hernia repair. Surgery. 2010;148(5):969-975.
280 CATEGORY 4- PART II
4. Carlson TL, Lee KW, Pierce LM. Effect of cross-linked and non-cross-linked acellular
dermal matrices on the expression of mediators involved in wound healing and matrix
remodeling. Plast Reconstr Surg. 2013; 131(4):697-705.
CRITIQUE6
Urinary retention is reported to occur in up to 22% of patients after laparoscopic hernia
repair. Risk factors include age greater than 60, a history of benign prostatic hypertrophy,
and operating time greater than 2 hours. Urinary retention can still occur in young patients
and seems to be more prevalent after laparoscopic repairs compared with open ones.
When comparing laparoscopic repairs with open repairs, laparoscopic repairs tend to have
longer operating times but similar recurrences and a faster return to work. Currently, there
is no definitive evidence that one method (open, total extra-peritoneal, or trans-abdominal)
shows superiority over the others, so the approach can be determined by surgeon preference
and patient choice, even in asymptomatic patients. Regarding asymptomatic patients,
surgical repair is not mandatory, because only approximately 17% of patients will develop
symptoms within I year from diagnosis, and this is typically not an incarcerated hernia.
Chronic postoperative pain is a concern after inguinal repairs. Chronic pain can occur
in up to 11 % of patients, but only about 3-4o/o of patients have severe or activity-limiting
symptoms. There is no definitive evidence that dividing the ilioinguinal nerve routinely
during an open repair results in a reduction in the incidence of this chronic pain syndrome.
Answer: (C) Urinary retention is a common complication after laparoscopic hernia repair.
References
1. Sivasankaran MY, Pham T, Divino CM. Incidence and risk factors for urinary retention
following laparoscopic inguinal hernia repair. Am J Surg. 2014;207(2):288-292.
2. Nienhuijs S, Staal E, Strobbe L, Rosman C, Groenewoud H, Bleichrodt R. Chronic
pain after mesh repair of inguinal hernia: a systematic review. Am J Surg. 2007; 194(3):
394-400.
CRITIQUE7
Idiopathic thrombocytopenic purpura (ITP) is a disease in which antibodies are formed
against platelets, which results in consumption of the platelets and thrombocytopenia. These
antibodies are created in the spleen, and a splenectomy is curative in approximately 75-85%
of patients. ITP is associated with mucosa! bleeding, petechiae, purpura, and ecchymoses.
The diagnosis is made by the presence of antiplatelet antibodies seen on serum testing.
For ITP, first-line medical therapy is the use of steroids to prevent the formation of
antibodies; 50-75% of adults will respond to the initial use of steroid therapy, but long-
term response rates are much lower at 15-20%. If steroids do not improve the symptoms,
a recurrence occurs, or serious bleeding occurs, then anti-IgG antibody can be used. In
pediatric patients, ITP is self-limited in the majority of patients, especially by the age of 5.
Transfusion, steroids, and anti-IgG antibody therapy is usually avoided in pediatric patients
unless the patient has life-threatening or symptomatic bleeding. When the platelet count is
less than 20,000/mm3 (l 4,000-44,000/mm3), pediatric patients are at risk for intracranial
hemorrhage.
By contrast, thrombocytopenic purpura (TTP) causes thrombocytopenia, because the
spleen sequesters the platelets and results in splenomegaly. TTP is associated with a
low platelet count, hemolytic anemia, neurologic complications, renal failure, mental
status changes, and bilateral lower-extremity petechiae. Here the diagnosis can be made
by blood smear testing, which will show schistocytes, nucleated red blood cells, and
basophilic stippling. The treatment is usually plasmapheresis and transfusion of fresh
CATEGORY 4- PART II
281
frozen plasma (not platelets), but splenectomy can be considered when the platelet
count and symptoms are refractory to plasmapheresis. Splenectomy is curative in only
approximately 40% of patients.
Answer: (E) The disease is usually self-limited for children younger than 5 years.
References
1. George JN. Idiopathic thrombocytopenic purpura: current issues for pathogenesis, diag-
nosis, and management in children and adults. Curr Hematol Rep. 2003;2(5):381-387.
2. Schifferli A, Kuhne T. Chronic immune thrombocytopenia in children: who needs sple-
...
nectomy? Semin Hematol. 2013;50 (suppl l):S58-S62.
CRITIQUES
Sigmoid diverticulitis is common, accounting for nearly 300,000 US hospital admissions
annually. Evidence-based guidelines continue to evolve as our understanding of the disease
process improves. Management has shifted from urgent operations for acute diverticulitis
and mandatory resection with colostomy formation to elective operations when the sur-
geon is able to maintain intestinal continuity.
Prior guidelines supported routine elective resection in patients younger than 50 and in
patients with 2 or more episodes of uncomplicated diverticulitis. Newer evidence suggests
that these groups are not at an increased risk of complications with successive episodes
and that the decision to proceed with elective sigmoidectomy should be individualized. An
additional recommendation is that colonoscopy be performed within 6-8 weeks after an
acute episode of diverticulitis if it is the first episode or if colonoscopy was not performed
recently. Although this practice is debated, the concern is that a small subset of patients
(1-2%) will in fact have cancer, ischemia, or inflammatory bowel disease.
Laparoscopic lavage is gaining popularity as an alternative surgical approach for pa-
tients with Hinchey type II and III disease by CT imaging (table 8.1).
are much lower than ileostomy closure rates. Thus, colectomy with a primary anastomosis
and proximal ilea] diversion for Hinchey types Ill or IV disease may be optimal in select
individuals versus subjecting all patients to a routine Hartmann procedure.
Answer: (C) Colectomy followed by primary anastomosis with ileostomy may be the
optimal strategy for selected patients with perforated diverticulitis.
References
I. Feingold D, Steele SR, Lee S, et al. Practice parameters for the treatment of sigmoid
diverticulitis. Dis Colon Rectum. 2014;57(3):284--294.
2. Regenbogen SE, Hardiman KM, Hendren S, Morris AM. Surgery for diverticulitis in
the 21st century: a systematic review. JAMA Surg. 2014;149(3):292-303.
3. Angenete E, Thornell A, Burcharth J, et al. Laparoscopic lavage is feasible and safe for
the treatment of perforated diverticulitis with purulent peritonitis: the first results from
the randomized controlled trial DILALA. Ann Surg. 2016;263(1):117-122.
4. Wasvary H, Turfah F, Kadro 0, Beauregard W. Same hospitalization resection for acute
diverticulitis. Am Surg. l 999;65(7):632-635; discussion 636.
CRITIQUE9
Comparing open and laparoscopic repair of incisional hernias with mesh, there is no dif-
ference in the early outcomes ofseroma, reoperation (about 13o/o), readmissions (2%), or
mortality (<l %). The risk ofrecurrence is higher with open repair (approximately 20% vs.
15%). Laparoscopic hernia repair is associated with increased operating room costs (sup-
plies and longer operative times) but has shorter total hospital length of stay compared with
open repair. The net effect is that total costs for both procedures are similar.
References
1. Helgstrand F, Rosenberg J, Kehlet H, Jorgensen LN, Bisgaard T. Nationwide prospec-
tive study of outcomes after elective incisional hernia repair. J Am Coll Surg.
2013;216(2):217-228.
2. Bower C, Roth JS. Economics of abdominal wall reconstruction. Surg Clin North Am.
2013;93(5): 1241-1253.
3. Awaiz A, Rahman F, Hossain MB, et al. Meta-analysis and systematic review of lapa-
roscopic versus open mesh repair for elective incisional hernia. Hernia. 2015;19(3):
449-463.
CRITIQUE 10
Surgical site infection (SSI) are a common hospital-acquired infection, causes increased
morbidity, and adds significant costs to care. Colorectal procedures are associated with a
relatively high SS! rate (5-30%) and thus are a prime target for reducing SSL The Surgical
Care Improvement Project is a joint effort between the Centers for Disease Control and
Prevention and the Centers for Medicare and Medicaid Services to develop a program to
reduce SSL
CATEGORY 4- PART II
283
In a Cochrane database analysis of 680 trials involving more than 43,000 patients, the
investigators concluded that combined oral and intravenous antibiotic prophylaxis reduced
SSI compared with intravenous antibiotics alone or oral antibiotics alone.
In a review of elective colorectal procedures using the Veterans Affairs Surgical Quality
Improvement Program, 12% of almost 6000 patients developed SSL Oral plus intravenous
antibiotics (n = 2426) had a lower SS! rate than intravenous antibiotics alone (n = 3324; 6.3%
vs 16.7%, p <.0001). A first-generation cephalosporin plus metronidazole was associated
with significantly lower SS! rates (6%) compared with ampicillin/sulbactam, second-
generation cephalosporins, or fluoroquinolone plus anaerobic coverage (each> 11 %). Oral
antibiotic use was associated with a lower SSI rate for every class of antibiotics used.
References
I. Deierhoi RJ, Dawes LG, Vick C, Itani KM, Hawn MT. Choice of intravenous antibiotic
prophylaxis for colorectal surgery does matter. J Am Coll Surg. 2013;217(5):763-769.
2. Nelson RL, Gladman E, Barbateskovic M. Antimicrobial prophylaxis for colorectal
surgery. Cochrane Database Syst Rev. 2014;5:CD001 !81.
CRITIQUE 11
Colorectal cancer is the third most common cancer in the United States, and liver metasta-
ses occur in up to 30% of patients. Hepatic resection of colorectal metastases may improve
survival in select patients; however, early detection and accurate imaging is crucial to iden-
tifying the correct patients for treatment.
Due to its availability, ease of use, and relatively low cost (compared with MRI), mul-
ti detector CT scan (MDCT) is generally the test of choice for screening and staging of
liver metastases. These metastases are best detected by dynamic CT scanning during the
portovenous phase.
Contrast enhanced MRI is the most sensitive imaging modality to detect small (<I
cm) liver lesions due to its superior soft tissue resolution and the ability to use diffusion-
weighted imaging. The sensitivity of contrast-enhanced MRI to detect liver metastases is
90-95% compared with 70-75% for MDCT. However, the increased costs and limitations
of MRI (e.g., metal implants, claustrophobia) limit its use as a screening tool. The main
role of MRI is in detection and characterization of small lesions. In this respect, MRI
outperforms both FDG-PET scan PET-CT scan in the detection of small liver metastases.
Transabdominal ultrasound fails to detect more than 50% of liver metastases. Contrast-
enhanced ultrasound may improve performance; however, this method is not yet available
in the United States.
References
1. Sahani DV, Bajwa MA,Andrabi Y, Bajpai S, Cusack JC. Current status of imaging and
emerging techniques to evaluate liver metastases from colorectal carcinoma. Ann Surg.
20 l 4;259(5):861-872.
2. Bonanni L, De'liguori Carino N, Deshpande R, et al. A comparison of diagnostic imag-
ing modalities for colorectal liver metastases. Eur J Surg Oncol. 2014;40(5):545-550.
3. Adam R, De GramontA, Figueras J, et al. The oncosurgery approach to managing liver
metastases from colorectal cancer: a multidisciplinary international consensus. Oncolo-
gist. 2012;17(10):1225-1239.
284 CATEGORY 4- PART 11
CRITIQUE 12
The most common postoperative complication of distal pancreatectomy is a leak from the
cut edge of the pancreas, occurring in 20-30% of patients. To avoid this leak and subsequent
development of a pancreatic fistula, many closure techniques and adjuncts were investigated.
Direct suture ligation is the most reliable method to reduce pancreatic leak after pancre-
atectomy. Although initially thought to be promising, stapler transection of the pancreas
(with or without stapler reinforcement) does not reduce leak rate. Postoperative pancreatic
stent placement might be useful treatment for a known leak; however, prophylactic use of
this technique does not reduce leak rate. Routine use of fibrin glue sealant, closed suction
drains, and postoperative somatostatin analogue do not reliably reduce leak rates after dis-
tal pancreatectomy.
References
I. Pannegeon V, Pessaux P, Sauvanet A, Vullierme MP, Kianmanesh R, Belghiti J. Pancre-
atic fistula after distal pancreatectomy: predictive risk factors and value of conservative
treatment. Arch Surg. 2006; 141(11): I 071-1076; discussion I 076.
2. Goh BK, Tan YM, Chung YF, et al. Critical appraisal of232 consecutive distal pancre-
atectomies with emphasis on risk factors, outcome, and management of the postop-
erative pancreatic fistula: a 21-year experience at a single institution. Arch Surg.
2008; 143(10):956-965.
3. Carter Tl, Fong ZV, Hyslop T, et al. A dual-institution randomized controlled trial of
remnant closure after distal pancreatectomy: does the addition of a falciform patch and
fibrin glue improve outcomes? J Gastrointest Surg. 2013;17(1):102-109.
4. Hackert T, Buchler MW. Remnant closure after distal pancreatectomy: current state and
future perspectives. Surgeon. 2012; I 0(2):95-10 I.
CRITIQUE 13
This patient is clinically hemodynamically abnormal and should not undergo definitive
closure. Component separation would increase operative time and not be appropriate given
the patient's hemodynamic status. Polytetrafluoroethylene (PTFE) or bridging with a bio-
logic mesh is also not indicated based on the patient's hemodynamics. Progressive fascial
closure would require some material being sewn to the fascial edges, which would again
increase operative time. Leaving the patienfs abdomen open with a temporary closure is
the best choice.
Patients requiring temporary abdominal closure have significant morbidity and an in-
hospital mortality rate of 33%. Complications of open abdomens include abdominal ab-
scesses, enterocutaneous fistulas, and chronic hernias. Eventual closure techniques may
include tissue rearrangement and use ofbiosynthetic agents.
Temporary closure of an open abdominal wound is commonly accomplished with a
vacuum dressing. When a patient requires open abdomen management, negative pressure
therapy is becoming the method of choice, especially in trauma patients and for planned re-
exploration in other general and vascular surgery patient populations. Temporary negative
pressure abdominal wound closure is associated with patient comfort, low complication
rates, and low costs. Eventual primary closure of open abdominal wounds after negative
pressure treatment exceeds 70o/o.
A well-designed Cochrane review using search criteria (including open abdomen, fas-
cial closure, vacuum, reapproximation, and ventral hernia) suggested that vacuum closure
is associated with the highest fascia! closure rates and the lowest mortality rates.
References
I. Barker DE, Green JM, Maxwell RA, et al. Experience with vacuum-pack temporary
abdominal wound closure in 258 trauma and general and vascular surgical patients.
J Am Coll Surg. 2007;204(5):784--792; discussion 792-793.
2. Boele van Hensbroek P, Wind J, Dijkgraaf MG, Busch OR, Goslings JC. Temporary
closure of the open abdomen: a systematic review on delayed primary fascial closure
in patients with an open abdomen [published correction appears in World J Surg.
2012:36(2):493]. World J Surg. 2009;33(2): 199-207.
3. Stone PA, Hass SM, Flaherty SK, DeLuca JA, Lucente FC, Kusminsky RE. Vacuum-
assisted fascia! closure for patients with abdominal trauma. J Trauma. 2004;57(5):
1082-1086.
CRITIQUE 14
Asplenic or hyposplenic patients have increased risk for infection and death from encapsu-
lated organisms, often pneumococcus, Haemophilus influenza type b, or meningococcus.
However, patients who have a splenectomy to treat an underlying hematologic outcome are
at greater risk for postsplenectomy infection. Splenic implants are not considered adequate
protection against overwhelming postsplenectomy infection (OPS!). OPS! can progress
rapidly from a mild flu-like illness to fulminant sepsis that is fatal in up to 70% of cases
with delayed or inadequate treatment. Because of the high mortality and fulminant course
associated with OPSI, vaccination and antibiotic prophylaxis are the basis of the manage-
ment of asplenic or hyposplenic patients (figure 14.1 ).
Pneumococcal, H. influenzae type b, meningococcal, and influenza virus vaccinations
are recommended for asplenic patients. preferably 2 weeks before an elective surgery. A
booster dose should be administered every 5 years. Children who receive their dose before
age 7 should receive a booster dose 3 years later with subsequent doses every 5 years.
Asplenic patients with a fever should receive empirical antimicrobial therapy.
Rapid identification of patients at risk for OPS! and administration of vancomycin and
ceftriaxone to cover Streptococcus pneumonia, H. influenzae, Neisseria meningitidis,
and many community-acquired Gram-negative bacilli can improve survival. Prophylactic
antimicrobial therapy is recommended for asplenic children younger than age 5 and may be
considered for older children and adults for 1-2 years after splenectomy. Patients who have
had postsplenectomy infection should have lifelong prophylaxis. Asplenic patients should
be educated about the risk oflife-threatening infection with any illness with fever or severe
flu-like symptoms without fever.
Answer: (C) Appropriate initial antibiotic coverage for an asplenic patient with a fever
includes empiric treatment with vancomycin and ceftriaxone.
References
!. Morgan TL, Tomich EB. Overwhelming post-splenectomy infection (OPS!): a case
report and review of the literature. J Emerg Med. 20!2;43(4):758-763.
2. Edgren G, Almqvist R, Hartman M, Utter GH. Splenectomy and the risk of sepsis: a
population-based cohort study. Ann Surg. 20 I 4;260( 6): I 081-1087.
3. Rubin LG, Schaffuer W. Clinical practice. Care of the asplenic patient. N Engl J Med.
20 I 4;371 (4):349-356.
4. Di Sabatino A, Carsetti R, Corazza GR. Post-splenectomy and hyposplenic states.
Lancet. 2011 ;378(9785):86-97.
286 CATEGORY 4 - PART II
l
Patient is able to get to a medical
facility that can administer parenteral
antimicrobial therapy within 2 hr
I
Yos '
No
t t
I
I Medical evaluation and testing,
including blood culture
I
Patient should take a single dose
of an oral antimicrobial agent
(ideally, prescribed previously
CRITIQUE 15
Cholecystokinin (CCK) cholescintigraphy is an appropriate step in diagnosing the patient
with typical gallbladder symptoms who has a negative ultrasound evaluation. CCK or a
synthetic derivative (sincalide) is injected 30-60 minutes before the administration ofTc-
99m labeled disofenin (DISIDA, 2,6-diisopropyl acetanilidoiminodiacetic acid) or mebro-
fenin (BRIDA, bromo-2, 4,6-trimethyl acetanilidoiminodiacetic acid). Imaging starts after
the injection and continues for 60 minutes. When acute cholecystitis is suspected and the
gallbladder is not seen within 60 minutes, imaging should be continued for up to 3-4 hours.
Imaging for 18-24 hours may be necessary in some cases (e.g., a severely ill patient, severe
hepatocellular dysfunction, or suspected common bile duct obstruction). If the patient is
being studied for a biliary leak, 2- to 4-hour delayed imaging should be obtained.
The Society of Nuclear Medicine defines an ejection fraction of less than 38% as ab-
normal. This is a calculated number designated as 2 standard deviations from the mean and
there will be "normal" patients who have ejection fractions less than 38% and abnormal
patients who have ejection fractions greater than 38%. Thus, specificity of this test for
gallbladder disease is low.
CATEGORY 4- PART IT
287
The use of CCK cholescintigraphy for diagnosis of biliary tract disease should be lim-
ited to those patients who meet ROME III criteria (table 15.1) for functional gallbladder
disorders. It should not be used to evaluate atypical abdominal pain, because many medical
conditions, including diabetes, irritable bowel disease, and celiac disease, can produce an
abnormal gallbladder ejection fraction.
Several medications may reduce gallbladder contractility and should be withheld before the
test. These include atropine, calcium channel blockers, octreotide, progesterone, indometha-
cin, theophylline, benzodiazepines, H2 antagonists, and all opioids. Although many physicians
rely on reproduction of the patient's symptoms by the CCK infusion to recommend cholecys-
tectomy, this is a very nonspecific finding. Several small retrospective studies recommended
removal of the gallbladder for symptoms induced by CCK infusion, but at this time it is not
considered a reliable test for the effectiveness of cholecystectomy in curing symptoms. Repro-
duction of symptoms is quite often the result ofrapid infusion of the CCK analog.
Answer: (A) It is useful when patients describe recurrent epigastric or right upper quad-
rant pain episodes lasting 30 minutes or longer.
Table IS.I The Rome III Criteria for Functional Gallbladder and Sphincter of
Oddi Disorders.
Reprinted from Journal of the American College of Surgeons, 217. BK Richmond, J Di Baise. H Ziessman.
Utilization ofCholecystokinin Cholescintigraphy in Clinical Practice. p. 320. Copyright ©2013 American
College of Surgeons. Published by Elsevier Inc, All rights reserved.
288 CATEGORY 4- PART 11
References
1. Richmond BK, DiBaise J, Ziessman H. Utilization of cholecystokinin cholescintigra-
phy in clinical practice. J Am Coll Surg. 2013;217(2):317-323.
2. Tulchinsky M, Ciak BW, Delbeke D, et al; Society of Nuclear Medicine. SNM prac-
tice guideline for hepatobiliary scintigraphy 4.0 [published correction appears in J
Nucl Med Technol. 2012;40(3):17 A. Dosage error in article text]. J Nucl Med Technol.
2010;38(4):210-218.
3. Behar J, Corazziari E, Guelrud M, Hogan W, Sherman S, Toouli J. Functional gallblad-
der and sphincter of Oddi disorders. Gastroenterology. 2006;130(5):1498-1509.
CRITIQUE 16
The most appropriate time to (semi-) electively operate on a pregnant women with cho-
lecystitis is in the second trimester (13-26 weeks), because there is a lower incidence of
premature labor compared with the third trimester. At 32 weeks, conservative manage-
ment of her cholecystitis should be attempted initially. If an operation is mandated, the
mother should be pretreated with betamethasone 24 hours and 12 hours preoperatively.
This approach enhances surfactant production and reduces the need for neonatal respira-
tory support.
At 32 weeks, any radiographs that are clinically needed are safe. This includes intra-
operative cholangiography as well as endoscopic retrograde cholangiopancreatography
(ERCP). The fetus should be shielded when possible. External fetal monitoring is indicated.
Laparoscopic surgery is as safe or safer than an open procedure in pregnancy. An open
cutdown to the peritoneal cavity is indicated (Hasson technique). Gallstone pancreatitis is
associated with a fetal loss of 4. 7% compared with a fetal loss of 2.8% with non-gallstone
pancreatitis. A patient with gallstone pancreatitis should be treated with a cholecystectomy
and clearing of stones with operative common duct exploration or postoperative ERCP.
Both are associated with lower rates of fetal loss than conservative management.
References
I. Dietrich CS 3rd, Hill CC, Hueman M. Surgical diseases presenting in pregnancy. Surg
Clin North Am. 2008;88(2):403-419, vii-viii.
2. Cappel! MS. The safety and efficacy of gastrointestinal endoscopy during pregnancy.
Gastroenterol Clin North Am. l 998;27(1):37-71.
3. Lee JJ, Lee SK, Kim SH, et al. Efficacy and Safety of Pancreatobiliary Endoscopic
Procedures during Pregnancy. Gut Liver. 2015;9(5):672--678.
4. Friedel D, Stavropoulos S, Iqbal S, Cappel! MS. Gastrointestinal endoscopy in the preg-
nant woman. WorldJ Gastrointest Endosc. 2014;6(5):156--167.
5. Al-Hashem H, Muralidharan V, Cohen H, Jamidar PA. Biliary disease in pregnancy
with an emphasis on the role ofERCP. J Clin Gastroenterol. 2009;43(1):58--62.
CRITIQUE 17
The only definitive treatment for end-stage hepatic failure is liver transplantation. Portacaval
shunting, splenorenal shunting, and a transjugular intrahepatic portosystemic shunt (TIPS)
will all reduce portal hypertension, which will reduce gastrointestinal bleeding, but they
do not treat the underlying liver failure and will exacerbate encephalopathy. Although
protein restriction was previously recommended for hepatic encephalopathy, it is no longer
indicated and may be harmful. High-protein diets are well tolerated in patients with cirrhosis;
they should receive 1-1.5 g/kg of protein and 25-40 kCal/kg per day. Most patients with
cirrhosis have a deficiency of branched chain amino acids. These are commonly found
CATEGORY 4- PART II 289
in dairy products and vegetables. Augmentation of the diet with these proteins may be
useful. Although there is anecdotal evidence in favor of the use of purgative agents such
as nonabsorbable disaccharides (such as lactulose), no comprehensive meta-analysis has
shown them to be effective.
References
1. Cordoba J, Minguez B. Hepatic encephalopathy. Semin Liver Dis. 2008;28(1):70-80.
2. Frederick RT. Extent of reversibility of hepatic encephalopathy following liver trans-
plantation. Clin Liver Dis. 2012;16(1): 147-158.
3. Wright G, Chattree A, Jalan R. Management of hepatic encephalopathy. Int J Hepatol.
2011 ;2011 :841407.
4. Als-Nielsen B, Koretz RL, Kjaergard LL, Gluud C. Branched-chain amino acids for
hepatic encephalopathy [published update appears in Cochrane Database Syst Rev.
20l5;2:CDOO1939]. Cochrane Database Syst Rev. 2003;(2):CDOO 1939.
CRITIQUE 18
This patient has clinical evidence of familial adenomatous polyposis (FAP). Patients with
classic FAP are at high risk of developing abdominal desmoid tumors, occurring in ap-
proximately 10-15% of patients with FAP. Attenuated FAP is a subtype and recognized
by fewer polyps, presentation at an older age and less risk for developing desmoid tumors.
Desmoid tumors are considered benign, because they do not metastasize. However,
despite having a benign designation, abdominal desmoid tumors can cause significant lo-
cal problems as they grow into surrounding tissues. Therefore, while considered "benign;·
some abdominal desmoid tumors clearly have a "malignant" effect on patient's lives and
can result in death.
The natural history of these tumors is unpredictable, with a small number regressing
completely, some waxing and waning, many staying stable over time, and a small num-
ber rapidly progressing. The management of these lesions is multidisciplinary. Surgery is
reserved for tumors that are symptomatic and resectable, because recurrence is common.
Simple observation is reasonable for small, asymptomatic lesions. As part of this strategy,
serial CT scans to identify complications, such as ureteral obstruction, is reasonable.
First-line pharmacologic agents include nonsteroidal anti-inflammatory medications
(e.g., sulindac) and antiestrogens (e.g., tamoxifen). Cytotoxic agents can be used for ad-
vanced disease that is not responding to these other, less-toxic choices. Surgery is often
complicated and associated with frequent complications. Therefore, it is done only for
significant symptoms. When necessary, resection is the best option if possible. However,
many patients will have unresectable disease, so a bypass can be done to relieve a bowel
obstruction.
This patient has an incidental finding of a mesenteric mass. While at risk for metastatic
colon cancer, the biopsy is most consistent with a desmoid tumor. Resection is not indicat-
ed at this time and should not be done. Imatinib is a tyrosine kinase inhibitor used to treat
gastrointestinal stromal tumors. FOLFOX (5-fluorouracil, leucovorin, and oxaliplatin) is
a combination chemotherapeutic regimen used for the adjuvant treatment of colon cancer.
Radiation is not used in the treatment of desmoid tumors, because it appears ineffective
and is associated with a high rate of complications. Therefore, sulindac is the best option
at this time.
References
I. Sakorafas GH, Nissotakis C, Peros G. Abdominal desmoid tumors. Surg Oneal.
2007;16(2):131-142.
2. Latchford AR, Sturt NJ, Neale K, Rogers PA, Phillips RK. A 10-year review of surgery
for desmoid disease associated with familial adenomatous polyposis. Br J Surg.
2006;93(10): 1258-1264.
CRITIQUE 19
Acute portal vein thrombosis often presents with onset of vague abdominal pain. Imag-
ing studies such as CT will typically define the significance of the abnormality. In most
patients, the portal vein and its tributaries and branches are involved. One study found that
more than 50% of the patients had a prothrombotic state, and 35% had a myeloproliferative
disorder as a precipitating factor in the development of portal vein thrombosis.
Imaging often shows ascites, but it is rarely clinically significant. Gastroesophageal
varices develop in up to 50o/o of patients without recanalization, although this is rarely an .
initial presentation. Studies suggest that gastroesophageal varices may develop as early as
a month after thrombosis but more commonly develop later over time. Early anticoagula-
tion is associated with recanalization in 40o/o of patients studied, but no patients recanalized
when anticoagulation was not used. Another study found similar findings in patients who
underwent early anticoagulation that continued for a mean of 8 months; 39% patients de-
veloped recanalization. Early anticoagulation of patients with acute portal vein thrombosis
leads to a better outcome with a reduced risk of gastrointestinal complications.
References
I. Turnes J, Garcia-Pagan JC, Gonzalez M, et al. Portal hypertension-related complica-
tions after acute portal vein thrombosis: impact of early anticoagulation. Clin Gastroen-
terol Hepatol. 2008;6(12):1412-1417.
2. Plessier A, Darwish-Murad S, Hernandez-Guerra M, et al; European Network for
Vascular Disorders of the Liver (EN-Vie). Acute portal vein thrombosis unrelated to
cirrhosis: a prospective multi center follow-up study. Hepatology. 2010;51 (1):210-218.
3. Loss M, Lang SA, Uller W, Wohlgemuth WA, Schlitt HJ. Combined surgical and
interventional therapy of acute portal vein thrombosis without cirrhosis: a new effec-
tive hybrid approach for recanalization of the portal venous system. J Am Coll Surg.
20 l 4;2 l 8(3):e79-e86.
CRITIQUE20
This patient presents with acute symptoms related to her polycystic liver disease. Indica-
tions for intervention include acute pain from potential bleeding into the cyst, vague mass
effect from the size of the cyst, and, rarely, liver dysfunction. Fenestration is most effective,
can be done laparoscopically, and often is performed as an outpatient procedure. There is
minimal risk of bleeding or postoperative complications with this approach.
Hepatic resection is not necessary in the management of this disease. Sclerotherapy is
ineffective with a high rate of recurrence. Observational studies suggest that pregnancy
and estrogen replacement therapy can increase the number and volume of cysts. Soma-
tostatin analogues may reduce the overall volume of the liver but have no effect on the
size of the cysts.
.1
References
1. Abu-Wasel B, Walsh C, Keough Y, Molinari M. Pathopbysiology, epidemiology, clas-
sification and treatment options for polycystic liver diseases. World J Gastroenterol.
20I3; 19(35):5775-5786.
2. Chen MF. Surgery for adult polycystic liver disease. J Gastroenterol Hepatol.
2000;15(11 ):1239- 1242.
CRITIQUE 21
The management of acute necrotizing pancreatitis continues to evolve. A multitude of dif-
ferent modalities are now available that affect the timing and management approach fo r
patients with walled-off pancreatic necrosis (WOPN). This patient continues to be symp-
tomatic a month after his acute process. The CT scan suggests a small fleck of air in one of
the pockets of the WOPN (figure 21.3).
Figure 21.3 CT scan suggesting a small fleck of air in one of the pockets
of the walled-off pancreatic necrosis.
Expectant management of this patient may continue to work, although bis recovery
is likely to be slow. The gas within the fluid collection raises concern for bacterial
contamination and infection. There is not an obvious endoscopic site for transluminal
drainage of this fluid collection, and the multiple loculated areas would make this less
likely to work effectively. Percutaneous aspiration is a diagnostic test to exclude an infected
fluid collection. Percutaneous drainage is possible and may expedite resolution of the
patient's symptoms, but often the material cannot be completely evacuated. Laparoscopic
debridement runs the risk of further seeding infected fluid throughout the peritoneal cavity,
which would be less than ideal.
292 CATEGORY 4- PART II
References
1. Papachristou GI, Takahashi N, Chahal P, Sarr MG, Baron TH. Peroral endoscopic drain-
age/debridement of walled-off pancreatic necrosis. Ann Surg. 2007;245(6):943-951.
2. Kumar N, Conwell DL, Thompson CC. Direct endoscopic necrosectomy versus step-up
approach for walled-off pancreatic necrosis: comparison of clinical outcome and health
care utilization. Pancreas. 2014;43(8): 1334--1339.
CRITIQUE22
The Child-Turcotte-Pugh (CTP) score was originally developed to evaluate the risk
of portocaval shunting procedures in patients with portal hypertension. It is also used
to evaluate surgical risk in other intra-abdominal procedures performed in cirrhotic
patients. Overall surgical mortality is 10% for patients with class A cirrhosis, 30% for
those with class B cirrhosis, and 75-80% for those with class C cirrhosis. Components
of the CTP score are international normalized ratio (INR), bilirubin level, albumin level,
encephalopathy, and ascites (table 22.1). The CTP score is criticized for its 2 subjective
variables (degree of encephalopathy and ascites), narrow range (5-15), and the equal
weight given to each variable.
The Model for End-Stage Liver Disease (MELD) is a linear regression model based on
3 laboratory values (INR, bilirubin, and creatinine level). It was initially developed as a
model to predict mortality after transjugular intrahepatic portosystemic shunt but has now
become the method used for liver transplant allocation in the United States. The MELD
formula is shown in figure 22.2.
Note. Ln =natural logarithm: SCr =serum creatinine (mg/dL): Tbili =total bilirubin (mg/dL)~
INR =international normalized ratio
Figure 22.2
CATEGORY 4 - PART II 293
50
45
40 /
~ 35
/
0
~ 30
/
9 25
/
w
:::!: 20
/
15
/
10
/
_/
5
0 -
0 5 10 15 20 25
Mortality Percentage
References
I. Geller DA, Goss JA, Tsung A. Liver. In: Brunicardi FC, Andersen, DK, Billiar TR, et
al, eds. Schwartz's Principles of Surgery. 9th ed. New York, NY:McGraw Hill;2010:
1093-11 33.
2. Asrani SK, Kim WR. Model for end-stage liver disease: end of the first decade. Clin
Liver Dis. 2011 ; 15(4):685-698.
3. Northup PG, Wanamaker RC, Lee VD, Adams RB, Berg CL. Model for End-Stage
Liver Disease (MELD) predicts nontransplant surgical mortality in patients with cirrho-
sis. Ann Surg. 2005;242(2):244-25 1.
CRITIQUE 23
This patient presents with a small bowel obstruction secondary to an obturator hernia. An
obturator hernia is a rare pelvic hernia with an incidence of approximately I%. A weakening
of the obturator membrane can result in an enlargement of the obturator canal and herniation
of small bowel contents adjacent to the obturator vessels and nerve, leading to an obstruction.
Obturator hernias are most commonly seen in elderly women with prior weight loss and no
history of abdominal surgery. Patients can present with signs of a small bowel obstruction:
nausea, vomiting, obstipation, and abdominal pain. Thigh or knee pain on the affected side
are commonly seen. The Howship-Romberg sign refers to ipsilateral groin pain radiating
down the thigh caused by irritation of the obturator nerve. The referred pain is relieved by
flexion of the thigh and aggravated by extension, abduction, and medial rotation.
294 CATEGORY 4 ~PART II
This patient presents with signs of compromised bowel, tenderness on exam, and an
elevated white blood cell count. Therefore, observation would not be appropriate manage-
ment. Surgery through a midline incision is preferred for an obturator hernia, because it
gives the best exposure, allows reduction of hernia content, and facilitates bowel resection
if necessary. Resection of the involved portion of the bowel is sometimes required because
of gangrenous changes or perforation. The defect in the membrane can usually be closed
with interrupted nonabsorbable sutures. On occasion, a mesh repair is required.
An obturator hernia cannot be repaired through a femoral or inguinal groin exploration. In
this patient with signs of compromised bowel, a preperitoneal approach is not recommended.
References
1. Lin HH, Juan CW, Chew BT, Chao JH: Obturator hernia. Int J Gerontol. 2010;4(2):
104-106.
2. Kulkarni SR, Punamiya AR, Naniwadekar RG, et al. Obturator hernia: A diagnostic
challenge. Int J Surg Case Rep. 2013;4(7):606--{;08.
3. Kammori M, Mafune K, Hirashima T, et al. Forty-three cases of obturator hernia. Am J
Surg. 2004;187(4):549-552.
4. Chang SS, Shan YS, Lin YJ, Tai YS, Lin PW. A review of obturator hernia and a
proposed algorithm for its diagnosis and treatment. World J Surg. 2005;29(4):450-454;
discussion 454.
CRITIQUE24
The CT scan of the chest reveals a left diaphragmatic hernia most likely through the previ-
ous site ofinjury (figure 24.2). The patient presents with symptoms ofa bowel obstruction,
abnormal vital signs, and an elevated white blood cell count. The suspicion for strangulated
bowel is high.
The patient will need an exploration to reduce the bowel, a resection if the bowel is
indeed gangrenous, and repair of the diaphragm. This can best be accomplished through an
abdominal incision using a midline exploratory laparotomy.
The thoracotomy incision, whether anterolateral or posterolateral, would make abdomi-
nal exploration of the bowel with possible resection quite challenging.
There is no role for placement of a chest tube or upper endoscopy in this patient, who
presents with a strangulated diaphragmatic hernia.
References
1. Cao C, Manganas C, Ang SC, Yan TD. A systematic review and meta-analysis on
pulmonary resections by robotic video-assisted thoracic surgery. Ann Cardiothorac
Surg. 2012;1(1):3-10.
2. Louie BE, Farivar AS, Aye RW, Vallieres E. Early experience with robotic lung resection
results in similar operative outcomes and morbidity when compared with matched
video-assisted thoracoscopic surgery cases. Ann Thorac Surg. 2012;93(5):1598-1604;
discussion 1604-1605.
CATEGORY 4 - PART ll 295
('J
"'O ~
- ..,
:.:> ('!)
.., IJtl
- 0
--'<
.j:>..
CRITIQUE25
Acute calculous cholecystitis is a common surgical emergency, often affecting elderly pa-
tients and those with serious medical comorbidities. Emergency cholecystectomy in this
high-risk group of patients is associated with increased morbidity up to 4 I% and with mor-
tality up to 4.5%. An alternative treatment is image-guided percutaneous cholecystostomy
under local anesthetic, which has less morbidity and mortality than open or even lapa-
roscopic cholecystectomy in the emergency setting. Once the acute episode is resolved,
cholecystectomy may or may not be considered as an elective intervention.
Magnetic resonance cholangiopancreatography (MRCP) requires no radiation and is a
noninvasive way to visualize the biliary tree. This study would not be done before chole-
cystostomy, but may be indicated to exclude choledocholithiasis later.
Hepatobiliary iminodiacetic acid (HIDA) scan is a nuclear medicine study that uses the
nuclear colloid, iminodiacetic acid. The nuclear colloid is injected intravenously, processed
in the liver, and secreted within the bile. Acute cholecystitis is confirmed if there is no vi-
sualization of the gallbladder or failure to fill the gallbladder within 2 hours after injection,
which is considered evidence that the cystic duct is obstructed. HIDA scan is not needed in
this patient to make the diagnosis of acute cholecystitis given the CT findings.
Endoscopic retrograde cholangiopancreatography (ERCP) is an invasive procedure in
which contrast is injected through the ampulla to image the biliary tree. ERCP can both
diagnose and treat biliary obstruction secondary to choledocholitbiasis, but it will not treat
acute cholecystitis.
References
!. Byrne MF, Suhocki P, Mitchell RM, et al. Percutaneous cholecystostomy in patients
with acute cholecystitis: experience of 45 patients at a US referral center. J Am Coll
Surg. 2003;197(2):206-211.
2. Kortram K, de Vries Reilingh TS, Wiezer MJ, van Ramshorst B, Boerma D. Percutane-
ous drainage for acute calculous cholecystitis. Surg Endosc. 20 l l ;25(11 ):3642-3646.
3. McKay A, Abulfaraj M, Lipschitz J. Short- and long-term outcomes following percu-
taneous cholecystostomy for acute cholecystitis in high-risk patients. Surg Endosc.
2012;26(5): 1343-1351.
4. Jang WS, Lim JU, Joo KR, Cha JM, Shin HP, Joo SH. Outcome of conservative percu-
taneous cholecystostomy in high-risk patients with acute cholecystitis and risk factors
leading to surgery. Surg Endosc. 2015;29(8):2359-2364.
5. Jackson PG, Evans SRT. Biliary system. In: Townsend CM, Beauchamp RD, Evers
BM, Mattox KL, eds. Sabiston Textbook ofSurgery. 19th ed. Philadelphia, PA: Elsevier
Saunders; 2012:1476-1514.
CRITIQUE26
Liver metastases from colorectal cancer are the most frequent hepatic malignancies in the
United States. Radiofrequency ablation (RFA) is a local therapy that uses a form of alter-
nating electrical current to achieve thermal destruction. RFA is performed using several
techniques: percutaneous, open, or laparoscopic. Tumor size and location can preclude
effective RFA when used as curative treatment. Tumor sizes larger than 4-5 cm are associ-
ated with an increased incidence of recurrence.
Traditionally, the presence of 4 or more hepatic metastases was a contraindication to
hepatectomy. Current data reveal that hepatectomy for 4 or more metastases is associated
with an approximate 5-year survival rate of33%. Resectability is based on volume ofliver
remaining after resection and not the actual number of tumors. In a patient with normal
liver function, a 20% remnant is recommended. In a patient who has undergone neoadju-
vant chemotherapy, a 33% remnant is recommended.
Neoadjuvant chemotherapy can yield good results when used for multiple colorectal
metastases. Chemotherapy and complete resection of hepatic metastases are associated
with long-term survival in up to 50--{)0% of patients.
Answer: (B) Resectability is often based on the volume of liver remaining after resection.
References
I. Geller DA, Goss JA, Tsung A. Liver. In: Brunicardi FC, Andersen, DK, Billiar TR, et
al, eds. Schwartz s Principles of Surgery. 9th ed. New York, NY:McGraw Hill;2010:
1093-1133.
2. Choti MA. Radiofrequency ablation of colorectal liver metastases. In: Cameron JL,
Cameron AM, eds. Current Surgical Therapy. I 0th ed. Philadelphia, PA: Elsevier Saun-
ders; 2011:298-300.
3. Sicklick JK, D'Angelica M, Fong Y: The liver. In: Townsend CM, Beauchamp RD,
Evers BM, Mattox KL, eds. Sabiston Textbook of Surgery. 19th ed. Philadelphia, PA:
Elsevier Saunders; 2012:1411-1475.
CRITIQUE27
This patient has acalculous cholecystitis, and no further imaging is necessary to confirm
the diagnosis. Acalculous cholecystitis is typically associated with critical illness, and most
of these patients are poor surgical candidates. Mortality associated with this disease is
41 %. Acalculous cholecystitis is generally the result of biliary stasis and gallbladder isch-
CATEGORY 4- PART II
297
References
I. Knab LM, Boller AM, Mahvi DM. Cholecystitis. Surg Clin North Am. 2014;94(2):
455-470.
2. Anderson JE, Inui T, Talamini MA, Chang DC. Cholecystostomy offers no survival
benefit in patients with acute acalculous cholecystitis and severe sepsis and shock.
J Surg Res. 2014;190(2):517-521.
CRJTIQUE28
Indirect hernias are associated with a patent processus vaginalis. Patients with a patent pro-
cessus vaginalis have a lifetime incidence of inguinal hernia of 5%. Direct hernias occur in
an anatomic area called Hesselbach triangle, which is bordered by the inguinal ligament in-
feriorly, the inferior epigastric vessels laterally, and the lateral edge of the rectus medially.
Femoral hernias occur inferior to the inguinal ligament and medial to the inferior epigastric
vessels and are more common in women than men. The most commonly occurring hernia
in female patients, however, is an indirect inguinal hernia.
During inguinal hernia repair, the most commonly injured nerve is the ilioinguinal. This
nerve is close to the external inguinal ring and is a sensory nerve to the pubic region and
the upper portion of the scrotum or labia majora. The genital branch of the genitofemoral
passes with the spermatic cord to the scrotum, and the lateral branch forms the lateral
femoral cutaneous nerve and provides sensation to the lateral portion of the anterior thigh.
Although most series report chronic pain limiting quality of life as occurring in only 5o/o
of postoperative hernia repair patients, up to 30% of patients report some long-term pain
or discomfort when asked on a confidential questionnaire. Chronic pain may persist for
several years and is the most serious long-term complication.
References
I. Jenkins JT, O'Dwyer PJ. Inguinal hernias. BM!. 2008;336(7638):269-272.
2. Gardner RM, Boyd CR. Examining modern approaches to inguinal and femoral herni-
orrhaphy. JAAPA. 2008;21(6):35-36, 39-41.
- - - - · - - -..- - · · - - - · - · - - · - · - - - - - - - , - - - - - - · - - - - - - - - - - - - · - - ·
Category 4 - Part III
Abdomen
Items 1-28
DIRECTIONS: Each of the questions or incomplete statements is followed by 5 suggested
answers or completions. Select the answer that is the BEST in each case and fill in the
space containing the corresponding letter on the answer sheet.
2. A 75-year-old man presents with new onset left groin bulge. He reports that he noticed
the bulge 2 months ago and experienced no pain or other symptoms. He has normal
bowel habits and no previous hernia surgery. He is othenvise healthy, except for
hypertension. He lives alone and works as a hospital volunteer 3 days a week. In this
patient, watchful waiting is
(A) contraindicated due to the acute nature of the hernia.
(B) contraindicated due to the presence of a bulge.
(C) contraindicated due to the age of the patient.
(D) at high risk of bowel obstruction.
(E) likely to fail due to progression of symptoms.
5. Men with asymptomatic or minimally symptomatic inguinal hernias are most likely to
(A) experience a high rate of bowel obstruction or strangulation of hernia contents.
(B) experience symptoms of pain some time in their lives if they choose
"watchful waiting."
(C) fail "watchful waiting'' if they are younger compared with older patients.
(D) undergo operative therapy with a frequency of20% over their lifetimes.
(E) remain asymptomatic throughout their lives.
6. Which of the following statements is true about carcinoid tumors of the appendix?
(A) The majority present with carcinoid syndrome.
(B) The depth of tumor invasion is the most accurate predictor oflymph
node metastasis.
(C) The status ofregional lymph nodes is the single most important factor
influencing 10-year survival.
(D) The tumor size is the most accurate predictor of lymph node metastasis.
(E) Appendectomy alone is adequate treatment for tumors less than 4 cm
in diameter.
7. A 44-year-old man with poorly controlled diabetes was diagnosed with severe
pancreatitis and associated pancreatic necrosis (figure 7 .1 ). He was managed without
intervention and discharged after 6 days. He re-presents 2 weeks later with increasing
diffuse abdominal pain. He is normotensive with a heart rate of I 05 beats per minute
and a temperature of 38.3°C. He has diffuse abdominal tenderness without peritoneal
signs. His white blood cell count is 24,000/mm3 (3600-l I ,200/mm3), his hemoglobin
is 10 g/dL (13.5-17.5 g/dL), and his serum glucose is 380 mg/dL (70-100 mg/
dL). Repeat imaging is seen in figure 7 .2. Which of the following is the next most
appropriate step in this patient's management?
(A) Admission with nasogastric tube placement, bowel rest, and octreotide
(B) Retroperitoneal debridement
(C) Percutaneous drain placement
(D) Exploratory laparotomy
(E) Endoscopic placement of distal feeding access for enteral nutrition
Figure 7.2
8. A 60-year-old man presents with epigastric pain radiating to his back and nausea. On
exam, he has focal epigastric tenderness. He has elevated transaminases, a lipase of
1800 units/L (I 0-40 units/L), and normal bilirubin. Right upper quadrant ultrasound
demonstrates cholelithiasis and a common bile duct measuring 9 mm. After admission
to the hospital, intravenous fluid resuscitation, and pain medication, his pain resolves
and laboratory values normalize. Which of the following is the next step?
(A) Laparoscopic cholecystectomy
(B) Endoscopic retrograde cholangiopancreatogram
(C) Discharge home with outpatient follow-up
(D) Magnetic resonance cholangiopancreatogram
(E) Laparoscopic cholecystectomy at 6 weeks
9. A 60-year-old man with known chronic liver disease secondary to alcohol abuse pres-
ents with acute onset right upper quadrant abdominal pain. He is diaphoretic. His
blood pressure is 90/40 mm Hg, his heart rate 120 beats per minute, and his respiratory
rate 20. Resuscitation is initiated. After improvement of his vital signs, he undergoes
a CT scan that shows hemorrhage from a 4-cm liver mass (figure 9.1). Therapy should
begin with which of the following?
(A) Biopsy of the mass
(B) Transjugular intrahepatic portosystemic shunt
(C) Embolization
(D) Laparotomy and packing
(E) Mass resection
CATEGORY 4 - PART III 303
10. A 55-year-old man had an esophagectomy for adenocarcinoma of the esophagus (stage
Ila). Six months later, he is found to have 3 technically resectable liver metastases 3-6
cm in diameter. Which of the following statements is true about liver-directed therapy
for this patient's presentation?
(A) Resection confers a survival advantage over chemotherapy.
(B) Radiofrequency ablation has a low local failure rate.
(C) External beam radiation is curative.
(D) Systemic chemotherapy should be offered.
(E) Transarterial chemoembolization is the preferred care for hepatic metastases of
noncolorectal, nonneuroendocrine cancers.
11. A 41-year-old man with Child class A alcoholic cirrhosis undergoes his first screening
upper endoscopy. He has no history of upper-gastrointestinal bleeding. Upper endos-
copy identifies the presence of large varices with no red wheals. Initial primary pro-
phylaxis against variceal hemorrhage for this patient is
(A) observation.
(B) beta-blockade.
(C) endoscopic variceal sclerotherapy.
(D) endoscopic variceal ligation and beta-blockade.
(E) spironolactone.
12. A 24-year-old man undergoes an ultrasound of the abdomen for nonspecific general-
ized abdominal pain. Three 7- to 8-mm polyps are identified in the gallbladder. The
next step in the management of this :finding would be
(A) CT scan of the abdomen.
(B) endoscopic ultrasound.
(C) laparoscopic cholecystectomy.
(D) open cholecystectomy with lymph node sampling.
(E) repeat ultrasound in 6 months.
304 CATEGORY 4- PART 111
13. A 47-year-old woman presents with fever and left upper quadrant pain. A CT scan is
obtained and shown in figure 13.1. The most appropriate treatment for this lesion is
(A) intravenous antibiotics.
(B) intravenous antifungals.
(C) percutaneous drainage with intravenous antibiotics.
(D) splenectomy.
(E) splenic embolization.
Figure 13.1
14. A 38-year-old man underwent an open right inguinal hernia repair. The Lichtenstein
technique was used. He had significant pain immediately after the procedure. This
pain has persisted for 6 months. He describes the pain as a shooting or '"electric shock"
type pain that starts in the right lower quadrant and radiates to the right testicle. On
exam, palpation over the internal ring reproduces his symptoms. He does not have a
recurrent hernia. Which of the following would be the next step in his treatment?
(A) Pulse steroids with rapid taper
(B) Physical therapy for scar release
(C) Mesh excision
(D) Triple neurectomy
(E) Diagnostic laparoscopy
15. A 23-year-old male soccer player presents with left groin pain. On exam, he does not
have an inguinal or femoral hernia. There is pain with palpation of the inguinal floor
and the pubic tubercle. Dynamic ultrasound is normal. MRI shows edema of the mar-
row under the tubercle and mild inflammation at the insertion of the conjoint tendon.
Initial treatment for these findings is
(A) monthly steroid injections for 3 months.
(B) monthly platelet rich protein injections for 3 months.
(C) no weight bearing with the left lower extremity for 6 weeks.
(D) physical therapy to address muscle imbalances and flexibility.
(E) surgery with mesh insertion and adductor tenotomy.
CATEGORY 4- PART III 305
16. Two weeks ago, a 69-year-old man with a remote history of right hemicolectomy
for a T2NO colon cancer was admitted with right upper quadrant pain, fever, and
jaundice. On presentation, he was hemodynamically abnormal but responsive to fluid
resuscitation. Laboratory values were as follows: white blood cell count of24,000/mm3
(3600-11,200/mm3), bilirubin of 14 mg/dL (0.2-1.9 mg/dL), alkaline phosphatase
of 670 U/L (<95 IU/L), glutamic-oxaloacetic transaminase of 250 U/L (<42 U/L),
gamma-glutamyl transferase of 175 U/L ( 10-70 U/L). Gallbladder ultrasound showed
a thickened gallbladder with a positive Murphy sign but without cholelithiasis and a
nonnal caliber common bile duct. The patient initially underwent magnetic resonance
cholangiopancreatography (MRCP; figure 16.1 ), followed by a cholecystostomy that
drained bile and pus. Despite broad-spectrum antibiotics for 7 days, his white blood
cell count remains at 15,000/mm3 and his bilirubin is 9 mg/dL. Which of the following
would you recommend?
(A) Laparoscopic cholecystectomy
(B) Liver biopsy
(C) Serum IgG4
(D) Cancer antigen 19-9 level
(E) Endoscopic retrograde cholangiopancreatography with stent placement
Figure 16.1
17. A 48-year-old man with known cirrhosis due to hepatitis C is being screened for hepa-
tocellular carcinoma. His CT scan shows portal vein thrombosis. Which of the follow-
ing would you recommend?
(A) Splenectomy
(B) Immediate liver transplant
(C) Observation
(D) Splenorenal shunt
(E) Anticoagulation
306 CATEGORY 4- PART Ill
18. A 54-year-old woman presented to her primary care provider with intermittent. right
upper quadrant pain associated with fatty foods. Ultrasound showed a distended
gallbladder without evidence of cholelithiasis. Blood work included an alkaline
phosphatase of 245 U/L (<95 U/L) associated with a normal bilirubin and serum
glutamic-oxaloacetic transaminase of 28 U/L (10-55 U/L). Based on the magnetic
resonance cholangiopancreatography (MRCP) scan shown (figure 18.1), which of the
following would you recommend?
(A) Endoscopic retrograde cholangiopancreatography with biopsy and stent
(B) Laparoscopic cholecystectomy with cholangiogram and common duct
exploration
(C) Whipple (pancreaticoduodenectomy)
(D) Cholecystectomy with bile duct excision and Roux-en-Y hepaticojejunostomy
(E) Cholecystectomy with choledochoduodenostomy
Figure 18.1
19. A 75-year-old woman on warfarin for management of atrial fibrillation presents to the
emergency department complaining of abdominal pain. Her blood pressure is 100/70
mm Hg, her heart rate is 110 beats per minute, and she is afebrile. Physical examina-
tion demonstrates a tender mass in the right lower quadrant, which does not change in
character when she flexes her abdominal wall musculature. Which of the following is
the next most appropriate step in management?
(A) Ultrasound of the right lower quadrant
(B) Broad-spectrum intravenous antibiotics
(C) CT scan of her abdomen and pelvis
(D) Mesenteric angiogram
(E) Urgent exploratory laparotomy
CATEGORY 4 - PART III 307
20. A 50-year-old man with alcoholic cirrhosis and ascites presents with abdominal pain.
fever. and leukocytosis. His blood pressure is 88/50 mm Hg. and his heart rate is I 00
beats per minute. Laboratory data are as follows:
21. Laparoscopic cholecystectomy carries a higher risk of a bile duct injury than open
cholecystectomy. One approach to mitigate this risk is to achieve the "critical view of
safety." Which of the following best describes the critical view of safety?
(A) Visualizing contrast in the duodenum on cholangiogram
(B) Visualizing 2 tubular structures entering the neck of the gallbladder and the
liver/cystic plate behind
(C) Visualizing the cystic duct--<:ommon bile duct junction
(D) Dissecting the gallbladder free from the liver bed in a top-down fashion
(E) Visualizing 2 tubular structures in the triangle of Calot
22. A 52-year-old woman with ulcerative colitis underwent a total colectomy with end
ileostomy many years ago. She was subsequently diagnosed with primary sclerosing
cholangitis, and she developed cirrhosis. She now presents with copious bleeding from
her ileostomy with bright red blood and clots in the bag. She reports intermittent epi-
sodes of bright red bleeding into her ileostomy bag over the past 3 months. She is pale
and diaphoretic. Her heart rate is 130 beats per minute, and her blood pressure is 80/30
mm Hg. On abdominal examination, she has a caput medusae and moderate ascites by
percussion and bedside ultrasound. After successful resuscitation, she undergoes an
upper endoscopy, which is negative for esophageal varices. What is the next best step
in her management?
(A) Transjugular intrahepatic portosystemic shunt
(B) Stomal enteroscopy
(C) Laparotomy and resection of the terminal ileum
(D) Portacaval shunt
(E) Beta-blockade
------·------·---
23. An 18-year-old Afghan woman presents with upper abdominal fullness and pain. A
chest x-ray (figure 23.1) and CTs of her chest and abdomen are obtained (figures 23.2
and 23.3). What is her diagnosis?
(A) Echinococcal cysts
(B) Amoebic cysts
(C) Polycystic disease
(D) Cystadenomas
(E) Retained fragments/foreign bodies
Figure 23.1
Figure 23.2
CATEGORY 4- PART III 309
Figure 23.3
24. A 52-year-old man is referred for evaluation of progressive fatigue, 2.7-kg weight
loss, jaundice, and pruritus. His medical history is significant for ulcerative colitis
diagnosed at age 28, which was followed by a total proctocolectomy with ileoanal
pouch anastomosis at age 46. After his operation, he has done well with 1 episode of
"pouchitis" 8 months ago. This episode resolved promptly with conventional therapy.
Physical examination is negative other than some mild scleral icterus. Laboratory
serum abnormalities include the following: Alkaline phosphatase = 422 IU/L (<95
IU/L), serum glutamic oxaloacetic transaminase= 70 IU/L (<42 IU/L), and serum
alanine transaminase= 65 IU/L (7-55 IU/L). The most likely study to confirm the
diagnosis is
(A) cholescintigraphy (HIDA scan).
(B) transabdominal ultrasound.
(C) magnetic resonance cholangiography.
(D) upper gastrointestinal endoscopic ultrasound.
(E) serum antinuclear cytoplasmic antibody determination (p-ANCA).
25. A 35-year-old man presents to your office with complaints of discomfort and dyses-
thesia at the base of his penis after open mesh-based inguinal hernia repair. These
symptoms are most consistent with the entrapment of which of the following nerves?
(A) Ilioinguinal nerve
(B) Lateral femoral cutaneous nerve
(C) Genital branch of the genitofemoral nerve
(D) Femoral branch of the genitofemoral nerve
(E) Obturator nerve
26. The most consistent predictor ofresponse to splenectomy for idiopathic thrombocyto-
penic (ITP) purpura is
(A) response to steroids.
(B) younger age.
(C) larger spleen size.
(D) degree ofthrombocytopenia.
(E) duration of ITP.
31 Q CATEGORY 4 - PART Ill
27. A 56-year-old man presents with fatigue, right upper quadrant abdominal pain and
tenderness, and leukocytosis. Amoebic serology is negative. CT scan of the abdomen
with intravenous contrast shows a 5-cm, round, fluid-filled lesion with an enhancing
rim in the right lobe of the liver. Appropriate treatment includes
(A) antibiotics only.
(B) antibiotics and percutaneous drainage.
(C) needle aspiration.
(D) right hepatectomy.
(E) percutaneous drainage only.
28. A 43-year-old woman with a history of 3 midline cesarean sections and diabetes pres-
ents with a I 0-cm reducible hernia defect just below her umbilicus. She undergoes
workup and is brought to the operating theater for an open repair of her defect. During
the case, a small enterotomy is made. It results in minimal spillage and is repaired
primarily. Which type of implant should be used to repair the hernia in this clinical
scenario?
(A) Polyester
(B) Polypropylene
(C) Polyglactin
(D) Polytetrafiuoroethylene
(E) Biologic
CATEGORY 4 - PART Ill 311
Critiques 1-28
CRITIQUE 1
Inguinal herniorrhaphy is one of the most commonly performed surgical procedures today.
Incidence of"'groin hernias'' is substantially higher in male patients (2-5o/o vs 0.3°/o in wom-
en). Although femoral hernias comprise only a small minority of groin hernias in men, they
represent nearly one-third of the groin hernias in women. After open mesh repair, a "'recur-
rence'' in women is often seen in the femoral canal. The most likely explanation is that these
are not recurrences but rather errors in diagnosis leading to the incorrect operation.
A traditional, open, tension-free, mesh repair (Lichtenstein) will address both the direct
and indirect defects but does not cover the femoral canal. Both Mc Vay and open preperi-
toneal repairs will address the femoral canal, but these are less commonly performed than
open Lichtenstein procedures. A laparoscopic, extraperitoneal herniorrhaphy allows for
exploration and coverage of the entire myopectineal orifice, addressing both inguinal and
femoral hernias. In women, laparoscopic preperitoneal repair is associated with a lower
recurrence rate than open mesh repair.
Answer: (C) The increased incidence of femoral hernias may explain why female sex is
a risk factor for hernia recurrence.
References
1. Andresen K. Bisgaard T, Kehlet H, Wara P, Rosenberg J. Reoperation rates for lapa-
roscopic vs open repair of femoral hernias in Denmark: a nationwide analysis. JAMA
Surg. 2014;149(8):853-857.
2. Schouten N, Burgmans JP, van Dalen T, et al. Female 'groin' hernia: totally extraperi-
toneal (TEP) endoscopic repair seems the most appropriate treatment modality. Hernia.
2012; 16(4):387-392.
CRITIQUE2
Management of asymptomatic or minimally symptomatic hernias is evolving as long-term
randomized controlled study data for watchful waiting is reported. Since their original
report in 2004 demonstrating safety of watchful waiting during short-term follow-up
(median 3.2 years), Fitzgibbons et al. followed patients up to 11.5 years and found that the
overall crossover rate to surgery is 68%. Men older than 65 are significantly more likely
to crossover to surgery than are younger men (79% vs 62%). The most common reason
for surgery was pain (54%). Only 2.4% of watchful waiting patients required emergency
operation for complications such as strangulation or obstruction. The authors concluded
that watchful waiting is therefore safe, regardless of the timing of onset of the hernia,
presence of a bulge, or age of the patient, but most patients will experience a progression
of symptoms and ultimately need surgical repair. A similar study in the United Kingdom
found that 72% of the watchful waiting cohort came to operative therapy at 7.5 years of
follow-up.
References
I. Fitzgibbons RJ Jr, Ramanan B, Arya S, et al; Investigators of the Original Trial. Long-
term results of a randomized controlled trial of a nonoperative strategy (watchful wait-
ing) for men with minimally symptomatic inguinal hernias. Ann Surg. 2013:258(3):
508-515.
312 CATEGORY 4- PART III
2. Ramanan B, Maloley BJ, Fitzgibbons RJ Jr. Inguinal hernia: follow or repair? Adv Surg.
2014:48:1-11.
3. Chung L, Norrie J, O'Dwyer PJ. Long-term follow-up of patients with a painless ingui-
nal hernia from a randomized clinical trial. Br J Surg. 2011 :98(4):596--599.
CRITIQUE3
In the United States, nonparasitic splenic cysts may be posttraumatic or primary (figure
3.1). Rupture of splenic cysts either spontaneously or posttraumatically is rare. Studies
suggest that fewer than 60% of posttraumatic cysts are symptomatic. Few data support the
use of cyst size as an indication for splenectomies in asymptomatic patients. Postaspira-
tion cyst and symptom recurrence are very common; however, symptomatic relief with
aspiration can suggest a benefit of operative management of splenic cysts. Because of the
benign nature of the majority of splenic cysts, partial splenectomies or partial cystectomies
(unroofing) are reported in most patients.
Answer: (C) Symptom relief after percutaneous needle aspiration may predict response
to operative management.
References
I. Kenney CD, Hoeger YE, Yetasook AK, et al. Management of non-parasitic splenic
cysts: does size really matter? J Gastrointest Surg. 2014;18(9):1658-1663.
2. Wu HM, Kortbeek JB. Management of splenic pseudocysts following trauma: a retro-
spective case series. Am J Surg. 2006;191(5):631-<534.
CATEGORY 4- PART !II
313
CRITIQUE4
Per-oral endoscopic myotomy (POEM) divides only the circular muscle fibers of the
lower esophagus and stomach. It is proposed that this incomplete ablation of the lower
esophageal complex compared with the Heller procedure lowers the 20-100% incidence
of gastroesophageal reflux that accompanies the Heller procedure and necessitates a
gastroesophageal fundoplication in combination with a full-thickness myotomy.
A 2014 study compared consecutive groups of patients treated with the 2 procedures.
Technical complication rates were similar with the 2 procedures. Successful treatment of
dysphagia with POEM, and rarely symptomatic gastroesophageal reflux, was comparable
to the results with Heller myotomy and partial gastric fundoplication. The purpose of adding
partial gastroesophageal fundoplication to a Heller myotomy (circular and longitudinal
muscle layers) is to prevent gastroesophageal reflux after this procedure.
Answer: (D) POEM can achieve similar rates of postprocedural symptomatic esophageal
reflux to those obtained with laparoscopic Heller myotomy and partial gastric fundoplication.
References
1. Bhayani NH, Kurian AA, Duns! CM, Sharata AM, Rieder E, Swanstrom LL. A compar-
ative study on comprehensive, objective outcomes of laparoscopic Heller myotomy
with per-oral endoscopic myotomy (POEM) for achalasia. Ann Surg. 2014;259(6):
1098-1103.
2. Hungness ES) Teitelbaum EN, Santos BF, et al. Comparison ofperioperative outcomes
between peroral esophageal myotomy (POEM) and laparoscopic Heller myotomy.
J Gastrointest Surg. 2013; 17(2):228-235.
CRITIQUES
Two important studies have appeared over the last several years comparing the British and
US results from expectant management of asymptomatic inguinal hernias. Both studies
extend earlier analyses of asymptomatic patients, and both studies reached the same
conclusions. Seventy-two percent of patients who chose '"watchful waiting" in the British
trial went on to operative management over a median of 7.5 years) and 68o/o of patients who
chose ·<watchful waiting'~ in the US trial went on to operative management over a median
of 10 years. Among patients who were 65 years of age or older, the operative conversion
rate was 79% in the US study. The main reason for choosing operations was pain. Both
studies demonstrated less than 3% rates of hernia complications (bowel obstruction or
strangulation of hernia contents) during periods of"watchful waiting."
Answer: (B) experience symptoms of pain some time in their lives if they choose
''watchful waiting."
References
1. Chung L, Norrie J, O'Dwyer PJ. Long-term follow-up of patients with a painless ingui-
nal hernia from a randomized clinical trial. Br J Surg. 2011:98(4):596-599.
2. Fitzgibbons RJ Jr, Ramanan B, Arya S, et al; Investigators of the Original Trial. Long-
term results of a randomized controlled trial of a nonoperative strategy (watchful wait-
ing) for men with minimally symptomatic inguinal hernias. Ann Surg. 2013;258(3):
508-515.
314 CATEGORY 4 - PART Ill
CRITIQUE6
The majority of appendiceal carcinoid tumors are small, and they are discovered inci-
dentally. Lymph node metastases from the tumors are predicted most accurately by size,
with tumors less than 1 cm in diameter and confined to the appendix not associated with
lymph node metastases and treated with appendectomy alone. Tumors greater than 2 cm in
diameter are treated with right hemicolectomy. Controversy surrounds the management of
patients with tumors 1-2 cm in diameter as well as carcinoids at the base of the appendix.
The status of regional lymph nodes does not influence 10-year survival. Because the ma-
jority of tumors are less than I cm in diameter when they are discovered and unlikely to
be associated with metastatic disease, the incidence of carcinoid syndrome is very rare in
patients with appendiceal carcinoid tumors.
Answer: (D) The tumor size is the most accurate predictor of lymph node metastasis.
References
I. Murray SE, Lloyd RV, Sippel RS, Chen H, Ollmann SC. Postoperative surveillance of
small appendiceal carcinoid tumors. Am J Surg. 2014:207(3):342-345; discussion 345.
2. Mullen JT, Savarese DM. Carcinoid tumors of the appendix: a population-based study.
J Surg Oneal. 2011; I 04(1):41-44.
CRITIQUE7
This patient has evidence ofinfected pancreatic necrosis (figure 7.3). Up to 30% of patients
develop infection after necrotizing pancreatitis, with presentation on average 3-4 weeks
after the original management of pancreatitis. In this case, the extraluminal gas evident on
CT imaging is suggestive of the diagnosis, even without culture results.
surgical intervention should be entertained. In those cases, the percutaneous drain can serve
as a ''road map" to direct a minimally invasive approach, such as the step-up approach or
video-assisted retroperitoneal debridement. Minimally invasive surgical approaches have
lower morbidity (e.g., multiple organ failure, incisional hernia, new onset diabetes) and
mortality rates compared with open necrosectomy.
Enteral nutrition tends to improve outcomes for pancreatitis, so indefinite bowel rest is
not advocated. Antibiotics alone for a fluid collection of this size, particularly in a patient
with sepsis, is not advocated. Distal feeding access may be desirable, particularly for those
patients with a gastric ileus, but that would not be the first step in management.
References
I. Karakayali FY. Surgical and interventional management of complications caused by
acute pancreatitis. World J Gastroentero/. 2014;20(37):13412-13423.
2. van Baal MC, van Santvoort HC, Bollen TL, Bakker OJ, Hesselink MG, Gooszen HG;
Dutch Pancreatitis Study Group. Systematic review of percutaneous catheter drainage
as primary treatment for necrotizing pancreatitis. Br J Surg. 2011 ;98( I): 18-27.
3. van Santvoort HC, Hesselink MG, Bakker OJ, et al; Dutch Pancreatitis Study Group.
A step-up approach or open necrosectomy for necrotizing pancreatitis. N Engl J Med.
20I0;362(16): 1491-1502.
CRITIQUES
Gallstones and alcohol are the most common causes of acute pancreatitis. The presence of
high serum lipase and typical abdominal pain suggests the diagnosis in the absence of a
CT scan. The presence of gallstones by abdominal ultrasound suggests the etiology. which
can be further supported by the presence of a dilated common bile duct and the finding of
elevated serum transaminases. The vast majority of stones pass spontaneously within 48
hours of presentation.
Mild cases can be treated with hospital admission, fluid resuscitation, and pain control.
In patients with more severe disease~ cholangitis, persistent hyperbilirubinemia, clinical
deterioration, or detection of a persistently impacted common bile duct stone, endoscopic
retrograde cholangiopancreatogram (ERCP) is warranted within 24-48 hours. In patients
with resolving symptoms of gallstone pancreatitis, cholecystectomy is indicated to remove
the source of stones and prevent recurrence. Delayed cholecystectomy is associated with a
high rate of recurrence of pancreatitis.
A randomized trial demonstrated that early laparoscopic cholecystectomy is associated
with a shorter overall length of hospital stay without increasing operative complications
or conversion rates, compared with delayed cholecystectomy. Magnetic resonance cholan-
giopancreatogram (MRCP) does not offer therapeutic options and is reserved for cases that
have an equivocal diagnosis or anatomy.
References
I. Fogel EL, Sherman S. ERCP for gallstone pancreatitis [published correction appears in
N Eng/J Med. 2014;370(5):488]. N Eng/J Med. 2014;370(2):150-157.
2. Aboulian A. Chan T, YaghoubianA, et al. Early cholecystectomy safely decreases hospi-
tal stay in patients with mild gallstone pancreatitis: a randomized prospective study.Ann
Surg. 2010;251(4):615-619.
316 CATEGORY 4- PART Ill
CRITIQUE9
This individual is in a high-risk group for hepatocellular carcinoma (HCC), and this pre-
sentation is likely a ruptured HCC. A biopsy is unnecessary and unsafe at this point. The
immediate goal is control of the bleeding. The diagnosis may be confirmed based on the
result of the recommended therapeutic arteriography.
The patient's abnormal hemodynamics and active bleeding warrant urgent intervention.
HCCs have a rich arterial supply. and embolization is the preferred first intervention in the
setting of acute rupture. The expected success rate would be better than 80%.
The bleeding is clearly arterial, as evidenced by the blush seen on CT scan (figure
9.2), so transjugular intrahepatic portosystemic shunt (TIPS) would not be indicated. TIPS
would be considered in the setting of bleeding from esophageal varices refractory to endo-
scopic intervention. Surgical approaches to this presentation-resection or packing----carry
a significant risk of morbidity and mortality in the acute setting in patients with chronic
liver disease and should be reserved for patients failing embolization.
Figure 9.2 CT scan showing hemorrhage from a 4-cm liver mass. The
bleeding is clearly arterial, as evidenced by the blush seen on CT scan.
References
1. European Association for the Study of the Liver; European Organisation for Research
and Treatment of Cancer. EASL-EORTC clinical practice guidelines: management of
hepatocellular carcinoma [published correction appears in J Hepatol. 2012;56( 6): 1430].
J Hepatol. 2012;56(4):908-943.
2. El-Serag HB. Hepatocellular carcinoma. N Eng!J Med. 2011;365(12):1118-1127.
3. Damis B, Rode A, Mohkam K, Ducerf C, Mabrut JY. Management of bleeding liver
tumors. J Vise Surg. 2014;151(5):365-375.
CATEGORY 4 - PART III 317
CRITIQUE IO
In this patient, there are several contraindications to liver-directed therapy. The short
disease-free interval and presentation with multiple lesions suggests poor tumor biology,
or inadequate initial staging, or both. Metastatic esophageal adenocarcinoma is rarely
confined to the liver; therefore, systemic therapy is indicated. There is no evidence that
resection has added benefit over systemic chemotherapy.
Noncolorectal, nonneuroendocrine cancers metastatic to the liver are a heterogeneous
group of diagnoses with highly variable biology. Few tumor types, other than colorectal
carcinoma, are known to metastasize to the liver in a manner amenable to curative intent
intervention. Most studies show no benefit of resection or ablation of liver metastases in
the treatment of gastrointestinal malignancies.
The interest in expanding the indications for liver-directed therapies has increased with
the advent of new technologies and the improved safety of liver resection. Tumor biology
is still the principle harbinger of successful treatment in any given patient. The treating
physicians should have a clear understanding of the goals of treatment (curative vs. pallia-
tive intention) as well as the patient's preferences. The ability to perform a procedure alone
is insufficient to serve as an indication.
Radiofrequency ablation and other percutaneous thermal ablation techniques enjoyed
popularity in the past decade due to ease of delivery and generally favorable safety profiles.
The limitations of the technology include tumor number and size. The ability to reliably
and safely destroy tumors larger than 3 cm is in question, with local failure rates in tumors
larger than 5 cm being greater than 30o/o.
External beam radiation is an attractive option given recent advancements in tumor
localization and delivery. The literature supports a role in palliation of symptomatic le-
sions but not in curative intent treatment. Likewise, transarterial chemoembolization has
a theoretical role in palliation of vascular lesions but is not a standard for any broad set of
diagnoses.
References
I. Groeschl RT, Nachmany I, Steel JL, et al. Hepatectomy for noncolorectal non-
neuroendocrine metastatic cancer: a multi-institutional analysis. J Am Coll Surg.
20 l 2;214(5):769-777.
2. Earle SA, Perez EA, Gutierrez JC, et al. Hepatectomy enables prolonged survival in
select patients with isolated noncolorectal liver metastasis. J Am Coll Surg. 2006;203(4):
436-446.
3. Reddy SK, Barbas AS, Marroquin CE, Morse MA, Kuo PC, Clary BM. Resection of
noncolorectal nonneuroendocrine liver metastases: a comparative analysis. J Am Coll
Surg. 2007;204(3):372-382.
4. Mancosu P, Castiglioni S, Reggiori G, et al. Stereotactic body radiation therapy for
liver tumours using flattening filter free beam: dosimetric and technical considerations.
RadiatOnco/. 2012;7:16.
5. Yamashita H, Onishi H, Matsumoto Y, et al: Japanese Radiological Society multi-insti-
tutional SERT study group (JRS-SBRTSG). Local effect of stereotactic body radio-
therapy for primary and metastatic liver tumors in 130 Japanese patients. Radial Oneal.
2014;9:112.
318 CATEGORY 4 ~PART Ill
CRITIQUE 11
The initial diagnosis and classification of non bleeding esophageal varices is usually made
on esophagogastroduodenoscopy. In most centers, varices are classified by a semiquantita-
tive morphological assessment into 3 sizes or grades:
• Small (grade I), generally defined as minimally elevated veins above the esophageal
mucosal surface normal in color, straight, and compressible.
• Medium (grade II), medium varices defined as tortuous veins occupying less than
one-third of the esophageal lumen, with or without red wheals, and noncompressible.
• Large (grade III) defined as those occupying more than one-third of the esophageal
lumen, with or without red wheals, and noncompressible.
Nonselective beta-adrenergic blockers are the foundation of therapy for primary
prophylaxis for preventing variceal bleeding. Patients with cirrhosis and varices that have
not bled should be started on primary prophylaxis. A meta-analysis of 11 trials (1189
patients) evaluated nonselective beta-blockers (e.g., propranolol, nadolol) versus nonactive
treatment or placebo in preventing first variceal hemorrhage. In patients with large- or
medium-sized varices, risk of first variceal bleeding was significantly reduced by beta-
blockers (30% in controls vs. 14% in patients treated with beta-blockers). One bleeding
episode was avoided for every l 0 patients treated.
Two invasive endoscopic approaches to manage esophageal varices are available:
sclerotherapy and banding. Endoscopic sclerotherapy is associated with a higher risk of
side effects compared with variceal band ligation (VBL). Therefore, variceal band ligation
is the preferred method for endoscopic prophylaxis of variceal bleeding. A meta-analysis
of 5 randomized clinical trials comparing VBL with no treatment showed a decreased risk
of first variceal bleeding and a lower mortality rate in the VBL group.
Spironolactone in combination with beta-blockade shows no benefit over beta-blockade
alone for primary prophylaxis of variceal bleeding.
Nonselective beta-blockers or variceal band ligation may be an appropriate first-line
choice for primary prophylaxis of variceal bleeding. However, the combination of both
therapies is not more effective and is associated with increased side effects.
References
1. Torres E, Barros P, Calmet F. Correlation between serum-ascites albumin concentra-
tion gradient and endoscopic parameters of portal hypertension. Am J Gastroenterol.
1998;93(11):2172-2178.
2. Bai M, Qi X, Yang M, Han G, Fan D. Combined therapies versus monotherapies for the
first variceal bleeding in patients with high-risk varices: a meta-analysis of randomized
controlled trials. J Gastroenterol Hepatol. 2014;29(3):442-452.
3. Simonette DA, Shah VH. Primary prophylaxis of esophageal variceal bleeding. Clin
Liver Dis (Hoboken). 2012;1(5):147-150.
4. Garcia-Tsao G, Sanyal AJ, Grace ND, Carey W; Practice Guidelines Committee of the
American Association for the Study of Liver Diseases: Practice Parameters Committee
of the American College of Gastroenterology. Prevention and management of gastro-
esophageal varices and variceal hemorrhage in cirrhosis [published correction appears
in Hepatology. 2007;46(6):2052]. Hepatology. 2007;46(3):922-938.
CATEGORY 4 - PART III
319
CRITIQUE 12
Gallbladder polyps are identified in 1.5-6.9% of the population. They can be classified as
"pseudotumors" (cholesterol polyps, adenomyomas, or inflammatory polyps), epithelial
(adenomas), mesenchymal (leiomyomas), and malignancies (adenocarcinomas). The main
concern when a polyp is identified is whether it is malignant. Single polyps are more likely
to be malignant than multiple polyps. The feature most predictive of malignancy is the size
of the polyp. A polyp larger than I cm in size has a risk of malignancy of 43-77%, whereas
polyps larger than 2 cm are nearly always malignant. Age also needs to be considered;
patients older than 50-60 years have a higher risk of malignancy. Finally, the presence of
gallstones along with polyps or a diagnosis of sclerosing cholangitis and polyps increases
the likelihood of malignancy.
With the knowledge of these risk factors, a care plan algorithm can be designed. In a
patient with true biliary symptoms, cholecystectomy is indicated based on the symptoms.
The presence of polyps does not influence that decision. In patients with no symptoms and
polyps 5 mm or smaller, a follow-up ultrasound in 6-12 months is indicated. If the polyps
have not increased in size, no further imaging is needed. Polyps 6-9 mm in size can be
followed with ultrasound. Typically, a follow-up study is obtained at 6 months and I year
and then yearly thereafter if polyp size is stable. If the polyps decrease in size, no further
imaging is needed. If they increase in size, a cholecystectomy is indicated. An exception
could be a patient over the age of 50 with a single polyp larger than 5 mm. In a patient older
than 50, cholecystectomy is indicated.
All polyps 10 mm or greater in size require surgery. CT scanning should be considered
in patients with polyps greater than 10 mm and age younger than 60 and in all patients with
polyps greater than 20 mm to evaluate for invasive cancer.
The role of endoscopic ultrasound in the management of gallbladder polyps is not well
defined. Studies show it can be used to predict pathology at the time of cholecystectomy in
97% of patients. However, it is more invasive and expensive, and it requires endoscopists
who have advanced endoscopic ultrasound training. Thus, although it may be useful in
evaluating and staging larger polyps, it may not be the most cost-effective way to follow
patients with smaller polyps. Open, nonradical cholecystectomy with lymph node sam-
pling is not a procedure used for benign or malignant gallbladder pathology.
References
I. Andren-Sandberg A. Diagnosis and management of gallbladder polyps. N Am J Med
Sci. 2012;4(5):203-211.
2. Park JY, Hong SP, Kim YJ, et al. Long-term follow up of gallbladder polyps. J Gastro-
enterol Hepatol. 2009;24(2):219-222.
3. Zakko WF. Zakko SF. Gallbladder polyps and cholesterolosis. In: Chopra S, Travis AC,
eds. UpToDate. Waltham, MA: UpToDate; 2014. Updated: Jan 24, 2014. Accessed:
Sept 30, 2014.
CRITIQUE 13
Splenic abscesses are rare, but if not treated appropriately, they have a high mortality rate
(figure 13.2). The most common etiology is hematogenous spread~ such as from infective
endocarditis. Splenic trauma is another common etiology, as is splenic infarction. Up to
50% of these infections are polymicrobial. For this reason, empiric broad-spectrum antibi-
otics should be initiated as soon as the diagnosis is made. Primary fungal splenic abscesses
are rare and anti fungal therapy would not be initiated as part of empiric therapy. Antibiotics
alone are not considered definitive treatment.
320 CATEGORY 4 - PA RT Ill
Percutaneous drainage is now the procedure of choice for splenic abscess. Aspiration al-
lows for culture-directed antibiotics in addition to preserving the spleen. Contraindications
to percutaneous drainage include multiple abscesses and cysts that have features that make
a diagnosis of Echinococcus more likely, such as a calcified cyst wall and the presence of
other cysts in the abdominal cavity, particularly in a patient from the Mediterranean basin
or Eastern Europe. Additionally, patients who are coagulopathic or those without safe per-
cutaneous access to the abscess are not candidates for percutaneous drainage.
Splenectomy is reserved for patients who are not candidates for or who have fai led
percutaneous drainage. There is no indication for the use of sple nic embolization in splenic
abscess, and it is one of the etiologies of splenic abscess.
References
I. Ferraioli G. Brunetti E, Gulizia R, Mariani G, Marone P, Filice C. Management ofsplen-
ic abscess: report on 16 cases from a single center. Int J Infect Dis. 2009;13(4):524--530.
2. Losanoff JE, Sasson MD. Splenic abscess. In: Geibel J, Talavera F, Morris DL, eds.
Medscape. New York, NY: WebMD LLC. Updated: May 1, 2014. Accessed: Jan 7,
2016.
CRITIQUE 14
Pain is to be expected after any operation. It is important to recognize when pain has tran-
sitioned from acute to chronic. For a n inguinal hernia repair, pain that exists beyond 3-6
months, whic h would be expected for the normal postoperative inflammatory response,
is defined as chronic pain. It is estimated that 10-12% of patients will suffer chronic pain
after inguinal hemiorrbaphy (CPIH) and that 0.5-6% of p atients will have pain that affects
their daily life or employment.
Evaluation ofCPIH requires the surgeon to know the di fferent types of pain that patients
experience. Nociceptive pain is acute pain associated with acute stimulation or damage to
sensory nerves. Touching a bot stove or a pain associated with a surgical incision are com-
CATEGORY 4- PART III 321
mon causes. This type of pain often transitions to inflammatory nociceptive pain, in which
inflammation leads to stimulation of the sensory nerves and the sensation of pain. An ex-
ample would be pain that persists after spraining an ankle. These 2 types of pain represent
the normal response to surgical injury.
Etiologies of chronic pain can be simple to complex. Inflammatory pain can persist.
This can occur if there is ongoing inflammation of the tissue and or a nerve in the region.
Nerves injured at the time of surgery can form a neuroma that is hypersensitive. Ongoing
pain stimuli can lead to a peripheral neuropathy, where there is peripheral amplification of
normal nociceptive signals. Ongoing pain stimuli can lead to a central neuropathy where
spinal cord neurons send increased pain signals to the brain. This is seen in allodynia
(normal stimulus leads to the sensation of pain) and hyperalgesia (amplified response to a
painful stimulus).
For CPIH, it is important to differentiate hernia recurrence, direct nerve injury associ-
ated pain, chronic inflammatory pain, and neuropathic pain. The first 3 have surgical op-
tions, whereas a pain management specialist best treats the fourth.
The pain associated with hernia recurrence is typical of an inguinal hernia. If physical
exam is negative, an ultrasound exam done with the patient standing and performing a
Valsalva maneuver can help to identify an occult recurrence. If the patient seems to be de-
scribing a recurrence, and evaluation is negative., diagnostic laparoscopy can be performed.
Treatment is repair of the hernia.
Direct nerve injury can lead to neuroma formation. Stimulation of the neuroma will
cause an electric shock type pain. In the groin, the ilioinguinal, iliohypogastric, and genito-
femoral nerves are all susceptible to injury. Because of the overlapping areas these nerves
supply sensation to, any history of this type of pain or physical exam that elicits this type
of pain (Tine! sign) should lead to nerve injury as the cause ofCPIH. Nerve blocks can be
attempted to confirm the diagnosis, but failure of the block could be based more on injec-
tion technique than wrong diagnosis. For this reason, with this type of pain, the surgeon can
make a direct recommendation for triple neurectomy for treatment.
Chronic inflammatory pain can occur due to the mesh, inciting chronic inflammation.
Additionally, the mesh can contract, forming a ·•meshoma," which can lead to chronic tis-
sue inflammation and irritation. This pain typically remains in the groin and can be worse
with movement. There is usually pain on palpation of the inguinal floor, but no electric
shock type pain. Treatment can be initiated with anti-inflammatory agents, including ste-
roids, in severe cases. Ifthe mesh is stiff on exam, physical therapy, including scar release,
can be attempted. If these fail, then mesh excision is considered.
The most difficult diagnosis is that of chronic neuropathic pain. On exam, the physician
will find a severe reaction to normal stimuli or to minimally painful stimuli. Pain will not
be localized to the groin and may be described as the entire lower abdomen. Investigation
for hernia recurrence is important. If a recurrent hernia is not identified, these patients are
best treated by a pain specialist.
References
1. Chen DC, Hiatt JR, Amid PK. Operative management of refractory neuropathic inguino-
dynia by a laparoscopic retroperitoneal approach. JAMA Surg. 2013;148(10):962-967.
2. Amid PK, Chen DC. Surgical treatment of chronic groin and testicular pain after lapa-
roscopic and open preperitoneal inguinal hernia repair. J Am Coll Surg. 2011;213(4):
531-536.
3. Bjurstrom MF, Nicol AL, Amid PK, Chen DC. Pain control following inguinal hemior-
rhaphy: current perspectives. J Pain Res. 2014;7:277-290.
322 CATEGORY 4- PART III
CRITIQUE 15
Pain in the inguinal region without evidence of hernia is known by many names. Sports
hernia and athletic pubalgia are used interchangeably in the literature, even though they are
different entities. The term pubic inguinal pain syndrome (PIPS) is now used to describe
the various forms of non-hernia-associated inguinal pain.
Most important in any discussion of PIPS is to ensure that the patient does not have
an inguinal or femoral hernia. This is done by exam and, if negative, a standing Valsalva
ultrasound (dynamic ultrasound) of the groin. If a hernia is not identified with either, then
PIPS should be considered.
PIPS has 2 main etiologies. The first is weakness or laxity of the inguinal floor. This
is the classic "sports hernia." Patients will describe more classic hernia symptoms and on
exam, with palpation of the inguinal floor, pain is reproduced. Dynamic ultrasound can
show weakness and bulging of the floor. Surgical reinforcement of the inguinal floor is the
treatment of choice.
The second etiology is inflammation and injury at the pubic tubercle and the associated
musculotendinous insertions. Patients describe pain in the proximal medial thigh, below
the scrotum, in the lower abdomen, and at the pubic bone. On physical exam, there is
tenderness to palpation of the pubic tubercle and there may be tenderness along the thigh
adductor tendons, the lower rectus muscle, or the medial aspect of the conjoined tendon.
Diagnosis can be confirmed with MRJ scanning, which will demonstrate inflammation
of these areas. The etiology of these findings is thought to be muscular imbalances of the
abdomen, pelvis, and proximal lower extremity. For this reason, physical therapy is the
initial treatment of choice. If physical therapy fails, surgery to off-load tension on the pubic
tubercle in addition to adductor tenotomy is indicated.
Steroid and platelet-rich protein injections are used to treat osteitis pubis and would not
be indicated for PIPS. IfMRI identified a pelvic stress fracture, limitation of weight bear-
ing would be considered. It is not used for PIPS.
References
1. Sheen AJ, Stephenson BM, Lloyd DM, et al. 'Treatment of the sportsman's groin':
British Hernia Society's 2014 position statement based on the Manchester Consensus
Conference. BrJ Sports Med. 2014;48(14):1079-1087.
2. Campanelli G. Pubic inguinal pain syndrome: the so-called sports hernia. Hernia.
2010;14(1):1-4.
3. Garvey JF, Hazard H. Sports hernia or groin disruption injury? Chronic athletic
groin pain: a retrospective study of 100 patients with long-term follow-up. Hernia.
2014; 18(6):815-823.
4. Rossidis G, Perry A, Abbas H, et al. Laparoscopic hernia repair with adductor tenoto-
my for athletic pubalgia: an established procedure for an obscure entity. Surg Endosc.
20 l 5;29(2):381-386.
CRITIQUE 16
This patient presents a picture of cholangitis without evidence of the distal bile duct ob-
struction that would be expected with choledocholithiasis, a stricture complicating pancre-
atitis, or obstructing pancreatic/common duct tumor. The incongruity of history/physical
and laboratory values should prompt further investigation before considering operative
intervention. The degree of hyperbilirubinemia and elevated alkaline phosphatase do not
support a diagnosis of acalculous cholecystitis. Because of its high sensitivity and specific-
ity, noninvasive magnetic resonance cholangiopancreatography (MRCP) has largely re-
placed endoscopic retrograde cholangiopancreatography (ERCP) as the initial diagnostic
CATEGORY 4 - PART Ill 323
test of choice. The MRCP images for this case show the classic "bead-on-a-string" ap-
pearance of sclerosing cholangitis (figure 16.2). Although placement of a stent might be
useful for biliary decompression in the face of a dominant extrahepatic stricture, significant
clinical improvement is unlikely in the presence of intrahepatic strictures. Because this
patient may ultimately be considered for liver transplant, cholecystostomy drainage and a
prolonged course of antibiotics is the most appropriate initial management for this patient.
Figure 16.2 MRCP image show ing the classic " bead-on-a-string"
appearance of sclerosing cholangitis.
References
1. Hirschfield GM, Karlsen TH, Lindor KD, Adams DH. Primary sclerosing cholangitis.
Lancet. 2013;382(9904):1587-1599.
2. Karlsen TH, Boberg KM. Update on primary sclerosing cholangitis. J Hepatol.
20 l 3;59(3):571-582.
3. Singh S, Talwalkar JA. Primary sclerosing cholangitis: diagnosis, prognosis, and
management. Clin Gastroenterol Hepatol. 2013;11(8):898-907.
CRITIQUE 17
Portal vein thrombosis (PVT) is a complication of cirrhosis commonly identified on im-
aging of patients with advanced liver disease and is associated with portal hypertension
and hypersplenism. Chronic partial PVT may be asymptomatic, whereas acute complete
obstruction may induce intestinal congestion with severe continuous or colicky abdominal
pain and occasionally nonbloody diarrhea. Although PVT and its attendant portal hyper-
tension pose a significant risk of variceal bleeding, a treatment algorithm combining an-
ticoagulation and transjugular intrahepatic portosystemic shunting (TIPS) offers the best
chance for restoring portal flow, reducing portal pressures, reducing thrombosis extension,
and reducing the risk of intestinal infarction.
Although appropriate for management of bleeding gastric varices due to splenic vein
thrombosis, splenectomy decreases portal vein flow, thus increasing the risk of PVT.
Splenorenal shunt, an option for management of bleeding varices, also decreases portal
blood flow. PVT alone is not an indication for liver transplantation nor is it necessarily a
contraindication. Its presence, however, is an independent risk factor for recurrent PVT
after transplant and decreased perioperative survival. Complete or partial recanalization is
associated with better survival rates and, therefore, anticoagulation is recommended in all
patients with PVT.
References
1. Kinjo N, Kawanaka H, Akahoshi T, et al. Portal vein thrombosis in liver cirrhosis. World
J Hepatol. 2014;6(2):64-71.
2. Plessier A, Rautou PE, Valla DC. Management of hepatic vascular diseases. J Hepatol.
2012;56 (suppl I):S25-S38.
3. Ponziani FR, Zocco MA, Senzolo M, Pompili M, Gasbarrini A, Avolio AW. Portal
vein thrombosis and liver transplantation: implications for waiting list period, surgical
approach, early and late follow-up. Transplant Rev (Orlando). 2014;28(2):92-101.
CRITIQUE 18
This patient's workup was initiated by a concern for biliary colic. The ultrasound did not
show cholelithiasis. Her laboratory picture of normal bilirubin and transaminase values
with a marked elevation in alkaline phosphatase supports further imaging of the biliary
tree. In this case, the magnetic resonance cholangiopancreatography (MRCP) shows a type
I choledochocyst (figure 18.2). There is no evidence for distal or common duct obstruc-
tion warranting endoscopic retrograde cholangiopancreatography (ERCP) with stent or
cholecystectomy with common duct exploration. A pancreaticoduodenectomy or Whipple
procedure is generally reserved for a distal bile duct or pancreatic head malignancy. In this
case, cholecystectomy with bile duct excision and Roux-en-Y hepaticojejunostomy is the
procedure of choice.
CATEGORY 4 - PART lll 325
Answer: (D) Cholecystectomy with bile duct excision and Roux-en-Y hepaticojeju-
nostomy
326 CATEGORY 4- PART III
References
I. Singham J, Yoshida EM, Scudamore CH. Choledochal cysts. Part 3 of3: management.
Can J Surg. 2010;53(1):51-56.
2. Soreide K, Korner H, Havnen J, Soreide JA. Bile duct cysts in adults. Br J Surg.
2004;91(12):1538-1548.
3. Todani T, Watanabe Y, Narusue M, Tabuchi K, Okajima K. Congenital bile duct cysts:
classification, operative procedures, and review of thirty-seven cases including cancer
arising from choledochal cyst. Am J Surg. l 977;134(2):263-269.
CRITIQUE 19
Rectus sheath hematoma (RSH) is a condition characterized by abdominal pain and an
abdominal wall mass. Fothergill sign is an abdominal wall mass that does not cross the
midline and remains present with contraction of the abdominal wall muscles. It is useful to
differentiate between a mass arising in the rectus muscle and intra-abdominal masses. This
sign would be positive with RSH.
Important risk factors for RSH include female sex, older age, anticoagulation therapy,
and cough or other abdominal trauma. A high index of suspicion in the appropriate clinical
setting can lead to rapid diagnosis via CT scan (figure 19.1) of the abdomen and pelvis,
which is the diagnostic modality of choice for RSH and is more accurate than ultrasonog-
raphy for this clinical entity. Most patients are successfully treated with symptom manage-
ment. Anticoagulation should be reversed, if at all possible.
CATEGORY 4- PART Ill
327
References
1. Salem is NS) Gourgiotis S, Karalis G. Diagnostic evaluation and management of patients
with rectus sheath hematoma. A retrospective study. Int J Surg. 2010;8(4):290-293.
2. Cherry WB, Mueller PS. Rectus sheath hematorna: review of 126 cases at a single insti-
tution. Medicine (Baltimore). 2006;85(2):105-110.
CRITIQUE20
Spontaneous bacterial peritonitis (SBP) is a bacterial infection of ascitic fluid without an
intra-abdominal source of infection. SBP is associated with underlying conditions such
as cirrhosis, nephrotic syndrome, and congestive heart failure. The patient will have ab-
dominal pain, fever, leukocytosis, and an ascitic fluid sample demonstrating more than 250
neutrophils/rnm3.
Broad-spectrum antibiotics, typically a third-generation cephalosporin, are started
immediately. Gram-positive and anaerobic organisms rarely cause SBP in cirrhotic patients
making vancomycin or metronidazole inappropriate. Because the patient is not bleeding,
neither fresh frozen plasma nor cryoprecipitate is necessary.
328 CATEGORY 4 - PART lll
References
I. Garcia-Martinez R, Caraceni P, Bernardi M, Gines P, Arroyo V, Jalan R. Albumin:
pathophysiologic basis of its role in the treatment of cirrhosis and its complications.
Hepatology. 2013;58(5):1836-1846.
2. Fernandez J, Navasa M, Garcia-Pagan JC, et al. Effect of intravenous albumin on
systemic and hepatic hemodynamics and vasoactive neurohormonal systems in patients
with cirrhosis and spontaneous bacterial peritonitis. J Hepatol. 2004;41(3):384-390.
CRITIQUE21
Laparoscopic cholecystectomy is associated with a 0.4% incidence of bile duct injury. Er-
rors in identification are the most common cause of bile duct injuries during laparoscopic
cholecystectomy. The '"critical view of safety" is advocated as a means of ensuring the
cystic duct and artery are clearly identified and that no other structures are adherent to or
mistaken for these structures before dividing them. The essential elements of this critical
view are to visualize 2 and only 2 tubular structures entering the neck of the gallbladder
with a view of the liver and cystic plate behind these cystic structures. Although intraopera-
tive cholangiography can be complementary to the critical view of safety, it is not required.
The cystic duct-common bile duct junction should not be routinely dissected, because
this maneuver can actually increase the risk of a bile duct injury. At times, a top-down
mobilization of the gallbladder may be necessary, but this is not part of or required for the
critical view of safety. Visualizing 2 tubular structures in the triangle of Calot alone does
not constitute this critical view.
Answer: (B) Visualizing 2 tubular structures entering the neck of the gallbladder and the
liver/cystic plate behind
References
1. Strasberg SM, Brunt LM. Rationale and use of the critical view of safety in laparoscopic
cholecystectomy. J Am Coll Surg. 2010;211(1): 132-138.
2. Wu YV, Linehan DC. Bile duct injuries in the era of laparoscopic cholecystectomies.
Surg Clin North Am. 2010;90(4):787-802.
3. Peitzman AB, Watson GA, Marsh JW. Acute cholecystitis: when to operate and how to
do it safely. J Trauma Acute Care Surg. 2015;78(1): 1-12.
CRITIQUE22
This patient is most likely bleeding from her peristomal varices given the negative upper
endoscopy. The patient should be aggressively resuscitated with blood products, and any un-
derlying coagulopathy should be corrected. A temporizing measure to consider would be to
apply traction to a balloon-tipped catheter inserted into her ileostomy to temporarily control
the bleeding. Although endoscopic management is often a first-line therapy in esophageal
variceal bleeding, enteroscopy through the stoma is not likely to be effective in managing
bleeding from peristomal varices, because they arise at the mucocutaneous junction.
If transjugular intrahepatic portosystemic shunt (TIPS) is available, this intervention
will significantly reduce the risk of current and future bleeding from peristomal varices.
Operative intervention with portacaval shunt carries a significantly higher mortality in this
case. Re-siting the ileostomy with or without ileum resection can be considered but should
be performed only if TIPS is not available or other factors indicate a need for a new ileos-
tomy site. Nonselective beta-blockers are useful in preventing initial variceal bleeds but are
not indicated for a patient who is actively bleeding and in hemorrhagic shock.
CATEGORY 4- PART III
329
References
I. Helmy A, Al Kahtani K, Al Fadda M. Updates in the pathogenesis, diagnosis and
management of ectopic varices. Hepatol Int. 2008;2(3):322-334.
2. Shibata D, Brophy DP, Gordon FD, Anastopoulos HT, Sentovich SM, Bleday R. Tran-
sjugular intrahepatic portosystemic shunt for treatment of bleeding ectopic varices with
portal hypertension. Dis Colon Rectum. 1999;42(12): 1581-1585.
3. Kwok AC, Wang F, Maher R, et al. The role of minimally invasive percutaneous emboli-
sation technique in the management of bleeding stomal varices. J Gastrointest Surg.
2013;17(7):1327-1330.
CRITIQUE23
Echinococcus is endemic in much of the underdeveloped world, including Afghanistan.
Four different species of this tapewonn infect humans, with Echinococcus granulosis be-
ing the most common. The x-ray and CT scan demonstrate classic findings of a calcified
cystic wall in the lungs and liver (figure 23.4). Amoebic cysts, polycystic disease, and
cystadenomas do not typically have calcified walls. Retained fragments or foreign bodies
would have more beam-hardening artifact scatter around the radiodense structures.
References
!. McManus DP, Zhang W, Li J, Bartley PB. Echinococcosis. Lancet. 2003;362(9392):
1295-1304.
2. Vachha B, Sun MR, Siewert B, Eisenberg RL. Cystic lesions of the liver. AJR Am J
Roentgenol. 2011;196(4):W355-W366.
3. Reid-Lombardo KM, Khan S, Sclabas G. Hepatic cysts and liver abscess. Surg Clin
North Am. 2010:90(4):679-697.
CRITIQUE24
Fatigue, weight loss, and pruritus are the classic presenting symptoms of primary scle-
rosing cholangitis (PSC). A history of ulcerative colitis, which is present in 75-80% of
patients with PSC, strongly suggests the diagnosis. Unlike primary biliary cirrhosis (PBC),
which is more common in women and affects the small and medium bile ducts, PSC has a
slight male predilection and can involve both intra- and extrahepatic ducts. Confirmation
of the diagnosis is by cholangiography, preferably magnetic resonance cholangiography,
which has the advantage of being noninvasive and cost-effective and has a sensitivity of
better than 90%.
Endoscopic retrograde cholangiography, which has a sensitivity approaching 1OOo/o, is
currently reserved for equivocal finding by magnetic resonance cholangiography. Cho-
lescintigraphy (HIDA scan) is not helpful in confirming the diagnosis, because this im-
aging modality lacks sufficient detail to show the classic "beads-on-a-string" anatomic
configuration of the ducts. Percutaneous liver biopsy is not necessary for the diagnosis.
Endoscopic or transabdominal ultrasound, while beneficial in the identification of cho-
ledocholithiasis or pancreatic pathology, is not recommended for suspected PSC, because
it cannot evaluate the intrahepatic ductal system. The determination of serum antibodies
(e.g., p-ANCA and ANA) is useful in suggesting autoimmune disorders but lacks the abil-
ity to differentiate the many autoimmune conditions, such as rheumatoid arthritis, Sjogren
syndrome, and celiac disease.
The elevation of serum alkaline phosphatase is the primary laboratory abnormality in
PSC patients. Most patients present with normal bilirubin levels; the marked elevation
of bilirubin in this case is suggestive of advanced disease or malignancy. Patients with
a history of ulcerative colitis have an increased risk of cholangiocarcinoma, and those
with a history of'"pouchitis" have an even greater risk. For patients with PSC, intact co-
lons, and ulcerative colitis, the risk of colorectal cancer, preferentially the right colon,
is increased by some reports 160-fold over normal individuals. The relationship between
PSC and ulcerative colitis remains obscure, and theories include genetic (variants in HLA
genes), geographical (prevalence in the United States and European countries 100-times
greater than in Asia), bacterial toxins (colonic bacterial overgrowth with exaggerated im-
mune response), and environmental factors (tobacco smoke and environmental toxins), or
combinations of these.
Endoscopic stenting of the bile ducts, preferentially with self-expanding metallic stents,
has some role in palliation in PSC. Some studies suggest that stenting of just a few of the
stenotic ducts provides symptomatic relief in most cases. Sphincterotomy and cholecys-
tectomy have no therapeutic or even palliative benefit. Corticosteroids and immunosup-
pressive agents are not of any value, either in providing symptomatic relief or in delaying
progression of the disease. However, in patients with symptoms overlapping autoimmune
hepatitis, corticosteroids may have a role.
The only effective therapy for end-stage PSC is liver transplantation. In fact, PSC is the
most common indication for liver transplantation in Scandinavia and the fifth most com-
mon indication in the United States. Unfortunately, transplanted livers have a 20% chance
of developing a recurrence of PSC, and this may ultimately lead to graft rejection.
References
1. Imam MH, Talwalkar JA, Lindor KD. Clinical management of autoimmune biliary
diseases. J Autoimmun. 2013:46:88-96.
2. Karlsen TH, Boberg KM. Update on primary sclerosing cholangitis. J Hepatol.
20 I 3;59(3):571-582.
3. Korn NS,Alkaade S. Role of endoscopy in primary sclerosing cholangitis. Curr Gastro-
enterol Rep. 2013;15(12):361.
4. Singh S, Talwalkar JA. Primary sclerosing cholangitis: diagnosis, prognosis, and
management. C/in Gastroenterol Hepatol. 2013;11(8):898-907.
5. Yimam KK, Bowlus CL. Diagnosis and classification of primary sclerosing cholangitis.
Autoimmun Rev. 2014: 13(4--5):445--450.
6. Fosby B, Karlsen TH, Melum E. Recurrence and rejection in liver transplantation for
primary sclerosing cholangitis. World J Gastroenterol. 2012; 18(1):1-15.
CRITIQUE25
Chronic pain after inguinal hernia repair is reported to occur in 5-30% of patients. Al-
though there is overlap in the different entrapment syndromes, detailed anatomic knowl-
edge of the innervation is critical to avoid entrapment and assist in diagnosis and manage-
ment of posthemiorrhaphy neuralgia when it occurs.
The ilioinguinal nerve and iliohypogastric nerves have similar anatomic origins, cours-
es, and sensory innervation. They provide sensory innervation to the proximal and medial
thigh, base of the penis, and upper scrotum.
The lateral femoral cutaneous nerve should be well away from the operative field. Its
sensory innervation is to the anterolateral thigh. The genital branch of the genitofemoral
nerve provides sensation to the scrotum. The femoral branch of the genitofemoral nerve
innervates the anterolateral thigh. The obturator nerve passes through the obturator canal
well away from the operative field (figure 25.1 ).
)'N
·:::::<'.\··.
lliolngulnal n.
lliohypogastric n.
Internal inguinal ring
Hernia sac
' ..;·,· Anterior scrotal
bro.nchos of the
ilioinguinal n.
References
I. Mui WL, Ng CS, Fung TM, et al. Prophylactic ilioinguinal neurectomy in open inguinal
hernia repair: a double-blind randomized controlled trial. Ann Surg. 2006;244(1):27-33.
2. Hsu W, Chen CS, Lee HC, et al. Preservation versus division of ilioinguinal nerve on
open ~esh repair of inguinal hernia: a meta-analysis of randomized c'Ontrolled trials.
World J Surg. 2012;36(10):2311-2319.
3. Ferzli GS, Edwards E, Al-Khoury G, Hardin R. Posthemiorrhaphy groin pain and how
to avoid it. Surg C/in North Am. 2008;88(1):203-216.
CRITIQUE26
Idiopathic thrombocytopenic purpura (ITP) occurs when autoantibodies against platelet
glycoprotein complexes result in varying degrees of thrombocytopenia. The spleen is the
predominant site of antibody-induced platelet sequestration and destruction. Splenectomy
is advocated in adults with thrombocytopenia from ITP with reported long-term remission
rates of 66-85o/o compared with much lower remission rates reported from medical man-
agement (<30% ). The benefits of laparoscopic splenectomy reinvigorated the interest in
splenectomy as a treatment for ITP.
Many studies examined factors that predict a sustained improvement in platelet counts
after splenectomy. Although the degree of response to steroids and shorter duration of
disease are positive predictors of response, the most consistent, independent predictor is
younger age. There is no specific age cut off; the younger the patient is, the more likely the
chance of complete remission. Lower platelet counts are a negative predictor. A few studies
show that spleen size does not predict response. In general, patients with ITP have normal
to slightly enlarged spleens. Splenomegaly is uncommon.
References
I. Kojouri K, Vesely SK, Terrell DR, George JN. Splenectomy for adult patients with
idiopathic thrombocytopenic purpura: a systematic review to assess long-term plate-
let count responses, prediction of response, and surgical complications. Blood.
2004; I 04(9):2623-2634.
2. Katkhouda N, Grant SW, Mavor E, et al. Predictors ofresponse after laparoscopic sple-
nectomy for immune thrombocytopenic purpura. Surg Endosc. 2001;15(5):484-488.
CRITIQUE27
Liver abscesses can be caused by fungus, bacteria, and amebae. Patients commonly present
with constant, dull right upper quadrant abdominal pain, probably due to stretching of the
liver capsule. Fever is common. Abscesses tend to occur in the right lobe, presumably due
to the larger hepatic volume and predominant flow of blood from the superior mesenteric
vein draining the gastrointestinal tract as opposed to the splenic vein. The presence of mul-
tiple abscesses suggests a bacterial or mixed source.
The treatment of liver abscesses has changed with the emergence of medical technol-
ogy, particularly imaging and image-guided percutaneous interventions. A century ago~
patients with multiple hepatic abscesses had a nearly universal mortality. Abscesses not
amenable to percutaneous drainage may be successfully treated with antibiotics alone. In
this case, the 5-cm fluid collection in the right lobe would be accessible by percutaneous
means. The enhancing rim suggests characteristics of a pyogenic abscess. The negative
amoebic serology eliminates amebae as the cause. Percutaneous drainage~ with or without
a catheter placement, improves the likelihood of treatment success. Occasionally, repeated
CATEGORY 4 - PART Ill 333
aspirations are needed and can still result in treatment success. The morbidity of surgical
drainage or even hepatectomy precludes its use.
ln the Western world, bacterial hepatic abscess is most common. In developing
countries, particularly Southeast Asia and Africa, amebic infection is the most frequent
cause. Amebic liver abscess is caused by the protozoa Entamoeba histolytica, and most can
be treated with amebicidal drug therapy alone.
References
1. Singh S, Chaudhary P, Saxena N, Khandelwal S, Poddar DD, Biswal UC. Treatment
of liver abscess: prospective randomized comparison of catheter drainage and needle
aspiration. Ann Gastroenterol. 2013;26(4):332-339.
2. Alvarez Perez JA, Gonzalez JJ, Baldonedo RF, et al. Clinical course, treatment, and
multivariate analysis of risk factors for pyogenic liver abscess. Am J Surg. 2001: 181 (2):
177- 186.
CRITIQUE28
To assist with risk stratification of surgical site events and operative decision making,
the Ventral Hernia Working Group (YHWG) developed a 4-tiered grading system (figure
28. l ). VHWG recommends the use of synthetic mesh for low-risk hernias (i.e., grade 1)
and bioprosthesis for potentially contaminated and infected hernias (i.e., grades 3 and 4,
respectively). Biologic meshes placed in between two layers ofvascularized tissue allow
rapid host cell and vascular infiltration, which is believed to help them resist infection more
effectively than synthetic meshes. In addition, biologic implants have high salvage rates in
the setting of infection and enteric contamination.
• Lowri.<kof
complications
• Smoker
• Obese
r.Previou~
infection
wound • lnfccrcd mc>h
• Septic dchisccncc
• No hisrory of • Di:lbctic • Stom!l prc.~nt
l•
wound infection • Viol:ition of the
• lmmunO!Cuppn:sscd
j:::l.'\trointc~tin:l1
COPD troct
Figure 28.1 Hernia grading system: assessment of risk for surgical site occurrences.
Wound infection defined as being contained within the skin or subcutaneous tissue
(superficial), or involving the muscle and/or fascia (deep).
Reprinted from Surgery. 148. by Ventral Hernia Working Group: K Breuing. CE Butler. S Ferzoco. et al.
lncisional Ventral Hernias: Review of the Literature and Recommendations Regarding the Grading and
Technique of Repair. p. 549. Copyright © 2010. with permission from Elsevier.
In this patient with a recurrent incisional hernia and violation of the integrity of the
small bowel (i.e., a YHWG grade 3 hernia), biologic mesh placement is the suggested
therapy. Placement of synthetic mesh is not recommended. Polyester, polypropylene, and
polytetrafluoroethylene are all permanent synthetic materials. Polyglactin is an absorbable
synthetic material, which is prone to recurrent hernias.
334 CATEGORY 4- PART Ill
References
1. Althubaiti G, Butler CE. Abdominal wall and chest wall reconstruction. P/ast Reconstr
Surg. 2014;133(5):688e--70le.
2. Butler CE, Baumann DP, Janis JE, Rosen MJ. Abdominal wall reconstruction. Curr
Prob/ Surg. 2013;50(12):557-586.
3. Slater NJ, van der Kolk M, Hendriks T, van Goor H, Bleichrodt RP. Biologic grafts for
ventral hernia repair: a systematic review. Am J Surg. 2013:205(2):220-230.
4. Itani KM, Rosen M, Vargo D, Awad SS, Denoto G 3rd, Butler CE; RICH Study Group.
Prospective study of single-stage repair of contaminated hernias using a biologic porcine
tissue matrix: the RICH Study. Surgery. 2012;152(3):498-505.
5. Rosen MJ, Denoto G, Itani KM, et al. Evaluation of surgical outcomes of retro-rectus
versus intraperitoneal reinforcement with bio-prosthetic mesh in the repair of contami-
nated ventral hernias. Hernia. 2013;17(1):31-35.
6. Kanters AE, Krpata DM, Blatnik JA, Novitsk)' YM, Rosen MJ. Modified hernia grad-
ing scale to stratify surgical site occurrence after open ventral hernia repairs. J Am Coll
Surg. 2012;215(6):787-793.
7. Ventral Hernia Working Group; Breuing K, Butler CE, Ferzoco S, et al. Incisional
ventral hernias: review of the literature and recommendations regarding the grading and
technique ofrepair. Surgery. 2010;148(3):544-558.
Category 5
Vascular
Items 1-39
2. The best option for superior mesenteric artery (SMA) bypass in a patient with chronic
mesenteric ischemia is from
(A) infrarenal aorta to SMA.
(B) splenic artery to SMA.
(C) left renal artery to SMA.
(D) right iliac artery to SMA.
(E) left iliac artery to SMA.
3. Which of the following statements regarding mortality and limb loss in isolated lower-
extremity vascular trauma is true?
(A) Mortality rate is higher after injury to the popliteal and tibial arteries than the
common and superficial femoral arteries.
(B) Mortality rate is higher after blunt injury than penetrating injury.
(C) The amputation rate is higher after injury to the popliteal and tibial arteries than
the common and superficial femoral arteries.
(D) The amputation rate is higher after penetrating trauma than blunt trauma.
(E) The presence of distal pulses after blunt injury to the knee excludes popliteal
artery injury.
4. A 24-year-old man presents with a gunshot wound to the right groin 4 inches below
the inguinal ligament and 1-cm lateral to the right femoral artery pulse. On physical
examination, he has a nonexpanding hematoma in the groin. The right common
femoral artery pulse is palpable without a bruit or thrill present. The next best step in
management of this patient is
(A) noninvasive Doppler pressure monitoring.
(B) duplex ultrasonography.
(C) measurement of the arterial pressure index.
(D) intra-arterial angiography.
(E) CT angiography.
5. A22-year-old man sustains a posterior dislocation of the right knee during a football
game. On admission, right dorsalis pedis and posterior tibial pulses cannot be palpated,
but triphasic flow is present on Doppler examination of both arteries. An angiogram of
the popliteal artery is obtained and is shown in figure 5.1. The best initial approach is
CATEGORYS 337
9. A 72-year-old woman with coronary artery disease is seen for left-sided abdominal
pain without peritoneal signs and passing of maroon stools for the past 24 hours. Her
temperature is 37.2°C, her heart rate is 92 beats per minute, and her blood pressure is
180/90 mm Hg. Her white blood cell count is l 8,000/mm3 (3600-11,200/mm3) with
a hematocrit of 28% (37-51 %). Her other laboratory values are normal. A CT scan
shows thickening in her colon near the splenic flexure. A colonoscopy is done the next
day. The colonoscopy is normal except for a 25-cm segment at the splenic flexure,
pictured in figure 9.1. Which of the following is the next best step in management?
(A) Bowel rest and intravenous fluids
(B) Arteriogram
(C) Intravenous steroids
(D) Fecal transplant
(E) Exploratory laparotomy
Figure 9.1
10. A 60-year-old asymptomatic man is noted to have a 50-69% right internal carotid artery
(ICA) stenosis on duplex exam for a bruit found on routine physical exam. Which of the
following is the most appropriate subsequent management of his ICA stenosis?
(A) Observation
(B) Medical management w ith antiplatelet agent
(C) Internal carotid artery stent placement
(D) Carotid endarterectomy
(E) Medical management with warfarin
CATEGORY 5 339
11. A 66-year-old woman has a right femoral popliteal below-knee bypass with greater
saphenous vein done for critical ischemia. Surveillance duplex shows 80°/o stenosis at
the distal anastomosis. The most appropriate management is
(A) an exercise program.
(B) clopidogrel.
(C) percutaneous transluminal angioplasty.
(D) surgical revision of anastomosis.
(E) by-pass graft.
12. A 68-year-old man with an 8-month history of postprandial abdominal pain and 9-kg
weight loss presents with 5 hours of acute severe, unrelenting, diffuse abdominal pain.
His surgical history is significant only for a carotid endarterectomy 4 years ago. He is
tachypneic and tachycardic, with a blood pressure of 107/82 mm Hg. His abdomen is
distended without peritoneal signs. His serum lactate is elevated. Electrocardiogram
reveals sinus tachycardia without myocardial changes. An abdominal x-ray series
demonstrates dilated small and large bowel, without free air. Which of the following is
his most likely diagnosis?
(A) Nonocclusive mesenteric ischemia
(B) Mechanical small bowel obstruction
(C) Mesenteric venous thrombosis
(D) Mesenteric arterial embolic disease
(E) Mesenteric arterial thrombotic disease
13. A 32-year-old woman who is 20 weeks pregnant presents 7 days after an uncompli-
cated laparoscopic cholecystectomy with an acute iliofemoral deep vein thrombosis.
Other than a markedly swollen limb, she is completely asymptomatic. Which of the
following would be the most appropriate therapy?
(A) Catheter-directed thrombolysis
(B) Intravenous unfractionated heparin and begin oral warfarin
(C) Intravenous unfractionated heparin and begin oral Factor Xa inhibitor
(D) Avena cava filter
(E) Subcutaneous low molecular weight heparin for the duration of pregnancy
14. Which of the following statements regarding vena cava filters is true?
(A) The rate of recurrent pulmonary emboli after inferior vena cava filter placement
is greater than 5%.
(B) They are indicated for iliofemoral venous thrombosis.
(C) They are associated with an increased incidence of recurrent deep vein
thrombosis compared with anticoagulation alone.
(D) Long-term risk ofvena cava thrombosis is less than 1%.
(E) Removal rate of retrievable filters in the United States is greater than 70%.
17. A 67-year-old homeless man presents with a venous leg ulcer. Which of the following
treatments effectively improves wound healing?
(A) Compression bandages
(B) Systemic antibiotics
(C) Silver-impregnated dressings
(D) Honey-based dressing
(E) Arterial bypass
18. With respect to arteriovenous (AV) hemodialysis access. which of the following state-
ments is true?
(A) Autogenous AV access should be delayed until the patient requires dialysis.
(B) Prosthetic access should be delayed until just before the need for dialysis.
(C) AV accesses should be placed as far proximal in the arm as possible.
(D) Lower-extremity sites are preferred over upper-extremity sites.
(E) Physical exam for fistula surveillance offers no benefit.
19. With regard to carotid stenosis. which of the following statements is true?
(A) Heterogeneous plaques increase the risk of stroke.
(B) Magnetic resonance angiography (MRA) underestimates the degree of
carotid stenosis.
(C) MRA cannot determine plaque morphology.
(D) MRA is the imaging method of choice when screening for carotid stenosis.
(E) Conventional digital angiography is required before carotid endarterectomy.
20. After endovascular repair of a blunt injury to the thoracic aorta, which of the following
statements is true?
(A) The patient needs to be monitored for right-arm ischemia.
(B) Covering the orifice of the left subclavian artery may result in vertebrobasilar
ischemia.
(C) Open repair has a lower mortality than an endovascular approach.
(D) Endovascular repair has no risk of spinal cord ischemia.
(E) Endovascular repair is preferable in pediatric patients.
CATEGORY5 341
21. A 54-year-old man presents with a !-week history of increasing abdominal pain,
which is made worse with meals. He notes increased abdominal distension, but his
bowel movements are normal. On examination, he has mild diffuse abdominal tender-
ness without peritoneal signs. His medical history reveals no prior abdominal surgery.
However, he has a history of nontraumatic deep venous thrombosis of his left leg 3
years ago for which he took warfarin for 6 months. A CT scan is shown in figures 21.1
and 21.2, What is the appropriate initial therapy for this condition?
(A) Intravenous heparin, followed by warfarin
(B) Intravenous tissue plasminogen activator (t-PA)
(C) Percutaneous thrombolysis
(D) Transjugular intrahepatic portal-caval shunt
(E) Exploratory laparotomy and thrombectomy
Figure 21.1
Figure 21.2
342 CATEGORY 5
22. A SO~year-old man undergoes intramedullary nailing for a right femur fracture com-
plicated by a femoral deep venous thrombosis (DVT). The patient was started on an-
ticoagulation therapy, which was prescribed at discharge, with planned duration for
3 months. Six weeks after discharge, he is ambulatory but presents with a painful,
swollen right leg that feels "heavy.'' On examination, the distal right leg is swollen and
tender with pitting edema but normal skin color and integrity. Distal pulses are normal.
Which of the following statements about his leg swelling is true?
(A) The risk of leg ulceration is approximately 50%.
(B) This patient should wear a graduated compression garment on his leg for
24 months.
(C) Leg swelling is reduced by the immediate use of compression stockings after
a diagnosis of DVT.
(D) This complication most commonly occurs in patients over the age of 65.
(E) The patient can expect a full recovery after treatment.
23. A 22-year-old man sustains a stab wound just above the middle of the left clavicle. He is
hemodynamically normal. Pulses in his left arm and wrist are diminished, but his pulses
are easily felt elsewhere. No other injuries are detected. His chest x-ray is normal. Which
of the following statements regarding a potential subclavian artery injury is true?
(A) A brachial-brachial index of 0.8 will exclude a vascular injury.
(B) Posterolateral thoracotomy provides adequate exposure of the suspected
injury site.
(C) Conventional angiography is more sensitive and specific than CT angiography
in evaluating for a subclavian artery injury.
(D) Duplex ultrasonography reliably demonstrates this vascular injury.
(E) Endovascular stenting is the preferred approach in this hemodynarnically
normal patient.
24. A 75-year-old man presents with hemodynamic instability and a mildly distended ab-
domen. He has a recent history of acute pancreatitis. After resuscitation, a CT scan
is performed (figure 24.1 ). The CT demonstrates a bleeding gastroduodenal artery
(GDA). Immediate next steps involve which of the following considerations?
(A) Laparotomy, drainage of the pancreas, and ligation of the GDA
(B) Selective transarterial embolization
(C) Somatostatin
(D) Endoscopic coagulation
(E) Palliative care consultation
Figure 24.1
CATEGORYS 343
25. In adults, the most common indication for surgical correction of an aberrant right sub-
clavian artery originating distal to the left subclavian artery is
(A) stridor.
(B) dysphagia.
(CJ aneurysm.
(D) right upper-extremity claudication.
(E) subclavian steal syndrome.
26. Carotid endarterectomy performed during the acute phase of a nondisabling stroke
(A) is commonly associated with conversion to hemorrhagic stroke.
(B) should be delayed until 2 weeks after the ischemic event.
(C) should be performed for stenoses greater than 70%.
(D) is associated with an in-hospital risk of myocardial infarction of I 0%.
(E) should not be performed if thrombolytics were given for the stroke.
27. Patients with peripheral artery disease who present with claudication symptoms
(A) will require surgical or radiologic intervention within 6 months of diagnosis.
(B) should be started on clopidogrel to improve walking distance.
(C) should be started on lipid-lowering therapy.
(D) have no increased risk of embolic stroke.
(E) have a risk of progression to critical ischemia and limb loss of I 0% per year.
30. Which of the following statements about thoracic aortic dissection is true?
(A) It is the most common type of acute thoracic aortic pathology.
(B) It occurs most commonly in the descending aorta in patients younger than 60.
(C) It is associated with marijuana use.
(D) Severe abdominal pain is the most common presentation.
(E) Medical management has a 20% 30-day mortality.
32. A 33-year-old woman was found unresponsive and lying on her right arm for an un-
known period of time. She is now awake and complaining of right arm pain. Physical
exam reveals a tense, discolored right upper extremity. There is no pulse and no motor
or sensory function. What is the next step?
(A) Venous duplex ultrasound
(B) CT angiography
(C) Axillary to brachia! artery bypass with greater saphenous vein
(D) Thrombolytics
(E) Upper-extremity compartment release
33. A 21-year-old college cross-country star comes to your clinic with the chief complaint
of aching, burning, and a feeling of fullness in the anterior aspect of her legs below the
knees. She thought she had "shin splints" and took 2 full weeks off training; however,
when she started running again, her symptoms recurred. They occur at 15 minutes into
her run, and subside with rest. Physical exam_. including pulses, was normal apart from
some weakness with dorsiflexion. X-rays showed no fracture. Which of the following
is the diagnostic test of choice?
(A) Bilateral lower-extremity CT angiography with 3-vessel runoff
(B) Angiography with the patient's feet in both dorsiflexion and plantar flexion
(C) MRI with the patient's feet in both dorsiflexion and plantar flexion
(D) Measurement of the anterior compartment pressures both pre- and postexercise
(E) Bilateral ankle-brachia! indices
34. Which ofthe following is associated with the greatest risk for venous thromboembolism?
(A) Medical patient at bed rest
(B) Morbid obesity (BM! >30)
(C) Acute spinal cord injury
(D) Current use of oral contraceptives or hormone replacement therapy
(E) Age 75 years
35. A patient presents after a motor vehicle collision with a cool foot and a posterior knee
dislocation. After reducing the knee dislocation, you note no distal pulses. Operative
exploration reveals injury to the popliteal artery and vein. After repair of the injuries
to the popliteal artery and vein, the right dorsalis pedis and posterior tibial pulses are
now palpable. The next best step in management of this patient is
(A) hourly confirmation of the presence of dorsalis pedis and posterior tibial pulses
in the intensive care unit.
(B) hourly measurement of compartment pressures in the intensive care unit.
(C) 2-compartment fasciotomy of the right calf if compartment pressures are greater
than 35 mm Hg.
(D) 3-compartment fasciotomy of the right thigh.
(E) 4-compartment fasciotomy of the right calf.
CATEGORYS 345
Items 36-39
(A) Acute embolic mesenteric ischemia
(B) Acute thrombotic mesenteric ischemia
(C) Both
(D) Neither
Critiques 1-39
CRITIQUE 1
Of the 3 types of thoracic outlet syndrome-neurogenic, arterial, and venous-neurogenic
comprises more than 95o/o of cases. Most patients with neurogenic thoracic outlet syn-
drome have a history of neck trauma. Evocative diagnostic maneuvers, such as simple arm
raising with head rotation, can elicit pain and paresthesia in many patients, whereas in oth-
ers with arterial symptoms from thoracic outlet syndrome, a palpable pulse at the wrist may
be lost. Most clinicians agree that for patients with neurogenic thoracic outlet syndrome,
conservative measures with physical therapy, ergonomic modifications to the work place,
and avoidance of activity that produces symptoms is indicated. Relief of symptoms may be
obtained in as many as 60-70%.
Venous thoracic outlet syndrome accounts for only 5% of all cases. Effort-related
thrombosis presents acutely as a blue, swollen, heavy, and painful arm. Treatment of the
venous occlusion with thrombolytic therapy is usually successful when administered
promptly. Without decompression of the vein, however, most patients experience a re-
currence of symptoms within 30 days. Most practitioners now recommend decompres-
sion with subclavius tendon division followed by venography 2-3 weeks later to assess
the need for venoplasty.
References
1. Brooke BS, Freischlag JA. Contemporary management of thoracic outlet syndrome.
Curr Opin Cardiol. 2010;25(6):535-540.
2. Sanders RJ, Hammond SL, Rao NM. Thoracic outlet syndrome: a review. Neurologist.
2008; 14(6):365-373.
3. Thompson RW. Challenges in the treatment of thoracic outlet syndrome. Tex Heart Inst
J. 2012;39(6):842-843.
CRITIQUE 2
Mesenteric ischemia remains a complex disease entity with highly variable mortality.
Acute mesenteric ischemia (AMI) carries a high perioperative mortality rate that can ap-
proach 52%, mainly due to ischemic bowel. Even if successfully treated, it can lead to
long-term intestinal malabsorption. Chronic mesenteric ischemia (CM!) is associated with
a history of vascular disease, with nearly half of patients having undergone prior vascular
operation. It is commonly associated with a history of smoking, hypertension, and diabe-
tes. Symptoms classically involve postprandial abdominal pain, fear of food, and uninten-
tional weight loss. CT angiography is used to confirm the diagnosis and to plan for surgical
revascularization.
The vast majority of patients with CMI have significant flow-limiting atherosclerotic
disease in both the superior mesenteric artery and celiac axis. Treatment options include
mesenteric artery endarterectomy, mesenteric artery reimplantation, or bypass grafting. All
of these interventions are associated with a lower operative mortality and incidence of
bowel resection than operative interventions for AMI. Revascularization can be accom-
plished using antegrade or retrograde techniques by bypassing single or multiple vessels
with either vein or prosthetic conduits. Depending on the amount of associated aortic ath-
erosclerotic disease, aortic replacement might be required. Because the infrarenal aorta
typically has a high degree of atherosclerotic burden, the suprarenal aorta or the right iliac
artery is a superior starting point for bypass grafting. The left iliac artery presents a techni-
CATEGORYS 347
cal challenge for such grafting. Neither the left renal artery nor the splenic artery provides
a suitable starting point for a bypass graft.
Endo vascular treatment of CMI is another therapeutic option at centers with the neces-
sary expertise. To date, randomized controlled trial data have failed to demonstrate a su-
periority of endovascular treatment over surgical revascularization for CMI; nonetheless,
treatment-related mortality, which is approximately 10%, does appear to be lower with
endovascular treatment. Restenosis rates approach 50o/o, however, and endovascular treat-
ment is not a viable option for treating nonatherosclerotic causes of CMI in which surgical
revascularization remains the gold standard therapy.
References
I. Valentine RJ, Wind GG. Celiac and mesenteric arteries In: Valentine RJ, Wind GG,
eds. Anatomic Exposures in Vascular Surgery. 2nd ed. Philadelphia, PA: Lippincott
Williams & Wilkins;2003:267-288.
2. Kazmers A. Operative management of acute mesenteric ischemia. Part I. Ann Vase
Surg. 1998; 12(2): 187-197.
3. Foley Ml, Moneta GL, Abou-Zamzam AM Jr, et al. Revascularization of the superior
mesenteric artery alone for treatment of intestinal ischemia. J Vase Surg. 2000;32(1):
37-47.
4. Cappel! MS. Intestinal (mesenteric) vasculopathy. II. lschemic colitis and chronic
mesenteric ischemia. Gastroenterol C/in North Am. I 998;27(4):827-860.
5. Mishkel, G. Treating mesenteric ischemia. Endovascular Today. 2012;02:81-86.
CRITIQUE3
The national trauma databank provides a comprehensive overview of the risk factors for
mortality and limb loss in patients with isolated lower-extremity vascular trauma. Half
of the deaths after isolated vascular trauma involve injury to the common femoral artery,
and more than 80% have injury to the common or superficial femoral arteries. Death is 3
times more frequent in patients with injuries to the common and superficial arteries than
in patients with injuries to the popliteal and tibial arteries. The mortality rate is higher with
penetrating injuries, present in almost 80% of deaths, compared with blunt injuries, which
account for the other 20% of deaths.
The popliteal artery is injured in half of the patients undergoing amputations. Three-
quarters of patients with limb loss have injury isolated to the popliteal or tibial arteries.
Amputation is Mice as frequent with injuries to the popliteal and tibial arteries compared
with the common and superficial femoral arteries. Amputation is also Mice as frequent
with blunt injury compared with penetrating injury. The presence of distal pulses after
blunt injury to the knee does not exclude the presence of an injury to the popliteal artery.
Answer: (C) The amputation rate is higher after injury to the popliteal and tibial arteries
than the common and superficial femoral arteries.
References
I. Kauvar DS, Sarfati MR, Kraiss LW. National trauma databank analysis of mortality
and limb loss in isolated lower extremity vascular trauma. J Vase Surg. 2011;53(6):
1598-1603.
2. Patel KR, Rowe VL. Extremity. In: Cronenwett JL, Johnston KW, Cambria RP, et
al, eds. Rutherford's Vascular Surgery 7th ed. Philadelphia, PA: Saunders Elsevi-
er:2010:2361-2373.
348 CATEGORYS
3. Sise MJ. Shackford SR. Peripheral vascular injury. In: Mattox KL, Moore EE, Feliciano
DV. eds. Trauma. New York, NY: McGraw-Hill;2013:816-847.
4. Rowe VL, Pourrabbani S, Weaver FA. Blunt popliteal artery injuries. In: Stanley JC,
Veith F, Wakefield TW, eds. Current Therapy in Vascular and Endovascular Surgery.
5th ed. Philadelphia, PA: Saunders Elsevier;2014:686-689.
CRITIQUE4
On physical examination, this patient has a penetrating wound with proximity to a major
vascular structure. In this setting, the arterial pressure index (API) may be used as an adjunct
to physical examination for the rapid triage of patients with penetrating trauma but without
hard signs of vascular injury. The arterial pressure index, defined as the Doppler arterial
pressure distal to the site of injury/Doppler arterial pressure in an uninvolved contralateral
extremity, is a noninvasive test that is useful when the physical examination is equivocal.
An AP! greater than 0.9 approaches a 100% predictive value for safe discharge to home.
An AP! less than 0.9 has a sensitivity of 95% and specificity of 97% for the detection of
arterial injury. Patients with an API less than 0.9 should undergo angiography. Screening
patients with measurement of the API and resenring angiography for limbs in which the
AP! is less than 0.9 is safe, accurate, and cost-effective. Hemodynamically normal patients
with lower-extremity gunshot wounds without fracture and an initial AB! of at least 0.9
can be discharged safely from the emergency department without additional diagnostic
imaging, potentially saving healthcare costs.
Noninvasive Doppler pressure monitoring and duplex ultrasonography have a much
lower sensitivity and specificity than measurement of the API in this setting. Although
highly accurate, intra-arterial contrast or CT angiography is costly, invasive, and time-
consuming and therefore should be reserved for patients with penetrating arterial trauma
with an AP! less than 0.9.
References
1. Fox N, Rajani RR, Bokhari F, et al; Eastern Association for the Surgery of Trauma.
Evaluation and management of penetrating lower extremity arterial trauma: an Eastern
Association for the Surgery of Trauma practice management guideline. J Trauma Acute
Care Surg. 2012;73(5 suppl 4):S315-S320.
2. Lynch K, Johansen K. Can Doppler pressure measurement replace "exclusion•· arteriog-
raphy in the diagnosis of occult extremity arterial trauma? Ann Surg. 1991;214(6):737-
741.
3. Johansen K, Lynch K, Paun M, Copass M. Non-invasive vascular tests reliably exclude
occult arterial trauma in injured extremities. J Trauma. 1991 ;31 (4):515-519; discussion
519-522.
4. Sadjadi J, Cureton EL, Dozier KC, Kwan RO, Victorino GP. Expedited treatment of
lower extremity gunshot wounds. J Am Coll Surg. 2009;209(6):740-745.
CRITIQUES
Blunt trauma of sufficient force to cause posterior dislocation of the knee can also cause
significant damage to the popliteal artery. The injury to the popliteal artery typically in-
volves disruption of one or more layers of the arterial wall and extends more than 5 cm in
length. The angiogram in this patient demonstrates complete occlusion of the right popli-
teal artery caused by the posterior dislocation of the right knee.
Given that the etiology of the occlusion of the popliteal artery is due to disruption of
one or more layers of the arterial wall and not thrombus formation alone~ catheter-directed
CATEGORYS
349
References
I. Patel KR, Rowe VL. Extremity. In: Cronenwett JL, Johnston KW, Cambria RP, et
al, eds. Rutheiford's Vascular Surgery. 7th ed. Philadelphia, PA: Saunders Elsevi-
er;20 l 0:2361-2373.
2. Sise MJ, Shackford SR. Peripheral vascular injury. In: Mattox KL, Moore EE, Feliciano
DV, eds. Trauma. New York, NY: McGraw-Hill;2013: 816-847.
3. Rowe VL, Pourrabbani S, Weaver FA. Blunt popliteal artery injuries. In: Stanley JC,
Veith F, Wakefield TW, eds. Current Therapy in Vascular and Endovascular Surgery.
5th ed. Philadelphia, PA: Saunders Elsevier;2014:686--{)89.
CRITIQUE6
Iliofemoral deep vein thromboses (DVTs) carry a worse prognosis than femoropopliteal
DVTs due to their association with long-tertn postthrombotic syndrome, which is related to
residual venous obstruction and valvular reflux. Iliofemoral DVTs are also associated with
at least a 2-fold higher risk of recurrent venous thromboembolism compared with femoro-
popliteal DVTs. Therefore, although anticoagulation alone is adequate for the treatment of
femoropopliteal DVTs, it is not adequate for iliofemoral DVTs.
A strategy of early thrombus removal is advocated for patients who meet the following
criteria for a first episode of acute iliofemoral DVT: symptom duration less than 14 days,
low risk of bleeding, ambulatory, and good functional capacity. Percutaneous techniques,
such as catheter-directed phartnacologic thrombolysis or phartnacomechanical thrombec-
tomy, are advocated as first-line therapy. Systemic anticoagulation or inferior vena cava
filter placement does not provide adequate protection against postthrombotic syndrome.
Systemic thrombolysis is associated with high rates of incomplete thrombolysis as well as
bleeding complications. The risk of bleeding complications in the early postoperative pe-
riod must be weighed against the benefit ofthrombolysis and clot clearance, thus systemic
thrombolysis is relatively contraindicated in the immediate postoperative period. Femoral
venovenous bypass is not indicated for the primary treatment of DVT.
CRITIQUE7
A vascular steal syndrome occurs in up to 10% of patients undergoing dialysis access sur-
gery. Predicting which patients will develop an ischemic steal is difficult, but certain patients
appear to be at higher risk than others. These include women, age at least 60 years, patients
with diabetes, patients with several access procedures on the same extremity, and use of au-
togenous vein or use of the brachia! artery. Resolution of symptoms with digital occlusion of
the fistula re-establishes native flow and confirms the diagnosis of ischemic steal.
Given a nonthreatened hand, a fistulogram will help define the cause of her symptoms.
Although the majority of steal syndromes occur secondary to high flow through the fis-
tula and inadequate collateralization. occasionally, proximal inflow obstruction may be the
cause. The fistulogram will be able to further define the actual etiology of the problem. If
inflow obstruction is present, the ischemic steal can oftentimes be managed by an interven-
tional radiologist without damaging the fistula.
An arteriovenous fistula (whether spontaneous, traumatic, or iatrogenic) creates a high-
flow. low-resistance state leading to a high cardiac output. Although ligation of the fistu la
wi II resolve the steal syndrome, if this is performed, the access is lost, and a new fistula
or graft will need to be placed. Several surgical procedures aim to preserve the fistula.
Outflow narrowing with banding has some success. More recent literature suggests a distal
revascularization with interval ligation (DRILL) procedure (figure 7.1). This procedure
effectively partially bypasses the fistula. preserving access while increasing blood flow to
the affected extremity.
References
l. Schanzer H, Eisenberg D. Management of steal syndrome resulting from dialysis access.
Semin Vase Surg. 2004; I 7( I ):45-49.
2. Mwipatayi BP. Bowles T, Balak.rishnan S, Callaghan J, Haluszkiewicz E, Sieunarine K.
lschemic steal syndrome: a case series and review of current management. Curr Surg.
2006:63(2): 130-135.
CATEGORYS
351
3. Suding PN, Wilson SE. Strategies for management of ischemic steal syndrome. Semin
Vase Surg. 2007;20(3): 184--188.
4. Wixon CL, Hughes JD, Mills JL. Understanding strategies for the treatment ofischemic
steal syndrome after hemodialysis access. J Am Coll Surg. 2000;191(3):301-310.
CRITIQUES
Peripherally inserted central catheters (PICCs) are frequently placed instead of traditional
central venous catheters (CVCs) for patient comfort and presumed safety. Like CVCs,
PICCs are associated with central line-associated bloodstream infections (CLABSis). In
the inpatient setting, the rate of CLABSI by catheter-days is similar between PICCs and
CVCs. Several studies looking only at inpatients failed to show any difference in the rate
of infections between eves and PieCs. Studies comparing outpatients noted a lower in-
fection rate in patients with PICCs compared with CVCs. PICC lines are associated with
higher DVT rates, likely due to their longer length. This is especially true in patients with
a malignant diagnosis.
Although ultrasound is becoming the standard of care for accessing the internal jugular
vein, most subclavian lines are still placed using a traditional landmark approach. Data
show the use of ultrasound to guide the cannulation of the subclavian vein is associated
with increased cannulation rate and decreased complication rates. Either a supraclavicular
or infraclavicular approach can be used.
An insertion bundle or checklist including infection-reducing practices reduces the rate
of CLABSis. These practices include handwashing, using a full barrier precaution, avoid-
ing the femoral site, using a chlorhexidine containing solution to prepare the insertion site,
and assessing daily for whether the line is still needed.
Answer: (E) PICCs are associated with increased catheter-associated venous thrombo-
embolism.
References
I. Chopra V, Anand S, Hickner A, et al. Risk of venous thromboembolism associated with
peripherally inserted central catheters: a systematic review and meta-analysis [published
correction appears in Lancet. 20l3;382(9901):1328]. Lancet. 20 l 3;382(9889):311-325.
2. Marsteller JA, Sexton JB, Hsu YJ, et al. A multicenter, phased, cluster-randomized
controlled trial to reduce central line-associated bloodstream infections in intensive care
units. Crit Care Med 2012;40(11):2933-2939.
3. Chopra V, O'Horo JC, Rogers MA. Maki DG, Safdar N. The risk of bloodstream
infection associated with peripherally inserted central catheters compared with central
venous catheters in adults: a systematic review and meta-analysis. Infect Control Hosp
Epidemiol. 20 l 3;34(9):908-918.
4. Fragou M, Gravvanis A, Dimitriou V, et al. Real-time ultrasound-guided subclavian
vein cannulation versus the landmark method in critical care patients: a prospective
randomized study. Crit Care Med 2011 ;39(7):1607-1612.
CRITIQUE9
This clinical scenario is most consistent with ischemic colitis. In the gastrointestinal tract,
the colon is the most common site of intestinal ischemia. The primary blood supply to the
colon comes from the superior mesenteric artery (SMA) and the inferior mesenteric artery
(IMA). The marginal artery of Drummond travels just adjacent to the colon and provides
collateral circulation between the SMA and the IMA. Compared with other portions of the
gastrointestinal tract, the blood supply to the colon is not as robust and therefore is subject
352 CATEGORY 5
to ischemic events. When the blood flow to the colon is reduced, either for occlusive or
nonocclusive reasons, colonic ischemia may result. Although colonic ischemia can occur
anywhere in the colon, the classic watershed areas considered most at risk are the splenic
flexure (Griffiths point) and the sigmoid colon (Sudeck point). Patients at risk for colonic
ischemia are typically elderly with other associated cardiovascular comorbidities, includ-
ing peripheral vascular disease and coronary artery disease. The disease can be occlusive
in nature. such as from atheroemboli or after vascular surgery with ligation of the IMA, or
nonocclusive. Nonocclusive disease is frequently associated with low flow states, such as
dehydration, sepsis, or cardiac failure.
Clinically~ patients present with the acute onset of abdominal pain, cramping, and
bloody diarrhea. Presentation is very similar to diverticulitis, except for the bloody diar-
rhea, which should prompt the clinician to consider ischemic colitis as the diagnosis. The
blood can be bright red or maroon, depending on the location of the ischemia. With reduced
blood flow, the colonic mucosa is the most susceptible, and the bloody diarrhea is second-
ary to mucosa! injury.
The outcome of patients with ischemic colitis depends on whether the ischemia is limit-
ed to the mucosa or becomes a full-thickness process. Most cases of colonic ischemia result
from a temporary low-flow srate that is restored with supportive measures. The ischemic
lesions are segmental and nongangrenous and will resolve with medical management in
24-48 hours. Approximately 20% of patients, however, have more extensive full-thickness
necrosis. These patients need urgent surgery with resection of the ischemic portion of the
colon. Under these circumstances, associated morbidity and even mortality can be quite
high. A few patients improve with medical management but develop a colonic stricture
several months later that may need surgical resection.
The diagnosis of ischemic colitis is based on clinical criteria as described here. Plain
films may show '"thumbprinting'' associated with mucosa! edema. CT scanning will usu-
ally show thickened colonic walls. A CT finding that is concerning is pneumatosis~ which
may indicate full thickness injury and is often used as an indication for surgical explora-
tion. Colonoscopy maybe a better diagnostic tool. Minimal insufflation should be used.
Mucosa changes will be apparent including edema, friability, and superficial ulceration,
which unfortunately resembles other forms of colitis. Necrotic-appearing mucosa implies
a full-thickness process. Unlike midgut mesenteric ischemia~ colonic ischemia is usually
associated with small vessels. Therefore, angiography is usually not helpful.
Treatment is supportive, including bowel rest and intravenous fluids. Antibiotics are
suggested to prevent bacterial translocation, but this approach is not routine. Steroids are
used to treat inflammatory colitis, such as ulcerative colitis or Crohn colitis, are not used
for ischemic colitis. With this approach, approximately 80% of patients with ischemic coli-
tis will improve rapidly. For patients who present with sepsis or who worsen with support-
ive care, surgery and resection is indicated. Fecal transplant is used to treat chronic and
recurrent Clostridium difficile colitis.
References
1. Washington C, Carmichael JC. Management of ischemic colitis. Clin Colon Rectal
Surg. 2012;25(4):228-235.
2. Beck DE, de Aguilar-Nascimento JE. Surgical management and outcome in acute isch-
emic colitis. Ochsner J. 2011; 11(3):282-285.
CRITIQUE IO
This patient has moderately severe, asymptomatic, carotid artery stenosis. At this level of
disease, appropriate management would be maximal medical therapy, including an anti-
platelet agent. There is no current indication for internal carotid artery stenting or carotid
CATEGORY5 353
endarterectomy. Consensus data suggests that observation alone is suboptimal therapy. Be-
sides anti-platelet agents, best medical therapy should control hypertension and diabetes
and use statins for hyperlipidemia.
Management of this patient with warfarin is not indicated either, because he has not had
any symptoms. The ACAS and ACST trials of the 1990s showed carotid endarterectomy
(CEA) provided significantly better reduction of stroke compared with medical therapy.
The annual risk of stroke in asymptomatic patients with greater than 60o/o stenosis was
found to be 2.2-2.4% in the medical arms of these trials. Since that time, however, interest
and understanding of the medical management of this disease has increased. Current data
suggest best medical therapy has a stroke risk less than 2%.
The CREST-2 trial (Carotid Revascularization and Medical Management for Asymp-
tomatic Carotid Artery Stenosis) is now open and enrolling patients. There are 2 main com-
ponents to this study. Patients who have greaterthan 70% internal carotid artery stenosis by
either angiography or duplex ultrasound are candidates for this trial. They are randomized
to (a) best medical therapy, (b) best medical therapy and CEA, or (c) carotid artery stenting
(CAS). Patients and their physicians will determine the preferred surgical approach, CEA
or CAS, and then patients will be randomized to either intervention or best medical therapy
among those groups. The intent of this trial is to detennine how best medical management
compares with the combination treatments. The trial plans to identify which patients can be
appropriately managed with best medical therapy alone.
References
1. Beckman JA. Management of asymptomatic internal carotid artery stenosis. JAMA.
2013;3l0(15):1612-1618.
2. Jonas DE. Feltner C,Amick HR, et al. Screening for asymptomatic carotid artery steno-
sis: a systematic review and meta-analysis for the U.S. Preventive Services Task Force.
Ann Intern Med 2014;161(5):336-346.
3. Bazan HA, Smith TA, Donovan MJ, Stembergh WC 3rd. Future management of carotid
stenos is: role of urgent carotid interventions in the acutely symptomatic carotid patient
and best medical therapy for asymptomatic carotid disease. Ochsner J 2014; 14(4):608-
615.
4. Brott TG. Carotid revascularization and medical management for asymptomatic carot-
id stenosis trial (CREST-2). Clinica/Trials.gov. Updated: Dec 30, 2015. Available at:
https://clinicaltrials.gov/ct2/show/NCT020892 l 7
CRITIQUE 11
The optimal treatment for a patient with asymptomatic critical stenosis of a distal anasto-
mosis found on duplex surveillance is percutaneous transluminal angioplasty (PTA). There
is debate regarding PTA versus open surgical revision, which was the gold standard for
years. A study from The Netherlands showed that, with a surveillance program and se-
lective use of PTA, assisted primary and secondary patency rates were well above 80%.
Furthermore, 28 of 43 bypasses after PTA intervention remained fully patent and free of
restenosis. In addition, the perioperative complication rate for PTA is significantly less
than that for open surgical revision. An exercise program would have a limited role in this
patient. Clopidogrel would not provide any significant benefit in controlling the risk of
graft failure, and additional bypass grafting is excessive and unnecessary in this patient.
An exercise program is helpful for all patients with pulmonary veno-occlusive disease, but
will not specifically help an anastomotic stenosis.
References
I. Baumann F, Engelberger RP, Makaloski V, Do DD, Baumgartner I, Diehm N. Single-
center experience in endovascular treatment for infrainguinal bypass obstructions. J
Vase lnterv Radial. 2012;23(8): 1055~1062.
2. van Oostenbrugge TJ, de Vries JP, Berger P, et al. Outcome of endovascular rein-
tervention for significant stenosis at infrainguinal bypass anastomoses. J Vase Surg.
2014;60(3):696-701.
CRITIQUE 12
This patient exhibits signs of chronic mesenteric ischernia complicated by sudden onset
of symptoms of acute mesenteric ischernia. His history of atherosclerotic disease mani-
fested by his endarterectomy and symptoms of chronic mesenteric ischemia make acute
mesenteric thrombotic disease a likely etiology. Mesenteric embolic disease would also
be in the differential diagnosis, but this is typically associated with cardiac arrhythmias
and history of myocardial infarction. In addition, it does not have a high association with
chronic abdominal pain, because the source of the ischemia is an embolus and not chronic
atherosclerotic disease.
Mesenteric venous thrombosis is typically a disease of slower onset with more insidi-
ous symptoms, associated with hypercoagulable states. This patient's x-ray, with a large
amount of large bowel air, is not typical of a small bowel obstruction. Nonocclusive mes-
enteric ischemia, which results in a low-flow state, is also possible in this patient. However,
there is no history of an inciting event causing volume depletion or a low flow state. Non-
occlusive mesenteric disease also tends to be associated with a prodromal acute illness and
has a high association with the use of vasopressors.
References
I. Lin PH, Poi MJ, Matos J, Kougias P, Bechara C, Chen C. Arterial disease. In: Brunicardi
FC, Andersen DK, Billiar TR, et al, eds. Schwartz's Principles a/Surgery. 10th ed. New
York, NY: McGraw-Hill;2014:827-914.
2. Hall JF, Ferguson CM. Mesenteric vascular disease of the small bowel. In: Cameron JL,
ed. Current surgical therapy. 9th ed. Maryland Heights, MO:Mosby;2008: 139-142.
3. Herbert GS, Steele SR. Acute and chronic mesenteric ischemia. Surg Clin North Am.
2007;87(5):1115-1134.
CRITIQUE 13
Deep vein thrombosis occurs in approximately 1 in 1000 pregnancies, which is 4 times
the rate of the nonpregnant population. The leading cause of maternal death in the United
States and Europe is pulmonary embolism. The 2012 American College of Chest Phy-
sicians Guidelines recommended treatment for deep vein thrombosis in the pregnant
patient is low molecular weight heparin for the duration of the pregnancy and for at least
6 weeks postpartum.
Because of its teratogenic effects, warfarin should be avoided during pregnancy. There
is little clinical experience with Factor Xa inhibitors, and they are not recommended for
this scenario at this time. In the setting of pregnancy, thrombolytic therapy should be re-
served for life-threatening thromboembolism. There is no indication for vena caval filter
placement for this patient.
Answer: (E) Subcutaneous low molecular weight heparin for the duration of pregnancy
CATEGORYS
355
References
1. Marik PE, Plante LA. Venous thromboembolic disease and pregnancy. N Engl J Med
2008;359(19):2025-2033.
2. Bates SM, Greer IA, Middeldorp S, Veenstra DL, Prabulos AM, Vandvik PO: American
College of Chest Physicians. VTE, thrombophilia, antithrombotic therapy, and preg-
nancy: antithrombotic therapy and prevention of thrombosis, 9th ed: American College
of Chest Physicians Evidence-Based Clinical Practice Guidelines. Chest. 2012; 141 (2
suppl):e69 l S-736S.
CRITIQUE 14
Permanent and retrievable inferior vena caval (IVC) filters are overused. In the setting of
deep vein thrombosis (DVT), IVC filter placement should be reserved for the patient with
absolute contraindications to anticoagulation or in the patient with progressive disease.
such as pulmonary embolism despite adequate anticoagulation.
The rate of pulmonary embolism after IVC filter placement is 0.7-4%. The risk of!VC
thrombosis after IVC filter placement is 2.8% in pooled data. Patients with IVC filters
consistently have a significantly increased incidence of subsequent DVT compared with
patients treated with anticoagulation only. The PREPIC study recommended warfarin anti-
coagulation with or without filter placement. However, recurrent DVT occurred in 36°/o of
patients with a filter despite anticoagulation. By contrast, 28% of patients with anticoagu-
lation alone had recurrent DVT. Although retrievable IVC filters have gained in popularity
in the United States, nearly two-thirds remain in place I year after placement.
Clot location is not an indication for IVC filter placement, including an ileofemoral
thrombosis.
Answer: (C) They are associated with an increased incidence of recurrent deep vein
thrombosis compared with anticoagulation alone.
References
I. PREPIC Study Group. Eight-year follow-up of patients with permanent vena cava filters
in the prevention of pulmonary embolism: the PREPIC (Prevention du Risque d'Embolie
Pulmonaire par Interruption Cave) randomized study. Circulation. 2005;112(3):416-422.
2. Bates SM, Greer IA, Middeldorp S, Veenstra DL, PrabulosAM, Vandvik PO; American
College of Chest Physicians. VTE, thrombophilia, antithrombotic therapy, and preg-
nancy: anti thrombotic therapy and prevention of thrombosis, 9th ed: American College
of Chest Physicians Evidence-Based Clinical Practice Guidelines. Chest. 2012; 141 (2
suppl):e691 S-736S.
3. Angel LF, Tapson V, Galgon RE, Restrepo Ml, Kaufinan J. Systematic review of the use
of retrievable inferior vena cava filters. J Vase Interv Radial. 2011 ;22(11):l522-l 530.e3.
CRITIQUE 15
True aneurysms develop in 5-10% of autogenous upper-extremity hemodialysis fistulae
(figure 15.1). Although not precisely defined in the literature, most practitioners consider a
vein segment greater than 2 cm in diameter or 3 times the diameter of the adjacent normal
vein to be aneurysmal. Years of repetitive clustered punctures is likely a contributing
factor in the development of these aneurysms; however, they may also occur in mature
fistulae that have never been punctured. Spontaneous rupture of these aneurysms has not
been described. Rupture or distal embolization could be induced by puncture and should
avoided; opt instead to puncture more normal segments of the fistulae.
356 CATEGORYS
1 f1J.,J,~-1 L
, ti/
• 11,/, 1, 111 ,/o
References
I. Pasklinsky G, Meisner RJ, Labropoulos N, et al. Management of true aneurysms of
hemodialysis access fistulas. J Vase Surg. 2011;53(5):1291- 1297.
2. Hossny A. Partial aneurysmectomy for salvage of autogenous arteriovenous fistula with
complicated venous aneurysms. J Vase Surg. 20 14;59(4): 1073-1077.
CRITIQUE 16
Central venous catheters are commonly left in place for months in patients with malignan-
cy. Thrombosis and phlebitis are complications of these catheters. Studies report clinically
significant upper-extremity deep vein thrombosis (DVT) as well as phlebitis. A few studies
have investigated thrombosis with duplex imaging.
Peripherally inserted catheters, especially those of larger diameter, are more frequent-
ly associated with venous thrombosis compared with central venous lines, which enter
through the subclavian or internal jugular veins. Venous thrombosis is also significantly
CATEGORY5 357
more likely when the tip of the catheter is positioned proximal to the superior vena cava
and in patients with a prior history of DVT. Increased duration of catheter placement is
associated with increased rates of infection but does not seem to be associated with throm-
bosis. Most upper-extremity DVTs occur within the first 10-14 days after insertion. Under-
lying conditions may have more to do with thrombosis, including bacteremia, which was
associated with a 70% rate of thrombosis.
Catheter materials vary in thrombogenicity, with polyethylene being the most thrombo-
genic. Polyethylene has a higher rate of thrombosis than polyurethane and silicone. Poly-
urethane is the most common material used for central venous catheters. Silicone is used
for many tunneled central lines and is the least thrombogenic. Despite the catheter mate-
rial, there is universal agreement that all catheters have fibrin deposition on them forming
a fibrin sheath. The entire length of a venous implanted catheter will be encased in fibrin
within 5-7 days of placement. The significance of fibrin sheath formation is unclear, al-
though some regard it as a prothrombotic state.
Phlebitis is a complication of peripherally inserted central catheter lines (PICCs) and
not reported as a common complication of centrally placed lines. Reported rates are con-
founded by a lack ofa standard definition, as silicone PICCs have rates reported as 4-36%.
Prospective reports suggest rates of 4-6%, with catheter removal being necessary in 6o/o
of patients.
References
1. Saber W, Moua T, Williams EC, et al. Risk factors for catheter-related thrombosis (CRT)
in cancer patients: a patient-level data (!PD) meta-analysis of clinical trials and prospec-
tive studies. J Thromb Haemost. 201! ;9(2):312-319.
2. Trerotola SO, Stavropoulos SW, Mondschein JI, et al. Triple-lumen peripherally insert-
ed central catheter in patients in the critical care unit: prospective evaluation. Radiology.
2010;256(1):312-320.
3. Chopra V, Ratz D, Kuhn L, Lopus T, Lee A, Krein S. Peripherally inserted central
catheter-related deep vein thrombosis: contemporary patterns and predictors. J Thromb
Haemost. 2014;12(6):847-854.
4. Turcotte S, Dube S, Beauchamp G. Peripherally inserted central venous catheters are
not superior to central venous catheters in the acute care of surgical patients on the ward.
WorldJ Surg. 2006:30(8):1605-1619.
5. Crowley AL, Peterson GE, Benjamin DK Jr, et al. Venous thrombosis in patients with
short- and long-term central venous catheter-associated Staphylococcus aureus bactere-
mia. Crit Care Med. 2008;36(2):385-390.
CRITIQUE 17
Venous stasis ulcers are relatively common, with a prevalence of 1.5 in 1000. Prevalence
increases with advancing age and in women. These ulcers are typically a chronic recurring
condition. Compression bandages have been used since the 17th century, with elastic
bandages being produced in the 19th century, and Unna boots used in the 21st century. In
a systematic review of 48 randomized controlled trials comparing compression dressings
with no compression, compression-particularly with dressings containing more layers of
elastic constituents-was the most effective method of increasing healing rates for venous
leg ulcers.
Current evidence does not support the routine use of systemic antibiotics. In an analysis
of 5 randomized controlled studies involving 233 patients, the use of systemic antibiotics
provided no benefit in wound healing. Similarly, there was no benefit with the use of topi-
cal antibiotics.
358 CATEGORY 5
Twelve randomized controlled trials with 1514 subjects, investigated the use ofsilver-
impregnated dressings but found no benefit with respect to wound healing (e.g., 15 vs 16
weeks). The use of honey-based products does not improve wound healing and increases
the number of adverse events.
Because this ulcer is the result of venous disease, an arterial bypass would be indicated
only if the patient had evidence of inadequate arterial inflow.
References
1. O'Meara S, Cullum N, Nelson EA, Dumville JC. Compression for venous leg ulcers.
Cochrane Database Syst Rev. 2012; 11 :CD000265.
2. O'Meara S, Al-Kurdi D, Ologun Y, Ovington LG, Martyn-St James M, Richard-
son R. Antibiotics and antiseptics for venous leg ulcers [published update appears
in Cochrane Database Syst Rev. 2014;l:CD003557]. Cochrane Database Syst Rev.
2013; 12:CD003557.
CRITIQUE 18
In patients with impending dialysis due to chronic renal disease, creation of an autogenous
arteriovenous (AV) fistula should be performed immediately to allow adequate time for the
fistula to mature. However, if the quality of the arterial and venous anatomy is poor and a
prosthetic AV fistula is required, this procedure should be delayed until just before the need
for chronic dialysis. In general, new AV fistulas in the upper extremity should be placed
as distal as possible, saving more proximal sites for future need. Upper-extremity sites,
particularly the nondominant arm, should be used before lower-extremity sites.
Clinical assessment of AV access requires experience in interpreting visual, auditory,
and tactile clues to impending AV access dysfunction. A normal autogenous access has a
soft pulse, is easily compressible, has a continuous low-pitched bruit, and has a palpable
thrill extended along the venous outflow. In experienced hands, physical exam is very ef-
fective and predictive for stenosis of the access in 80% of patients. However~ measurement
of access blood flow is the best determinant of access function. A decrease of more than
25% flow from baseline has a high predictive value for significant stenosis (87-100%).
Static dialysis venous pressure is useful only for prosthetic access, but the absolute
measurement depends on multiple factors, such as systolic blood pressure. Although the
absolute measurement is only of limited value, a trend demonstrating increased pressures
may be indicative of stenosis. Duplex ultrasound may be used for the early identification
of impending AV access failure and may allow for intervention before failure but is highly
operator dependent.
Answer: (B) Prosthetic access should be delayed until just before the need for dialysis.
References
1. Sidawy AN, Spergel LM, Besarab A, et al; Society for Vascular Surgery. The Society
for Vascular Surgery: clinical practice guidelines for the surgical placement and mainte-
nance of arteriovenous hemodialysis access. J Vase Surg. 2008;48(5 suppl):2S-25S.
2. Murad MH, Elamin MB, Sidawy AN, et al. Autogenous versus prosthetic vascular
access for hemodialysis: a systematic review and meta-analysis. J Vase Surg. 2008;48(5
suppl):34S-4 7S.
CATEGORY5 359
CRITIQUE 19
Factors other than degree of internal carotid artery stenosis may be critical in the produc-
tion of cerebrovascular symptoms. Carotid plaques cause cerebrovascular events through
either reduction in blood flow or embolization due to plaque disruption. Plaque morphol-
ogy may be as critical as severe stenosis in producing cerebrovascular events and therefore
may be important in selecting patients for surgical treatment to prevent stroke.
Plaque morphology is classified into heterogeneous or homogeneous based on echo-
genicity determined by ultrasound or magnetic resonance angiography (MRA). MRA is
better than ultrasound in determining plaque morphology, but duplex ultrasound is still the
best screening test for carotid stenosis. Angiography is no longer necessary before carotid
endarterectomy if duplex ultrasound demonstrates a significant stenosis.
Heterogeneous plaques consist of a mixture ofhypoechoic, isoechoic, and hyperechoic
lesions, whereas homogenous consist of one type. On pathologic examination, heteroge-
neous plaques correlate with intraplaque hemorrhage, ulceration, loose stroma-containing
lipids, cholesterol, and proteinaceous deposits. Heterogeneous plaques are associated with
cerebrovascular events. In addition to causing adverse neurologic events, heterogeneous
plaques are associated with higher degrees of stenosis of the internal carotid artery. Hetero-
geneous plaques were found in only 17% of patients with less than 59o/o stenosis compared
with 80% in those with more than 70% stenosis.
The sensitivity and specificity for diagnosing high-grade stenosis (70-99%) with MRA
or ultrasound are essentially identical, 88% and 84%, respectively. However, MRA over-
estimates carotid stenosis.
References
!. AbuRahma AF, Wulu JT Jr, Crotty B. Carotid plaque ultrasonic heterogeneity and
severity of stenosis. Stroke. 2002;33(7): 1772-1775.
2. Hatsukami TS, Ross R, Polissar NL, Yuan C. Visualization of fibrous cap thickness and
rupture in human atherosclerotic carotid plaque in vivo with high-resolution magnetic
resonance imaging. Circulation. 2000; 102(9):959-964.
3. Ricotta JJ, Aburahma A, Ascher E, Eskandari M, Faries P, Lal BK; Society for Vascular
Surgery. Updated Society for Vascular Surgery guidelines for management of extracra-
nial carotid disease [published correction appears in J Vase Surg. 2012;55(3):894]. J
Vase Surg. 2011;54(3):e!-e3 l.
CRITIQUE20
Endo vascular techniques for injury to the thoracic aorta have significantly decreased major
complications associated with open repair. Stroke and spinal cord ischemia can still occur,
and specific complications of thoracic endovascular aortic repair (TEVAR) are related to
covering the left subclavian artery when necessary.
Up to 40% of patients undergoing TEVAR have injury near the origin of the left subcla-
vian artery, which requires covering the orifice of the left subclavian artery while placing
the TEVAR graft. The left subclavian artery provides blood supply to the upper spinal cord
through the vertebral artery connection to the anterior spinal artery. Coverage of the orifice
of the left subclavian artery may result in paraplegia.
The left subclavian artery also provides blood flow to the left arm. Therefore, the left
arm rather than the right should be monitored for ischemia. The left subclavian artery pro-
vides blood flow to the left vertebral artery, and covering its orifice may result in reversal
of blood flow in the left vertebral, thereby stealing blood flow from the posterior circula-
tion. When this occurs, vertebrobasilar ischemia can follow, manifesting as syncope, diplo-
pia, or vertigo. Published reports show that when the left subclavian artery is covered, 6%
360 CATEGORY5
Answer: (B) Covering the orifice of the left subclavian artery may result in vertebro-
basilar ischemia.
References
I. Matsumura JS, Lee WA, Mitchell RS, et al; Society for Vascular Surgery. The Society
for Vascular Surgery practice guidelines: management of the left subclavian artery with
thoracic endovascular aortic repair. J Vase Surg. 2009;50(5): 1155- 11 58.
2. Matsumura JS, Cambria RP, Dake MD, Moore RD, Svensson LG, Snyder S; TX2 Clini-
cal Trial Investigators. International controlled clinical trial of thoracic endovascular
aneurysm repair with the Zenith TX2 endovascular graft: 1-year resu lts. J Vase Surg.
2008 Feb;47(2):247-257; discussion 257.
CRITIQUE 21
The contrast-enhanced CT scan shows a clot in the mesenteric veins (figure 21.3). This
imaging and the patient's symptoms suggest that be has mesenteric venous ischemia. This
condition has a slower course than mesenteric arterial ischemia, and the diagnosis is typi-
cally delayed, which often leads to extensive intestinal infarction. Cirrhosis and a throm-
botic disorder-such as Protein C, S, or anti-thrombin III deficiency or Factor V Leiden-
are predisposing factors. Hematologic workup for a prothrombotic condition is indicated.
References
1. Brandt LJ, Boley SJ. AGA technical review on intestinal ischemia. American Gastroin-
testinal Association. Gastroenterology. 2000;118(5):954-968.
2. Hamik JG, Brandt LJ. Mesenteric venous thrombosis. Vase Med 2010;15(5):407-418.
3. Kumar S, Sarr MG, Karnath PS. Mesenteric venous thrombosis. N Engl J Med
2001;345(23):1683-1688.
4. Zhou W, Choi L, Lin PH, Dardik A, Eraso A, Lumsden AB. Percutaneous transhepat-
ic thrombectomy and pharmacologic thrombolysis of mesenteric venous thrombosis.
Vascular. 2007:15(1 ):41-45.
CRITIQUE22
Postthrombotic syndrome (PTS) occurs after deep venous thrombosis (DVT) destroys the
competency of the deep venous valvular system in the leg. PTS causes reflux and hyperten-
sion in the venous system of the leg on standing. Patients complain ofa swollen, heavy, and
painful leg, which is worse with standing or walking. Skin changes may occur months later
with "bronzing" and hyperpigmentation, thinning of the skin, and venous skin ulcers. The
risk of PTS after DVT is 10-50%. The risk ofulceration in PTS is approximately 5-10%.
Although external compression stockings are used to help manage established PTS,
they do not prevent PTS after DVT diagnosis. PTS affects younger adults as well as the
elderly, with a mean age of 56. PTS significantly degrades quality of life and ability to
work for years. The economic burden of PTS in the United States may be as high as $200
million annually. Therapy for PTS relies on anticoagulation for up to 6 months after in-
jury and extended use of compression stockings. Current recommendations suggest com-
pression stockings should be worn for at least 24 months. Primary prevention is key and
accomplished with protocol-driven DVT prophylaxis regimens for appropriate patients.
Catheter-directed thrombolysis and endovascular approaches to acute DVT within 14 days
of occurrence may decrease the risk of later development of PTS.
Answer: (B) This patient should wear a graduated compression garment on his leg for
24 months.
References
1. Kahn SR, Comerota AJ, Cushman M, et al; American Heart Association Council on
Peripheral Vascular Disease, Council on Clinical Cardiology, and Council on Cardio-
vascular and Stroke Nursing. The postthrombotic syndrome: evidence-based preven-
tion, diagnosis, and treatment strategies: a scientific statement from the American Heart
362 CATEGORY5
CRJTIQUE23
The arterial pressure index (API), also called the ankle-brachia! index (ABI) or brachial-
brachial index (BBi), depending on which arterial pressures are being compared, is a useful
way to screen for arterial injuries. An API of less than 0.9 indicates that an injury is likely
and farther investigation is required. However, a nonnal API greater than 0.9 does not
exclude injury, because some injuries, such as intimal flaps or pseudoaneurysrns, may still
be found on imaging. Approximately l 0% of these will not require further intervention.
Venous injuries are not detected by APL Vascular injuries proximal to the axilla and groin
are also not detected by an abnormal API secondary to the excellent collateral blood flow.
CT angiography (CTA) has largely replaced traditional angiography in trauma centers.
It is faster, less expensive, and provides equivalent sensitivity and specificity in the de-
tection of arterial injuries. Duplex ultrasonography is noninvasive and can show many
peripheral injuries, but it is less sensitive than CTA or angiography and may not see the
subclavian vessels well due to the presence of the clavicle.
Hemodynamicall y nonnal patients may be candidates for endovascular approaches. Pa-
tients who are in shock with subclavian artery injuries should be taken to the operating
room. Proximal control of the injured subclavian artery can be achieved via several tech-
niques, but a posterolateral thoracotomy is not one of them. If control is sought close to the
origin of the vessel, partial or complete median sternotomy may be required. Anterolateral
thoracotomy is used to reach the proximal left subclavian. A supraclavicular incision and
approach may be possible, especially in nonobese individuals, on the right side or for distal
injuries. Rapid removal of the head of the clavicle by disarticulation of the clavicular head
and cutting the mid clavicle gives good exposure ofthe subclavian artery (figure 23.1).
Ref erences
1. Seamon MJ, Smoger D, Torres OM, et al. A prospective validation of a current prac-
tice: the detection of extremity vascular injury with CT angiography. J Trauma.
2009;67(2):238-243 ; discussion 243- 244.
2. Buscaglia LC, Walsh JC, Wilson JD, Matolo NM. Surgical management of subclavian
artery injury. Am J Surg. 1987;154(1 ):88-92.
3. Feliciano DY. Management of peripheral arterial injury. Curr Opin Crit Care.
201O; 16(6):602-608.
CRITIQUE 24
Hemorrhage is a life-threatening complication of severe acute pancreatitis and warrants
urgent management. The diagnosis is clear from the arterial phase of the CT scan (fig-
ure 24.2). In the setting of instability and acute pancreatitis, it is ill advised to attempt
to address this with laparotomy. Operative access to the gastroduodenal artery in acute
pancreatitis would be challenging, and such an approach is reserved for failure of other
options. The location and nature of bleeding in this setting is best managed with trans-
arterial embolization.
This patient is not a candidate for endoscopic coagulation because the bleeding is ex-
traluminal. Nonvariceal luminal upper gastrointestinal bleeding is a relatively heteroge-
neous group of diagnoses (gastric and duodenal ulcers, small-bowel diverticula, jejuna!
ulcer, Dieulafoy lesions, and pancreatic hemorrhage). Studies of bleeding duodenal ulcers
refractory to endoscopic intervention reveal that transarterial embolization carries a higher
re bleed rate compared with surgery (odds ratio = 0. 77 vs. 0.41 , p <.O I) but no difference
in mortality.
Somatostatin is used to treat variceal bleeding and has the physiological benefit oflow-
ering pancreatic exocrine secretions as well as splancbnic venous blood pressure. This is an
arterial bleed, and somatostatin would have no effect on the acute event.
364 CATEGORY5
References
I. Jairath V, Kahan BC, Logan RF, et al. National audit of the use of surgery and radiologi-
cal embolization after failed endoscopic haemostasis for non-variceal upper gastrointes-
tinal bleeding. Br J Surg. 2012;99(12): 1672-1680.
2. Beggs AD, Dilworth MP, Powell SL, Atherton H, Griffiths EA. A systematic review of
transarterial embolization versus emergency surgery in treatment of major nonvariceal
upper gastrointestinal bleeding. Clin Exp Gastroenterol. 2014;7:93-104.
CRITIQUE25
An aberrant right subclavian artery (ARSA) is the most common vascular ring abnormal-
ity, with a reported incidence of0.5-2%. Typically, the aberrant vessel is the last great ves-
sel on the aortic arch and originates posteriorly, distal to the left subclavian artery. To reach
the right thoracic outlet, the ARSA usually travels posterior to the esophagus.
Only 10-20% of adult patients with ARSA develop symptoms. Most commonly, the
symptoms are due to compression of the esophagus, and the affected patients complain of
dysphagia. This condition is often referred to as dysphagia lusoria based on the Latin term
lusus naturae, meaning '"freak of nature."
Adults typically complain of dysphagia for solid food, often associated with chest pain
and regurgitation of chewed, but undigested, food. Very rarely, adults present with rupture
of an aneurysmal ARSA. In infants, ARSA is associated with respiratory symptoms, par-
ticularly strider, due to the absence of tracheal rigidity in this age group. Once patients with
ARSA reach adulthood, it is extremely rare for them to present with respiratory symptoms.
Right upper-extremity claudication is a symptom of subclavian stenosis, either due to
atherosclerosis or thoracic outlet syndrome. This symptom is not associated with ARSA.
Subclavian steal syndrome is a constellation of symptoms arising from an occlusion or
high-grade stenosis in the proximal subclavian artery. This condition mandates that upper-
extremity circulation be supplied by retrograde blood flow through the vertebral artery
filling the subclavian. In patients with this condition, vigorous use of the upper extremity
can "'steal" optimal blood flow away from the vertebrobasilar cerebral circulation, resulting
in symptoms such as dizziness, syncope, vertigo, and visual disturbances.
References
I. Epstein DA, Debord JR. Abnormalities associated with aberrant right subclavian arter-
ies-a case report. Vase Endovascular Surg. 2002;36(4):297-303.
2. Levitt B, Richter JE. Dysphagia lusoria: a comprehensive review. Dis Esophagus.
2007;20(6):455-460.
CRITIQUE26
For patients with a nondisabling acute neurologic event, surgical revascularization with
carotid endarterectomy was controversial because of concerns about reperfusion injury or
worsening ischemia with arterial clamping. This controversy led many surgeons to believe
that a period of recovery was warranted to prevent both conversion to hemorrhagic stroke
CATEGORYS
365
and worsening ischemic stroke. In the 1990s, the North American Symptomatic Carotid
Endarterectomy Trial (NASCET) found no difference in perioperative stroke rates for pa-
tients operated on early (<30 days) versus late (>30 days), with stroke rates of 4.8% and
5.2%, respectively. Despite this finding, few patients received urgent endarterectomy.
In an international multicenter trial, patients with carotid artery stenosis greater than
70o/o who were within 2 weeks of acute neurologic event received carotid artery endar-
terectomy with a composite event rate of nonfatal stroke, nonfatal myocardial infarction,
or death of 5.5%. The overall incidence of periprocedural stroke, including hemorrhagic
stroke, increased from patients without acute preprocedure neurologic events, but this in-
crease was small (event rate of 4.8%). There was no difference in stroke rate for patients
who received an urgent versus delayed (>14 days) carotid endarterectomy. Urgent endar-
terectomy is now recommended for management of patients with acute stroke and carotid
disease as part of the American Heart Association and American Academy of Neurology
guidelines.
Immediate treatment of acute stroke patients with catheter-directed thrombolysis fur-
ther complicates decision making because of concerns about bleeding. Prospective studies
show that endarterectomy can safely be performed within the acute period after thrombo-
lytic therapy without major complications.
References
1. Thapar A, Jenkins IH, Mehta A, Davies AH. Diagnosis and management of carot-
id atherosclerosis [published correction appears in BMJ. 2013;346:12420]. BM!.
2013;346:f!485.
2. Ishishita Y, Toshikazu K, Yamaoka Y, Morita A. Surgical revascularization during the
acute phase of stroke caused by near occlusion of the cervical internal carotid artery.
Neurosurg Quart. 20l5;25(2):142-144.
3. Tsivgoulis G, Krogias C. Georgiadis GS, et al. Safety of early endarterectomy in
patients with symptomatic carotid artery stenosis: an international multicenter study.
EurJ Neural. 2014;21(10):1251-1257.
4. Crozier JE, Reid J, Welch GH, Muir KW, Stuart WP. Early carotid endarterectomy follow-
ing thrombolysis in the hyperacute treatment of stroke. Br J Surg. 2011 ;98(2):235-238.
CRITIQUE27
Claudication in patients with peripheral arterial disease presents as exertional pain in the
affected extremity. The majority of patients (>60%) have no progression of symptoms in
the 5 years after initial presentation. Limb loss or the need for revascularization occurs in
few patients (<10%). The 5-year risk of nonfatal stroke and myocardial infarction is 20%
and mortality is 30%. Risk factor modification in these patients is of critical importance
and includes smoking cessation, exercise, diet and weight control, blood pressure control,
modification oflow-density lipoprotein (LDL) and cholesterol levels, antiplatelet therapy,
angiotensin-converting enzyme inhibitor therapy, and glycemic control for diabetic patients
according to American College of Cardiology and American Heart Association guidelines.
The Adult Treatment Panel for the National Cholesterol Education Program equates
peripheral arterial disease to established coronary disease. Aggressive LDL and cho-
lesterol lowering should be mandatory in these patients. The Antiplatelet Trialists col-
laboration summarized all existing trials of aspirin therapy and found a 23% reduction
in serious vascular events for patients treated with antiplatelet therapy. Clopidogrel, a
second-generation thienopyridine derivative, has antiplatelet effects that show a 24%
composite risk reduction compared with aspirin in the Clopidogrel Versus Aspirin in
Patients at Risk of Ischemic Events (CAPRIE) study. It may provide some additional
366 CATEGORYS
benefit to patients with peripheral arterial disease compared with aspirin monotherapy.
Clopidogrel has no effect on walking distance.
References
I. Stoyioglou A, JaffMR. Medical treatment of peripheral arterial disease: a comprehen-
sive review. J Vase Interv Radio/. 2004;15(1l):1197-1207.
2. Friedell ML, Stark KR, Kujath SW, Carter RR. Current status oflower-extremity revas-
cularization. Curr Prob/ Surg. 2014;51(6):254-290.
3. Wennberg PW. Approach to the patient with peripheral arterial disease. Circulation.
2013; 128(20):2241-2450.
4. Knepper JP, Henke PK. Diagnosis, prevention, and treatment of claudication. Surg Clin
North Am. 2013;93(4):779-788.
CRITIQUE28
Nontraumatic acute arterial occlusion is usually embolic or thrombotic in etiology. Thera-
peutic decisions will depend on the etiology. Open surgical embolectomy or percutaneous
embolectomy with an aspiration catheter in relatively healthy arteries can restore perfusion
quickly. Acute thrombosis in arteries with atherosclerotic disease poses a greater challenge
and is associated with limb loss in 10-25% of cases. All patients in whom thrombosis
is suspected should initially be heparinized. Percutaneous catheter-directed thrombolysis
may be considered when the extremity is determined to be tolerant of the time of continued
ischemia required while the thrombolytic drugs are administered. All patients receiving
thrombolytic therapy should also receive heparin. Systemic heparinization is not a contra-
indication to thrombolysis.
In the STILE trial, 28% ofthrombolysis attempts were unsuccessful. Other series have
documented failure to restore patency rates ranging from 22-45o/o due to poor catheter
placement or extent of disease. The STILE study revealed a statistically significant dif-
ference in favor of operative intervention, with more patients experiencing death, major
amputation, ongoing ischemia, and major morbidity in the thrombolytic group. Secondary
analysis that stratified for duration of ischemia found an improved amputation-free sur-
vival and shorter hospital stay in patients with acute ischemia (<14 days) with thrombolysis
but better results with operative intervention for patients with acute on chronic ischemia
of longer duration.
Subsequent trials (TOPAS-1, TOPAS-11) showed equivalent results for operative
intervention and thrombolysis and failed to demonstrate a definitive answer as to which
approach is better. Amputation rates are statistically similar, and observed mortality rates
are complicated by myocardial infarction associated with operative intervention and the
increased bleeding complications or cerebrovascular events associated with thrombolysis.
Data from the Swedish Vascular Registry documented a 3-fold increase in risk of stroke in
patients treated with thrombolytic therapy.
References
1. Nordanstig J, Smidfelt K., LangenskiOld M, Kragsterman B. Nationwide experience of
cardio- and cerebrovascular complications during infrainguinal endovascular interven-
tion for peripheral arterial disease and acute limb ischaemia. Eur J Vase Endovasc Surg.
2013;45(3):270-274.
2. Clagett GP, Sobel M, Jackson MR, Lip GY, Tangelder M, Verhaeghe R. Antithrombotic
therapy in peripheral arterial occlusive disease: the Seventh ACCP Conference on Anti-
thrombotic and Thrombolytic Therapy. Chest. 2004;126(3 suppl):609S-626S.
CATEGORY5
367
CRITIQUE29
The retroperitoneal exposure of the iliac fossa is used for arterial bypass, excision of tumors,
renal transplantation, and abscess drainage. In this approach, the peritoneum is retracted
medially by blunt dissection. The ureter is elevated with the peritoneum. Dissection of the
length of the ureter is not necessary and should be avoided, because it may devascularize
the segmental blood supply. The iliac vessels are anterior to the psoas muscle. The
genitofemoral nerve is posterior to the area of dissection and lies on the surface of the
psoas muscle. Medial and posterior to the psoas is the sympathetic plexus. The vasculature
is anterior to these nerves, so division of the plexus is not required for exposure. The
gonadal artery and vein are anterior to the ureter and are seen near the level of the origin
of the hypogastric artery and vein. The gonadal vessels then course parallel to and anterior
to the external iliac artery. The right external iliac vein is posterior and lateral to the right
external iliac artery. The left external iliac vein is posterior and medial to the left external
iliac artery. The iliolumbar vein may actually be as many as 4 separate posterior branches
extending posterior and laterally from the proximal external iliac vein. Mobilization of the
artery does not require mobilization of these branches.
References
1. Mirilas P, Skandalakis JE. Surgical anatomy of the retroperitoneal spaces, Part V: surgi-
cal applications and complications. Am Surg. 2010;76(4):358-364.
2. Guerin P, Obeid I, Bourghli A, et al. The lumbosacral plexus: anatomic consider-
ations for minimally invasive retroperitoneal transpsoas approach. Surg Radio! Anat.
20l2;34(2):151-157.
CRITIQUE30
Aortic dissection is the most common acute thoracic aortic pathology. occuning in 5-30
per million people per year. Dissection involving the ascending aorta occurs more com-
monly in patients ages 50-60, whereas descending aortic dissections occur in patients over
age 60.
Congenital defects of the arterial wall, such as Marfan syndrome, and inflammatory
diseases, such as giant cell arteritis, are associated with aortic dissection. In addition, con-
ditions that significantly raise blood pressure and cause shear stress on the aortic wall, such
as severe essential or malignant hypertension, weight lifting, and cocaine use are associ-
ated with aortic dissection. Marijuana use is not associated with aortic dissection.
More than 90% of patients present with the complaint of sudden-onset acute back or
chest pain described as sharp or tearing. Pain may lessen with time, and patients may note
flank or abdominal pain late in the course of the dissection as the intimal tear occludes re-
nal or mesenteric vessels causing ischemia. For patients with a descending aortic dissection
that does not compromise blood flow to critical organs, medical management with aggres-
sive blood pressure control results in mortality of less than 10% at 30 days.
Answer: (A) It is the most common type of acute thoracic aortic pathology
References
1. Braverman AC. Aortic dissection: prompt diagnosis and emergency treatment are criti-
cal. Cleve C/in J Med. 2011;78(10):685-696.
2. Criado Fl. Aortic dissection: a 250-yearperspective. Tex Heart Inst J. 2011 ;38(6):694-700.
3. Hughes GC, Andersen ND, McCann RL. Management of acute type B aortic dissection.
J Thorac Cardiovasc Surg. 2013;145(3 suppl):S202-S207.
368 CATEGORYS
CRITIQUE31
Causes of visceral artery aneurysm include atherosclerosis (30-50%), medial degeneration
(25%), trauma (20%), congenital anomalies (15%), and mycotic infection (10%). The most
commonly occurring visceral aneurysm is splenic, accounting for 60-70% of all visceral
aneurysms. Most are solitary and saccular, and 75% occur in the distal third, whereas 20o/o
occur in the middle third of the splenic artery. The risk of rupture is 2-10% and is higher
in women than in men. Hepatic artery aneurysms are the second most common, accounting
for 20-40%. They may be classified as intra- or extrahepatic.
Management of visceral artery aneurysms depends on etiology as well as size. Infec-
tious or traumatic aneurysms may be considered for immediate intervention. Indications
for repair of visceral aneurysms include women of child-bearing age, size larger than 2 cm,
rapid growth, or pain on presentation. Endovascular embolization is successful in 75-90o/o
of cases of visceral artery aneurysms. Reperfusion is reported in 5o/o, and recanalization is
reported in 12%.
References
!. Finley JW, Franklin DP, Indeck M. Rupture of a traumatic common hepatic artery aneu-
rysm: case report and review of the literature. J Trauma. 1998;45(3):637-641.
2. Ha JF, Sieunarine K. Laparoscopic splenic artery aneurysm resection: review of current
trends in management. Surg Laparosc Endosc Percutan Tech. 2009; l 9(2):e67-e70.
3. Chadha M, Ahuja C. Visceral artery aneurysms: diagnosis and percutaneous manage-
ment. Semin Intervent Radio/. 2009;26(3):196-206.
CRITIQUE32
Compressive brachia! plexopathy and ischemia of the upper extremity is often termed "Sat-
urday night palsy." It can progress to full-fledged compartment syndrome with associated
myonecrosis and rhabdomyolysis. In the case presented, the patient has compartment syn-
drome with resultant muscle ischemia and possibly necrosis. Further imaging would delay
therapy for this patient with limb-threatening ischemia in which muscle and nerve function
will be restored only ifthe extremity compartments are released.
Upper-extremity compartment release can be performed using an incision from the
deltopectoral groove int_o the space between the biceps and triceps muscles. The incision
then tracks laterally across the antecubital fossa and along the border of the brachioradialis.
The carpal tunnel should also be released to decompress the median nerve. A separate
incision over the dorsal aspect of the forearm may be required for complete release. Once
the compartments are released, muscle viability can be assessed at both the index operation
and during subsequent returns to the operating room.
Vascular reconstruction would be performed only if the ischemia resulted from arterial
injury or thrombosis. Thrombolytics are not indicated in compressive ischemia resulting in
a compartment syndrome.
References
1. Ivatury RR. Pressure, perfusion, and compartments: challenges for the acute care
surgeon. J Trauma Acute Care Surg. 2014;76(6): 1341-1348.
2. Rush RM Jr, Arrington ED, Hsu JR. Management of complex extremity injuries: tour-
niquets, compartment syndrome detection, fasciotomy~ and amputation care. Surg Clin
North Am. 2012;92(4):987-1007.
CATEGORYS 369
CRITIQUE33
This patient has chronic exertional compartment syndrome (CECS). CECS is a condition
of athletes usually younger than 40 years of age. Men and women are affected equally, al-
though women may be more susceptible than men to lower leg CECS. CECS is recognized
by pain or tightness, cramping, burning, or aching over the affected compartment during
exercise. The affected extremity may feel weak.
The anterior and lateral compartments of the lower leg are commonly affected; the deep
and posterior compartments are less commonly involved. Similar to an anterior compart-
ment syndrome, an anterior compartment CECS will exhibit weakness on dorsiflexion and
loss of sensation in the web of the first toe secondary to deep peroneal nerve involvement.
Most patients will relate the pain occurring at the same time in their exercise routine.
Athletes may be forced to stop playing while runners will continue but their gait is changed
to include a modified flatfoot strike. The affected compartment will gradually feel full
and pain will increase will active and passive stretching. Paresthesia or dysesthesia may
develop in the distribution of the affected nerve. Symptoms tend to subside with rest. Any
persistent symptoms are usually minimal during normal daily activities. The clinician must
perform a full evaluation and assessment to appropriately diagnose CECS, because treat-
ment is a fasciotomy or fasciectomy.
There is significant overlap between CECS and other lower-extremity pain syndromes.
These include periostitis, tibial or fibular stress fracture, and anterior tibial pain syndrome
(i.e., medial tibial stress syndrome, lateral tibial stress syndrome). Diagnosis is confirmed
by measuring pre- and postexercise compartment pressures.
Bilateral lower-extremity CT angiogram with 3-vessel runoff would be an appropriate
initial imaging study for a patient with claudication or concern for lower-extremity
peripheral vascular disease. Bilateral ankle-brachia! indices and pulse volume recordings
are similarly appropriate for patients with lower-extremity vascular disease.
Angiography or MRI with the patient's feet in both dorsiflexion and plantar flexion are
provocative studies used in the diagnosis of popliteal artery entrapment syndrome. Patients
with popliteal artery entrapment syndrome do not present with pain in the anterior aspect
of their legs but rather calf pain with walking.
References
I. Turnipseed WD. Clinical review of patients treated for atypical claudication: a 28-year
experience. J Vase Surg. 2004;40(1):79-85.
2. Turnipseed WD. Functional popliteal artery entrapment syndrome: a poorly
understood and often missed diagnosis that is frequently mistreated. J Vase Surg.
2009;49(5): 1189-1195.
370 CATEGORY5
CRITIQUE34
The most significant preventable cause of death in hospitalized patients is venous thrombo-
embolism (VTE). Many risk-stratification models are described in the literature; however,
the most widely used is the Caprini model. The Caprini model was validated in several
large studies, most notably in a National Surgical Quality Improvement Program study in-
volving more than 8000 surgical patients. As an example of a different model, Kucher and
associates published a similar scoring system and showed an 8.2% incidence of confirmed
VTE at 90 days in patients with a score of 4 or greater who did not receive prophylaxis.
However, this model lacked low-end sensitivity, with 20% of patients scoring 0-2 still
developing VTE. The Caprini model was designed to risk stratify patients using a checklist
to simplify this process as much as possible while understanding the complex nature of the
risk for VTE and the interaction of multiple risk factors.
The patient is asked to fill out a simple questionnaire listing many of the major factors,
and this list can be inserted into the larger form (figure 34.1 ). Empowering the patient im-
proves the quality of the data and helps orient the patient and doctor toward prevention of
VTE. The risk assessment model form also lists a score that is a relative number based on
the likelihood of each risk factor to be associated with risk for VTE. The highest individual
score of 5 is assigned to the highest risk category, for which randomized controlled trials
have shown the incidence ofVTE to be 40-80%, with up to 5% mortality from pulmonary
embolism. Minor surgery has a score of 1 and major surgery a score of2. Acute spinal cord
injury is in the category of highest risk (5 points), along with elective major lower extrem-
ity arthroplasty; hip, pelvis, or leg fracture within 30 days; stroke within 30 days; multiple
trauma within 30 days; and major surgery lasting longer than 3 hours. Each of the remain-
ing choices is assigned a score of 1 point. Use of this model to identify and grade VTE risk
is important for prophylaxis for VTE.
References
1. Caprini JA. Risk assessment as a guide for the prevention of the many faces of venous
thromboembolism. Am J Surg. 201O;199(1 suppl):S3-SIO.
2. Bahl V, Hu HM, Henke PK, Wakefield TW, Campbell DA Jr, Caprini JA. A valida-
tion study of a retrospective venous thromboembolism risk scoring method. Ann Surg.
20 I 0;251 (2):344-350.
3. Kucher N, Koo S, Quiroz R, et al. Electronic alerts to prevent venous thromboembolism
among hospitalized patients. N Engl J Med. 2005;352( 10):969-977.
CATEGORY5 371
Dr.If Time
CRITIQUE35
Blunt injury of sufficient force to result in posterior dislocation of the knee can cause
disruption and occlusion of the popliteal artery, and in many circumstances, the popliteal
vein as well. The vascular repair should be performed first or, if absolutely necessary,
shunts should be placed while the orthopedic procedure is performed. In the setting of
combined popliteal arterial and venous injuries, both the arterial and venous injuries should
be repaired. Failure to restore venous continuity can lead to leg edema and compartmental
hypertension, and consequently, occlusion of the arterial repair.
Historically, the ligation of venous injuries was associated with higher amputation rates.
The goal of venous repair is to maintain early patency to augment venous return in the
early postoperative period while collateral venous channels develop. Four-compartment
calf fasciotomy should be added to the repair of most popliteal artery injuries, especially if
arterial and venous thrombosis is present or the warm ischemia time exceeds 1 hour. Four-
compartment calf fasciotomy through a 2-incision approach is preferred for the majority of
combined arterial and venous injuries of the lower extremity.
Hourly confirmation of the presence of dorsalis pedis and posterior tibial pulses in the
intensive care unit is not sensitive enough or specific enough to detect a compartment
syndrome before irreversible neurovascular injury occurs. The measurement of compart-
ment pressures in the intensive care unit is possible, but interpretation of the findings is not
always clear-cut and may delay detection and intervention for a compartment syndrome.
To ensure adequate treatment after acute arterial and venous interruption, all 4 fascial com-
partments in the calf need to be decompressed with fasciotomies. Decompression of only 2
or 3 compartments does not sufficiently protect against the development of a compartment
syndrome. A thigh compartment syndrome would not be associated with this combination
of injuries.
References
1. Patel KR, Rowe VL. Extremity. In: Cronenwett JL, Johnston KW, Cambria RP, et
al, eds. Rutherford's Vascular Surgery. 7th ed. Philadelphia, PA: Saunders Elsevi-
er:2010:2361-2373.
2. Sise MJ, Shackford SR. Peripheral vascular injury. In: Mattox KL, Moore EE, Feliciano
DV, eds. Trauma. New York, NY: McGraw-Hill;2013:816-847.
3. Rowe VL, Pourrabbani S, Weaver FA. Blunt popliteal artery injuries. In: Stanley JC,
Veith F, Wakefield TW, eds. Current Therapy in Vascular and Endovascular Surgery.
5th ed. Philadelphia, PA: Saunders Elsevier;2014:686--689.
CRITIQUE 36-39
Acute occlusive mesenteric ischemia can broadly be divided into 2 major categories: em-
bolic mesenteric ischemia and thrombotic mesenteric ischemia. Patients with an embolus
to the mesenteric vessels typically have no prodromal symptoms and have an associated
acute onset of severe and unrelenting abdominal pain. The embolus typically occludes the
superior mesenteric artery just distal to the middle colic artery. These patients often have
sparing of the proximal jejunum and transverse colon, because the middle colic artery
remains patent. The source of the emboli is often cardiac and has a high association with
patients who have history of atrial fibrillation or myocardial infarctions.
Acute thrombotic mesenteric ischemia is often an end event in a long-standing history of
progressive atherosclerotic disease of the superior mesenteric artery. These patients typically
have a higher incidence of chronic mesenteric ischemic symptoms. There may be a history of
weight loss and food aversion. The occlusion typically occurs very proximal in the superior
mesenteric artery, which does not allow for sparing of the proximal jejunum or the colon.
CATEGORYS 373
The initial treatment for both conditions includes fluid resuscitation and administra-
tion of intravenous heparin to mitigate any thrombus propagation. Definitive diagnosis
can be made by arteriogram, but CT angiography is increasingly used in delineating
acute occlusion of the superior mesenteric artery. Once the diagnosis is made, surgical
treatment is indicated.
The operative therapy varies with the etiology of the lesion. For an embolus, an embo-
lectomy may suffice without further arterial reconstruction. Acute thrombotic disease often
requires a bypass of the involved segment, because it is typically severely atherosclerotic.
Endarterectomy is associated with high rethrombosis rates. A vein graft is preferred in the
setting of nonviable bowel. The bypass graft can be taken either from the aorta or from the
iliac artery. The supraceliac aorta affords relative absence of atherosclerotic disease. How-
ever, exposure to this section of the aorta is technically more challenging and may require
extensive dissection that allows additional ischemic time for the bowel. The advantage of
the iliac artery is that it is less technically challenging to expose; however, it may be heav-
ily calcified and unsuitable for graft takeoff. After revascularization, all nonviable bowel
should be resected. Marginal bowel can be left in place and reassessed at a second-look
operation 12-24 hours after the revascularization. Marginally viable bowel should not be
removed before the bypass is complete, because this bowel may be viable with revascular-
ization procedures.
When available, endovascular techniques may allow for thrombolysis or balloon angio-
plasty, with or without stenting, to immediately restore flow. Laparotomy or laparoscopy
may still be indicated to assess bowel viability.
Answers:
36. (C) Both
37. (A) Acute embolic mesenteric ischemia
38. (A) Acute embolic mesenteric ischemia
39. (D) Neither
References
I. Lin PH, Poi MJ, Matos J, Kougias P, Bechara C, Chen C. Arterial disease. In: Brunicardi
s
FC, Andersen DK, Billiar TR, et al, eds. Schwartz Principles ofSurgery. 10th ed. New
York, NY: McGraw-Hill;2014:827-914.
2. Hall JF, Ferguson CM. Mesenteric vascular disease of the small bowel. In: Cameron JL,
ed. Current surgical therapy. 9th ed. Maryland Heights, MO:Mosby;2008: 139-142.
3. Herbert GS, Steele SR. Acute and chronic mesenteric ischemia. Surg Clin North Am.
2007;87(5): 1115-1134.
Category 6
Endocrine
Items 1-27
DIRECTIONS: Each of the questions or incomplete statements is followed by 5 suggested
answers or completions. Select the answer that is the BEST in each case and fill in the
space containing the corresponding letter on the answer sheet.
l. A 50-year-old man with hypertension diagnosed at the age of35, fairly well controlled
on 4 antihypertensive agents, is evaluated for secondary forms of hypertension. His
serum aldosterone is 34 ng/dL (3-16 ng/dL), and his plasma renin activity is 0.11 ng/
mL/hour (0.2-3 ng/mL/hour), for an aldosterone-to-renin ratio of309. A 24-hour urine
aldosterone after oral salt loading confirms a diagnosis of primary hyperaldosteronism.
A CT of his abdomen shows a 1.3-cm left adrenal nodule and normal right adrenal
gland. His father was diagnosed with hypertension atage 60. The next most appropriate
step in his management is
(A) MRI of the abdomen.
(B) iodocholesterol scintigraphy.
(C) adrenal vein sampling.
(D) laparoscopic left adrenalectomy.
(E) genetic testing for glucocorticoid-remediable aldosteronism.
3. A 30-year-old woman has a 2-cm left thyroid nodule palpated on physical exam. There
is no family history of endocrine disorders or malignancy. Laboratory values show a
normal thyroid-stimulating hormone level. Thyroid ultrasound shows a solitary 2.3-
cm hypoechoic, solid, left thyroid nodule without calcifications. Fine needle aspiration
biopsy under ultrasound guidance is negative for malignancy. Which of the following
is the next most appropriate step in her management?
(A) Serum thyroglobulin level
(B) Radionuclide thyroid scan
(C) Levothyroxine
(D) Repeat thyroid ultrasound in 6--18 months
(E) Hemithyroidectomy
4. A 45-year-old woman with a history of melanoma has a 3-cm left adrenal mass inci-
dentally discovered on a CT scan. It has Hounsfield units of20 on the precontrast scan,
classifying it as an indeterminate adrenal mass. Which of the following is the most
appropriate next step in her evaluation?
CATEGORY6
377
6. A 42-year-old woman presents with obtundation, nausea, and vomiting. Her serum
calcium level is 15.5 mg/dL (8.9-10.1 mg/dL), and her parathyroid hormone is
elevated. The first step in management should be
(A) intravenous mithramycin.
(B) resuscitation with normal saline.
(C) intravenous hydrocortisone.
(D) intravenous phosphate infusion.
(E) urgent parathyroidectomy.
7. A patient presents with upper abdominal discomfort and dyspepsia while receiving
a high dose of a proton pump inhibitor. He has a serum gastrin of 200 pg/mL and an
elevated level of5-hydroxyindoleacetic acid (5-HIAA) in his urine. Endoscopy identi-
fies a 2-cm solitary tumor in the antrum with ulceration and otherwise nonnal mucosa.
Which of the following is the most effective therapy for this patient?
(A) Endoscopic surveillance
(B) Endomucosal resection of the tumor
(C) Antrectomy
(D) Distal gastrectomy with lymphadenectomy
(E) Total gastrectomy with lymphadenectomy
Items 10--11
10. A 15-year-old girl is brought to clinic by her mother for evaluation of a thyroid nodule.
Which of the following statements is true?
(A) The incidence of thyroid nodules in children has been declining over the past
20 years.
(B) The risk of cancer in a thyroid nodule in children is greater than in adults.
(C) The initial step in the workup of this child is a fine needle aspiration biopsy of
the nodule.
(D) The risk of thyroid cancer in the pediatric population is greater for boys than
for girls.
(E) Fine needle aspiration biopsy of thyroid nodules in children has a specificity and
sensitivity of 50-70%.
11. The patient undergoes fine needle aspiration biopsy, which reports '"cells consistent
with papillary cancer." There is no palpable adenopathy, and a cervical ultrasound
shows no suspicious areas elsewhere in the neck compartments. The primary lesion
measures 1.3 cm. \Vhich of the following statements about thyroid masses is true?
(A) Axial CT imaging of the neck and chest is an important preoperative
investigation.
(B) A total thyroidectomy is appropriate.
(C) Central lymph node dissection in children provides prognostic information.
(D) Hashimoto thyroiditis is rarely associated with thyroid cancer.
(E) Compared with adults, recurrence within 5 years is associated with an
increased mortality.
12. Central lymph node dissection for a 2-cm thyroid cancer is indicated for all
patients with
(A) anaplastic thyroid cancer.
(B) Hilrthle cell thyroid cancer.
(C) papillary thyroid cancer.
(D) follicular neoplasms.
(E) medullary thyroid carcinoma.
(A) Cancellous bone is more affected than cortical bone by the catabolic effects
ofPTH.
(B) Patients with asymptomatic primary hyperparathyroidism who choose
observation over parathyroidectomy will have no significant change in bone
mineral density after 10-15 years of follow-up.
(C) Patients with asymptomatic primary hyperparathyroidism who choose
antiresorptive therapy over surgery will have no significant change in bone
mineral density after 10-15 years of follow-up.
(D) Patients with primary hyperparathyroidism who undergo successful
parathyroidectomy can expect at best stabilization of their preoperative bone
mineral density.
(E) Patients with primary hyperparathyroidism who undergo successful
parathyroidectomy will have a statistically significant improvement in their
preoperative bone mineral density.
14. A patient with resistant hypertension on 3 antihypertensive medications and with mul-
tiple episodes ofhypokalemia has a plasma aldosterone-to-renin ratio greater than 50.
Adrenal protocol CT scan demonstrates a 3-cm left adrenal adenoma and a 5-cm right
adrenal adenoma. The patient should undergo
A. 1-metaiodobenzylguanidine (MIBG) scan.
B. adrenal protocol MRI.
C. somatostatin scan.
D. adrenal vein sampling.
E. percutaneous biopsy of the right adrenal mass.
Items 15-19
(A) Failed pituitary resection
(B) Conn syndrome
(C) Pheochromocytoma
(D) Adrenocortical carcinoma
(E) Adrenal myelo!ipoma
Items 20-22
(A) Secondary hyperparathyroidism
(B) Tertiary hyperparathyroidism
(C) Both
(D) Neither
20. H ypocalcemia
22. Calciphylaxis
Items 23-27
(A) Primary hyperparathyroidism
(B) Secondary hyperparathyroidism
(C) Both
(D) Neither
23. 32-year-old woman without other medical history, elevated calcium found on routine
work-up with sestamibi scan revealing uptake in the left lower pole
24. 55-year-old man with history of end-stage renal disease and hemodialysis, persistently
elevated calcium levels after successful cadaveric renal allograft without rejection
25. 60-year-old woman on hemodialysis for polycystic kidney disease with elevated para-
thyroid hormone and sestamibi scan showing uptake in all 4 glands
26. 34-year-old man with celiac disease, elevated parathyroid hormone, and low albumin
adjusted calcium level
27. 23-year-old woman with elevated calcium, parathyroid hormone level, and calcium/
creatinine less than 0.01on24-hoururine
CATEGORY6 381
Critiques 1-27
CRITIQUE 1
Primary hyperaldosteronism is characterized by autonomous secretion of aldosterone that
is not suppressible by sodium loading. It is caused by a unilateral adrenal adenoma, uni-
lateral or bilateral adrenal hyperplasia, or, rarely, by the familial disorder glucocorticoid-
remediable aldosteronism.
The management of primary hyperaldosteronism depends on subtype classification,
because patients with a unilateral adenoma/hyperplasia are candidates for adrenalectomy,
and patients with bilateral hyperplasia should be treated with a mineralocorticoid receptor
antagonist (i.e., spironolactone or eplerenone). Adrenal vein sampling is advocated to
distinguish between a unilateral and bilateral sources of aldosterone overproduction,
because some studies suggest that CT findings alone are misleading. On the basis of
CT findings alone, 20-25% of patients may be excluded incorrectly as candidates for
adrenalectomy or may have unnecessary or inappropriate surgery.
MRI has no advantage over CT in the evaluation of the adrenal glands, lodocholes-
terol scintigraphy is rarely useful and is no longer used in most centers. Genetic testing
for glucocorticoid-remediable aldosteronism is indicated for patients with hypertension
diagnosed younger than the age of20 or with a family history of either primary hyperaldo-
steronism or stroke at a young age (<40 years).
References
I. Funder JW, Carey RM, Fardella C, et al; Endocrine Society, Case detection, diagno-
sis, and treatment of patients with primary aldosteronism: an endocrine society clinical
practice guideline, JC/in Endocrinol Metab, 2008;93(9):3266-3281.
2, Young WF, StansonAW, Thompson GB, Grant CS, Farley DR, van Heerden JA, Role for
adrenal venous sampling in primary aldosteronism, Surgery, 2004;136(6):1227-1235,
CRITIQUE2
Graves disease is one of the most common causes ofthyrotoxicosis in the United States. It
is an autoimmune disorder in which thyrotropin receptor antibodies (TRAb) stimulate the
thyroid-stimulating hormone receptor, resulting in increased thyroid hormone production.
The diagnosis can be made clinically based on a history of symptoms associated with hy-
perthyroidism and physical exam findings of exophthalmos and a symmetrically enlarged
thyroid gland,
If the clinical presentation ofthyrotoxicosis is not diagnostic of Graves disease, a radio-
active iodine uptake scan is indicated and will show diffusely elevated uptake, This test is
contraindicated in pregnancy and lactation, Measurement ofTRAb levels are an alternative
way to diagnose Graves disease if a radioactive iodine uptake scan is unavailable or contra-
indicated, These antibodies are not always detectable, and this is related to the sensitivity
of the assay,
Ultrasonography does not generally contribute to the diagnosis of Graves disease, Thy-
roglobulin levels can be measured if factitious ingestion of thyroid hormone is suspected.
Because thyroglobulin is released along with thyroid hormone, in cases of endogenous thy-
rotoxicosis, its release is suppressed in the setting of exogenous thyroid hormone ingestion.
In this case, the clinical findings and laboratory results essentially confirm the diagno-
sis, and no further diagnostic tests are needed.
References
1. Bahn Chair RS, Burch HB, Cooper DS, et al; American Thyroid Association; American
Association of Clinical Endocrinologists. Hyperthyroidism and other causes of thyro-
toxicosis: management guidelines of the American Thyroid Association and American
Association of Clinical Endocrinologists [published correction appears in Thyroid.
2011 ;21(10):1169. Thyroid. 2012;22(11): 1195]. Thyroid. 2011;21(6):593-646.
2. Paunkovic J, Paunkovic N. Does autoantibody-negative Graves' disease exist? A second
evaluation of the clinical diagnosis. Horm Metab Res. 2006;38(1):53-56.
CRITIQUE3
Thyroid nodules are common, particularly in women and in older patients. The risk of thy-
roid cancer is 5-15%, depending on factors such as personal history of radiation exposure
and family history of thyroid cancer. The work-up ofa thyroid nodule begins with the eval-
uation for potential hyperfunction with the measurement of the serum thyroid-stimulating
hormone (TSH). Radionuclide thyroid scans are indicated only if the TSH is subnormal,
indicating hyperthyroidism. The serum thyroglobulin level is proportional to the size of the
thyroid gland and is not useful in the evaluation of a thyroid nodule.
Fine needle aspiration (FNA) biopsy is the procedure of choice in the evaluation of a
thyroid nodule. Biopsy results can be categorized as nondiagnostic (insufficient cells), neg-
ative for malignancy, indeterminate (neoplasm, atypia/follicular lesion of undetermined
significance, or suspicious for malignancy), and malignant (table 3.1). If the FNA is nega-
tive, follow-up thyroid ultrasound in 6--18 months is recommended, as FNA has a false
negative rate of up to 5o/o. If the FNA is read as neoplasm or suspicious for malignancy,
then hemithyroidectomy is required for definitive diagnosis. Suppression of thyroid nodule
growth with levothyroxine is not indicated in patients who live in iodine-sufficient parts
of the world.
References
I. Haugen BR, Alexander EK, Bible KC, et al. 2015 American Thyroid Association
management guidelines for adult patients with thyroid nodules and differentiated
thyroid cancer: The American Thyroid Association Guidelines Task Force on Thyroid
Nodules and Differentiated Thyroid Cancer. Thyroid. 2016;26(1):1-133.
2. Cibas ES, Ali SZ; NCI Thyroid FNA State of the Science Conference. The Bethesda
System for reporting thyroid cytopathology. Am JC/in Pathol. 2009;132(5):658-665.
3. Baloch ZW, Alexander EK, Gharib H, Raab SS. Overview of diagnostic terminology
and reporting. In: Ali SZ, Cibas ES, eds. The Bethesda System for reporting thyroid
cytopathology: Definitions_. criteria, and explanatory notes. New York, NY: Springer
Science+ Business Media;2010;1-3.
CATEGORY6 383
I. Nondiagnostic or Unsatisfactory
Cyst fluid only
Virtually acellular specimen
Other (obscuring blood, clotting artifact, etc.)
II. Benign
Consistent with a benign follicular nodule (includes adenomatoid nodule, colloid nodule, etc.)
Consistent with lymphocytic (Hashimoto) thyroiditis in the proper clinical context
Consistent with granulomatous (subacute) thyroiditis
Other
III. Atypia of Undetermined Significance or Follicular Lesion of Undetermined Significance
IV. Follicular Neoplasm or Suspicious for a Follicular Neoplasm
Specify ifHOrthle cell (oncocytic) type
V. Suspicious for Malignancy
Suspicious for papillary carcinoma
Suspicious for medullary carcinoma
Suspicious for metastatic carcinoma
Suspicious for lymphoma
Other
VI. Malignant
Papillary thyroid carcinoma
Poorly differentiated carcinoma
Medullary thyroid carcinoma
Undifferentiated (anaplastic) carcinoma
Squamous cell carcinoma
Carcinoma with mixed features (specify)
Metastatic carcinoma
Non-Hodgkin lymphoma
Other
Table 3.1 The Bethesda System for Reporting Thyroid Cytopathology; recommended
diagnostic categories.
Reprinted from The Bethesda System/or Reporting Thyroid C.vtopatho!ogy: Definitions. Criteria, and
Explanatory Notes, Overview of Diagnostic Terminology and Reporting, 2010, ZW Baloch. EK Alexander, H.
Gharib, SS Raab, ©2010 Springer, Part of Springer Science+Business Media. With pennission of Springer.
CRITIQUE4
Incidental adrenal masses are common, occurring in approximately 4-6% of patients.
The majority of adrenal masses are benign, nonfunctional, cortical adenomas. However,
in a patient with a history of carcinoma, up to 75o/o of adrenal masses may be a metas-
tasis. The assessment consists of an evaluation for hormonal overproduction as well as
risk of malignancy.
Some adrenal tumors can have subclinical hyperfunction, and screening tests for hy-
percortisolism, pheochromocytoma, and hyperaldosteronism are necessary. Fluorodeoxy-
glucose PET scan is a reasonable test in a patient with a negative hormonal evaluation and
a history of malignancy, but a hormonal evaluation should be completed first. Metaiodo-
benzylguanidine (MIBG) scans are appropriate for patients with a biochemical diagnosis
of pheochromocytoma with bilateral adrenal masses or in whom metastatic pheochromo-
cytoma is suspected. Fine needle aspiration (FNA) biopsy may be useful in a patient with
a negative hormonal evaluation and a history of malignancy but can differentiate only
384 CATEGORY6
between adrenal and nonadrenal origin of the adrenal mass. Hormonal evaluation should
be negative for pheochromocytoma before attempting FNA of an adrenal mass to avoid a
potential hypertensive crisis.
Laparoscopic left adrenalectomy may ultimately be required for definitive diagnosis,
but a hormonal evaluation should be done first.
References
1. Zeiger MA, Siegelman SS, Hamrahian AH. Medical and surgical evaluation and treat-
ment of adrenal incidentalomas. J Cl in Endocrinol Metab. 2011;96(7):2004--2015.
2. Zeiger MA, Thompson GB, Duh QY, et al; American Association of Clinical Endocri-
nologists; American Association of Endocrine Surgeons. The American Association
of Clinical Endocrinologists and American Association of Endocrine Surgeons medi-
cal guidelines for the management of adrenal incidentalomas. Endocr Pract. 2009; 15
(suppl 1):1-20.
CRITIQUES
Well-differentiated thyroid cancer accounts for more than 90% of all thyroid cancers. Cer-
vical lymph node metastases may be found in as many as 20-50% of patients with papillary
cancer. Micrometastases are also frequently found. Some series show that nearly 90% of
the examined nodes in patients after resection for papillary carcinoma may contain micro-
metastatic deposits. The clinical significance of this finding is unknown. Papillary carci-
noma of the thyroid is associated with excellent long-term survival, and the use of central
lymph node dissection is controversial in patients with nonpalpable clinical nodes. The
application of central and modified neck dissection is quite clear in patients with positive
lymph nodes in conjunction with papillary carcinoma of the thyroid.
The lymph node compartments of the neck can be separated into several levels and
sublevels (figure 5.1). The level I node compartment includes the submental and sub-
mandibular nodes above the hyoid bone and anterior to the posterior edge of the subman-
dibular gland. This area is readily examined underneath the mandible. Levels II, III, and
IV nodes are arrayed along the jugular veins on either side bordered anteromedially by
level VI and laterally by the posterior border of the sternocleidomastoid muscle. Levels
II, III and IV descend superiorly to inferiorly along the sternocleidomastoid muscle. The
boundary between levels II and III are at the level of the hyoid bone and between level III
and IV are at the level of the cricoid cartilage. Level V lymph nodes are in the posterior
triangle lateral to the posterior border of the sternocleidomastoid muscle. Level VI con-
tains the classic "central compartment." It includes the thyroid gland and is classically
described as having its lateral borders at the carotid sheath. It extends inferiorly to the
level of the clavicular heads. The superior border of level VI is the hyoid bone (not the
laryngeal notch). Level VII includes lymph nodes immediately behind the manubrium in
the anterior mediastinum.
There are no data that midline cancers of the thyroid necessarily mandate level I, IA, or
IB dissection unless there is obviously palpable lymphadenopathy in the area. The spinal
accessory nerve is an important structure in modified neck dissection. It is found in level
VA and courses superiorly into level II. It is not found in level IV. The omohyoid muscle,
courses superiorly and medially in level V crossing level IV before attaching superiorly to
the hyoid bone in level VI. It does not course through level IL
Answer: (C) The superior border oflevel III is at the level of the hyoid.
CATEGORY6
385
Figure 5.1 Schematic right anterior oblique view indicating levels of the neck and upper
mediastinum relevant to neck dissection.
Consensus Statement on the Terminology and Classification of Central Neck Dissection for Thyroid Cancer: ('")
The American Thyroid Association Surgery Working Group with Participation from the American Association
of Endocrine Surgeons, American Academy of Otolaryngology-Head and Neck Surgery, and American Head
and Neck Society. Sally E. Carty, David S. Cooper, Gerard M. Doherty, Quan-Yang Duh, Richard T. Kloos.
i"
Susan J. Mandel. Gregory W. Randolph, Brendan C. Stack, Jr.. David L. Steward, David J. Terris. Geoffrey "
·~
B. Thompson, Ralph P. Tufano. R. Michael Tuttle, and Robert Ude!sman. Thyroid. 2009. 19(11 ):1153-1158.
doi: 10.1089/thy.2009.0159. Copyright ©2009 Mary Ann Liebert. Inc.
References
I. American Thyroid Association Surgery Working Group; American Association of
Endocrine Surgeons; American Academy of Otolaryngology-Head and Neck Surgery;
American Head and Neck Society, Carty SE, Cooper DS, Doherty GM, et al. Consensus
statement on the terminology and classification of central neck dissection for thyroid
cancer. Thyroid. 2009; 19( 11):1153-1158.
2. White ML, Gauger PG, Doherty GM. Central lymph node dissection in differentiated
thyroid cancer. World J Surg. 2007;31(5):895-904.
3. Popadich A, Levin 0, Lee JC. et al. A multicenter cohort study of total thyroidectomy
and routine central lymph node dissection for cNO papillary thyroid cancer. Surgery.
2011;150(6):1048-1057.
4. Smith PW, Salomone LJ, Hanks JB. Thyroid: figure 38-3. In: Townsend CM, Beau-
champ RD, Evers BM, Mattox K, eds. Sabiston Textbook ofSurgery. 19th ed. Philadel-
phia, PA: Elsevier Saunders:2012:889.
386 CATEGORY6
CRITIQUE6
Initial management of hyperparathyroid crisis should be infusion of normal saline. Once
urine output is adequate, furosemide should be given to promote calcium excretion. Bisphos-
phonates are superior to mithramycin in in lowering serum calcium, with much less toxicity.
Traditional management recommended operation within 72 hours of admission, but
modem therapy with normalization of physiologic parameters reduced mortality of this
condition from greater than 90% in the 1950s to less than 5% currently. Delayed operation
also allows time for preoperative imaging. Although 88o/o of patients present with an iso-
lated parathyroid adenoma, a significant number of glands are found in aberrant locations.
Severe postoperative hypocalcemia occurs in more than one-third of patients. Postoper-
ative monitoring of calcium with administration of 1-25 dihydroxyvitamin D3 and calcium
as needed is indicated. Steroids and phosphate infusion have no specific role in manage-
ment of this condition.
Cinacalcet is a first-in-class calcimimetic drug that works to directly inhibit parathyroid
hormone (PTH) in patients with chronic renal failure. Calcimimetics allosterically modu-
late the calcium-sensing receptor, increasing its sensitivity to extracellular calcium and
thereby lowering PTH secretion from the parathyroid gland. Cinacalcet is used for patients
with secondary hyperparathyroidism in chronic renal failure, parathyroid carcinoma, and
in those rare patients who are not candidates for surgery.
References
1. Phitayakom R, McHenry CR. Hyperparathyroid crisis: use of bisphosphonates as a
bridge to parathyroidectomy. J Am Coll Surg. 2008;206(6):1106-1115.
2. Makras P, Papapoulos SE. Medical treatment of hypercalcaemia. Hormones (Athens).
2009;8(2):83-95.
3. LeGrand SB, Leskuski D, Zama I. Narrative review: furosemide for hypercalcemia: an
unproven yet common practice. Ann Intern Med. 2008:149(4):259-263.
CRITIQUE?
Gastric carcinoid tumors are neuroendocrine tumors. There are 3 subtypes of gastric carci-
noids (types I, II, and III). Each type has a distinct pathophysiologic mechanism, different
clinical outcomes, and different management approaches.
Growth of type I and II carcinoids is gastrin dependent while type III is gastrin
independent. Gastrin is a hormone secreted by G cells in the gastric antrum and stimulates
parietal cells to produce hydrochloric acid. Proton pump inhibitors (PP!s) limit acid
secretion, and serum gastrin levels of200 pg/mL (0-100 pg/mL) are common in patients
on PP!s.
Type III is a sporadic disease associated with normal gastrin levels. These carcinoids
have the worst prognosis, because their rate of metastases is more than 50o/o, making sur-
veillance wrong. Type III tumors are solitary, ulcerated, and deeply invasive lesions, usu-
ally larger than 1-2 cm. An atypical carcinoid syndrome with itching, cutaneous wheals,
and bronchospasm is associated with type III lesions. This syndrome is thought to be at-
tributable to the high levels of histamine released from its enterochromaffin-like cells. True
carcinoid syndrome is most commonly recognized in patients who have metastatic disease.
Sporadic type III gastric carcinoids are treated with gastric resection and regional lymph-
adenectomy. Partial gastric resection is adequate, because these tumors do not spread in
a submucosal pattern, but most will have lymphatic or distant metastases at the time of
presentation. The spread of this tumor makes endomucosal resection and antrectomy alone
inadequate treatment. Although total gastrectomy is feasible, it would be overtreatment for
this lesion.
CATEGORY6 387
References
I. Massironi S, Sciola V, Spampatti MP, Peracchi M, Conte D. Gastric carcinoids: between
underestimation and overtreatment. World J Gastroenterol. 2009; 15(18):2177-2183.
2. Nikou GC, Angelopoulos TP. Current concepts on gastric carcinoid tumors. Gastroen-
terol Res Pract. 2012;2012:287825.
CRITIQUES
Fine needle aspiration (FNA) plays a pivotal role in the evaluation and management
of thyroid nodules. Up to 30% of patients may have an indeterminate FNA result. with
cytology reported as "atypia (follicular lesion) ofundetermined significance" or "suspicious
for follicular neoplasm." Diagnostic hemithyroidectomy is used in these patients because
of the low but potential risk for malignancy.
The use of a gene-expression classifier in patients with indeterminate thyroid nodules
can aid in the surgical decision-making process. The negative predictive value of a benign
gene expression classifier (GEC) result for "atypia (follicular lesion) of undetermined
significance" or "suspicious for follicular neoplasm" cytology nodules is 94-95%. This
approaches the same risk of patients whose initial FNA is reported to be benign. In such
cases-an indeterminate FNA combined with a benign GEC-a period of watchful waiting
can be considered rather than surgery, which carries a 2-1 Oo/o risk of operative morbidity.
Patients typically undergo repeat clinical examination and ultrasound at I year, because
the risk of malignancy is not zero in GEC benign patients. Use of GEC testing is not
advocated for either papillary or medullary thyroid cancer, and therefore should not be
used for nodules with this cytology.
References
1. Alexander EK, Schorr M, Klapper J, et al. Multicenter clinical experience with the
afirma gene expression classifier. J Clin Endocrinol Metab. 2014;99(1):119-125.
2. Alexander EK, Kennedy GC, Baloch ZW, et al. Preoperative diagnosis ofbenign thyroid
nodules with indeterminate cytology. N Engl J Med 2012;367(8):705-715.
3. Duick DS, Klapper JP, Diggans JC, et al. The impact of benign gene expression clas-
sifier test results on the endocrinologist-patient decision to operate on patients with
thyroid nodules with indeterminate fine-needle aspiration cytopathology. Thyroid.
2012;22(10):996--1001.
4. Tuttle RM, Haddad RI. Ball DW, et al. Thyroid carcinoma. Version 2.2014. J Natl
Compr Cane Netw. 2014;12(12):1671-1680.
CRITIQUE9
The most common malignancy of the thyroid gland is papillary thyroid carcinoma (PTC)
and its variants. Total thyroidectomy is recommended for tumors greater than I cm diam-
eter and central neck dissection is indicated only when cervical lymph nodes are involved.
A common variant of PTC is the follicular variant (FV-PTC). Its clinical behavior is
unique with clinical features between PTC and follicular thyroid carcinoma (FTC). Over-
all and disease-specific survival does not differ significantly, but both types have slightly
higher overall survival rates than patients with FTC. The strongest risk factor for mortality
for both PTC and FV-PTC is age older than 45 years
The long-term outcome of patients with FV-PTC remains excellent and similar to that
of patients with C-PTC. Extrathyroidal extension or distant metastases ofFV-PTC have a
388 CATEGORY6
higher risk of mortality compared to PTC regardless of any treatment. For these high-risk
FV-PTC patients, prophylactic lymph-node dissection, close monitoring after initial sur-
gery, and other interventions should be considered.
Radioactive iodine ablation, thyroxine, and systemic chemotherapy are not effec-
tive treatments.
References
I. Ustun B, Chhieng D, Prasad ML, et al. Follicular variant of papillary thyroid carcinoma:
accuracy of FNA diagnosis and implications for patient management. Endocr Pathol.
20 l 4;25(3):257-264.
2. Yu XM, Schneider DF, Leverson G, Chen H, Sippel RS. Follicular variant of papillary
thyroid carcinoma is a unique clinical entity: a population-based study of I 0, 740 cases.
Thyroid. 2013;23(10):1263-1268.
3. Lin HW, Bhattacharyya N. Clinical behavior of follicular variant of papillary thyroid
carcinoma: presentation and survival. Laryngoscope. 2010;120(suppl 4):Sl63.
4. Miller BS, Doherty GM. An examination of recently revised differentiated thyroid
cancer guidelines. Curr Op in Oneal. 2011 ;23( I): 1-6.
5. National Comprehensive Cancer Network. NCCN Guidelines Thyroid Carcinoma.
Version 2.2015. Fort Washington, PA: National Comprehensive Cancer Network.
CRITIQUES 10-11
Thyroid surgery in children has become an increasingly important issue, because the inci-
dence of thyroid cancer is increasing. Thyroid nodules are common in adults in the United
States, and clinicians and surgeons need to be aware of the appropriate workup, preopera-
tive evaluation, and surgical treatment. The widespread use of ultrasound has contributed
to the increased diagnosis of thyroid malignancy in adults and children. It is thought that
2--<i% of adults have palpable thyroid nodules and 20-35% may have ultrasonographically
detectable abnormalities in the thyroid. In the pediatric population, epidemiologic studies
are much less robust; however, it appears that the prevalence of thyroid cancer in children
may range anywhere from 0.2-So/o.
Considerable data exist to show that the cancer risk of a thyroid nodule is significantly
greater in children than in adults: 20-25% versus 5-10%, respectively. As with adults, thy-
roid carcinoma in children is more common in girls at a 1.5-2.0:1.0 ratio.
With regard to the workup of this patient, it is relatively well documented that thyroid
nodules should be initially worked up by a good history and physical, obtaining laboratory
values [e.g., thyroid-stimulating hormone (TSH) values] and ultrasound. Fine needle aspi-
ration biopsy should be used only in confirmed solid or solid-cystic nodules with normal
or high TSH values. Among all the diagnostic evaluations, fine needle aspiration offers the
best sensitivity (at 95%) and specificity (at 85-88%), including the pediatric population.
With a diagnosis of papillary cancer, it is important to clear local lymph nodes in levels
I-VI by a good physical exam and cervical ultrasound. In this circumstance, there are no
other suspicious areas, and the primary lesion measures 1.3 cm. New guidelines from the
American Thyroid Association suggest that total thyroidectomy should be done for papil-
lary thyroid cancer in children. The reason is to reduce the rate of recurrence. There is no
current evidence that central lymph node dissection would provide any increased prog-
nostic information either in the adult or pediatric group. No more preoperative imaging,
including CT scan, needs to be done at this time.
Patients with Hashimoto thyroiditis have almost 3 times greater chance of harboring a
cancer in a thyroid nodule. Finally, lymph node recurrence of the primary in 5-10 years is
not associated with an alteration of overall survival. Despite the fact that the patient might
CATEGORY6
389
present with initial extrathyroidal extension, local metastasis, or later metastasis. overall
survival is excellent and remains greater than 90-95% for 25-30 year survival.
Answers:
10. (B) The risk of cancer in a thyroid nodule in children is greater than in adults.
11. (B) A total thyroidectomy is appropriate.
References
1. Matsuzu K, Sugino K, Masuda K, et al. Thyroid lobectomy for papillary thyroid cancer:
long-term follow-up study of 1,088 cases. World J Surg. 2014;38(1 ):68-79.
2. Nixon IJ, Ganly I, Patel SG, et al. Thyroid lobectomy for treatment of well differenti-
ated intrathyroid malignancy. Surgery. 2012;151(4):571-579.
3. Ledbetter DJ. Thyroid surgery in children. Semin Pediatr Surg. 2014;23(2):60-65.
4. De Luca F, Aversa T, Alessi L, et al. Thyroid nodules in childhood: indications for biop-
sy and surgery. Ital J Pediatr. 20 l 4;40:48.
5. Vergamini LB, Frazier AL, Abrantes FL, Ribeiro KB, Rodriguez-Galindo C. Increase
in the incidence of differentiated thyroid carcinoma in children, adolescents, and young
adults: a population-based study. J Pediatr. 2014; 164(6): 1481-1485.
6. Francis GL, Waguespack SG, Bauer AJ, et al; American Thyroid Association Guidelines
Task Force. Management Guidelines for Children with Thyroid Nodules and Differenti-
ated Thyroid Cancer. Thyroid. 2015;25(7):716-759.
CRITIQUE 12
Medullary thyroid carcinoma (MTC), a neuroendocrine tumor, may occur sporadically or in
conjunction with multiple endocrine neoplasia (MEN) 2 syndromes. The incidence of nodal
metastasis is up to 80o/o for MTC. Preoperative evaluation with ultrasound and serum calcitonin
levels helps guide operative plauning. For patients with MTC on fine needle aspiration but with
a negative cervical ultrasound, total thyroidectomy with central lymph node dissection (CLND-
level VI) is performed. This operative approach is due to ( 1) the high rate of central lymph node
involvement and (2) the fact that up to one-third ofultrasounds in MTC may be false-negatives.
Concurrent dissection of the lateral nodal compartments may be considered in patients with
elevated calcitonin levels or palpable nodes. Patients with positive preoperative imaging of the
central compartment should undergo CLND plus ipsilateral neck dissection.
The role of CLND for papillary thyroid cancer is the most controversial. Although early
advocates of CLND reported an incidence of metastatic disease in up to 60% of patients,
the impact of performing CLND was questioned. Several studies reported prophylactic
CLND does not affect lymph node recurrence or survival for papillary thyroid cancer.
In addition, a higher incidence of postoperative hypoparathyroidism was reported in pa-
tients undergoing CLND. Therefore, selective CLND rather than prophylactic CLND is
recommended. Patients should undergo preoperative high-resolution ultrasound to assess
for nodal metastasis. CLND is performed in patients with concerning nodal disease by
examination, ultrasound imaging, or intraoperative findings.
The determination of malignancy for follicular neoplasms is not established until cap-
sular invasion is identified on pathologic review after resection. Less than I0% of patients
with follicular thyroid carcinoma have lymph node metastasis; prophylactic CLND does
not have a role in operative management. Hilrthle cell thyroid cancer is a subtype of fol-
licular neoplasms; as such, operative management is similar. Anaplastic thyroid cancer is
a rare but aggressive tumor; 10% are localized to the thyroid, 40% have extra-thyroidal
invasion or nodal metastasis, and the remainder have widely metastatic disease. In patients
with locoregionally confined tumors that are determined to be resectable by imaging and
endoscopy, thyroidectomy with therapeutic CLND should be performed.
-----------i-----------
390 CATEGORY6
References
1. Wu LS. Roman SA, Sosa JA. Medullary thyroid cancer: an update of new guidelines
and recent developments. Curr Opin Oneal. 2011;23(1):22-27.
2. Giordano D, Valcavi R, Thompson GB, et al. Complications of central neck dissection
in patients with papillary thyroid carcinoma: results of a study on I 087 patients and
review of the literature. Thyroid. 2012;22(9):911-917.
3. Cisco RM, Shen WT, Gosnell JE. Extent of surgery for papillary thyroid cancer: preop-
erative imaging and role of prophylactic and therapeutic neck dissection. Curr Treat
Options Oneal. 2012;13(1):1-10.
4. Mathur A, Olson MT, Zeiger MA. Follicular lesions of the thyroid. Surg Clin North Am.
20 l 4;94(3):499-513.
5. Lal G, Clark OH: Thyroid, parathyroid, and adrenal. In: Brunicardi FC, Andersen
DK, Billiar TR, et al, eds. Schwartz's Principles of Surgery. 10th ed. New York, NY:
McGraw-Hi11;2014: 1521-1596.
6. Sancho JJ, Lennard TW, Paunovic I, Triponez F, Sitges-Serra A. Prophylactic central
neck dissection in papillary thyroid cancer: a consensus report of the European Society
of Endocrine Surgeons (ESES). Langenbecks Arch Surg. 2014;399(2):155-163.
7. American Thyroid Association Surgery Working Group; American Association of
Endocrine Surgeons; American Academy of Otolaryngology-Head and Neck Surgery;
American Head and Neck Society, Carty SE, Cooper DS, Doherty GM, et al. Consensus
statement on the terminology and classification of central neck dissection for thyroid
cancer. Thyroid. 2009; 19(11): 1153-1158.
CRITIQUE 13
Primary hyperparathyroidism affects cortical bone more than cancellous bone, thus the
catabolic effects of parathyroid hormone are more commonly observed in the femoral neck
and distal radius, which contain more cortical bone than the lumbar spine.
Patients with asymptomatic primary hyperparathyroidism who were prospectively fol-
lowed without surgery for 10-15 years showed that bone mineral density began to fall at
cortical sites before I 0 years, ultimately decreasing by I 0% at the femoral neck and 35%
at the distal radius at 15 years of follow-up. Patients treated with antiresorptive therapy
showed no difference in bone mineral density at 15 years compared with patients who
were not treated with antiresorptive therapy. Patients with primary hyperparathyroidism
who underwent successful parathyroidectomy and were prospectively followed for 10-15
years postoperatively were found to have sustained increases in bone mineral density at all
3 skeletal sites.
Answer: (E) Patients with primary hyperparathyroidism who undergo successful para-
thyroidectomy will have a statistically significant improvement in their preoperative bone
mineral density.
References
1. Silverberg SJ, Shane E, Jacobs TP, Siris E, Bilezikian JP. A 10-year prospective study of
primary hyperparathyroidism with or without parathyroid surgery [published correction
appears in N Engl J Med. 2000;342(2):144]. N Eng/J Med. 1999;341(17):1249-1255.
2. Rubin MR, Bilezikian JP, McMahon DJ, et al. The natural history of primary hyperpara-
thyroidism with or without parathyroid surgery after 15 years.JC/in Endocrinol Metab.
2008;93(9):3462-34 70.
CATEGORY6 391
CRITIQUE 14
Primary aldosteronism, or Conn disease, is an uncommon but important cause of hyperten-
sion. Conn disease may be present in 3-20% of adults with difficult-to-treat high blood
pressure that is often associated with hypokalemia. Normally, an increase in angiotensin II,
in response to elevated renin, results in aldosterone secretion from the zona glomerulosa
of the adrenal cortex. Thirty percent of hypersecreting adrenal adenomas result in Conn
disease. A serum aldosterone-to-renin ratio greater than 25 suggests the diagnosis.
Adrenal protocol CT scan and MRI can both identify adrenal masses, with an adenoma
having a characteristic appearance. However, these imaging studies do not differentiate
functional from nonfunctional adenomas. In the setting of bilateral adenomas, localization
of the hypersecreting adenoma is important, because treatment depends on whether the
disease is unilateral or bilateral. Although it can be technically difficult to perform, adrenal
venous sampling should be done to identify the hypersecreting adenoma before adrenalec-
tomy. Unilateral disease can be effectively treated with laparoscopic adrenalectomy.
Functional localization of pheochromocytomas arising from the adrenal medulla can be
accomplished with nuclear scintigraphy using I-metaiodobenzylguanidine (MIBG) scan.
A somatostatin scan is used in the evaluation of carcinoid, but has no role in localizing
functional adrenal adenomas.
Diagnosis and treatment of an adrenal mass can often be accomplished without biopsy,
especially in the setting of benign disease. Thus, percutaneous biopsy of the adrenal mass
would not change the course of treatment in this case.
References
1. Weiner ID. Endocrine and hypertensive disorders of potassium regulation: primary
aldosteronism. Semin Nephrol. 2013;33(3):265-276.
2. Chao CT, Wu VC, Kuo CC, et al. Diagnosis and management of primary aldosteronism:
an updated review. Ann Med. 2013;45(4):375-383.
3. Young WF, StansonAW, Thompson GB, Grant CS, Farley DR, van Heerden JA. Role for
adrenal venous sampling in primary aldosteronism. Surgery. 2004;136(6): 1227-1235.
CRITIQUE 15--19
The initial treatment for classic Cushing disease was pituitary resection; however, fail-
ures of this surgery result in residual tumor with increased adrenocorticotropic hormone
(ACTH) levels (Nelson syndrome). In this circumstance, bilateral adrenalectomy is very
effective. More recently, laparoscopic approaches demonstrated equal outcomes and per-
haps increased patient satisfaction compared with open adrenalectomy. After bilateral
adrenalectomy, a syndrome of increased ACTH secretion (figure 15-19.1) can occur,
requiring clinical intervention in as many as 5- I 0% of patients. Persistently elevated
ACTH secretion causes significant skin darkening and may require further pituitary sur-
gery or irradiation.
The syndrome of primary aldosterone secretion must be worked up thoroughly to sepa-
rate the 2 subtypes: unilateral cortical adenoma verses bilateral adrenal hyperplasia. In
patients older than 40, a mass found in the adrenal gland may not be the source of in-
creased aldosterone secretion, and selective venous sampling is strongly advocated to rule
out bilateral adrenal hyperplasia. Patients younger than 40 with a well-defined hypodense
adrenal mass I cm or greater and significantly marked plasma aldosterone concentrations
might not need selective venous sampling (figure 15-19 .2).
392 CATEGORY6
Since the classic article on Sipple syndrome was published nearly 40 years ago, there has
been an expanding knowledge base concerning genetic markers co-existing with pheochro-
mocytoma and the MEN2 syndrome, specifically parathyroid disease and medullary thyroid
carcinoma. Patients with the MEN2 syndrome have a high incidence of paroxysmal hyper-
tensive attacks and a high prevalence of hypertension and other cardiovascular problems,
which takes priority for diagnosis and treatment. Patients with pheochromocytoma should
be considered for timely resection of their pheochromocytoma and then follow-up genetic
screening and marker determination (calcitonin) for thyroid malignancy (figure 15- 19.3).
CATEGORY6
393
Adrenal cortical carcinoma (ACC) is a rare and aggressive tumor. It commonly occurs
in patients before age 5 or between ages 40 and 50. The clinical presentation ofACC varies.
Most patients are asymptomatic or have vague symptoms of abdominal discomfort. More
than 50% of adrenal cortical carcinomas produce hormones and may result in symptoms
related to the excess amount of that particular secretion. Cushing syndrome due to excess
cortisol secretion is the most common presentation in patients with hormonally active ACC
tumors, and it may occur in 30-40% of such patients (figure 15-19.4).
Answers:
15. (E) Adrenal myelolipoma
16. (B) Conn syndrome
17. (C) Pheochromocytoma
18. (A) Failed pituitary resection
19. (D) Adrenocortical carcinoma
References
l. Thompson SK, Hayman AV, Ludlam WH, Deveney CW, Loriaux DL, Sheppard BC.
Improved quality of life after bilateral laparoscopic adrenalectomy for Cushing's
disease: a JO-year experience. Ann Surg. 2007;245(5):790- 794.
2. Mattsson C, Young WF Jr. Primary aldosteronism: diagnostic and treatment strategies.
Nat Clin Pract Nephrol. 2006;2(4): 198- 208.
3. Keiser HR, Beaven MA, Doppman J, Wells S Jr, Buja LM. Sipple's syndrome: medul-
lary thyroid carcinoma, pbeochromocytoma, and parathyroid disease. Studies in a large
family. NIH conference. Ann Intern Med. 1973;78(4):561- 579.
4. Pacak K, Eisenhofer G, llias I. Diagnosis ofpheochromocytoma with special emphasis
on MEN2 syndrome. Hormones {Athens). 2009;8(2): 111- 116.
5. Rose, J, Guerrero MA. Adrenal cortical carcinoma: an updated review. Internet Jour of
Endo. 20 13:8 (1).
CATEGORY6 395
CRITIQUE 20-22
Increased parathyroid gland activity involves either intrinsic glandular activity that alters
parathyroid hormone (PTH) excretion (primary or tertiary hyperparathyroidism) or an ex-
trinsic calcium homoeostasis abnormality that stimulates PTH production (secondary hy-
perparathyroidism).
Primary hyperparathyroidism is the most common type of hyperparathyroidism, pri-
marily affecting postmenopausal women. Excessive PTH secretion by an autonomous
gland results in hypercalcemia. Although primary hyperparathyroidism historically pre-
sented with renal stones and metabolic bone disease, patients now present with minimal
sympto1ns, such as weakness, fatigue, and mental dysfunction, or no symptoms at all. Lab-
oratory values confirm hyperparathyroidism, and treatment is adenoma removal, which is
done increasingly by minimally invasive parathyroidectomy.
Vitamin D deficiency and hypocalcemia, which stimulate PTH synthesis. represent the
most common cause of secondary hyperparathyroidism, particularly in the elderly and in
patients with chronic renal failure. Abnormal renal conversion of25-dihydroxycholecalcif-
erol to 1,25-dihydroxycholecalciferol causes decreased intestinal calcium absorption. This,
together with reduced phosphate excretion and hyperphosphatemia result in hypocalcemia.
Correction of the underlying systemic metabolic insufficiency can often treat secondary
hyperparathyroidism.
Tertiary hyperparathyroidism represents a condition in which patients with longstand-
ing secondary hyperparathyroidism develop autonomous functioning glands; as such, cal-
cium levels may be normal or elevated. Surgery is usually indicated in patients with tertiary
hyperparathyroidism, and only rarely in secondary hyperparathyroidism when medical
therapy is failing.
Calciphylaxis is a life-threatening condition that results in painful, ischemic cutaneous
wounds characterized by tissue calcification, dry gangrene, and, occasionally, wound sep-
sis. Calciphylaxis develops when increased circulating calcium and phosphate levels lead
to calcium-phosphate precipitation, which compromises the microvasculature. Although
calciphylaxis is reported in primary hyperparathyroidism, it is most commonly associated
with secondary and tertiary hyperparathyroidism and is an indication for urgent parathy-
roidectomy. Patients with calciphylaxis who undergo total parathyroidectomy demonstrate
improved wound healing and survival, as well as decreased pain.
Answers:
20. (A) Secondary hyperparathyroidism
21. (C) Both
22. (C) Both
References
I. Pitt SC, Sippel RS, Chen H. Secondary and tertiary hyperparathyroidism, state of the art
surgical management. Surg Clin North Am. 2009;89(5): 1227-1239.
2. Fraser WD. Hyperparathyroidism. Lancet. 2009;374(9684): 145-158.
3. Ahmad R, Hammond JM. Primary, secondary, and tertiary hyperparathyroidism.
Otolaryngol Clin North Am. 2004;37(4):701-713.
396 CATEGORY6
CRITIQUE 23-27
Hyperparathyroidism is the result of increased activity of the parathyroid glands. These
glands secrete parathyroid hormone (PTH), which helps to maintain ionized calcium and
phosphate within appropriate levels throughout the body. Hyperparathyroidism can be
classified into 3 distinct types.
Primary hyperparathyroidism is the third most common endocrine disorder and results
from the excess secretion of PTH from one or more of the 4 parathyroid glands. The elevated
PTH leads to hypercalcemia. Patients typically are asymptomatic, but, in advanced stages,
patients will present with abdominal pain, psychiatric manifestations, bone disorders, and
renal calculi. In approximately 85% of cases, a solitary parathyroid adenoma is responsible.
In approximately 15% of cases, multiple glandular hyperplasia/adenomas are the culprit.
Rarely, in less than I% of cases, parathyroid carcinoma is responsible for the hyperparathy-
roidism. Sestamibi exam will show uptake in the hyperplastic/adenomatous glands. Excision
of the adenoma(s) is curative. Primary hyperparathyroidism should be differentiated from
familial hypocalciuric hypercalcemia (FHH), a benign inherited condition that manifests
with hypercalcemia and hypocalciuria (unlike the hypercalciuria seen in primary hyperpara-
thyroidism). Screening involves obtaining a 24-hour urine collection and determining the
calcium/creatinine ratio. If it is less than or equal to 0.1, FHH is diagnosed.
Secondary hyperparathyroidism results from excess secretion of PTH from normal
parathyroid tissue in response to systemic perturbations in calcium metabolism. It is most
frequently seen in the setting of chronic renal failure, but it can also develop from inad-
equate calcium ingestion/absorption, Vitamin D deficiency, and idiopathic hypercalciuria.
Ultimately, the hyperparathyroidism develops as a result of chronic hypocalcemia leading
to overproduction of PTH by the parathyroid glands. Consequently, 4-gland hyperplasia
is often encountered. Treatment typically entails 31/2-gland excision with reimplantation
of the remaining Yz gland in either the stemocleidomastoid or forearm muscles to prevent
development ofhypoparathyroidism.
Tertiary hyperparathyroidism refers to the autonomous release of PTH from the para-
thyroid glands as a result of chronic overstimulation due to secondary hyperparathyroid-
ism. This disorder typically occurs in patients with chronic kidney disease who receive
a renal transplant. In this setting~ the parathyroid glands persist in oversecretion of PTH
despite normalization of calcium metabolism due to the functioning kidney. Treatment fol-
lows that undertaken for secondary hyperparathyroidism.
The 32-year-old woman with elevated calcium and a solitary adenoma on sestamibi
scan has primary hyperparathyroidism. The 55-year-old man with elevated PTH levels af-
ter successful kidney transplant has tertiary hyperparathyroidism. The 60-year-old woman
with renal failure from polycystic disease and 4-gland uptake has secondary hyperparathy-
roidism. The 34-year-old with celiac disease has secondary hyperparathyroidism second-
ary to malabsorption of calcium. The 23-year-old woman with a calcium/creatinine ratio
less than 0.1 has FHH.
Answers:
23. (A) Primary hyperparathyroidism
24. (D) Neither
25. (B) Secondary hyperparathyroidism
26. (B) Secondary hyperparathyroidism
27. (D) Neither
References
I. Fraser WD. Hyperparathyroidism. Lancet. 2009;374(9684): 145-158.
2. Michels TC, Kelly KM. Parathyroid disorders. Am Fam Physician. 2013;88(4):249-257.
3. Prescott JD, Stephen A. Secondary and tertiary hyperparathyroidism. In: Cameron JL,
Cameron AM, eds. Current Surgical Therapy. I I th ed. Philadelphia, PA: Elsevier Saun-
ders;2014:673--676.
SESAP STAFF AND AUTHORS-I
SESAP 16 Staff
Chrysa M. Cullather
SESAP & Review Courses Medical Editor
Chicago, fl
SESAP STAFF AND AUTHORS-2
SESAP 16 Authors
The members of the 8 SESAP 16 authoring committees are listed. The members of the
SESAP Advisory Committee served as co-chairs of each committee.
'
Richard M. Bel!, MD, FACS*
Camden, SC
David C. Borgstrom. MD. FACS"'
ll1organtown, WV
Jay J. Doucet, MD, MSc, FACS. Dan Eisenberg, MD. MS. FACS
FRCSC.RDMS Palo Alto, CA
San Diego, CA
SESAP STAFF AND AUTHORS--4
'
Jason B. Fleming. MD. FACS"' Charles M. Friel. MD. FACS. FASCRS
Houston. TX Charlottcsvitle, VA
COL Robert 8. Lim. MD. FACS Lind::! L. Maerz. MD. FACS. FCCM
Honolulu, Hf New Haven. CT
- - - -------------
INDEX-I
Index
Numbers in boldface indicate category number/item. Numbers in normal typeface
indicate page numbers. Page numbers followed by f and t indicate figures and tables,
respectively. For example, "Splenic abscesses, 4-III/13: 304, 319-320, 320f' indicates
that splemc abscesses are discussed in category 4 part III item 13, on pages 304 and
319-320 and in a figure on page 320.
574. 609; 9/7: 670. 683-684: 9/23: 673 Biliary strictures, 3-11/34: 111
risk factors for. 9/23: 673 Bilious emesis. in infants, 10/14: 713. 734-736
Atypical ductal hyperplasia (ADH). 2/25: 32. Biologic matrices. 4-1/8-11: 246, 251-252
53-54; 2/30-32: 33. 57-58: 2/47-49: 35. 61-62 Biologic mesh. 4-111/28: 333-334
Atypical lipomatous tumors (ALT). 11/4: 763. "Bird's beak" sign. 3-11/14: 125, 126/
763/. 776 Bivalirudin. 8-11/20: 5S8. 5S9t
Atypical lobular hyperplasia (ALH). 2/30-32: 33. Black widow spider. 7-11/31: 467. 468/
57-58
Black widow spider bites. 7-1/4-6: 39S. 40S-409
Auriculotcmporal nerve (A TN). 111: S Bladder rupture. extraperitoneal. 7-111/25: 515-516.
Auto-positive end-expiratory pressure (auto-PEEP). 515J,- 10/36: 721. 755-756
9/11: 671. 686-687
Blast injury. 7-111116: 4SO. 504-505: 7-111/27: 4S7.
Autogenous hemodialysis access fistulas. 5/15: 517
339.355-356.356/ Blatchford score. 3-11114: 166. 167t
Autonomy. 13/10: 840-841 Bleeding. See also Hemorrhage
Awake fiberoptic intubation. 9/12: 687-688 arterial. 7-1122: 402. 418: 7-1111: 432. 444:
Axillary radiation therapy. 2/16: 47 7-lll/7, 477. 498: 7-III/27: 487. 517
Axillary surgery, 2/16: 47 after colonoscopy. 3-11/23: 105-106, 133-134
duodenal. 3-1/26-28: S5
B gastrointestinal. in children. 10/13: 713. 734
8-ce!l lymphoma intraperitoneal. 7-1/28: 404. 422
cutaneous-leg type. 12/5: 807. 816-817. S 16/ intratracheal.1111: 4. 15-16
smal! bowel. 3-111/32: 160. 188-189 lower gastrointestinal. 3-11118: 105. 130
Bacterial liver abscesses. 4-111127: 333 management of, 8-11112: 627-62S
Bacterial peritonitis. spontaneous (SBP). 4-111/20: pelvic. 7-1/36: 428
327-328 source diagnosis of, 7-111/17: 505. 505f
Balloon dilation in trauma. 7-11/12: 453
for achalasia. 3-II/3: 116 upper-extremity. 7-11/1: 432. 444
endoscopic pyloric. 10/23: 746 upper gastrointestinal, 3-1/20-22: 82-S3:
Banding. 3-113: 75 3-1/26-28: 68. 84: 3-11/38: 113. 146-147:
Bariatric surgery 3-111/4: 150. 166. 1671; 3-lll/9: 152.
obstructive sleep apnea in, 8-11/24: 592-593 169-170: 3-lll/24: 157. 182: 3-111/26: 158.
pregnancy after. 10/1: 710. 724 183-184: 3-IV/25: 229
venous thromboembolic event risks. 8-11/23: Bleeding duodenal ulcers. 3-111/3: 150. 165:
568,592 3-IV/25: 204
Barium enema. double contrast. 3-IV/6: 215 Bleeding gastroduodena! artery (GDA). 5/24: 342,
Barium csophagography. 3-IV/7: 215-216 363-364, 363/
Barrett csophagitis. 3-11/3; 116 Bleeding liver masses, 4-111/9: 303, 303.f. 316. 3 l 6f
Barrett esophagus. 3-1/1: 64: 3-111/29: 159. 186 Bleeding peptic ulcer disease. 3-111/9: 152. 169-
cancer in. 3-111/6: 168 170: 3-III/24: 157. 182; 3-IV/25: 229-230
with high-grade dysplasia. 3-1/l: 74 Bleeding varices. 4-111/22: 307. 328-329
Basal cell carcinoma (BCC). 1117: 6. 6/, 20-21 acute. 9/27: 674, 697-698
Beger procedure. 4-1/30: 259-261. 260/ esophageal, 3-113: 64. 75-76: 3-11/9: 103.
Benzodiazepines 121-122: 3~1V/25: 230
adverse reactions. 9/16: 672. 690 gastroesophageal. 3-IV/36: 240
for alcohol withdrawal. 8-111/42: 657-658 gastrointestinal. 3-11/9: 103. 121-122
risk ratio for delirium. 14/5: 850-851 management of, 3-IV/36: 240: 9/27: 697-69S
Beta-blockers pcristomal. 4-111/22: 307. 328-329
perioperative. 8-1/14: 533; 8-111/20: 641-642; from portal hypertension. 3-IV/36: 240
8-III/47: 625. 661 prophylaxis against. 3-IV/25: 230: 4-111/11:
for prophyla'\is against variceal bleeding. 303.318
4-111/11: 318 Blood component resuscitation. 9/39-41: 703-705
Bethesda criteria Blood glucose control. 8-11/36: 571. 600-601;
for colorectal cancer. 3-111/31: I SS 8-111/40: 624. 656
for thyroid cancer, 118: 13. 131: 6/3: 382. 3831 Blood transfusions
Bevacizumab, 11/25; 795: 11136-38: 772. SO I: and colorectal cancer recurrence risk. 3~1V/34:
11/46, 773. 803 208. 239
Bicycle crashes. 7-111/33: 490, 523: 7-111/39: 494. massive. 7-11/12: 435. 453: 9/41: 676, 703-705
529-530 perioperative.. 3-IV/34: 208. 239
Bile duct injury. 13/5: 829. 829/, 836-S37 red blood cell. 8-11/15: 565. 584
Bile leaks. 7-111119: 483. 509-510 Bloodstream infection
Biliary anatomy, intrahepatic. 7-111/19: 509 central line-associated (CLABSI). 5/8: 351:
Biliary colic. 4-111/18: 306 8-11/33: 570. 598-599
Biliary disease. in pregnancy. 10/2: 710. 725 central line-related (CR-BSl). 9/20: 692
Biliary sepsis, 8-1/40: 537. 552-553: 8-1/44: Bloody diarrhea. 3-IV/16: 202. 223
555-556 Bloody stools and abdominal pain/discomfort.
3-IV/15: 202. 222-223
INDEX-5
Blunt abdominal trauma, 7-11/34: 442, 442/ 471- ductal. 2/1-2: 26: 2/14: 29
472. 471/,· 7-111/34: 491, 491/ 523-525. 524( ER-positive, 2/1: 26, 36: 2/26: 54: 2/33-37: 34.
Blunt aortic injury (BAI), 7-1115: 414. 415; 7-1/28: 58-59
404,422 HER2-negative, 2/17: 29. 47-48; 2/22: 51
Blunt cardiac injury, 7-11/35: 443, 472: 7-11118: 477, hereditary. 2/11: 28. 42-43: 2/19: 30, 48-49
498-499 honnone replacement therapy and, 2/13: 28-29.
Blunt cerebrovascular injury (BCVI). 7-1/16: 401, 44
414. 415; 7-1/27: 403. 421-422: 7-11128-30: 440. inflammatory, 2/20: 49-50
465-467 invasive, 2/16: 29
grading scale for. 7-11128-30: 466! lobular carcinoma in situ (LCIS). 2/27: 32-33, 55
Blunt cervical spine injury. 7-1133: 405. 425 locally advanced, 2/20: 30. 30/, 49-50
Blunt chest trauma. 7-11/7: 433, 449: 7-11/35: 443, in men. 2/18: 30. 48
472: 7-111135: 492, 525-526 metastatic. 11/5: 777
Blunt diaphragmatic rupture. 7-111/34: 491. 523-525. node-positive. 2/5: 27, 38-39
524[ pleomorphic lobular carcinoma in situ (PLCIS).
Blunt liver trauma. 7-111115: 480, 503-504 2/4: 26. 37-38
Blunt pancreatic trauma. 7-111139: 494, 529-530. pregnancy-associated, 2/22: 31. 51; 10/5: 711.
529/ 726
Blunt popliteal artery injury. 7-1126: 403. 421 prevention of, 2/19: 30. 48-49: 11/33: 772. 799
Blunt renal trauma, 7-1120: 402. 417; 7-1132: 405, prophylactic contralateral mastectomy for. 2/14:
424-425; 7-11132: 441. 470; 7-111121: 484. 510- 45: 2/29: 33. 57
511, 511/ recurrence risk, 2/26: 32, 54--55
Blunt splenic injury (BSI). 7-1/30: 404. 423~ 7-1135: risk factors for. 2/7: 27. 40: 2/18: 30. 48
406,427 treatment of. 10/5: 71 I. 726
in adults, 7-111/24: 485, 514-515 triple-negative, 2/17: 29. 47-48: 2/28: 33. 56
angioembolization in. 7-11/3: 432, 445-446 Breast carcinoma in situ. lobular. 2/27: 32-33. 55
management of. 7-11/3: 445-446; 7-111/24: Breast conservation therapy. 10/5: 711. 726
514-515 Breast imaging. screening. 2/3: 26, 37
Blunt subclavian artery injury. 7-1117: 401. 415 Breast Imaging Reporting and Data System
Blunt thoracic aorta injury, 5/20: 340. 359-360 (BI-RADS), 2/44-46: 60-61, 611
Blunt thoracic trauma. 7-111/34: 491, 491/ 523-525. Breast pain. 2/8: 28; 2/24: 32. 52-53
524( Bronchodilators. 8-11/10: 565. 578-579
Boari flap, 7-11/36: 473 Bronchopulmonary carcinoid tumors, 10/30: 751-752
Body core temperature, 7-11/33: 470 Brown recluse spider. 7-11131: 467, 468/
Boerhaave syndrome, 10/45--47: 722, 759 Brown recluse spider bites. 7-114-6: 398, 408-409.
Bone mineral density, 6/13: 390 409f 7-Il/31: 467-470, 469/
Botulinum toxin Bundles, 8-1149: 539, 560
for achalasia. 3-11/3: 116 Bupivacaine toxicity, 8-111/28: 621. 646-647
for esophageal achalasia, 3-1123-35: 89 Bums
for gastroparesis. 3-111/27: 184 in children. 7-117-10: 398. 399/, 400/ 410-413,
Bowel intussusception, 3-IV/29: 205, 233-234. 234[ 410/ 41lf412/ 7-III/18: 481/-482/ 506-509,
Bowel preparation 506f 507f 508/
for elective colorectal surgery, 8-111/16: 618. combined with inhalation injury. 7-11/9: 434,
638-639 450-451
mechanical. combined with oral antibiotics. delayed excision of, 7-111/2: 476. 495
8-11/38: 602-603: 8-111/16: 638-639 esophageal. 7-11/23-24: 463
Brachial-brachial index (BBI). 5/23: 362 flame, 7-IIl/18: 506, 507/
Bradycardia. 9/35-38: 676. 702-703 hypermetabolic response to, 7-1119: 402. 416
BRAF inhibitors.11/9: 780~ 12/6: 817 immersion. 7-111118: 506. 506/
BRAF mutations. 12/16: 811. 823-824 nutritional support in. 9/32-34: 702: 9/42-44:
Brain failure. acute, 8-111/22: 642-643 705
BRCAl mutations, 2/19: 48-49 perioral, 7-111118: 506, 507/ 508/
BRCA2 mutations, 2/11: 42-43: 2/19: 30, 48-49 scald. 7-11112: 495; 7-111/18: 506, 506/
Breast, 2/1-49: 25-62 Button battery ingestion. 10/12: 712. 712/, 733-734
fat necrosis of, 2/7: 40
fibrocystie changes, 2/7: 40 c
intraductal papilloma of. 2/21: 31. 49 13
C-urea breath testing. 3-11119: 169-170
Paget disease of, 2/6: 39-40, 39/ '"<C>ABC" acronym, 7-111/27: 517
Breast abscesses, subareolar, 2/23: 31. 3 If 51-52. Cabozantinib. 11/32-35: 772. 800
52/ Ca!ciphylaxis. 6/20-22: 395
Breast cancer, 2/3-29: 26-33 Calcitonin, 11/1: 774
adjuvant treatment for. 2/33-34: 34; 2/33-37: Calculous cholecystitis. acute, 4-11/25: 272. 295-296
58-59 Calgranulin C (S100Al2). 3-IV/16: 223
chronic lymphedema after, 11/5: 764, 764/ Caloric requirements. 9/42-44: 677. 705
776-777, 777/ Calprotectin. 3-IV/16: 223
contralateral. 2/1: 26. 36; 2/14: 29 Cameron lesions, 3-111/17: 154, 175
INDEX--<i
Canadian C-Spine Rule, 7-1/33: 425 Cefotetan, 7-1114: 433t; 8-1/25-28: 534. 545-546
Canadian Critical Care Nutrition Guidelines, 9/29: Cefoxitin. 7-11/4: 433t
700 Celiac disease. 3-111128: 158. 185
Cancer, 11/1-47: 761-803. See also specific sites Cellulitis. 12/3: 806
in Barrett esophagus, 3-111/6: 168 Centers for Disease Control and Prevention (CDC).
staging. 3-111/28: 185 4-11/10: 282
Capacity, 13/10: 840--841 Centers for Medicare and Medicaid Services (CMS).
Caprini model, 5/34: 370 4-11110: 282
Carcinoembryonic antigen. 3-11/1: 100. 114/ Central line-associated bloodstream infections
Carcinoid crisis. 3-11/5: 118; 11113: 766. 782 (CLABSls), 5/8: 351: 8-11/33: 570, 598-599
Carcinoid syndrome, 11/13: 782 Central line-related bloodstream infections
causes of. 3-111/20: 156. 178 (CR-BSls), 9/20: 692
treatment of. 3-111115: 173 Central lymph node dissection (CLND). 6/12: 378.
Carcinoid tumors, 4-11/4: 277; 11113: 782: 11/15: 389-390
784 Central venous catheters. 5/8: 338, 351: 5/16: 356-
antral. 3-11/21: 105. 132 357: 9/20: 673, 692
appendiceal, 4-111/6: 301, 314; 11/30: 77L Cerebral monitoring, 7-11/6: 433. 448
798-799 Cerebrovascular injury. blunt (BCVI). 7-1/16: 401.
discriminating features of, 4-11/4: 278t; 11/15: 414. 415: 7-1/27: 403. 421-422; 7-11/28-30: 440.
784, 7851 465-467
gastric. 3-11/36: 112. 145-146; 6/7: 377. 386-387 grading scale for. 7-11/28-30: 466t
gastrointestinal, 3-1115: 117-118. 118/; 3-111/20: Certification, 13/12: 831. 841-842
178 Cervical esophageal cancer. 3-111/36: 191-192
pulmonary. 10/30: 720. 751-752 Cervical fractures. 8-1/48: 539. 559-560
Cardiac box. 7-11/26: 439. 464 Cervical spine injury. 7-1133: 405. 425
Cardiac events. 8-11/21: 567. 589-591 Cervical tuberculous lymphadenitis, 1/3: 2, 9
Cardiac injury. blunt 7-11135: 443. 472; 7-11118: 477. Cesarean section
498-499 complications of, 4-111128: 310
Cardiac risk assessment, 8-11121: 589-591. 590/; urgent, 10/4: 710. 726
8-III/6: 615, 630; 8-III/37: 624, 654: 8-lll/49: Cetuximab. 11/25: 795
626, 663: 8-lll/SI: 626, 664-665: 10/33: 753-754 CHA,DS,-VASC scoring system, 8-111/32: 650.
with CHADS/CHA,DS,-VASC scores. 8-111/32: 656t; 9i23: 694. 694t
650. 650t: 9/23: 694. -694t CHADS, scoring system, 8-111132: 650t; 9/23: 694t
in elderly. 8-11116: 615. 630 Charlson Comorbidity Index. 8-11/32: 598: 8-111113:
Cardiogcnic shock. 8-1147: 539. 559: 9/22: 673. 635; 8-111122: 643
693-694 Chemotherapy. See also specific agents
Cardiopulmonary resuscitation. 13/13: 832. 842 for esophageal adenocarcinoma metastatic to
Carotid cndarterectomy (CEA) liver. 4-111/10: 317
for carotid artery stenosis. 5/10: 353 for gastric cancer. 3-IV/2: 212~ 3-IV/23: 227-
complications of. 1/20: 7. 22-23; 8-111137: 624. 228: 3-IV/31: 236-237: 11/36: 772, 800-SOI
654 hyperthermic intraperitoneal (HIPEC). 11/27:
for stroke. 5/26: 343. 364-365 771. 796--797
Carotid plaques. 5/19: 359 neoadjuvant. 3-111/33: 160. 189; 4-11126: 296;
Carotid stenosis. 5/19: 340. 359 11/6: 778; 11/45: 773. 802-803
Catastrophic hemorrhage. 7-111127: 487. 517 Chest pain
Catheter-administered thrombolytic therapy. 5/28: after football injury, 7-111/20: 483. 510
343. 366 after left cxtrap\eural pneumonectomy. 10/27:
Catheter-associated upper-extremity deep vein 718, 748, 749/
thrombosis. 5/16: 340. 356-357 after motor vehicle collision. 7-11/35: 443, 472
Catheter-associated venous thromboembolism. 5/8: Chest trauma
351 blunt, 7-11/7: 433. 449: 7-11/35: 443. 472
Catheter-based thrombectomy. 5/6: 349 stab wounds. 7-1/18: 401. 415-416; 7-11126: 439.
Catheter-induced pulmonary artery perforation. 9/5: 464
669,682-683 Chest wall. flail. 7-11115: 435. 455-456
Caustic ingestion Chest wall pain. 7-1129: 404
in adults. 3-11/8: 102-103. 120-121: 7-11/23-24: Chest wall trauma. blunt, 7-111135: 492. 525-526
439. 462. 463; 10/32: 720. 753 Child abuse, 7-111/38: 493. 528-529
classification of. 7-11/23-24: 463 burn injury from. 7-117-10: 410. 41 lf, 413;
esophageal injuries. 10/32: 720. 753 7-III/18: 481/-482/ 506, 506/ 509
grading of injury, 3-11/8: 12lt nonaccidental head injury (NAHI). 7-IJ37: 407.
Cecal bascule. 3-11/14: 125 428-429. 429!
Cecal volvulus. 3-11114: 104. 125-126 Child-Pugh B cirrhosis. 4-1127: 249. 257
CT findings in. 3-11/14: 125. 126/ Child-Turcotte-Pugh (CTP)
radiographic findings in, 3-11114: 125. 125/ Calculator. 4-11/22: 292!; 9/45-48: 706t
Cefazolin. 8-11137: 602 Score. 4-11122: 271, 292: 9/45-48: 677,
706--707
INDEX-7
surgery in. 8-11/11: 565, 579-580: 8-11/32: analgesia: 8-111/24: 620. 644
570.598 benefits of. 8-11/9: 578~ 8-11/22: 591-592
surgical risk factors. 8-111/46: 625. 660-661 for colorectal surgery. 8-1119: 564. 578:
upper gastrointestinal bleeding in, 8-111/2: 614, 8-11/12: 565. 580-582: 8-11/22: 568,
627-628 591-592
Electrocardiography. 8-111/49: 663 contraindications to. 8-1/36: 535, 548-549
Electrocautery. 14/8: 852-853 cost-effectiveness of. 8-11/12: 565. 580-582
Electrosurgery, 8-111/3: 614. 628 Enoxaparin, 8-1/21-24: 534. 544-545
Embolic mcsenteric ischemia, acute. 5/36-39: 344, Enteral nutrition
372-373 in critical care. 9/29: 700
Embryonal sarcoma. 10/21: 742 in intensive care unit, 8-111/17: 639
Emergence reactions. 9/35-38: 676. 702-703 for nccrotizing pancreatitis. 4-11/2: 274
Emergency cholecystectomy. 4-11125: 295 after open small and large-bowel operations.
Emergency department thoracotomy. 7-1112: 432. 8-III/14: 617. 635
444-445 Enteric duplication cysts. 4-11/4: 278!; 11115: 785t
Emergency laparotomy. 7-111/34: 524, 524( Enteritis
Empyema C/ostridium difficile, 3-IV/15: 222-223
postpneumonia. 8-1139: 537. 552 radiation, 11139-42: 773. 801-802
posttraumatic, 7-11/18: 437. 458-459 Enteroclysis, 11/42: 773. SOI
Encephalopathy, 3-IV/36: 209. 240 Enteroco!itis. neutropenic, 3-IV/1: 211-212: 11121:
hepatic. 4-11/17: 268. 288-289 770, 792
End-of-life care. 13/16: 844 Enterocutaneous fistulas. 3-11137: 113, 146
End-stage liver failure Eosinophilic esophagitis. 3-IV/12: 219. 220/,·
Model for End-Stage Liver Disease (MELD) 3~1V/13: 221
scoring system. 4-11122: 271. 292-293, 292/, Epididymitis. 10/40: 758
293[: 8-111151: 664: 9145-48: 677. 706-707. Epidural analgesia. postoperative. 8-111/24: 644
706/ Epidural catheters. 7-11/7: 449
treatment of. 4-11/17: 288-289 Epinephrine. 8-1118--20: 533. 544
End-stage primary sclerosing cholangitis, 4-111/24: drug interactions, 8-111/28: 621. 646-647
330 for septic shock. 9/6: 683
Endarterectomy Epirubicin. 11/36-38: 772, 800-801
carotid (CEA). 5/26: 343, 364-365 Ertapenem.8-1125--28: 534.545-546
urgent, 5/26: 364-365 Esophageal achalasia, 3~1/23-35: 89-90
Endocrine disorders. 6/1-27: 375-396 Esophageal atrcsia. 10/19: 714. 739-740. 740/
Endoscopic cystgastrostomy. 3-111/38: 195 Esophageal bums. 7-11/23-24: 463
Endoscopic mucosa! resection (EMR). 3-1/1: 74; Esophageal cancer
3-III/36: 192 adenocarcinoma. 3-111/29: 159
Endoscopic myotomy. peroral (POEM). 3-1/23-35: adenocarcinoma stage Ila, 4-111110: 303. 317
90: 3-11/3: 116: 4-IJl/4: 313 advanced,3-1113-16:81-82
Endoscopic pyloric balloon dilatation, 10/23: 746 cervical, 3-111/36: 191-192
Endoscopic retrograde invasive. 3-111136: 161" 191-192
cholangiography. 4-111124: 330 metastatic to liver. 4-111110: 303. 317
cholangiopancreatography (ERCP). 4-11125: neoadjuvant chemoradiotherapy for. 3-111/33:
295; 4-111/16: 322-324 160. 189
cholangiopancreatography with sphincterotomy preoperative therapy for, 11123: 770. 793-794
(ERCP/ES). HV/17: 202. 224: 8-11/31: 570. squamous cell carcinoma. 3-111136: 161. 191-
597-598 192: 3-IV/38: 209. 241
complications of. 3-11/34: 111. 144: treatment of, 3-111/29: I 86: 3-IV/20: 203.
8-1138: 536. 536/ 550-551. 550/ 225-226
551/· 8-11/31: 570. 597-598 Esophageal manometry. 3-IV/7: 216
during pregnancy. 4-11/16: 288 Esophageal perforation. 3-11122: 105. 132-133:
Endoscopic transpapillary gallbladder stcnt 3-11/34: 111. 144; 3-IV/10: 200, 218-219;
(ETGS). 8-1140: 552-553 10145-47: 759
Endoscopy Esophageal stenting. 3-11122: 105. 132-133
before antireflux surgery, 3-IV/7: 215-216 Esophageal tears. 10/45-47: 759
for bleeding esophageal varices. 3-1/3: 75 Esophageal varices
upper. 3-IV/7: 215 bleeding. 3-113: 64, 75-76: 3-1119: 103. 121-
Endotracheal intubation. 8-1/48: 559-560 122; 3-IV/25: 230
Endovascu!ar aortic repair, thoracic (TEVAR), classification of. 4-111111: 318
5/20: 359-360 management of. 4-111/11: 318
Endovascular balloon occlusion. resuscitative, of prophylaxis. 3-IV/25: 230
the aorta (REBOA). 7-11/14: 435. 454-455. 455/ Esophagectomy
Endovascular stenting. 5/23: 362-363 complications of. 3-111/33: 189
Energy devices. 8-11125: 568, 593-594 Ivor-Lewis. 8-111/27: 621
Enhanced recovery after surgery (ERAS) programs, Esophagitis
3-11/16: 105, 128-129 Barrett. 3-1113: 116
INDEX-12
Food bolus impaction, 3-11/11: 103, 123-124; Gardner syndrome, 3-111/21: 179: 4-11/4: 276
3-IV/12-13: 201. 219-221 Gastrectomy
Food intolerance. 3-1/5: 65 for gastric cancer, 3-IV/23: 227-228: 3-IV/31:
Football injury 236-237
chest pain after, 7-111/20: 483. 510 palliative. 3-IV/23: 227-228: 3-IV/31:
right knee dislocation. 5/5: 336-337, 337/. 236-237
348-349 sleeve. 3-11/4: 101, 117: 3-IV/8: 216
Foreign body obstruction Gastric adenomatous polyps. 3-11/30: 139
esophageal. 10/12: 712, 712/. 733-734 Gastric antral vascular ectasia. 3-1/26--28: 85
rectal. 3-1/45: 73, 98; 3-111114: 153. 153/. 172 Gastric band slippage. 3-1/5: 65. 77-79, 77/, 78/
Fractures Gastric bypass
acetabular, 7-11111: 452 marginal ulcers after, 3-11139: 113. 147-148
cervical, 8-1/48: 539. 559-560 Roux-en-Y. 3-11/4: 101. J 17; 5/6: 337
femoral. 7-1/11-14: 401. 413-414; 7-111131: Gastric cancer, 3-IV/23: 227
488, 521-522 adjuvant therapy for. l l/24: 770. 794
humeral, 7-11/l: 432, 444 advanced, 3-IV/23: 227-228
long bone, 7-111/31: 521-522 chemotherapy for. 11/36: 772, 800-801
after motor vehicle collision, 7-111/11: 478, distal. 3-11116: 168
500-501 HER2-positive, 3-IV/2: 198. 212: 3-IV/23: 228
open. 7-1/11-14: 401. 413--414 management of, 3-IV/2: 212; 3-IV/31: 207,
pelvic, 7-1136: 407. 428: 7-11111: 452: 7-111122: 236-237
484.485/ 512-513. 512/ 513/ 10/36: 721. metastatic, 3-IV/23: 227-228: 3-IV/31: 207.
755-756 236--237
rib. 7-1129: 404, 422--423: 7-1/36: 407, 428: proximal. 3-111/6: 168
7-11/7: 433, 449; 7-11/15: 435, 455-456; stage IV. 3-IV/23: 227: 3-IV/31: 236-237
7-11/18: 437. 458-459: 7-11/27: 440. 465: tumor markers. 11/25: 770. 795
7-III/6: 477. 497-498: 7-111/33: 490. 523: Gastric carcinoid tumors, 3-11/36: 145-146; 6/7:
7-111/35: 492. 492/. 525-526. 526/ 377, 386--387
sternal. 7-1130: 404, 423: 7-111/8: 477. 498-499 Gastric emptying study. 3-IV/7: 216
tibial, 7-1/11-14: 401. 413--414 Gastric lavage, 3-111/26: 183
upper extremity. 7-1/22: 402. 418 Gastric outlet obstruction. 3-111/28: 158
wrist. 7-1/33: 405. 425 benign. 3-111/3: 165
Frailty management of. 10/23: 717, 746
assessment of. 8-1/41: 537. 553: 8-11/19: 588: peptic ulcer associated. 3-IV/37: 240--241
8-11/32: 598: 8-III/13: 635 Gastric polyps. 3-11130: 139-140
definition of. 8-11/32: 598 Gastric prolapse. 3-1/5: 77-79. 77/, 78/
surrogate markers for. 8-111122: 643 Gastric resection. 3-IV/23: 227-228
Frailty Index, 8-11/19: 588 Gastric ulcers. 3-1/2: 75
Fresh frozen plasma (FFP). 7-11/13: 435, 453--454; Gastric varices. 3-1/26--28: 85
8-1/4-8: 532. 541: 9/39: 676. 704 Gastroduodcnal artery (GOA). bleeding. 5/24: 342.
Frey procedure, 4-1130: 259-261. 260/ 363-364, 363/
Frey syndrome, 111: 8 Gastroesophageal adenocarcinoma. 3-111/6: 15 J,
Frostbite injury 168
basic principles and categorization of, 7-11/8: Gastroesophageal reflux (GER)
449 acid suppression therapy for. 10/10: 729
fourth-degree. 7-11/8: 434. 434f. 450. 450/ differential diagnosis of. 3-11/3: 116: 3-111/35:
Full disclosure, 13/8: 830, 839: 13/11: 831, 841 191
Fundamentals of Use of Surgical Electricity course Gastroesophageal reflux disease (GERD). 3-IV/22:
(FUSE). 8-II/25: 594 203. 227
Fundic gland polyps (FGPs). 3-11/30: 139 in children, 10/10: 729
Fundoplication management of. 3-IV/22: 227; 10/10: 729
Nissen. slipped. 3-11/35: 112. 144--145. 145/ preoperative evaluation of. 3-IV/7: 199, 215-
partial. 3-IV/22: 203. 227 216; 3-IV/22: 227
total. 3-IV/22: 203, 227 Gastrointestinal bleeding
in children, 10/13: 713, 734
G variceal. 3-11/9: 103. 121-122
Gallbladder disorders, functional. 4-11/15: 286-288. Gastrointestinal cancer, advanced, 3-1/13-16:
2871 81-82
Gallbladder polyps. 4-1/6: 246. 251: 4-111112: 303. Gastrointestinal carcinoid tumors. 3-1115: 117-118.
319 118/
Gallstone pancreatitis. 4-111/8: 315 Gastrointestinal stromal tumors (GISTs). 3-1/43:
mild.3-11/2: 100.115 73. 96-97: 3-11/I 7: 105. 129-130; 3-11/27: 108.
in pregnancy. 4-11116: 288 136-137: 3-111/7: 151. 168-169: 3-IV/35: 209.
treatment of, 3-IV/17: 202. 224 239-240
Gallstones. 4-111/8: 315 advanced, 11134: 772, 800
c-kit-positive. 3-IV/8: 200, 216-217
INDEX-14
discriminating features of. 4-11/4: 278!; 11/15: thoracoabdominal. 7-11112: 435. 453
785r ureteral injury after, 8-11/42: 573. 606--607
recurrence. 3-IV/8: 216 Gut suture. 4-1116-18: 247. 253
resection of, 3-11/31: 109, 140 Gynecomastia. 2/10: 28. 42: 2/18: 48
in small bowel. 3-III/15: 173
treatment of. 3-11/31: 140: 3-IV/8: 216-217: H
3-IV/35: 239 H,-receptor blockers, 2/10: 42: 3-11138: 147: 9/14:
Gastrojejunostomy, 3-11/15: 127: 3-11113: 165: "689
3-IIl/28: 185: 3-JV/37: 240: 10123: 746 Haemophilus influen=ae type b (Hib) vaccine.
Gastroparesis, 3-11/20: 105. 131; 3-111127: 158. 184 7-111/3: 496: 7-111129: 519-520. 520t
Gastroschisis.10/11: 712. 730-733, 730/, 731/, Haloperidol. 8-11148: 610
732/, 733/: 10141-44: 722. 758-759 Hamartoma, oral. 1/9: 4. 14. 14/
sutureless abdominal wall closure.10/11: 712. Hamman sign. 7-111/20: 510
730-733. 730[, 731/, 732/, 733[,· Hartford Consensus. 9/30: 675, 700-701
Gastrostomy, percutaneous endoscopic (PEG). Hashimoto thyroiditis. 6/10-11: 388
3-IIIll: 150. 164 Hasson technique. 4-11/16: 288
Gelatinous ascites. 11/19: 768. 769/ Head and neck.111-21: l-24
Gene-expression classifier (GEC) testing, 6/8: 377. Head and neck masses. 1/16: 5. 18-20
387 Head injury. nonaccidental (NAHi). 7-1/37: 407.
Genetic testing 428-429. 429r
for colorectal cancer. 3-IV/6: 215 Health Information Technology for Economic and
for familial adcnomatous polyposis. 3-111121: Clinical Health Act (HlTECH). 13/14: 843
179 Health Insurance Portability and Accountability Act
for glucocorticoid-remediable a\dosteronism. (HlPAA). 13114: 832, 843
6/1: 381 Health privacy. 13/14: 832. 843
for Lynch syndrome. 3-111/31: 187-188 Healthcare
Gcnitofemoral nerve. 4-111/25: 331. 331/ decent minimum of. 13/4: 828, 836
Gentamicin. 8-11/28: 569, 595-596 standard of care. 13/5: 837
Glasgow-Blatchford score, 3-111/4: 166. 167t: value in, 14/9: 853
3-ill/26: 183 Healthcare power of attorney. 13/16: 833
Glasgow Coma Scale (GCS), 7-11/28: 465: 8-1141: Heating blankets. 9/21: 693
553 Helicobacter p_vlori infection. 3-111/24: 182: 11/29:
Global Trigger Tool (GTT), 14/10: 847. 854 771. 798
Glossophaf)1lgeal nerve injury. 1120: 23 Heller myotomy
Glucagon, 3-IV/13: 221 for achalasia. 3-1113: 116: 3-111/35: 191
Glucocorticoid-remediable aldosteronism. 6/1: 381 laparoscopic, 3-1123-35: 90: 4-111/4: 313
Glucose control. 8-11/36: 571. 600--601: 8-111140: postprocedure follow-up, 3-111135: 161. 191
624. 656 Hemangioendothelioma. infantile hepatic, 10/21:
Gonadal artery. 5/29: 367 742
Gonadal vein. 5/29: 367 Hemangioma. hepatic, 4-1/14: 247. 252
Graft-versus-host disease, 3-IV/l: 211, 21 lf Hcmatemesis. 3-111/26: 158
Graham patch closure. 3-IV/37: 240-241 Hematoma
Granuloma fungoidcs. 11/3: 775 perinea!. 7-111/25: 486. 515-516
Granulomatous mastitis. idiopathic. 2/9: 28. 41 rectus sheath. 4-111/19: 306. 326-327. 327f
Graves disease. 6/2: 381-382 rctroperitoneal. 7-11132: 441, 470: 7-111/33:
Greater auricular nerve (GAN), 1/1: 2. S. Sf 490. 523
Griffiths point. 5/9: 352 subdural. 7-11120: 437. 460
Groin hernias. 4-111/2: 300. 311-312 Hematuria. 7-11/16-17: 436. 456-458
Groin injury. 5/4: 336, 348 Hemobilia. 7-111115: 503-504
Groin masses. 12/1: 806. 812 Hemodialysis
Groin pain, 4-111115: 304, 322 arteriovenous access for. 5/18: 340, 358
Group A Streptococcus infection. 8-1127: 534. autogenous access fistulas for. 5/15: 339.
545-546 355-356. 356/
Gum chewing, postoperative. 8-111125: 645 Cimino fistula for. 5/7: 337
Gunshot wounds, 5/4: 336. 348: 7-1131: 405, 423- continuous venovenous, 8-11111: 627
424; 7-111132: 488, 489f, 522-523 intermittent, 8-11111: 627
abdominal. 7-11/36: 443. 472--473; 7-111136: Hemophilia A. 9/53: 677. 708
492.527 Hemophilia B. 9/53: 677, 708
left mid-thigh. 7-11111: 476. 495 Hemorrhage. See also Bleeding
lower abdomen. 7-1114: 433: 7-11122: 438. 438f, catastrophic. 7-111127: 487, 517
461--462, 462f hepatic adenoma with, 4-1115: 247. 252
perioperative antibiotics for. 7-11/4: 433. 433t intraparenchymal, 7-11121: 437. 461
postoperative management of, 8-11119: 616, 632 from liver mass. 4-111/9: 302. 303/, 316, 316f
recommendations for active shooter events. massive, 7-111/28: 487. 517-519
9130: 700-701
INDEX-15
management of. 6/6: 377. 386: 6/13: 378-379. repair of. 4-11/9: 266, 282; 4-111128: 310.
390 333-334
primary, 6/13: 390: 6/20--22: 395: 6/23-27: 380. ventral. 4-111/28: 310. 333-334. 333/
396 Incompetence. 13/10: 840-841
secondary. 6/6: 377. 386: 6/20-22: 380. 395: Infantile choriocarcinoma. 10/21: 742
6/23-27: 380, 396 Infantile hepatic hemangioendothe!ioma. 10/21: 742
tertiary. 6/20-22: 380. 395: 6/23-27: 396 Infants
Hyperp!astic polyps. 3-11/30: 139: 4-1/19: 247, 254 aberrant right subclavian artery. 5/25: 364
Hypertension bilious emesis in, 10/14: 713, 734-736
intra-abdominal (IAH). 8-1/46: 538. 557-558, exposure to anesthesia. 14/7: 847. 851-852
557t; 9/25: 695--696 inguinal hernias in, 10/15: 713. 736
portal. 3-1/3: 64, 75-76: 3-111/39: 163. 195-196: liver masses in. 10/21: 716. 716/, 742
3-IV/36: 209, 240 Infection(s). See also specific infections
resistant. 6/14: 379 central line-associated bloodstream infections
secondary forms. 6/1: 376 (CLABSls), 5/8: 351
Hypcrthcrmia, malignant. 8-111/23: 620, 643--644 hospital-acquired multi-drug resistant. 8-11134:
early clinical features of. 8-111/18: 618, 640 571, 599
treatment of. 8-111/18: 640 hypothenar space. 12/21-23: 8 IL 825
Hyperthermic intra.peritoneal chemotherapy (HlPEC). midpalmar space. 12/21-23: S11. 825
11/27: 771, 796-797 overwhelming postsplenectomy (OPSI). 4-11/14:
Hypertriglyceridemia. 9/35--38: 676, 702-703 267, 285; 7-IJJ/3: 496; 8-JJ/43: 573, 607
Hypocalcemia, 6/6: 386 pancreatic. 4-111/7: 314-315. 314{
Hypocalciuric hypercalcemia. familial (FHH). prophylactic perioperative antibiotic regimens
6/23-27: 396 for. 4-11/10: 266. 282-283
Hypoglossal nerve injury. 1/20: 22 soft tissue, 8-1/27: 534, 545-546: 8-111/7: 615,
Hypokalemia. 6/14: 379 630-631
Hyponatremia, 9/24: 695 Surgical Infection Prevention (SIP), 14/1: 848
Hyposplenia. 4-11/14: 285 surgical site. See Surgical site infection (SSI)
Hypotension thenar space, 12/21-23: SJ 1. 825
supine hypotensive syndrome. 10/3: 725 thyroglossal duct cyst. 1/12: 4. 16
in trauma patients. 7-11/32: 441. 470 urinary tract 8-1125: 534, 545-546
treatment of. 9/1: 668. 679 Inferior vena cava filters. 5/14: 339. 355
Hypothenar space infections. 12/21-23: 811. 825 for pulmonary embolism. 8-111/41: 656-657
Hypothermia. 7-11/33: 442. 470-471 retrievable. 8-11/12: 565. 581
active rewarming for, 9/21: 673, 693 Infiltrating ductal breast cancer, 2/1-2: 26
Hypovolemic shock. 7-1111: 432. 444: 7-111/10: 500 Inflammatory bowel disease. 3-IV/16: 223
Hysterectomy. 4-1123: 248 Inflammatory fibroid polyps (IFPs). 3-11/30: 140
Inflammatory pain. 4-111/14: 321
Inflammatory phlegmon. 3-1123-25: 68. 83-84
Idarucizumab. 8-1/15-17: 543; 8-11/5-8: 577; Infliximab. 3-IV/4: 199: 3-IV/27: 231
8-JJ/30: 597; 8-III/29: 648, 6471; 8-III/32: 650 Influenza vaccine, 7-111129: 519-520, 5201
Idiopathic thrombocytopenic purpura (ITP). 4-1117: Information security. 13/14: 843
265. 280-281; 4-111126: 309. 332 Inguinal hernia, 4-11/28: 273, 297
Ileitis, regional. See Crohn disease in infants and children. 10/8: 711. 728: 10/15:
Ileocolonic intussusception. long segment. 3-IV/29: 713. 736
205 in men. 4-111/5: 301. 313
lieus. 8-111/25: 644--645 pain associated with. 4-111/14: 321
Iliofemoral deep vein thrombosis. 5/6: 337. 349: recurrent. 4-1/1-3: 250; 4-111114: 321
5/13: 339 Inguinal hernia repair. 4-1/1-3: 246. 250: 4-11/6: 265.
Iliohypogastrie nerve. 4-111/25: 331. 331/ 280: 4-11/28: 297
Ilioinguinal nerve. 4-111/25: 331. 331/ in infants and children. 10/15: 736
"I'm sorry" laws. 13/11: 83 l. 841 pain after. 4-111125: 331-332
Imatinib, 3-11/17: 105. 129; 3-IV/8: 216-217 robotic. 8-11112: 565, 581
Imipenem. 8-11128: 569. 595-596 Inguinal hemiorrhaphy, 4-111/1: 311: 4-111/25: 309
Immersion bums. 7-111/18: 506. 506/ anterior approach. 4-111125: 331, 331/
Immunohistochemistry (IHC) analyses. 3-111131: chronic pain after. 4-111/14: 304. 320-321
187-188 Inhalation injury, 7-11/9: 434. 450-451
Immunosuppression, 3-IV/4: 199 Inherited colorectal cancer syndromes. 3-111/31: 160.
perioperative, 3-IV/4: 213-214 187-188
skin cancer in. 12/7: 808, 817 Innovations. 13/15: 832. 843-844
Incidental adrenal masses. 6/4: 376-377. 383-384 lnspiratory resistance training. 8-11/10: 565. 578-579
Ineisional hernias Institute for Healthcare Improvement
complex. 4-11/5: 279-280 bundles. 8-1/49: 539, 560
grading system. 4-111/28: 333. 333/ Global Trigger Tool (GTT).14/10: 847. 854
after laparotomy. 4-1/23: 248. 254-255 Institute of Medicine
definition ofquality.14/9: 847. 853
INDEX-17
infectious complications of. 4-11117: 314-315, to cardiac box. 7-11/26: 439. 464
314[ hematuria in, 7-11116--17: 436. 456--457
management of. 3-111/38: 193-J 95 indications for resuscitative thoracotomy.
mild, 3-11/2: JOO. 115 7-11/2: 444-445
necrotizing. 4-11/2: 264. 274-275: 4-11121: 270. to kidney. 7-1132: 405. 424-425
291-292. 291/ resuscitative thoracotomy in. 7-1/23: 419
in pregnancy, 4-11/16: 288 Penetrating wounds. 7-11/l: 432. 444
Revised Atlanta Classification System for, Penicillin, 8-1/25-28: 534, 545-546
3-111138: 193. 194! antibiotic allergic cross-reactivity. 8-11/28: 569,
severe, 4-111/7: 30L 301/, 302/ 595-596
Papillary thyroid carcinoma (PTC), 1/4: 2; 1/9: Peptic ulcer disease (PUD), 3-1/2: 64. 75: 3-1/26--
13-14 28: 84; 3-11/13: 104. 124-125: 3-111/3: 165
in children, 6/10-11: 378. 388-389 bleeding. 3-111/9: 152. 169-170: 3-111124: 157.
follicular variant (FV-PTC), 6/9: 387-388 182: 3-IV/25: 229-230
management of, 6/9: 378. 387-388 perforated. 3-IV/ll: 201. 219
metastatic. 1/18: 6 recurrent. 3-IV/37: 209. 240-241
Papilloma., intraductal. of breast. 2/7: 40: 2/21: 31. surgical management of. 3-IV/37: 240-241
49 Percutaneous cholecystectomy. 4-1/4-7: 246.
Paraccntesis. large-volume. 3-111/39: 195 250-251
Paradoxical upg;ading reaction (PUR). 1/3: 9 Percutaneous cholccystostomy. 8-1140: 537.
Paraesophagcal hernias, 3-111/17: 154. 154/, 552-553
175-176, 175/ Percutaneous coronary intervention (PCI). 8-llJJ48:
repair of. 3-11/33: 143: 3-111/34: 189-190 661-663. 662/
typeIII.3-11133: 111:3-III/34: 160.161/, Percutaneous drain. 3-1123-25: 83
189-191, 190/ Percutaneous endoscopic gastrostomy (PEG).
Parastomal hernias. 3-11112: 150. 164-165; 10/38: 3-111/1: 150, 164
722. 757 Percutaneous thoracic duct embolization (PTDE).
after enterostomy. 3-11/10: 103, 122-123 8-llI/27: 646
keyhole repair of. 3-11110: 122. 123/ Percutaneous tracheostomy. 9/17: 672. 690--691
Sugarbaker repair of. 3-11/10: 122, 122/ Percutaneous transluminal angioplasty (PT A). 5/11:
synthetic mesh repair of, 3-111/2: 165 353-354
Parathyroid adenoma., 6/13: 378-379. 390 Periareolar mammary duct fistula. 2/15: 46-47. 46/
Parathyroid hormone, 6/6: 377. 386 Perinea! hematoma. 7-111/25: 486. 515-516
Parathyroidectomy. 6/13: 378-379, 390 Perioperative care. 8-111-50: 531-561: 8-11/1-51:
Parenteral nutrition 563--012: 8-Ill/1-51: 613--065
in critical care, 9/29: 700 acute statin therapy cessation, 8-111/44: 625,
for necrotizing pancreatitis. 4-11/2: 274 658-659
Paronychia., 12/21-23: 81 J. 825 analgesia. 8-111/24: 620. 644
Parotid gland, 1/16: 19, 19/ antibiotic prophylaxis. 8-11137: 571. 601---602:
Partial splenectomy, 10/18: 714, 738-739 8-llI/35: 623. 652---653
Partial thromboplastin time (PTT), 8-1132-34: 535. anticoagulation, 8-11/29: 569. 596: 8-111/2:
547-548: 8-11/44: 573. 607--008 627-628; 8-111/8: 615. 631
Patent processus vaginalis. 4-11/28: 297 anti platelet therapy, 8-111/8: 615. 631: 8-111/48:
Patient-controlled analgesia (PCA), 8-11/27: 569, 626,661-663
595: 8-11/49: 575. 611: 8-III/24: 620, 644 in atrial fibrillation. 8-1135: 535. 548; 8-11/47:
Patient safety, 14/1-10: 845-854 574. 609: 8-III/20-21: 619. 641-642:
Patient safety indicators (PS!s). 8-1/12-14: 533, 8-111/29: 621. 647---648; 8-111132: 622.
542-543 650--051
Pedunculated polyps. 3-JV/28: 232, 233/ blood transfusion, 3-IV/34: 208, 239
Pelvic bleeding, 7-1/36: 428 cardiac assessment, 8-11121: 589-591, 590/,·
Pelvic fluid collections. 7-11/22: 461-462, 462/ 8-III/6: 615. 630
Pelvic fractures. 7-1136: 407. 428 in diabetes mellitus. 8-11/36: 571. 600---601
management of. 7-111122: 512-513. 512/; glucose control. 8-11/36: 571, 600-601;
10/36: 756 8-111/40: 624. 656
after motorcycle crash, 7-111/22: 484. 485/, guidelines for cardiac assessment. 8-11121: 589.
512-513. 513.f.· 10/36: 721. 755-756 590/
in pregnancy. 7-11/11: 452 immunosuppression. 3-IV/4: 213-214
Pelvic injury. 7-llI/11: 478, 500-501 in metoprolol therapy. 8-111147: 625. 661
Penetrating colorectal injuries. 7-111/36: 492, 527 oxygen supplementation. 8-111150: 626.
Penetrating esophageal injury. 7-111/32: 488. 663---664
522-523 patient-controlled analgesia. 8-11/27: 569. 595:
Penetrating neck trauma, 7-111130: 488. 521 8-11/49: 575, 611: 8-111/24: 620. 644
Penetrating subclavian artery injury, 7-1117: 401. preoperative assessment of obstructive sleep
415 apnea (OSA), 8-111/11: 616. 634
Penetrating trauma. See also Gunshot wounds: Stab preoperative MRI. 2/2: 26. 36-37
wounds
INDEX-24
preoperative nutritional assessment. 8-111/10: Pneumonectomy, left extrapleural. 10/27: 718, 748,
616.632-633.633/ 749f 10128: 749. 750/
Perioral burns, 7-111/18: 506, 507/, 508/ Pneumonia
Peripheral arterial disease, 5/27: 343. 365-366 postpneumonia empyema, 8-1139: 537, 552
Peripherally inserted central catheters (PICCs). 5/8: ventilator-associated.1111: 4: 8-1/26: 534.
351: 5116: 356-357 545-546; 8-11/50: 575. 611
Peristomal varices. bleeding, 4-111/22: 307, Pneumothorax
328-329 accidental. 8-1/13: 533
Peritoneal adenomucinosis, disseminated (DPAM). after fall. 7-11116: 477. 497-498
11119: 790-791 occult. 7-1130: 404. 423: 7-111/6: 497-498
Peritoneal carcinomatosis. 11/27: 771. 796-797 spontaneous. 10/34: 72L 754-755
Peritoneal mucinous carcinomatosis (PMCA). Polycystic liver disease, 4-11/20: 268. 290-291
11/19: 790-791 Polydioxanone, 4-1116-18: 247. 253
Peritoneovenous shunts, 3-111/39: 196 Polyglactin 910. 4-1/16-18: 247. 253
Peritonitis. 3-111/15: 154 Polyglecaprone 25. 4-IJ16-18: 247. 253
bacterial. 4-111/20: 327-328 Polypectomy
feculcnt. 3-IV/18: 202. 224 follow-up, 3-1/38: 93
localized. 8-1/28: 534, 545-546 postpolypectomy coagulation syndrome. 3-1/4:
purulent. 3-11/6: 119. 119t: 3-IV/18: 224 76
Peroral endoscopic myotomy (POEM). 3-1/23-35: Polyps
90: 3-ll/3: 116: 4-IIU4: 313 adenomatous, 3-IV/28: 232
Petechial rash. 7-111/31: 488. 522: 8-111/15: 617, classification of. 3-IV/28: 232
617/, 635-636. 636/ colonic, 3-1/4: 64; 3-IJ38: 71: 3-11/28: 108,
pH testing. 3-IV/7: 216 137-138: 3-IV/33: 238: 4-1/19: 247, 254;
Pharyngeal cancer. 1/15: 1S 4-1/28: 249
Pharyngolaryngocsophagectomy (PLE). 3-111/36: duodenal. 3-11129: 138, 138t; 3-IV/9: 200.
191-192 200/, 217. 218!
Phenothiazines. 8-11/48: 610 fundic gland, 3-11/30: 139
Phenylephrine, 8-1/18--20: 533, 544 gallbladder. 4-1/6: 246. 251; 4-111/12: 303. 319
Pheochromocytoma. 6/14: 391; 6/15-19: 379, 392, gastric. 3-11130: 139-140
393/ hyperplastic. 3-11/30: 139
malignant. 4-1113: 264. 275-276. 275/ inflammatory fibroid, 3-11130: 140
Phlebitis malignant. 3-IV/28: 232. 233/
with central venous catheters. 5/16: 356 pedunculated. 3-IV/28: 232. 233/
with peripherally inserted central catheters. rectal. 3-11/7: 102, 120
5116: 357 rectosigmoid, 3-1117: 102. 120
postphlebitic syndrome, 8-1/42: 554 sessile. 3-IV/28: 232. 233/; 4-1/28: 249
Phlegmon. 3-1/23-25: 68. 83-84; ! 19t; 3-IV/5: Polytrauma. 8-111/41: 624, 656-657
214: 4-ll/2: 281. 28lt: 10116: 737: 10125: 747 Polyuria. 9/31: 675, 701-702
Photodynamic therapy. 3-1/1: 74 Popliteal artery
Phyllodes tumor. 2/38-43: 34. 59-60 in amputation. 5/3: 347
Physician-assisted death, 13/2: 828. 834-835 blunt injury, 7-1126: 403, 421
Physician impairment, 13/3: 828. 835-836 femoropopliteal, 5/6: 337. 349
Physician-patient confidentiality. 13/14: 832. 843 in knee dislocation, 5/5: 336-337. 337/, 348-
Pilonidal disease. 12/4: 806. 807/ 349; 5/35: 344. 372
treatment of. 12/4: 815; 12/8: 808. SIS: 12/14: popliteal artery entrapment syndrome. 5/33:
810.823 369
Pituitary resection. failed. 6/15-19: 379 Popliteal vein injury. 5/35: 344, 372
Plaques, carotid, 5/19: 359 Portal hypertension, 3-IV/36: 209, 240
Platelet dysfunction, 8-1/32-34: 547-548 Portal hypertension ascites. 3-111139: 163. 195-196
Platelet transfusion. 8-111/21: 642; 9/41: 676, Portal hypertension varices. 3-1/3: 64, 75-76
703-705 Portal vein thrombosis. 4-11119: 268, 290: 4-111117:
Platinum-based agents, 11/38: 772. 801 305,324
Pleomorphic lobular carcinoma in situ (PLCIS), Portal venous gas. 8-11/16: 585, 585/
2/4: 26, 37-38 Positive end-expiratory pressure (PEEP), 8-IJ50:
Pleural effusion. 7-11/20: 437, 460 539,560-561
Pleurodesis, mechanical. 10/34: 754-755 auto-PEEP. 9/11: 671. 686-687
Pneumatic dilatation. 3-1/23-35: 89-90 Postoperative care
Pneumatosis intestinalis. 3-11/12: 103. 124: in alcohol withdrawal. 8-111/42: 625, 657--658
3-IV/32: 237 early feeding. 8-111/25: 644--645
after cardiac surgery, 8-11/16: 585, 585/ pain control. 8-11/11: 565. 580: 8-11/49: 575.
risk factors for. 3-11/12: 124 610-611
Pneumococcal vaccine. 7-111/3: 496: 7-111/29: telehealth visits. 8-111/26: 620. 645
519-520. 520t Postoperative pulmonary complications (PPCs).
Pneumomediastinum, spontaneous (SPM). 7-111/20: 8-ll/10: 565. 578-579
510
INDEX-25
Postoperative thromboembolic events, 8-1112: 533, Proximal gastric cancer, 3-111/6: 168
542-543 Pseudoan;urysms. 7-11/16-17: 436, 436/, 457-458,
Postparacentesis circulatory dysfunction (PCD). 457/ 458/
3-III/39: 195 Pseudocysts
Postphlebitic syndrome, 8-1142: 554 discriminating features of. 4-11/4: 278!; 11/15:
Postpolypectomy coagulation syndrome. 3-1/4: 76 785!
Postthrombotic syndrome (PTS). 5/22: 361-362; symptomatic, 3-111/38: 194
8-1/42: 537, 554 Pseudomyxoma peritonei (PMP), 11/19: 790-791
Potassium chloride, 8-11113: 565, 582-583 Pseudotumors. 4-111/12: 319
Pouchitis. 4-111/24: 309 Psoas hitch, 7-11136: 473
Povidone-iodine. 8-1/9-11: 532, 541-542 PTEN mutations, 2/11: 43
Prednisone, 3-IV/4: 199 Pubic inguinal pain syndrome (PIPS). 4-111115: 322
Pregnancy. 7-11/25: 463-464 Pubic ramus fracture. 7-111/11: 478, 500-501
acetabular fractures in. 7-11111: 452 Puestow procedure. 4-1/30: 259-261, 260/;
breast cancer in. 10/5: 711. 726 8-111/42: 625
cardiovascular changes during. 9/19: 691-692: Pulmonary artery rupture. 9/5: 669. 682-683
10/3: 725 Pulmonary carcinoid tumors, 10/30: 720, 751-752
cholecystitis in. 4-11/16: 267. 288 Pulmonary edema. noncardiogenic, 9/49: 677. 707
cholelithiasis in, 10/2: 710, 725 Pulmonary embolism. 9/9: 671/
deep vein thrombosis in. 5/13: 339. 354-355 postoperative, 8-111/33: 623
gallstone pancreatitis in. 4-11/16: 288 treatment of. 8-111141: 624. 656-657: 9/9: 670.
motor vehicle collisions in. 7-11/11: 435, 452: 685
7-11/25: 439,463-464; 9/19: 672; 10/3: 710, Purulent peritonitis, 3-IV/18: 224
725 Purulent skin and soft tissue infection, 12/3: 806,
pelvic fractures in. 7-11/11: 452 813, 814(
postbariatric surgery. 10/1: 710. 724 Pyloric balloon dilatation, endoscopic. 10/23: 746
pulmonary changes during. 10/3: 725 Pyodenna gangrenosum, 3-1129-32: 88. 89/;
recommendations for seat belt use in. 7-11/11: 8-III/30: 622, 648-049, 648/
452
recommendations for total weight gain in. 10/1: Q
724. 724t QT prolongation, drug-induced, 8-111119: 618,
trauma in, 7-11/11: 435, 452 619/ 640-641
ultrasound in. 7-11/11: 452 Quality. 14/9: 847, 853
Pregnancy-associated breast cancer. 2/22: 31. 51 Quality adjusted life years (QALYs). 8-11/12: SSL
Prehabilitation. 14/2: 846. 848-849 582
Preoperative care Quality guidelines. 14/9: 847. 853-854
in diabetes mellitus. 8-11116: 630 Quality improvement (QI), 13/1: 828. 834
for elderly. 8-11/32: 570. 598; 8-111/6: 615. 630 National Surgical Quality Improvement
nutritional assessment. 8-111110: 616. 632-633, Program (NSQIP"°). 8-111/20: 641-642;
633/ 8-III/51' 664; 13/1: 834
recommendations for testing. 8-11116: 630
Pressure control inverse ratio ventilation (IRV),
9/11: 686-<587 R
Primary sclerosing cholangitis (PSC), 4-111/16: Radial scars. 2/7: 40
323; 4-111/24: 330-331 Radiation enteritis. 11/39-42: 773, 801-802
Pringle maneuver, 7-111117: 505, 505/ Radiation sickness, 7-1121: 402. 417-418
Privacy. 13/14: 832. 843 Radiation therapy
Probiotics. 3-11124: 134: 8-111112: 634 axillary, 2/16: 47
Promethazine. 8-11148: 610 for breast cancer. 2/16: 47
Promotility agents. 3-1120-22: 83 for gastric cancer, 3-IV/2: 212
Propofol. 9/16: 672. 690; 9/36: 676. 702-703 Radiofrequency ablation (RFA). 3-111: 74; 4-11126:
Proportional assist ventilation (PAV). 9/11: 687 296
Propranolol. 7-1/19: 416 Radiotherapy. neoadjuvant. 3-111/33: 160. 189:
Protein requirements, 9/32-34: 676. 702 11/6: 778; 11/450 773, 802-803
Rapid~shal!ow breathing index (RSBI). 9/4:
Prothrombin complex concentrate (PCC), 7-1/1-3:
398, 408; 7-11/21: 461 681-082
comparison with fresh frozen plasma, 7-11/13: Rapid thromboelastography (r-TEG). 7-111/28:
435,453-454 517-519
4-factor, 8-111/2: 627-628 common findings in, 7-111/28: 518t
3-factor. 8-111/2: 627-628 therapeutic interventions based on. 7~111/28:
Prothrombin time. 8-1/32-34: 547-548 519/
Proton pump inhibitors (PPis). 8-11/5-8: 564 Rash
for GER and GERD, 10/10: 712. 729 antibiotic, 8-111115: 637, 637/
side effects of. 6/7: 377. 386 petechial, 7-111131: 488, 522: 8-111/15: 617,
for upper gastrointestinal bleeding. 3-1/20-22: 617/ 635-036, 636/
83; 3-11/38: 146-147; 3-IV/25: 229 Rattlesnake bite, 7~111/4: 476. 496-497
INDEX-26
Rectal cancer. 3-1/6: 65. 79: 3-11/26: 107, 135-136: Retroperitoneal sarcoma (RPS). 11114: 766. 783:
11/43-47: 773, 802-803 11/18: 768. 768/ 788-789. 789/
adenocarcinoma. 11/6: 764/; 11117: 767, Revised Cardiac Risk Index (RCRI). 8-111120:
787-788 641--642: 8-111/51: 664
advanced, 3-1/13-16: 81-82 Rewanning. 7-11133: 470
follow-up. 3-1142: 96 active. 9/21: 673. 693
local excision of. 3-11/19: 105, 130-131 Rhabdomyolysis. 5/32: 344. 368
management of. 11/6: 764. 778: 11128: 771. Rho(D) immune globulin, 7-11/11: 452
797 Rib fractures, 7-11/7: 449
staging evaluation of, 3-IV/19: 203, 225 after ATV crashes. 7-1136: 407, 428
T3N 1. 3-1/42: 72. 96 in elderly. 7-1129: 404. 422-423: 7-111/6: 477.
upper-third. 11/6: 764. 778 497-498
Rectal dysplasia. 3-111/21: 156. 179 after fall, 7-11116: 477, 497-498: 7-111135: 492,
Rectal foreign bodies. 3-1/45: 73. 98: 3-111/14: 153. 492/ 525-526, 526/
153.f.172 initial evaluation of, 7-111133: 490. 523
Rectal polyps. 3-1117: 102, 120 after motor vehicle collisions, 7-1129: 404,
Rcctosigmoid cancer. 3-11126: 135-136; 3-IV/30: 422-423; 7-1117: 433. 449: 7-11/18: 437.
206 458-459
Rectosigmoid polyps. 3-11/7: 102. 120 multiple, 7-11127: 440, 465
Rectus sheath hematoma. 4-111/19: 306. 326-327. stabilization of. 8-11112: 565. 581
327/ surgical fixation of. 7-11115: 435. 455-456;
Red blood cell transfusion. 8-11/15: 565. 584 7-11/27: 465
Reflux, 3-1/5: 65; 3-111/29: 159 Right femoral popliteal below-knee bypass, 5/11:
gastroesophageal. See Gastroesophageal reflux 339
(GER) Right hepatic artery, 3-1/40: 94-95
laryngopharyngeal, 3-IV/7: 216 Right knee dislocation. 5/5: 336--337, 337/,
Renal cell carcinoma. 10/37: 721. 756 348-349
Renal failure, 9/32-34: 702 Right laryngeal nerve. 1/4: 10
Renal retroperitoneal hematoma. 7-111/33: 490, 523 Right subclavian artery
Renal trauma. 7-1132: 405, 424--425 aberrant (ARSA). 5/25: 343, 364
American Association for the Surgery of course of, 1/4: 2, 9-10; 1/5: 3
Trauma grading system for. 7-1132: 405: indications for surgical correction. 5/25: 343
7-111/21: 484. 511/ Risk-benefit ratios. 8-11/12: 582
after all-terrain vehicle (ATV) crash. 7-11/32: RittLximab. 12/5: 816
441, 470 Rivaroxaban, 7-11121: 461; 8-1115-17: 533, 543;
blunt 7-1120: 402. 417: 7-1132: 424-425: 8-1/21-24: 534, 544--545; 8-11/5-8: 564, 577
7-11/32: 441. 470: 7-III/21: 484, 510-511. for acute venous thromboembolism. 8-11120:
511/ 588. 5891
classification of. 7-1132: 405, 424 anticoagulation effects of. 8-111/29: 647t
penetrating. 7-1132: 405, 424--425 laboratory evaluation of. 8-11114: 565. 583-584
pseudoaneurysm in. 7-11/16--17: 457-458. 457/, Robotic surgery. 8-11112: 565. 581
458/ Rockall score. 3-11114: 167!
stab wounds, 7-11116--17: 436. 436/, 456-458, Rome III criteria, 4-11115: 287, 287t
457/ 458/ Rotational thromboelastography. 8-11114: 583-584.
Resident duty hour restrictions. 13/6: 829. 837-838 583/
Resistance training, inspiratory. 8-11110: 565. Rotational thromboclastometry (ROTEM).
578-579 7-111/28: 517-519
Respiratory quotient. 8-111-3: 540; 8-1129-31: 534. Roux-en-Y
546-547. 5461 cystojcjunostomy. 3-111138: 194
Resuscitation esophagojejunostomy, 3-111/37: 192
cardiopulmonary, 13/13: 832. 842 gastric bypass. 3-1114: 101, 11 7
damage control. 7-111/10: 500 gastrojejunostomy. 3-111/3 165:
""do not attempt resuscitation" (DNAR) orders. hepaticojejunostomy. 4-111118: 324-325
13/7: 830. 838-839
for septic shock, 9/28: 674, 698 s
in trauma patients. 7-1125: 403. 420; 7-1131: Sl00A12 (calgranulin C). 3-IV/16: 223
405. 423-424 Sacroiliac joint disruption. 7-1/36: 407. 428
Resuscitative endovascular balloon occlusion of the Saddle embolism. 9/9: 685
aorta (REBOA), 7-11/14: 435. 454-455, 455/ Safe Practices for Better Healthcare, 8-1112-14:
Resuscitative thoracotomy. 7-11/2: 432 542-543
indications for. 7-1123: 403. 418-419: 7-1112: Safety. 14/1-10: 845-854
444-445 critical view of safety. 4-111121: 307. 328; 13/5:
after prehospital CPR. 7-1/23: 418-419 837
RET mutation analysis. 11/1: 774 patient safety indicators (PSis). 8-1/12-14: 533,
Retroperitoneal hematoma, 7-11/32: 441. 470: 542-543
7-111/33: 490. 523
INDEX-27
Surgical Safety Checklist (WHO). 14/3: 846. Sequential Organ Failure Assessment (SOFA),
849 8-1/41: 553
Sandwich sign. 4-11/4: 277: 11/15: 784 Serotonin. 3-111/5: 118; 3-111120: 178; 10/30: 751;
Saphenous vein grafts. 7-1/26: 421 11/13: 782:
Sarcoma Sessile polyps, 3-11/28: 108. 137; 3-JV/28: 232
dermatofibrosarcoma protuberans. 12/18-20: anatomic landmarks, 3-IV/28: 233/
811.824 colonic, 4-1/28: 249
embryonal. 10/21: 742 Sezary syndrome. 11/3: 775
liposarcoma (LPS), 4-11/4: 278t; 11/4: 776; SF-36 testing, 14/2: 846, S49
11115: 784, 7851 Shaken baby syndrome. 7-111/38: 52S
retroperitonea!. 11/14: 766. 783: 11/18: 768. Shared decision making. 13/8: 830. 839
768/ 788-789, 789/ Shin splints, 5/33: 344, 369
well-differentiated Jiposarcoma (WDLPSs). Shock
11/4: 776 cardiogenic. 8-1147: 539. 559; 9/22: 673,
Saturday night palsy. 5/31: 368 693-694
Scald burns. 7-111/2: 495; 7-111/18: 506. 506/ differential diagnosis of. 9/6: 6S3
Scars, radial. 2/7: 40 hemorrhagic. 7-11/34: 442, 471-472; 7-111127:
Sc\erosing adenosis, 2/7: 40 487, 517
Sc\erosing cholangitis, 4-111/16: 322-324 hypovolemic. 7-11/1: 432. 444; 7~111/10: 500
MRCP findings in, 4-111/16: 323/ neurogenic. 7-111/10: 500
primary, 4-111/16: 323: 4-111/24: 330-331 septic, 4-1/4: 246; 8~111/36: 623, 653--654; 9/1:
Sc\erosing mesenteritis. 4-11/4: 278!; 11/15: 7851 668, 679: 9/6: 683: 9/28: 674, 698, 699/
Sclerotherapy Shunts
for bleeding esophageal varices. 3-1/3: 75 intravascular. 7-1122: 402, 41 S
for hydrocele. 10/39: 722. 757 peritoneovenous. 3-111/39: 196
Scopolamine. &-11/48: 610 temporary arterial. 7~11/1: 444
Screening temporary vascular, 7-1/22: 402, 418; 7-1111:
for blunt cardiac injury, 7-111/8: 477, 498-499 444
for blunt cerebrovascular injury. 7-1/27: 421- transjugular intrahepatic portosystemic (TIPS).
422; 7-11/28-30: 465-466 3-111/39: 196: 3-IV/36: 240: 4-111/9: 316;
for colon cancer. 3-111/25: 157; 3-IV/6: 199. 4-111/22: 328-329
215 Sigmoid cancer, 4-1/28: 249. 258
for colorectal cancer. 3-IV/16: 223 Sigmoid diverticulitis, 3-1116: 102. 119-120, 119/
for lung cancer. 10/24: 717. 746--747 with abscess. 3-111118: 155. 155/, 176--177.
for obstructive sleep apnea. 8-11124: 592-593: 176/
8-111/11: 616, 634 guidelines for management of, 4-11/8: 281
Screening breast MRI. 2/3: 26, 37 Hinchey classification of. 3-1116: l 19t; 4-11/8:
Screening colonoscopy. 3-111/25: 182-183: 4~1119- 2Si, 2Slt
22: 247, 254 management of. 4-11/8: 266. 2S l-2S2
Scrotal pain, 10/40: 722, 757-758 Sigmoid volvulus, recurrent. 3-111122: 156. 179-
Seat belt sign. 7-111/14: 479. 479/, 502-503 180, 180/
Second-look laparotomy. 10/6: 727 Silk suture, 4-1/16-18: 247. 253
Security of information, 13/14: 843 Skin. 12/1-23: S05-S25
Self-determination/autonomy. 13/10: 840-841 Skin and soft tissue infection (SSTI), 8-1/27: 534,
Self-expanding metal stents (SEMS). 3-11/32: 141. 545-546
142/ management of, 12/3: 813. S14f
Sentinel lymph node biopsy (SLNB) necrotizing, 8-111/7: 615, 630-631
complications of, 2/16: 47 necrotizing soft tissue infection (NSTI). 12/9:
in melanoma. 11/9: 780; 12/17: 81l.824; 808,818-821,819/
12/18-20: 81 I. 824 purulent.12/3: 806. 813. Sl4f
in Merkel cell carcinoma. 12/17: SI 1. 824 Skin antiseptics, 8-1/9-11: 541-542
in node-positive breast cancer, 2/5: 27, 38-39 Skin cancer, See also specific cancers
Sepsis in immunosuppressed patients, 12/7: SOS, SI 7
biliary. 8-1/40: 537. 552-553; 8-1/44: 555-556 Merkel cell carcinoma. 11/7: 765, 77S-779
nutritional support in. 9/42--44: 705 Skin grafts, 11112: 766. 766/ 781-782
overwhelming postsplenectomy (OPSS). Skin lesions. stoma~associated. 8-111130: 622.
7-111129: 519-521 648-<i49, 648/
prevention of, 7~111/29: 488, 519-521. 5201 Skin rash
Surviving Sepsis Guidelines, &-1144: 555: 9/10: antibiotic. 8-111/15: 637. 637/
670, 686: 9/28: 698 petechial. 7-111/31: 488. 522: 8-111/15: 617.
Septic shock, 4-1/4: 246 617/, 635-<i36, 636/
early goal-directed therapy (EGDT) for. 9/28: Sleep apnea. obstructive (OSA). 8-11/24: 568.
698, 699/ 592-593; 8-111/11: 616. 634
management of. &-111/36: 623. 653-654; 9/1: Sleep deprivation, 13/3: S2S. 835-836
668, 679: 9/6: 669, 683: 9/28: 674, 698 Sleeve gastrectomy. 3-11/4: I 01, 117; 3-IV/8: 216
INDEX-28
Slipped Nissen fundoplication. 3-11/35: 112. blunt. 7-1130' 404. 423: 7-1/35' 406. 427:
144-145, 145/ 7-1113, 432, 445-446; 7-111/24, 485. 514-515
Small bowel cancer, 3-111/28: 185; 11/31: 799 grade III laceration. 7-111/5: 476-477. 497
adenocarcinoma, 3-111/8: 152, 169; 3-111/15: grade IV. 7-11/34' 442, 442/, 471-472. 47lf
173 7-111/3, 476. 496; 7-111112, 478. 478/
8-ce!l lymphoma. 3-111/32: 160, ISS-189 high-grade. 7-111/12: 501-502
metastasis. 3-111/15: 172 isolated, 7-1/35: 406, 427
Smal! bowel intussusception. 3-111/19: 177-178. after motor vehicle collision. 7-111112: 478,
177/ 501-502
Small bowel obstruction. 3-1115: 101, 117-118; traumatic. 7-111/12: 501
3-111/10: 152. 170 Spontaneous bacterial peritonitis (SBP). 4-111/20:
due to incarcerated ventral hernia. 3-IV/39: 327-328
210.242 Spontaneous pneumomediastinum (SPM). 7-111120:
due to long segment ileocolonic 510
intussusception. 3-IV/29: 205 Spontaneous pneumothorax. 10/34: 72L 754-755
secondary to intussusception. 3-111119: 155, Sports hernia, 4-111115: 322
155/, 177-178.177/ Sports injury
secondary to obturator hernia. 4-11/23: 271. chronic exertional compartment syndrome.
293-294 5133, 344, 369
signs of. 4-11123: 293 right knee dislocation. 5/5: 336-337. 337/.
Small bowel operations. 8-111/14: 617. 635 348-349
Small bowel pneumatosis, 3-IV/32: 237 shin splints, 5/33: 344, 369
Small bowel tumors Squamous cell carcinoma
GISTs, 3-111115: 173 esophageal. 3-111/36: 161. 191-192: 3-IV/38:
ncurocndocrine. 3-111/15: 172. 173. 173/. 174 209.241
Small bowel volvulus. 3-IV/32: 237-238. 237/ oropharyngeal. 1115: 5. 18
Small cell lung cancer. 11110: 765. 780-781 Squamous intraepithelial lesions, high-grade,
Smoking cessation. 8-11110: 565, 578-579: 3-II1116' 154, 174
8-111/34: 623. 652 Stab wounds. 7-1112: 432: 7-11119: 437. 459-460
Snake bite, 7-111/4: 476. 496-497 to anterior neck. 7-111/13: 479. 502
Social media. 13/9: 840; 13/14: 832. 843 in cardiac box. 7-11126: 439. 464
Society of American Gastrointestinal and CT imaging. 7-11119: 459. 459/
Endoscopic Surgeons (SAGES), 8-11125: 594 indications for resuscitative thoracotomy.
Soft tissue, 12/1-23: 805-825 7-1112, 444-445
Soft tissue infections. See Skin and soft tissue to neck, 7-111130: 488. 521
infection (SSTI) renal. 7-11116-17' 436. 436/, 456-458. 457/
Spherocytosis. hereditary. 10/18: 714. 738-739 458/
Sphincter of Oddi disorders. 4-11/15: 287. 287t subclavian artery, 5/23: 342. 362-363, 362/
Sphincterotomy. endoscopic retrograde survival after thoracotomy, 7-1112: 445
cholangiopancreatography with (ERCP/ES). thoracoabdominal. 7-1/18: 401. 415-416:
3-IV/17: 202. 224: 8-11/31: 570. 597-598 7-1124: 403, 419-420; 7-11/5: 433, 447-448
Spider bites, 7-1/4-6: 398, 408-409, 409/,· 7-11/31: Staffing, 14/6: 84 7. SS I
441.467-470,469/ Standard of care. 13/5: 837
Sp;decs. 7-11/31' 441/, 467. 468/, 469/ Standard procedures: modifications to. 13/15: 832,
Spigelman scoring system. 3-11/29: 138. 138t; 843-844
3-IV/9: 217, 218! Staphylococcus aureus
Spinal accessory nerve. 1/6: 3. 11 breast abscess. 2/21: 51
Spindle cells, 4-11/18: 268 central-line-related bloodstream infections.
Splenectomy. 4-11/13: 266 9/20: 692
for hereditary spherocytosis, 10/18: 714, methicillin-resistant (MRSA). 8-1125-28:
738-739 545-546; 8-1143, 537, 554-555: 8-1137, 549:
for ITP. 4-111/26: 309, 332 8-11134, 599.12/3, 813-814. 814fi 12/11-12,
overwhelming postsplenectomy infection 821
(OPS!). 4-11/14' 267. 285; 7-IIl/3' 496: methicillin-susceptible (MSSA). 8-1143: 554-
8-11/43, 573. 607 555, 1213, 813-814. 814/
overwhelming postsplenectomy sepsis (OPSS). monomicrobial infections. 12/9: 820
7-II1129' 519-521 Starvation, 8-1/1-3: 532. 540
partial. 10/18: 714. 738-739 Statin therapy. 8-111/21: 642: 8-111144: 625,
postsplenectomy vaccines, 7-111/3: 476, 496; 658-659
7-111/29, 488. 519-521, 5201; 8-11/43, 607 Stenosis, anastomotic. 5/11: 353-354
total. 10/18: 714. 738-739 Stents and stenting
Splenic abscesses. 4-111/13: 304. 319-320. 320/ coronary, 8-111148: 661-663, 662/
Splenic cysts. nonparasitic. 4-111/3: 300. 312. 312/ drug-eluting coronary stents. 8-11/29: 596;
Splenic injury 8-11118, 615. 631; 8-111/21, 619, 642
after all-terrain vehicle (A TV) crash, 7-11/32: duodenal. 3-1/39: 94
441,470 endoscopic transpapillary gallbladder stent
INDEX-29
Tranexamic acid (TXA). 7-1/1-3: 398, 408: 7-1/31: Tubular adenoma, 3-1/38: 71
405. 423--424: 7-11/10: 434. 451--452 Tumors
indications for, 8-11/41: 572. 606: 9/41: 676, of anus, 3-IV/3: 212-213
704 atypical lipomatous (ALT), 1114: 763, 763/,
Transesophageal echocardiography, 9/22: 693-694 776
Transfusion-related acute lung injury (TRALI), carcinoid. See Carcinoid tumors
9/49-52: 677. 707 desmoid. See Desmoid tumors
Transfusion-related circulatory overload (TACO). fibrous, solitary. 4-11/4: 2781; 11/15: 785t
9/49-52: 677. 707 gastric. 6/7: 377, 386--387
Transjugular intrahepatic portosystemic shunt gastrointestinal stromal. See Gastrointestinal
(TIPS). 3-1/3: 75: 3-111/39: I 96 stromal tumors (GISTs)
for bleeding varices, 4-111/22: 328-329 liver. 10/21: 716. 742-745. 742/ 743/ 744[
indications for. 4-111/9: 316 MALT-type. 3-111/32: 189
for portal hypertension, 3-IV/36: 240 neuroendocrine. See Neuroendocrine tumors
Transthoracic echocardiography. 7-1/25: 403, 420 neurogenic. 4-11/4: 2781
Transverse colon cancer, 4-11/18: 268 pseudotumors. 4-111/12: 319
Trastuzumab, 3-IV/2: 212: 11/25: 795: 11/36-38: Vanek. 3-11130: 140
772. SOI Turcot syndrome. 3-111/21: 179
Trauma, 7-1/1-37: 397--429: 7-11/1-37: 431--473: Turner syndrome. 10/22: 745
7-11111-39: 475-530 Typhlitis. 3-IV/I: 198. 211-212; 11121: 792
acute coagulopathy of (ACOT). 9/39-41: Tyrosine kinase inhibitors
703-705 for advanced GISTs, 11134: 772. 800
bleeding in, 7-11/12: 453 for GISTs. 3-11/17: 105. 129-130
blunt abdominal, 7-11/34: 442. 442/, 471--472, for medullary thyroid cancer. 11/32: 772.
47If· 7-III/34: 491. 491/ 523-525. 524f 799-800
blunt chest, 7-1117: 433, 449; 7-11/35: 443, 472:
7-111/35: 492, 525-526 u
blunt diaphragmatic rupture. 7-111134: 491, Ulcerative colitis. 3-1/8-12: 66, 80--81; 3-IV/27:
523-525,524/ 205. 231-232; 3-IV/33: 208. 238-239; 4-111/24:
blunt liver. 7-111/15: 480, 503-504 309. 330-331
blunt pancreatic, 7-111/39: 494, 529-530, 529/ Ulcers
blunt renal. 7-1/20: 402, 417: 7-1132: 405, duodenal, 3-11/38: 113; 3-IV/25: 204: 3-IV/37:
424-425; 7-11/32: 441. 470; 7-111/21: 484. 209. 240-24 l
510-511. 51 lf duodenal bulb, 3-111/3: 150, 165
blunt thoracic, 7-111/34: 491. 491/, 523-525. gastric. 3-1/2: 75
524[ after gastric bypass, 3-11/39: 113, 147-148
esophageal injury. 7-111/32: 488, 522-523 peptic. See Peptic ulcer disease (PUD)
gunshot wounds. 5/4: 336, 348 stress, 9/14: 672. 689
hcmaturia in. 7-11/16-17: 436. 456-458 venous leg, 5/17: 340. 357-358
intraosseous access in. 7-11119: 477. 499 venous stasis. 5/17: 357-358
multisystem, 10/36: 721. 755-756 Umbilical hernia. 14/7: 847
nonaccidental (NAT), 7-111/18: 506. 506/; Underfeeding. 8-1/29-31: 546-547
7-111/38: 493, 528-529 Unfractionated heparin (UFH). 8-1/21-24: 534,
nonaccidental head injury (NAHI). 7-1/37: 407, 544--545; 8-11/17: 567. 586
428--429. 429! Unilateral vocal cord paralysis (UVCP). 1113: 4.
nutritional support in. 9/42-44: 705 16-17
penetrating. See Gunshot wounds; Penetrating Upper-extremity arterial injury, 7-11/1: 432, 444
trauma: Stab wounds Upper-extremity compartment release. 5/32: 368
polytrauma, 8-111/41: 624. 656--657 Upper-extremity compartment syndrome, 5/32:
during pregnancy. 7-11/11: 435. 452 344.368
renal. 7-1132: 405, 424-425 Upper-extremity deep vein thrombosis. 5/16: 340.
resuscitation in, 7-1125: 403, 420; 7-1131: 405, 356-357
423--424 Upper gastrointestinal bleeding. 3-1/20-22: 82-83:
thromboelastography in. 8-11151: 575, 612 3-1/26-28: 68. 84: 3-11/38: 113. 146-147;
vascular. 5/3: 336, 347-348 3-111/4: 150. 166: 3-111/9: 152. 169-170;
Traumatic brain injury (TBI) 3-111124: 157. 182: 3-IIl/26: 158. 183-184
after fall, 7-111/35: 526/ in elderly. 8-111/2: 614. 627-628
guidelines for management of. 7-11/6: 448 Glasgow-Blatchford score, 3-111/4: 166. 167t
after motor vehicle collision, 9/31: 675, management of, 3-111/4: 166; 3-IV/25: 229
701-702 pre-endoscopic Rockall score, 3-111/4: 167t
after motorcycle crash. 7-11/6: 433, 448 Upper gastrointestinal injury, caustic, 3-11/8: 102-
severe. 7-11/6: 448 103, 120-121. 121/; 7-11/23-24: 439, 462, 463
Triage, 7-111/37: 528 Upper rectal/rectosigmoid cancer. 3-11/26: 107.
Tuberculous lymphadenitis 135-136
cervical. 1/2-3: 2, 9 Ureteral injury, 7-11/22: 461--462, 462/,· 13/11: 831
treatment of. 1/3: 9
INDEX-32
x
X-rays, American Society of Anesthesia
recommendation for smokers, 8-111/6: 630
z
Zenker diverticulum. 3-IV/12: 219
Zollinger-Ellison syndrome. 3-11/36: 145
------~------